Download as pdf or txt
Download as pdf or txt
You are on page 1of 605

PREPARED BY:-

Dr. Dler R. Abdulkareem


1
‫ميحرلا نمحرلا هللا مسب‬
‫و‌ميهرهبان‬
‫‌‬ ‫خشنده‌‬
‫‌‬ ‫‌‬
‫اوى‌خوای‌به‬ ‫به‌ن‬
‫‌‬

‫‪In the name of ALLAH, most gracious,‬‬


‫‪most merciful‬‬

‫‪2‬‬
3
Educational Testing
and Assessment
Systems
Derm In-Review | Board Review Study Questions
www.dermatologyinreview.com

Prepared by:-
Dr. Dler R. Abdulkareem

Written and Developed by

4
Board Prep: Custom Quizzes & Timed
Practice Exams

Chapter No. of Page


CHAPTERS
No. Questions No.

All Questions 3451 604

8 Infectious Diseases of the Skin 291 7

9 Pediatric Dermatology 128 115

10 Cutaneous Manifestations of Systemic Disease 128 161

11 Disorders of the Hair and Nails 192 215

12 Bullous and Vesicular Dermatoses 155 282

13 Photobiology and Photosensitivity Disorders 137 338

14 Plants and Creatures of Dermatologic Significance 134 386

15 Dermatologic and Cosmetic Surgery 182 434

16 Dermatopharmacology 296 500

5
6
Chapter -8-
Infectious Diseases of the Skin
1
This HPV strain is responsible for Bushke-Lowenstein tumor and also for anogenital warts:
A. HPV 6, 11
B. HPV 1
C. HPV 2, 4
D. HPV 3, 10
E. HPV 7
►A

Anogenital and Buschke-Lowentein tumor is caused by HPV 6 and 11. HPV 1 is from
palmoplantar warts. HPV 2, 4 causes common warts HPV 7 causes Butcher's warts.

2
The causative organism that is responsible for pitted keratolysis is:
A. Micrococcus sedentarius
B. Streptobacillus moniliformis
C. Klebsiella pneumonia
D. Neisseria meningitides
E. Burkholderia mallei
►A

Pitted keratolysis is caused by Micrococcus sedentarius. The clinical features are shallow 1-3mm
pits on plantar surface of feet. The treatment is topical erythromycin, clindamycin, or benzoyl
peroxide.

3
Most common causative agent for the lesion shown in this 5-year old child is
A. Staphylococcus aureus
B. group A β-hemolytic streptococci
C. Pseuomonas
D. kleibsiella
E. H.influenza
►A

The picture shown is Impetigo, which is a common, contagious superficial skin infection caused
most commonly by staphylococci. Although seen in all age groups, the disease is most common
in infants and children. Lesions may involve any body surface but occur most frequently on the

7
exposed parts of the body, especially the face, hands, neck, and extremities. There are two classic
forms of impetigo, bullous and nonbullous. Nonbullous impetigo accounts for more than 70% of
cases. Historically was caused primarily by group A β-hemolytic streptococci (GABHS), but
now appears to be most commonly caused by S. aureus. Anaerobic organisms may also be
recovered from lesions of nonbullous impetigo. (Reference: Brook I, Frazier EH, Yeager JK.
Microbiology of nonbullous impetigo. Pediatr Dermatol 1997;14(3):192-195.)

4
A 4 year-old boy is diagnosed with Staphylococcal Scalded-Skin Syndrome. Vesiculation in this
disease is secondary to exfoliative toxins binding what target protein?
A. Plectin
B. Bullous pemphigoid antigen 2
C. Bullous Pemphigoid antigen 1
D. Desmoglein 3
E. Desmoglein 1
►E

Staphylococcus Scalded-Skin Syndrome (SSSS) is caused by Staphylococcus exfoliative toxins


A and B binding desmoglein 1. This results in the intraepidermal split at the granular layer seen
in this illness. Targeting of desmoglein 3 would result in a suprabasilar split. Targeting of
plectin, BPAG1, or BPAG2 would result in a subepidermal split. This is not seen in SSSS.

5
Tick-borne Relapsing Fever is caused by:
A. B. burgdorferi
B. B. recurrentis
C. B. duttonii
D. B. henselae
E. B. bacilliformis
►C

Relapsing Fever can be either Louse-borne or Tick-borne. Tick-borne Relapsing Fever is caused
by B. duttonii or B. hermsi, while Louse-borne Relapsing Fever is caused by B. recurrentis.

6
Staphylococcal scalded skin syndrome is caused by exfoliative toxins binding to:
A. Desmoglein-1
B. Desmokolin-1
C. TSST-1
D. Laminin-5

8
E. Desmoglein-3
►A

Staph scaled skin syndrome is caused by Et-A and ET-B that bind to desmoglein-1. Children
under 5 years of age, adults with renal failure or immunodeficiency. Also a Nikolsky's sign is
seen in lesional and non-lesional skin with a subgranular blister.

7
Which human papillo virus (HPV) type is implicated in papillomatosis cutis carcinoides di
Gottron?
A. 7
B. 11
C. 18
D. 57
E. 60
►B

Papillomatosis cutis carcinoides di Gottron, also known as Gottron's tumor, is a verrucous


carcinoma of the skin. In 1948, Ackerman first described verrucous carcinoma (VC), a low-grade
tumor that generally is considered a clinicopathologic variant of squamous cell carcinoma.
Verrucous carcinomas are caused by HPV types 6 and 11. When they occur on the feet they are
called epithelioma caniculatum. When they occur on the genitals they are refered to as Buschke-
Lowenstein tumors. When they occur in the mouth they are called Ackerman's tumors or oral
florid papillomatosis.

8
Which of the following is a characteristic of papilloma viruses?
A. enveloped viruses
B. single stranded RNA viruses
C. replicate independent of the host cell
D. controlled by host antibody regulated response
E. can cause a condition that manifests with stridor and hoarseness in children
►E

Papilloma viruses are non-enveloped,double stranded DNA viruses. Cell mediated responses are
primarily responsible for controlling papillomavirus infections. Unlike viruses such as HSV,
HPV does not have enzymes required for replication of viral DNA, and is entirely dependent on
the host cellular machinery. Recurrent respiratory papillomatosis can be caused by HPV-6 and
11, with verrucous lesions of the airways. It can occur as a juvenile or adult-onset form and
present with hoarseness in children. Unlike viruses such as HSV, HPV does not enzymes

9
required for replication of viral DNA, and is entirely dependent on the host cellular machinery
for these functions.

9
The dominant organism present on oily regions of the skin is:
A. Gram positive coagulase negative staphylococcus
B. Staphylococcus aureus
C. Propionibacterium acnes
D. Corynebacterium spp.
E. Propionibacterium avidum
►C

The most common organism on sebaceous regions of human skin is Propionibacterium acnes.
This is an aerotolerant, anaerobic gram positive club-shaped diptheroid that lives in the depths of
the sebaceous follicle. It has a potent lipase which cleaves sebaceous triglycerides into glycerol
and free fatty acids. The free fatty acids lower the pH and are thought to retard the growth of
pathogens such as S. aureus. Other residents of oily regions are: Malassezia furfur, S.
epidermidis, Micrococcus luteus and other catalase positive aerobic cocci. S. aureus is a
pathogen, and is not considered normal flora of any region of the skin. The other options are
found on dry or moist regions of the skin.

10
Measles is caused by which type of virus?
A. Paramyxovirus
B. Togavirus
C. Picornavirus
D. Rhabdovirus
E. Parvovirus
►A

Measles is caused by an RNA virus known as paramyxovirus. The illness is characterized by


high fever, cough, coryza, conjunctivitis, and Koplik's spots. Viral exanthem characterized by
erythematous macules and papules beginning on the face and spreading caudally 3-4 days after
prodromal symptoms.

11
A 12 year old girl presents complaining of multiple warts on the fingers that have been
recalcitrant to common over-the-counter treatments. What is the clearance rate of untreated
common warts in children at 2 years after diagnosis?
A. 25-30%

10
B. 50-55%
C. 70-75%
D. 1%
E. 5-10%
►C

While warts can be very difficult and frustrating to treat, the spontaneous clearance rate at two
years is actually quite high (70-75%).

12
A 28 year old sexually active female presents to clinic complaining of new growths of her labia
majora. Examination reveals condyloma acuminatum, and biopsy with immunohistochemistry
confirms the diagnosis of human papilloma virus infection. Condyloma accuminatum is most
commonly caused by what HPV type?
A. 6 and 11
B. 16 and 18
C. 1 and 3
D. 31 and 33
E. 48 and 60
►A

While many subtypes of HPV are associated with genital warts, 6 and 11 are the most common.
In general, potentially oncogenic HPV subtypes such as 16 and 18 cause sessile genital warts.

13
What is the most common subtype of molluscum contagiosum virus infection seen in HIV
patients?
A. MCV-1
B. MCV-2
C. MCV-3
D. MCV-4
E. MCV-5
►B

The most common subtype of molluscum contagiosum virus is MCV-1. In HIV patients,
however, MCV-2 is more common.

14
The treatment of choice for Loiasis is:
A. Ivermectin

11
B. Diethylcarbamazine (DEC)
C. Permethrin
D. Malathion
E. Chloramphenicol
►B

The recommended treatment for loiasis is Diethylcarbamazine (DEC).

15
Congenital Varicella Syndrome occurs after maternal varicella infection during which stage of
pregnancy?
A. First 20 weeks
B. Third trimester
C. 5 days before and 2 days after delivery
D. 20-24 weeks
E. None of these answers are correct
►A

Congenital Varicella Syndrome occurs after maternal varicella-zoster virus infection early in
pregnancy (up to 20 weeks gestation).

16
The most common agent of cutaneous and furuncular myiasis in North America is:
A. Dermatobia hominis
B. Gasterophilus intestinalis
C. Wohlfahrtia magnifica
D. Hypoderma hominis
E. Cuterebra intestinalis
►A

Dermatobia hominis is most common agent of cutaneous and furuncular myiasis in North
America, but Cuterebra spp. are also a frequent cause of furuncular myiasis in the US.
Complications of myiasis are tetanus and secondary bacterial infection.

17
A 28 year-old man presents with fever, chills, arthralgias and the cutaneous lesions shown. The
most likely diagnosis is:
A. Purpura fulminans
B. Rickettsialpox
C. Anthrax

12
D. Gonoccemia
E. Primary herpes simplex infection
►D

The image shows sparsely distributed hemorrhagic vesiculopustules with erythematous bases on
an acral surface. These features are characteristic of gonococcemia.

18
A 48 year-old woman presents to the Emergency Room with a 2 day history of fever and a well
demarcated erythematous plaque on her right cheek. What is the treatment of choice?
A. Prednisone
B. Intravenous penicillin
C. Cephalexin
D. Fluconazole
E. Potent topical corticosteroid
►B

The most likely diagnosis is Erysipelas which is caused by beta-hemolytic group A


streptococcus. The treatment of choice is intravenous penicillin.

19
Orf virus infection is also known as:
A. Ecthyma
B. Ecthyma contagiosum
C. Echtyma gangrenosum
D. Pseudocowpox
E. Milker's nodule
►B

Orf virus infection is also known as Ecthyma contagiosum. Ecthyma Gangrenosum is caused by
Pseudomonas aeruginosa. Ecthyma is caused by Staphylococcal or streptococcal pyoderma.
Pseudocowpox and Milker's nodule are also known as paravaccinia and caused by udders of
infected cows.

20
A patient develops herpes zoster in the V1 distribution. A few days after development of the
classic vesicular rash, her family brings her to the emergency room because of new onset
seizures. She had been complaining of confusion, headache, fatigue, and loss of appetite prior to
the convulsions. What is the diagnosis?
A. Syndrome of inappropriate anti-diuretic hormone

13
B. Ramsay-Hunt syndrome
C. Delayed contralateral hemiparesis
D. Post-zoster neuropathy
E. Intracerebral vasculitis
►A

The question stem describes signs and symptoms of hyponatremia. The syndrome of
inappropriate antidiuretic hormone secretion is a known, albeit rare, complication of varicella-
zoster virus infection. The other four answer choices are also known complications of varicella-
zoster virus infection, but do not cause the disease presentation as described.

21
What laboratory finding is commonly seen in children with measles virus infection?
A. Lymphopenia
B. Thrombocytosis
C. Polycythemia
D. Anemia
E. Lymphocytosis
►A

Children who are actively infected with the measles virus are commonly found to have
lymphopenia. While an abnormality of any of the blood cell lines is possible during viral
infections, in the case of measles infections, lymphopenia is most common. However, cbc may
reveal a relative lymphocytosis and thrombocytopenia.

22
The vector for relapsing fever that causes paroxysmal fevers, headaches, mylagias and
erythematous or petechia macules in refugees is:
A. Pediculus humanus
B. Ornithodoros
C. Ixodes dammini
D. Ixodes pacificus
E. Ixodes ricinus
►A

Relapsing fever is caused by the human body louse Pediculus humanus and the borrelia species
is B. recurrentis. The clinical features are paroxysmal fevers, headache, lymphocytoma,
mylagias, erythematous or petechial macules on trunk and extremities.Louse-borne relapsing
fever is transmitted by the human body louse and is generally restricted to refugee settings in
developing regions of the world.Tick-borne relapsing fever has the same symptoms occurs in the

14
western United States and is usually linked to sleeping in rustic, rodent-infested cabins in
mountainous areas. The vector for tick borne is ornithodoros and it is due to Borrellia duttonii.

23
A 19 year old sexually active male presents with a painful ulcer on the shaft of his penis. A
biopsy specimen stained with Giemsa reveals a "school of fish." Which of the following is the
causative organism?
A. Hemophilus ducreyi
B. Treponema pallidum
C. Calymmatobacterium granulomatis
D. Chlamydia trachomatis L1
E. Herpes simplex virus
►A

Chancroid is a sexually transmitted disease caused by the bacteria Hemophilus ducreyi. It is one
of many organisms that can cause a genital ulcer. Chancroid is characterized by a soft, painful
chancre with ragged edges. Giemsa stained sections reveal a "school of fish" configuration which
is typical. Treatment is with azithromycin, ceftriaxone, ciprofloxacin, or erythromycin.

24
Epithelioma cuniculatum is an HPV-associated verrucous carcinoma involving the:
A. Oral mucosa
B. Scalp
C. Sole
D. Penis
E. Trunk
►C

Epithelioma cuniculatum is a form of verrucous carcinoma involving the sole.

25
A patient had a severe febrile illness with hemolytic anemia and now has exophytic nodules on
the trunk, extremities and face. Which of the following organisms is the cause?
A. Bartonella henselae
B. Bartonella quintana
C. Bartonella bacilliformis
D. Rickettsia rickettsii
E. Rickettsia akari
►C

15
The first stage of this condition is known as Carrion‟s disease, which can be fatal. It was in Dr.
Carrion (as a medical student), who self inoculated himself to prove that the cause of verruga
peruana (the second stage described above) was the same as that of the first stage. The cause of
both Carrion‟s disease and verruga peruana is Bartonella bacilliformis. B. henselae is the cause
of Cat Scratch disease while B. quintana causes Trench Fever. Rickettsia rickettsii is the cause of
Rocky Mountain Spotted Fever and Rickettsia akari causes Rickettsial pox.

26
The diagnosis of leishmaniasis can be confirmed by which of the following techniques?
A. Weil-Felix test
B. Warthin-Starry stain
C. Culture in Sabourad's agar
D. Culture in Novy-MacNeal-Nicolle (NNN) medium
E. Culture in Michel's medium
►D

Confirmation of infection with Leishmaniasis is performed by culture in Novy-MacNeal-Nicolle


(NNN) medium.

27
What is the most common presentation of mucous membrane involvement of EBV infection?
A. Buccal ulcers with superficial slough
B. Erosions of the gingivae
C. Plication of the tongue
D. Palatal petechiae
E. Oral hairy leukoplakia
►D

The most common presentation of intra-oral involvement of EBV infection is pinpoint petechiae
of the junction of the hard and soft palate. The associated eponym is Forchheimer spots.
Forchheimer spots are not pathognomonic for EBV infection, as they can be seen in measles,
rubella, and scarlet fever. While oral hairy leukoplakia is associated with EBV infection, it is not
the most common mucous membrane sign.

28
A 4 year old girl experiences systemic symptoms including fever, fatigue, headache, and
vomiting. An associated exanthem consists of a non-pruritic eruption including the formation of
blisters on the palms and soles and an intra-oral ulcer. Hand-foot-mouth syndrome is diagnosed.
What nail finding can be seen approximately 1 month after infection with Coxsackie virus and
other Enteroviruses?

16
A. Onychomadesis
B. Onycholysis
C. Trachyonychia
D. Terry's nails
E. Paronychia
►A

Although rare, onychomadesis has been associated with enteroviral infections, including hand-
foot-mouth syndrome due to Coxsackie virus. Onychomadesis is an idiopathic shedding of the
nails beginning at the proximal end. It may be due to temporary arrest of the matrix during the
infection. In most cases, the nails return normally.

29
A 1 year old female patient is brought in by her mother because of the development of warty
growths peri-anally. A clinical examination and biopsy confirms the diagnosis of peri-anal warts.
In 11 month old patients, what is the most common cause of acquisition of human papilloma
virus.
A. Vertical transmission peri-natally
B. Sexual abuse
C. Contact with a wart-containing finger
D. Contact with a fomite
E. Aerosol transmission
►A

In any child with peri-anal warts, the possibility of sexual contact must be excluded. Fortunately,
in infants up to 12 months of age, the most common route of acquisition of the human papilloma
virus is vertical transmission at the time of delivery.

30
Epidermodysplasia verruciformis is caused by which human papilloma virus (HPV) types?
A. 6 and 11
B. 16 and 18
C. 5, 8, and 9
D. 3 and 10
E. 13 and 32
►C

Epidermodysplasia verruciformis is caused by HPV types 5, 8, 9, 12, 14, 15, 17, 19-26.

31

17
A child presents with fever, cutaneous tenderness and erythema of flexural and periorifacial
areas. Within 24 hours the erythema progresses to flaccid blisters and erosions. No organisms are
cultured and Nikolsky's sign is positive. First line therapy includes:
A. IV penicillinase resistant penicillin with clindamycin
B. Aspirin
C. IV IgG
D. PO tetracylcine
E. PO cephalexin
►A

First line therapy for SSSS includes IV penicillinase resistant penicillin. Second line therapy is
IV macrolide treatment. Third line therapies are cephalosporin and vancomycin. Clindamycin is
added to curtail toxin production.

32
Name the syndrome defined by gonorrhea with perihepatitis and liver adhesions.
A. Lhermitte-Duclos Syndrome
B. Turcot Syndrome
C. Fitz-Hugh-Curtis Syndrome
D. Leser-Trelat Syndrome
E. Clutton's Syndrome
►C

Gonorrhea, caused by a gram negative intracellular diplococcus, often presents cutaneously with
hemorrhagic vesiculopustules of the distal extremities. Septic arthritis is also an important
presentation of gonococcemia, most commonly involving the knee.

33
A 48-year-old Latin American female with a diagnosis of leprosy develops bullous lesions that
appear rapidly and ulcerate, especially below the knee. This most likely represents:
A. Reversal reaction
B. Upgrading reaction
C. Erythema nodosum leprosum
D. Lucio's phenomenon
E. Lofgren's syndrome
►D

This patient has a type III reactional state to leprosy called Lucio's phenomenon. It mostly occurs
in people of Latin American descent and is characterized by rapidly ulcerating and bullous
lesions usually on the legs. It has a high mortality rate. A type I reaction, or a reversal/upgrading

18
reaction, can occur after treatment and are associated with a change in cell-mediated immunity
and result in accelerated destruction of bacilli. This is manifested as inflammation of existing
lesions with no systemic symptoms. A type II reaction, or erythema nodosum leprosum,
represents an immune complex deposition and features widespread erythematous nodules with
systemic features. Lofgren's is associated with sarcoidosis.

34
An AIDS patient develops a smooth papule with a collarette of scale with vascular proliferation
seen on biopsy and is seen with those in contract with cats. The most likely cause of this is:
A. Bartonella hensalae
B. Brucellosis hensale
C. Pseudomonas mallei
D. Bacillus anthracis
E. Pasteurella
►A

This is bacillary angiomatosis and is caused by Bartonella hensalae and can be associated with
contact with cats in 20% of cases and occurs in patients with AIDS. The disease is treated with a
macrolide antibiotic or doxycycline.

35
A 27 year-old HIV positive presents with this pruritic eruption? What is the characteristic cell
type found on histopathology?
A. Neutrophil
B. Plasma Cell
C. Basophils
D. Eosinophil
E. Lymphocyte
►D

Eosinophilic pustular folliculitis is a pruritic disorder that may be seen in HIV positive patients.
Typically, the CD4 count is < 300. In patients with HIV, pruritus tends to improve after
institution of anti-retroviral therapy.

36
A 43 year-old man with uncontrolled HIV disease presents with yellowish, red papules, draining
sinuses and ulcers perianally and periurethrally. Which histopathological or
immunohistochemical stain may aid in diagnosis?
A. CD3
B. CD20

19
C. Von Gieson (EVG)
D. Von Kossa
E. Steiner
►D

The most likely diagnosis is malacoplakia. Malacoplakia, an infection usually attributed to S.


aureus, P. aeruginosa, or E. coli is characterized histopathologically by Michelis-Gutmann
bodies. These are foamy eosinophilic macrophages containing calcified, laminated,
intracytoplasmic bodies. They can be highlighted by a calcium stain such as von Kossa. CD3 and
CD20 are lymphocyte markers, von Gieson is an elastic tissue stain, and Steiner stain is used to
highlight spirochetes. These would not aid in the diagnosis of malacoplakia.

37
The main deterrent to staphylococcal infection of the skin is:
A. Complement
B. Neutrophils
C. Macrophages
D. Immunoglobulin
E. CD8+ T-cell mediated immunity
►B

The main deterrent to staphylococcal infection is the neutrophils. These contain bactericidal
chemicals and proteins such as myeloperoxidase-H202-halide, lysozyem, serine proteases,
defensins and bacterial permeability increasing peptide. Complement serves as an effective
opsonin for phagocytosis by circulating neutrophils and macrophages. Immunoglobulin response
hastens phagocytosis but is not absolutely required. Cytotoxic T-cells do not play a major role in
controlling S. aureus infections.

38
What named nerve is involved in the formation of Hutchinson's sign (vesicles at the nasal tip) in
herpes zoster?
A. Ophthalmic nerve
B. V2 (maxillary)
C. Nasociliary
D. Infratrochlear nerve
E. Labial nerve
►C

20
Hutchinson's sign is defined by vesicles at the nasal tip in a patient with facial herpes zoster. It is
caused by VZV invovlement of the nasociliary nerve. Patients suffering with this sign should be
screened for ophthalmologic herpes zoster invovlement.

39
Cat scratch disease is from transmission from infected cats to humans and the treatment for the
disease is:
A. Spontaneous resolution in the majority of cases
B. Erythromycin
C. Doxycycline
D. Cloramphenicol
E. Minocycline
►A

Cat scratch disease is caused by B. henselae. It is from the flea-Ctenocephalides felis. It


spontaneous resolution in most of cases.

40
A goatherder develops this painless pustule that subsequently forms a black eschar. What causes
the edema to form in this lesion?
A. Increased levels of cAMP
B. Release of TNF-alpha
C. Release of IL-1beta
D. Inhibition of phagocytosis
E. C1 esterase inhibitor
►A

Anthrax is caused by Bacillus anthracis, a gram positive spore-forming rod. Anthrax is primarily
seen in individuals who are in contact with wild or domestic animals. Recent cases have been
linked with bioterrorism. B. anthracis produces edema toxin and lethal toxin. Edema toxin is
comprised of edema factor and protective antigen. Edema factor is a calmodulin dependent
adenyl cyclase. The increased cAMP induces the gelatinous edema of anthrax skin lesions.

41
Which of the following vectors is responsible for transmitting Chagas‟ disease?
A. Tstse fly
B. Reduviid bug
C. Black fly
D. Mosquito
E. Sandfly

21
►B

The reduviid big is the vector of American trypanosomiasis (Chagas disease). The vector of
African trypanosomiasis is the tstse fly.

42
What is the causative agent of this parasitic eruption which also causes patchy pulmonary
infiltrate and eosinophilia?
A. Dracunculiasis
B. Gnathostomiasis
C. Ground itch
D. Larva migrans
E. Larva currens
►C

Larva migrans is caused by Ancylostoma braziliense, a hookworm of cats and dogs. It does not
leave the epidermis because it lacks collagenase. Larva currens is due to strongyloides and
presents with serpiginous lesions on the buttock, thighs or perianally. Ground itch is due to
ancylostoma duodenale and necator americanus which clinically present like cutaneous larva
migrans but are able to leave the epidermis, therefore causing fuplomany infiltrates and
eosiniophilia. Loeffler's syndrome, is the name for the patchy infiltrate of the lungs with
eosinophilia. Treatment for this parasitic infection is ivermectin or albendazole or thiabendazole.

43
Treatment of severe nodulocystic acne with which of the following vitamin derivatives may
completely arrest the disease process through decreasing P. acnes?
A. Vitamin D
B. Vitamin A
C. Vitamin K
D. Vitamin E
E. Vitamin B3
►B

13-cis-retinoic acid (isotretinoin/Accutane) is a derivative of vitamin A. It has several


mechanisms of action in acne. It rapidly suppresses sebum production, causing a decrease in P.
acnes populations. It also decreases follicular plugging by an uncertain mechanism. Vitamins D,
K and E are fat soluble vitamins, but are not able to completely arrest the disease process in acne.
Vitamin B3 is niacin and is associated with a diet of exclusively of corn, millet or sorghum.
Other sources: carcinoid, Hartnup disease, parasites, GI disorders, IV nutrition, and psychiatric
disease. Findings include the 3 D‟s: Diarrhea, Dementia, Dermatitis.

22
44
The Jarisch-Herxeimer reaction is most closely associated with the release of which of the
following cytokines?
A. IFN-alpha
B. IL-12
C. TNF-alpha
D. IL-4
E. IL-10
►C

TNF-alpha. The Jarisch-Herxheimer reaction is associated with increased plasma concentrations


of TNF-alpha after antibiotic treatment of syphilis. The Jarisch-Herxheimer reaction has also
been described following treatment of louse-borne relapsing fever, lyme disease and
leptospirosis.

45
A patient with HIV/AIDS presents with vesicles in a disseminated distribution. A biopsy is
performed. Which histologic finding favors infection with varicella-zoster virus over herpes
simplex virus?
A. Pseudoepitheliomatous hyperplasia
B. Margination of keratinocyte nuclei
C. Multinucleated cells
D. Superficial dermal leukocytoclastic vasculitis
E. Molding of nuclei
►D

Herpes simplex virus and disseminated varicella-zoster virus infection can be difficult to
distinguish clinically and histologically. These viruses share the histologic findings of
margination of nuclei, multinucleated cells, and nuclear molding. The presence of
leukocytoclastic vasculitis favors a diagnosis of disseminated varicella-zoster virus.

46
Woolsorter‟s disease is caused by:
A. Erysipelothrix rhusiopathiae
B. Listeria monocytogenes
C. Corynebacterium minutissimum
D. Corynebacterium tenius
E. Bacillus anthracis
►E

23
Anthrax, also known as woolsorter‟s disease or malignant pustule is caused by Bacillus
anthracis, a gram positive rod. It is primarily caused by contact with infected wild or domestic
animals or their products (such as wool). This organism has 3 clinical forms: inhalation,
gastrointestinal and cutaneous. The primary lesion is a “malignant pustule” which begins as a
painless papule that evolves into a hemorrhagic bulla. Ultimately, a black eschar forms. The lack
of pain is helpful in distinguishing between anthrax and other infections of the skin which are
painful, such as ecthyma. The other listed organisms are not causes of this disease.

47
A patient with known chronic Hepatitis C virus infection presents with hyperkeratotic plaques of
the bilateral feet. Biopsy reveals psoriasiform changes with superficial epidermal necrosis. Diet
supplementation with what mineral has been shown to improve the appearance and symptoms of
this cutaneous finding?
A. Zinc
B. Biotin
C. Magnesium
D. Calcium
E. Chromium
►A

Necrolytic acral erythema is characterized by hyperkeratotic well-defined plaques on the lower


extremities, seen occasionally in patients with hepatitis C virus infection. Treatment consists
primarily of treating the underlying infection. Diet supplementation with zinc has been shown to
improve necrolytic acral erythema, even in the presence of normal serum zinc levels.

48
A patient is experiencing posterior cervical lymphadenopathy and has been diagnosed with
African trypanosomiasis. This sign is known as:
A. Winterbottom's sign
B. Hutchinson sign
C. Romana's sign
D. Sleeping sickness sign
E. Tsetse sign
►A

This patient has African trypanosomiasis. It is caused by T. brucei gambiense and T. brucei
rhodesiense and the vector is the Tsetse fly. Posterior cervical lymphadenopathy is known as
winterbottom's sign. Romana's sign is eyelid edema and conjunctivitis at the site of inoculation
for American trypanosomiasis.

24
49
Which of the following is true regarding lymphogranuloma venereum?
A. The primary stage presents as a painful herpetiform ulcer.
B. Proctocolitis can be seen in the late stages of this disease.
C. It is caused by Chlamydia psittaci.
D. Buboes should be incised and drained.
E. Transmission does not occur via skin-to-skin contact.
►B

Lymphogranuloma venereum presents as a painless ulcer and is caused by Chlamydia


trachomatis serotypes L1, L2, L3. While buboes can be aspirated, they should not be incised and
drained. Transmission occurs through sexual contact, but can occur via skin-to-skin contact.
Complications of late stage disease include proctocolitis, perirectal abscesses, rectovaginal
fistulas, and anal fistulas.

50
Which of the following tests is the most sensitive serologic test in late primary syphilis?
A. FTA-ABS
B. VDRL
C. ELISA
D. RPR
E. MHA-TP
►A

The fluorescent treponemal antibody absorption test is the most sensitive serologic test in
primary syphilis. The ELISA is the test of choice in early primary and congenital syphilis
because it is the first to become reactive. However, it is less sensitive in late primary syphilis
because of reduction of IgM production. The ELISA is 100% specific. MHA-TP is less sensitive
than FTA-ABS during primary syphilis.

51
A 16 year old female presents with a sore throat, painful cervical lymphadenopathy, and an
enlarged spleen. An empiric trial of amoxicillin leads to the development of a diffuse
morbilliform rash. What is the incubation period of the most likely viral cause of this patient's
symptoms?
A. 3-7 days
B. 7-10 days
C. 2-3 weeks
D. 3-7 weeks

25
E. 6 months
►D

Infectious mononucleosis is characterized by a sore throat, painful lymphadenopathy, fatigue,


and splenomegaly. The formation of a morbilliform rash upon challenge with ampicillin or
related antibiotics is a classic finding. The most common virus associated with infectious
mononucleosis is Epstein-Barr virus. The incubation period is relatively long, 3-7 weeks.

52
A 39 year old male goatherder presents with a necrotic eschar which is found to be caused by
cutaneous anthrax. Bacillus anthracis produces edema factor, which causes gelatinous edema of
skin by lesions by increasing what?
A. Cyclic adenosine monophosphate
B. Tumor necrosis factor alpha
C. Interleukin-1 beta
D. Transforming growth factor
E. Endothelial growth factor
►A

Edema factor is a portion of the edema toxin, one of two exotoxins secreted by Bacillus
anthracis. Edema factor causes gelatinous edema of anthrax skin lesions by inducing an increase
in cyclic adenosine monophophate levels. Lethal toxin, the other exotoxin, works by releasing
tumor necrosis factor-alpha and interleukin-1 beta.

53
A 20-month old child develops a high fever followed 2 days later by a sudden eruption of rose
pink macules and papules with white halos as the fever subsides. What is the most likely
diagnosis?
A. Measles
B. Rubella
C. Scarlet fever
D. Exanthem Subitum
E. Erythema infectiosum
►D

The cutaneous features of Exanthem Subitum (Roseola infantum, Sixth Disease)as described in
the question develop as the fever subsides. While the other diagnoses are part of the differential
diagnosis of "fever and rash" in children, only roseola infantum has this characteristic clinical
course.

26
54
A 69 year old man who had recently returned from a boating trip in the Atlantic developed pain
with hemorrhagic bullae of his right leg. He had substained a cut on his right leg prior to leaving
for the trip, but swam in the water anyway. Which of the following treatments should be started?
A. Doxycycline
B. Erythromycin
C. Penicillin
D. Rifampin
E. Dapsone
►A

The case presentation describes an infection with Vibrio vulnificus, a Gram-negative rod, which
may lead to a rapidly expanding cellulitis, with hemorrhagic bullae and necrosis, and life-
threatening septicemia. This infection mainly occurs along the Atlantic seacoast. It may be
acquired after ingesting raw oysters or other seafood. Alternatively, localized skin infection may
result after exposure of an open wound to seawater. Doxycycline together with ceftazidime is the
treatment of choice. In patients with preexisting hepatic dysfunction or immunocompromise and
whose wounds are exposed to or acquired in saltwater, prophylactic antibiotic coverage with
doxycycline, 100 mg every 12 h, and cleansing with 0.025% sodium hypochlorite solution may
prevent progressive infection.

55
Which of the following is not a tick born disease?
A. Lyme disease
B. Tularemia
C. Rocky Mountain spotted fever
D. Yaws
E. Ehrlichiosis
►D

Ticks are capable of transmitting:Lyme disease-Borrelia burgdorferi, a tickborne spirochete.


Tularmia-Francisella tularensis, bite of a tick. Rocky Mountain spotted fever-Rickettsia
rickettsii, tick typhus. Yaws-Treponema Palladium, subspecies pertunue-transmitted
nonsexually, by contact with infectious lesions. Ehrlichiosis-Ehrlichia chaffeensis, tickborne

56
Herpetic whitlow is usually a localized infection to the index finger. The most common cause of
Herpetic whitlow is from:
A. HSV2
B. HSV1

27
C. EBV
D. CMV
E. VZV
►A

Herpetic whitlow is more commonly caused by HSV2 from digital/genital contact. It can also
occur in dentist.

57
A patient with atopic dermatitis presents with frequent flares of dermatitis. They have multiple
lichenified plaques, some with serum crust. Which of the following organisms is most likely
colonizing these areas?
A. Aerobic diphtheroids
B. Gram positive coagulase negative cocci
C. Staphylococcus epidermidis
D. Staphylococcus aureus
E. Propionibacterium avidum
►D

S. aureus rapidly becomes a member of the cutaneous flora in locations where there is serum or
plasma on the skin. Often patients with atopic dermatitis will have colonization of this organism
in lesions and may benefit from topical or oral antibiotics. P. avidum and aerobic diphtheroids
are present in moist regions of the skin. Gram positive coagulase negative cocci are the dominant
organism in the dry regions of the body. S. epidermidis is most commonly found in oily body
regions.

58
Focal epithelial hyperplasia (Heck‟s Disease) is associated with which of the following human
papillomavirus types?
A. 3
B. 4
C. 7
D. 10
E. 13
►E

Focal epithelial hyperplasia is caused by HPV-13 and 32.

59

28
A 23 year old intravenous drug user presents with the onset of purpura on the legs. Biopsy
reveals vasculopathic changes with intravascular thrombi seen in superficial dermal vessels.
Serum studies for cryoglobulins and Hepatitis C virus are both positive. Patients with HCV and
mixed cryoglobulinemia are more likely to develop what malignancy?
A. Non-Hodgkins lymphoma
B. Cutaneous T-cell lymphoma
C. Anaplastic large cell lymphoma
D. Kaposi's sarcoma
E. Acute myelocytic leukemia
►A

Mixed cryoglobulinemia is characterized by LCV and the finding of monoclonal or polyclonal


IgG and IgM cryoglobulins in the blood. Mixed cryoglobulinemia is commonly associated with
HCV infection. Patients with HCV-associated mixed cryoglobulinemia are at an increased risk of
developing non-Hodgkins lymphoma, approximately 35 times the general population.

60
Cat scratch disease is caused by infection with:
A. Bartonella species
B. Gram-positive cocci
C. Gram-positive bacilli
D. Rickettsiae
E. Spirochetes
►A

Cat scratch disease is caused by infection with B. henselae - a gram-negative bacillus of the
genus Bartonella.

61
Guarnieri bodies are associated with:
A. Measles
B. Orf
C. CMV
D. Smallpox
E. Anthrax
►D

Guarnieri bodies are cytoplasmic eosinophillic inclusions found on light microscopy.

62

29
A patient from India presents with fever, hepatosplenomegaly, and darkening of the skin. Which
of the following is a cause of visceral leishmaniasis?
A. L. donovani
B. L. major
C. L. mexicana
D. L. aethiopica
E. L. tropicana
►A

Visceral leishmaniasis is the most severe form of leishmaniasis. Complications can be fatal, and
include pneumonia, tuberculosis, and dysentery, which are potential opportunistic infections.
Visceral leishmaniasis is most commonly caused by L. donovani and L. infantum.

63
Which of the following factors is most important in the skin providing protection against
infectious agents?
A. The mechanical barrier of intact skin
B. The resident flora present on intact skin
C. The relative dryness of intact skin
D. The cornified cell envelope
E. The presence of immunoglobulins on intact skin
►A

The most important determinant of the skin‟s protection against infectious agents is the
mechanical barrier of intact skin. The dryness of the skin is also important in retarding infection.
The resident flora may help prevent colonization by pathogenic organisms, but is not the most
important factor. Immunoglobulins are present on intact skin in miniscule amounts. The
cornified cell envelope is not a major determinant of infection prevention.

64
Which exotoxin(s) are involved in mediating the effects of Bacillus anthracis - the causative
organism for anthrax?
A. Exotoxin B
B. Edema Toxin
C. Alpha toxin
D. Lethal Toxin
E. Edema Toxin and Lethal Toxin
►E

30
B. anthracis has 2 exotoxins: edema toxin and lethal toxin, each comprised of a pair of
noncovalently linked proteins: “ Edema toxin = Edema Factor (EF) + Protective Antigen (PA) “
Lethal toxin = Lethal Factor (LF) + PA “ EF causes gelatinous edema of anthrax skin lesions by
inducing an increase in cyclic adenosine monophosphate (cAMP) levels “ LF causes shock and
death in disseminated anthrax via the release of tumor necrosis factor-alpha (TNF-a)and
interleukin-1b (IL-1b) “ PA is responsible for entry of exotoxins into the cell by receptor-
mediated endocytosis.

65
A rabbit farmer presents with fevers, myalgia, and a rash. A serum study reveals antibodies to
Francisella tularensis. Which of the following is the most common form of tularemia?
A. Ulceroglandular
B. Chancriform
C. Oculoglandular
D. Typhoidal
E. Meningeal
►A

Tularemia represents a wide range of clinical syndromes caused by Francisella tularensis. The
bacterium is most commonly acquired through contact with infected animals, including rabbits.
There are a wide variety of variants, with the ulceroglandular form being most common.
Ulceroglandular tularemia is characterized by a chancre-like ulcer with raised borders and
regional lymphadenopathy, most commonly on the finger or hand. Treatment is with
streptomycin.

66
Which human papillomavirus type is associated with giant condylomata of Bushke and
Lowenstein (Bushke-Lowenstein tumor)?
A. 1
B. 2
C. 6
D. 16
E. 31
►C

The Bushke-Lowenstein tumor is caused by HPV-6 and 11.

67

31
A 48 year-old man develops headache, myalgias, and high fever 5 days after a hunting trip. On
examination he has a tender ulcer with raised margins and an eschar on his index finger, as well
as tender axillary lymphadenopathy. The most likely diagnosis is:
A. Anthrax
B. Orf
C. Glanders
D. Tularemia
E. Erysipeloid
►D

Tularemia, which is caused by Francisella tularensis, occurs after exposure to infected animals,
including, rabbits, foxes, and squirrels. The clinical description is that of the ulceroglandular
form, which is the most common presentation.

68
The treatment of choice of the tropical infectious disease shown is:
A. Ivermectin
B. Permethrin
C. Diethylcarbamazine
D. Pentavalent antimony
E. Mebendazole
►D

The ulcerated nodule shown here is characteristic of cutaneous leishmaniasis (CL).


Leishmaniasis is treated with pentavalent antimony.

69
A homeless patient with a history of diabetes and alcoholism presents with chronic suppurative
nodules with granular bodies on light microscopy. Culture rapidly grows Staphylococcus aureus
colonies. What is the clinical term describing this type of infectious lesion?
A. Botryomycosis
B. Scrofuloderma
C. Rhinoscleroma
D. Ecthyma gangrenosum
E. Erysipeloid
►A

Chronic purulent nodules with granular bodies seen in patients with immunosuppresion,
diabetes, and alcoholism is called botryomycosis. Causative organisms include S. aureus, P.
aeruginosa, E. coli, and Proteus species. Scrofuloderma is cutaneous extension of underlying

32
Mycobacteria tuberculosis infection. Rhinoscleroma is caused by Klebsiella pneumoniae
rhinoscleromatis. Ecthyma gangrenosum is typically seen following Pseudomonas septicemia.
Erysipeloid is caused by Erysipelothrix rhusiopathiae.

70
Corynebacterium tenuis is responsible for which of the following infections?
A. Majocchi's granuloma
B. Trichomycosis axillaris
C. Erysipelas
D. Malakoplakia
E. Erysipeloid
►B

Corynebacterium tenuis is responsible for causing trichomycosis axillaris, a superficial bacterial


overgrowth of axillary hairs characterized by circumferential yellow (most common), red, or
black granular concretions. Majocchi's granuloma is a dermatophyte infection. Erysipelas is
caused by beta-hemolytic group A streptococcus. Malakoplakia is usually caused by S. aureus, P.
aeruginosa, or E. coli. Erysipeloid is caused by Erysipelothrix rhusiopathiae. Corynebacterium
minutussimum is the causative organism in erythrasma and pitted keratolysis.

71
A 32 year-old woman presents with meningitis and palpable purpura. A diagnosis of
meningococcemia is confirmed via culture of cerebrospinal fluid. Which complement
components are most likely to be deficient in this patient?
A. C1
B. C2
C. C3
D. C4
E. C5
►E

Patients with meningococcemia often demonstrate deficiencies of late complement components


C5-9.

72
Trichomycosis axillaris has a yellowish brown concretions on axillary hair shaft and is caused
by:
A. Corynebacterium tenuis
B. Francisella tularensis
C. Epstein-Barr Virus

33
D. Staphylococcal aureus
E. Micrococcus sedenatrius
►A

Trichomycosis axillaris is caused by Corynebacterium tenius. It is a yellowish brown concretions


on axillary hair shafts. Treatment is shaving, benzoyl peroxide gel, and topical erythromycin.

73
What is the vector for this nematodal infection?
A. Water flea (Cyclops)
B. Tsetse fly (Glossine morsitans)
C. Reduviid bug
D. Mango fly (Chrysops)
E. Black fly (Simulium)
►D

Loiasis is caused by the nematode loa loa. It is transmitted by the mango fly (Chrysops dimidia
or CHrysops silacea). It may manifest with painful, localized subcutaneous nonpitting edema
called calabar swellings. Conjunctival migrations are also common.

74
Human bites result in infection with this gram negative bacillus that is the part of the normal
flora of the human mouth. The infection can also follow fist fights from blows to the mouth. The
infection is caused by:
A. Eikenella corrodens
B. Bartonella hensalae
C. Pastuerella mutocida
D. Pseudomonas
E. Crotalidae
►A

Eikenella corrodens is common in human bite infections. Pasteurella multocida is usually from
cat bites and Pasteurella canis is from dog bites.

75
A patient is suspected to have contracted rabies after a dog bite. The best site for diagnostic
biopsy would be:
A. Neck
B. Acral area
C. Site of bite

34
D. Buccal mucosa
E. Buttocks
►A

The best site for diagnosis of rabies is a biopsy from the neck. This virus has an incubation
period of 5 days to 1 year and demonstrates retrograde axoplasmic flow until it reaches the spinal
cord. It subsequently undergoes centrifugal spread along the peripheral nerves to the skin,
intestine, and into the salivary glands where it is shed.

76
What is the annual risk of developing herpes zoster in an HIV infected patient?
A. 3% per month
B. 10% per month
C. 3% per year
D. 10% per year
E. 25% per year
►C

The annual risk of developing herpes zoster in an HIV infected individual, regardless of the CD-
4 count, is approximately 3%.

77
A 59-year-old male returns from traveling abroad where he ate many fish. Since then he
complaints of recurring migratory 1 cm/day erythematous urticarial plaques. Biopsy shows
eosinophilic panniculitis. What is the most likely condition?
A. Drancunculiasis
B. Gnathostomiasis
C. Enterobiasis
D. Hookworm
E. Larva currents
►B

This patient has gnathostomiasis caused by Gnathostoma dolorosi or spinigerum. It is commonly


acquired from freshwater fish. It manifests with migratory intermittent erythematous urticarial
plaques that recur every 2-6 weeks. The plaques move about 1 cm/day. Histology shows
eosinophilic panniculitis. Treatment is surgical removal or albendazole.

78
Foamy macrophages containing Klebsiella pneumoniae are called:
A. Virchow cells

35
B. Hansemann's cells
C. Mikulicz's cells
D. Michaelis-Gutman cells
E. Russell cells
►C

Mikulicz”s cells are foamy macrophages found in Rhinoscleroma “ a chronic, inflammatory,


granulomatous disease of the upper respiratory tract. Virchow cells are found in Hansen”s
disease. Hansemann cells and Michaelis-Gutman bodies are found in Malakoplakia.

79
A 37 year old male with a history of HIV/AIDS presents complaining of the new onset of
purplish nodules of the feet and lower legs. He lost his insurance eight months ago and has been
unable to afford the HAART therapy that he was prescribed. HHV-8 is found in the semen of
what percent of men with this disease?
A. 5%
B. 20%
C. 35%
D. 50%
E. 65%
►B

Kaposi's sarcoma is a vascular neoplasm seen most commonly on the lower extremities, back,
and genitaliae. In the United States, this disease is most commonly seen in patients with AIDS.
Kaposi's sarcoma has been linked to infection with HHV-8. 20% of men with Kaposi's sarcoma
will have detectable HHV-8 in their semen.

80
Which bacterial disease carries a risk of mortality due to coinfection with salmonella?
A. Carrion's disease
B. Cat-scratch disease
C. Trench fever
D. Rocky Mountain Spotted Fever
E. Q fever
►A

Carrion's disease, or Oroyo fever, is caused by Bartonella bacilliformis and is transmitted via the
Lutzomyia sandfly. It is characterized by fevers, headache, arthralgias, and an acute eruptive
stage that resembles bacillary angiomatosis. It has an untreated fatality rate of 40% secondary to

36
salmonella infection; chloramphenicol is given with tetracycline for treatment. Epidemic typhus
may also coexist with salmonella.

81
Xenopsylla cheopis is the vector responsible for which infectious disease?
A. Human Monocytic Ehrlichiosis
B. Human Granulocytic Ehrlichiosis
C. Scrub typhus
D. Epidemic typhus
E. Endemic typhus
►E

Xenopsylla cheopis (rat flea) is the vector of endemic typhus, which is caused by R. typhi.

82
A 25 year old woman who is 13 weeks pregnant is exposed to cytomegalovirus (CMV). Three
weeks later, she develops infectious mononucleosis symptoms. What percentage of neonates
with congenital CMV infection are asymptomatic?
A. 10%
B. 25%
C. 50%
D. 75%
E. 90%
►E

Symptoms of congenital CMV infection include low birth weight, microcephaly, seizures,
petechial rash, and hepatosplenomegaly. Fortunately, the vast majority (90%) of congenital
CMV infections are asymptomatic.

83
Which of the extracellular enzymes of S. aureus plays a role in toxic shock syndrome (TSS)?
A. Coagulase
B. Penicillinase
C. Enterotoxin
D. Hemolysin
E. Exfoliatin
►C

In addition to TSST-1 which is present in 50% of non-menstrual cases of TSS, enterotoxins,


especially B & C, cause TSS. These enterotoxins are also superantigens, which recognize the V-

37
beta region of the T-cell receptor. Coagulase clots plasma, Penicillinase degrades penicillin,
Hemolysin lyses red blood cells and exfoliatin splits the epidermis (antigen is desmoglein 1).

84
A patient with primary syphilis is treated with Benzathine penicillin G. Soon after treatment, he
develops fever, headache, myalgias, and elevated white blood cell count, consistent with the
Jarisch-Herxheimer Reaction. This reaction is primarily mediated through which inflammatory
cytokine?
A. TNF-alpha
B. IFN-gamma
C. IL-6
D. EGFR
E. IL-10
►A

The Jarisch-Herxheimer Reaction is characterized by fever, headache, lymphadenopathy,


myalgias, and elevated white blood cell count. It is caused by the release of inflammatory
cytokines, particularly TNF-alpha, due to phagocytosis of spirochetes following antibiotic
administration.

85
A 23 year old Peace Corp volunteer returns from Africa with an extensive undermined, non-
healing ulceration on the right lower leg. Which of the following infections is the most likely
cause of this presentation?
A. M. marinum
B. M. kansasii
C. M. chelonei
D. M. ulcerans
E. Leishmania donovani
►D

The most likely cause of a non-healing ulceration in someone who has been in a tropical location
is M. ulcerans. M. marinum causes inflammatory nodules following injury in an aquatic
environment. M. chelonei is a rapidly growing organism most commonly found in soil, water,
dust and animals which presents as single/multiple erythematous SC nodules on an extremity or
following a surgical procedure. M. kansasii rarely causes skin lesions. L. donovani is a causse of
visceral leishmaniasis in India and Kenya (kala-azar).

86
Calabar swellings are a characteristic feature of which disease?

38
A. Onchocerciasis
B. Eumycetoma
C. Tertiary Syphilis
D. Carrion's Disease
E. Loiasis
►E

Calabar swellings are characteristic of Loiasis. They are transient, non-tender soft tissue
swellings found most commonly around joints.

87
A 8 year-old girl presents to the pediatrician with a poorly marginated blue-red single indurated
plaque on her right cheek. What is the most likely causative organism?
A. Beta-hemolytic group A streptococcus
B. Staphyloccus aureus
C. Pseudomonas aeruginosa
D. Haemophilus influenzae
E. Klebsiella pneumoniae
►D

The most likely diagnosis is H. flu cellulitis. This infection usually occurs in children and is
characterized by a single indurated plaque with poorly delineated margins. Erysipelas, an
infection caused by beta-hemolytic streptococcus group A, usually presents with well delineated
margins in adult patients.

88
Which virus is most commonly associated with oral hairy leukoplakia?
A. Pox virus
B. Herpes virus
C. Epstein-Barr virus
D. Human papilloma virus
E. Parvovirus
►C

Oral hairy leukoplakia is an oral mucosal infection most often caused by the Epstein-Barr virus,
which occurs in immunocompromised patients. Clinically, thick, white plaques are noted on the
lateral sides of the tongue, often fissured.

89

39
A 29-year-old woman presents with multiple crateriform pits on the soles of her feet following a
month of hiking on the Appalachian Trail. She wants a complete skin exam and wonders about
her feet. Which of the following do you plan to do next for her foot problem.
A. These pits are a variant of normal
B. You explain that she has a hereditary syndrome that causes these pits
C. You explain that this is a superficial bacterial infection and will resolve with
topical therapy
D. You ask for a skin biopsy
E. You use a Wood‟s lamp to confirm your diagnosis
►C

Multiple crateriform pits on the feet following suboptimal hygiene most likely is pitted
keratolysis caused by Micrococcus sedentarius. Treatment is topical erythromycin, clindamycin
or benzoyl peroxide. This is not Gorlin‟s syndrome (Basal Cell Nevus syndrome). While these
pits are common, it is not a variant of normal. A skin biopsy could confirm the diagnosis, but is
not necessary. A Wood‟s lamp would not be particularly helpful. Some believe that
Corynebacterium spp. also cause this condition, but that is still theory and not proven.

90
Roseola infantum is caused by which virus?
A. Coxsackie virus A16
B. Human herpes virus 6
C. Parvovirus B19
D. Epstein-Barr virus
E. Pox virus
►B

Roseola infantum may be cause be either human herpes virus types 6 or 7. It is the sixth of the
traditional exanthems of childhood. A diffuse, maculopapular eruption is typically preceded by a
prodrome of fever and either respiratory or gastrointestinal symptoms. The infection usually
occurs in the spring and the fall and nearly all children are seropositive for the virus by the age of
4.

91
Butcher's warts are most commonly caused by which human papilloma virus (HPV) type?
A. 7
B. 2
C. 3
D. 5
E. 13

40
►A

Butcher's warts are caused by HPV type 7.

92
For which of the following smallpox vaccination complications is the administration of vaccine
immune globulin indicated?
A. Post-vaccinal encephalitis
B. Erythema multiforme
C. Eczema vaccinatum
D. Vaccinia keratitis
E. Mild generalized vaccinia
►C

Vaccinia immune globulin can be administered to treat some of the complications associated
with vaccinia vaccination. Vaccinia immune globulin is indicated for inadvertent inoculation
with extensive involvement or ocular implantations, eczema vaccinatum, generalized vaccinia
(severe or recurrent), and progressive vaccinia. It is not recommended for inadvertent inoculation
with mild reaction, generalized vaccinia (mild or limited), post-vaccination encephalitis, and
isolated vaccinia keratitis.

93
Nonmenstrual cases of the staphylococcal toxic shock syndrome are mediated by which of the
following toxins?
A. Enterotoxins B and C
B. Exfoliative toxins A and B
C. Edema toxin
D. Epidermolytic toxins A and B
E. Pyrogenic exotoxins A, B, C
►A

Toxic shock syndrome (TSS) is mediated by Enterotoxins B and C (50% of nonmenstrual cases)
and TSS toxin 1 (TSST-1), which is the leading cause of menstrual cases.

94
Which of the following skin manifestations of M. tuberculosis infection is a tuberculid?
A. Erythema induratum of Bazin
B. Lupus vulgaris
C. Miliary TB
D. Scrofuloderma

41
E. Tuberculous gumma
►A

A tuberculid is a cutaneous immunologic reaction to tuberculosis elsewhere in the body. By


definition, cultures and stains for M. tuberculosis are negative. Erythema induratum of
Bazin/Nodular vasculitis present as erythematous or cyanotic nodules on the posterior calves.
85% of cases occur in women. The other listed presentations of TB are all due to secondary
spread from an established TB infection at another location. In some cases, organisms can be
recovered and cultures may be positive.

95
The treatment for patients that are pregnant or for children <9 for lyme disease is:
A. Doxycycline
B. Levoquin
C. Amoxicillin
D. Tetracycline
E. Penicillin
►C

Patients that are pregnant or for children <9 years old, the treatment for lyme disease is
amoxicillin. Lyme disease is caused by B. burgdorferi. Clinical signs are erythema migrans and
acrodermatitis chronica atrophicans.

96
A 35 year old male has had frequent sex with prostitutes. He presents complaining of the rapid
onset of a new rash on his legs. A review of symptoms is otherwise unremarkable. Examination
reveals palpable purpura and tender nodules of the lower legs. Biopsy reveals vasculitis of small-
and medium-sized vessels. What percentage of patients with this disease are found to be infected
with Hepatitis B virus?
A. 7-8%
B. 20%
C. 35%
D. 1%
E. 50%
►A

Polyarteritis nodosa is characterized by the onset of painful nodules on the lower extremities. It
can be limited to the skin (cutaneous PAN), or can involve a wide variety of organ systems
(systemic PAN). Approximately 7-8% of cases of polyarteritis nodosa cases are associated with
Hepatitis B virus infection.

42
97
A 43 year-old man presents with suppurative nodules and sinus tracts over the mandible. He also
is noted to have poor oral hygiene. What is the most likely causative organism?
A. Bartonella Henselae
B. Nocardia brasilensis
C. Streptococcus somaliensis
D. Actinomyces israelii
E. Staphylococcus aureus
►D

Actinomyces israelii is an anaerobic gram-positive rod which causes chronic suppurative nodules
and sinus tracts with an exudate containing sulfur granules. It most commonly effects the
cervicofacial area, especially near the mandible. The abdomen and thoracic areas can also be
affected. The source of infection is endogenous and patients with poor oral hygiene, penetrating
foreign bodies, and recent dental procedures are at increased risk. Treatment of choice is
penicillin.

98
A young male patient presents with a painful ulcer with a ragged edge after a new sexual
encounter. What is the treatment of choice?
A. Penicillin
B. Doxycycline
C. Azithromycin
D. Ciprofloxacin
E. Cephalexin
►C

Ulcers in the genital area in sexually active patient generally fall into two groups: painful and
painless. Painless ulcers include syphilllis, lymphogranuloma venereum, and granuloma
inguinale. The primary cause of painful erosion or ulcer is H. ducreyi or herpes simplex. An
ulcer with a ragged edge that is painful is characteristic of H. ducreyi. The treatment of choice is
azithromycin.

99
A 35 year-old woman presents with a painful vaginal erosion and tender left-sided inguinal
lymphadenopathy. Gram stain reveals clusters of coccobacilli in a “school of fish” pattern. What
is the most likely diagnosis?
A. Primary syphilis
B. Lymphogranuloma venereum

43
C. Granuloma inguinale
D. Chancroid
E. Secondary syphilis
►D

Chancroid classically presents with a soft, tender chancre with ragged edges and unilateral,
tender inguinal adenopathy (bubo). A “school of fish” pattern on Gram or Giemsa stain is
diagnostic.

100
Botryomycosis is:
A. A fungal infection caused by M. canis
B. A bacterial infection commonly associated with Clostridium botulinum
C. A bacterial infection commonly associated with Staphylococcus aureus
D. A fungal infection caused by T. tonsurans
E. An acute, disseminated infection involving the genitourinary system
►C

Botryomycosis is an uncommon, chronic, idonlent disorder characterized by nodular, crusted,


purulent lesions. Sinuses that discharge sulfur granules are present. These heal with atrophic
scars. The granules yield most commonly Staphylococcus aureus on culture, although cases
caused by Pseudomonas aeruginosa, E. coli, Proteus, Bacteroides, and Streptococcus have been
reported

101
Which of the following is not a DNA virus?
A. Herpesvirus
B. Poxvirus
C. Parvovirus
D. Paramyxovirus
E. Adenovirus
►D

Paramyxovirus is a single-stranded RNA virus. All other listed choices are DNA-based viruses.

102
A patient is diagnosed with a unilateral enlargement of the medial clavicle after being diagnosed
with congenital late syphilis. This is called the:
A. Higoumenaskis sign
B. Hutchinson's sign

44
C. Saber sign
D. Saddle sign
E. Gummas
►A

This patient has congenital late syphilis, seen in children over two years of age. The unilateral
enlargement of the medial clavicle is called the Higoumenaki's sign. These patients also can have
Hutchinson's teeth or widely-spaced, peg-shaped teeth in the upper incisors. They can also have
a saddle nose and saber shins.

103
Histoplasmosis is a self limiting pulmonary disease that is related to the dose of conidia and can
be seen in AIDS and immunosuppressed patients. It is associated with:
A. Bird and bat droppings
B. Rat droppings
C. Deer droppings
D. Deer mites
E. Tic bites
►A

Histoplasmosis is associated with bird and bat droppings and is seen in the mississippi and ohio
river valleys in conjunction with inhalation of microconidia.

104
A patient with HIV/AIDS presents complaining of an asymptomatic patch on the tongue.
Examination reveals a white patch with a corrugated surface on the lateral tongue. What
percentage of HIV patients will have this finding?
A. 5%
B. 20%
C. 33%
D. 50%
E. 67%
►C

Oral hairy leukoplakia is characterized by a white patch with a corrugated surface on the side of
the tongue. It is generally asymptomatic. Oral hairy leukoplakia is seen most commonly in
patients with defective cell-mediated immunity, in particular HIV infection. Oral hairy
leukoplakia is associated with EBV opportunisitc infection, among other viruses. 33% of patients
with HIV will have oral hairy leukoplakia.

45
105
The vector of New World Leishmaniasis is the:
A. Phlebotomus sandfly
B. Lutzomyia sandfly
C. Tsteste fly
D. Deer fly (Chrysops spp.)
E. Simulium black fly
►B

The sandfly belonging to the genus Lutzomyia is the vector of New World Leishmaniasis.
Sandflies of the genus Phlebotomus are the vectors of Old World Leishmaniasis.

106
A 34 year old woman complains of recurrent painful blisters on her lower lip. These occur 4
times per year, and are preceded by painful sensations for a day before the blisters appear. What
is the most common trigger of recurrent orolabial herpes simplex virus infection?
A. UVB radiation
B. UVA radiation
C. Visible light radiation
D. Emotional stress
E. Medication use, including opioids
►A

Recurrent orolabial herpes simplex virus infection is a very common problem. In many cases, the
cause of the recurrences is idiopathic. Identifiable triggers include light exposure and emotional
stress, among others. UVB is the most commonly implicated factor.

107
Verruga peruana is transmitted by:
A. Tick
B. Blackfly
C. Sandfly
D. Fecal oral contact
E. Fecal fecal contact
►C

Verruga peruana, a disease endemic to Peru and a few neighboring countries, is caused by
Bartonella bacilliformis. It is transmitted by the sandfly, Lutzomyia verrucarum. It is preceded
by an acute febrile stage called Oroyo fever or Carrion”s disease. The treatment of choice is
chloramphenicol.

46
108
Which of the following is the vector of lymphatic filariasis caused by Wuchereria bancrofti?
A. Black fly
B. Tsteste fly
C. Mosquito
D. Deerfly
E. Botfly
►C

Lymphatic filariasis is spread by mosquitoes belonging to genera Aedes, Anopheles, Culex, or


Mansonia.

109
A patient is diagnosed with Loiasis. This patient most likely has clinical features such as:
A. Adult worm migrating across the conjunctivae
B. Nodules over bony prominences on the extremities
C. Ocular involvement that leads to blindness
D. Depigmented macules/patches also known as "leopard skin"
E. Pruritic papules
►A

Loiasis is caused by the organism Loa Loa. The vector is mango flies. The clinical characteristic
is calabar swelling and adult worm migrating across conjunctivae or "eye worm". The treatment
is DEC or diethylcarbamazine.

110
A patient on the bone marrow transplant service has a fever, neutropenia, hemorrhagic bullae and
erythematous nodules with dusky gray centers. The organism most likely to be responsible is:
A. Streptococcus pyogenes
B. Mycobacterium tuberculosis
C. Candida albicans
D. Pseudomonas aeruginosa
E. Staphylococcus aureus
►D

Ecthyma gangrenosum is an infection of critically ill or immunocompromised individuals by


Pseudomonas aeruginosa. Clinically, patients develop opalescent, tense vesicles or pustules
surrounded by a narrow pink to violaceous halo. The lesions rapidly become hemorrhagic, then

47
violaceous and necrotic leaving ulcers. The most common location is on the buttocks and lower
extremities. Treatment is with aminoglycosides.

111
What is the principal vector of Lyme Disease in the Northeastern U.S.?
A. Ixodes ricinus
B. Soft-bodied ticks (Ornithodoros)
C. Ixodes dammini
D. Ambylomma americanum
E. Dermacentor variabilis
►C

The proncipal vector of Lyme Disease in the Northeastern U.S. is Ixodes dammini (Ixodes
scapularis).

112
Patients that have diagnosed with congenital syphilis have dental findings called:
A. Hutchinson teeth
B. Erythrodontia
C. Hypodontia
D. Peg-shaped teeth
E. Enamel hypoplasia
►A

Congential sphyilis can be passed from the mother to the child. Dental findings are called
hutchinson teeth and mulberry molars.

113
Each of the following is true regarding leishmaniasis except:
A. Arthropod vector is the sand fly
B. Mazzotti‟s test is diagnostic
C. Pentavalent antimony used for visceral disease
D. Cutaneous disease is the most common form
E. Espundia seen mucocutaneous disease
►B

Leishmaniasis is a parasitic infection born by a sandfly vector. No treatment is necessary for


cutaneous leishmaniasis, pentavalent antimony for visceral leishmaniasis and mucocutaneous.
Mazzotti reaction is used to test for onchocerciasis in which a single dose of

48
dimethycarbamazine is given for reaction. Amphotericin B is only used in pentavalent antimony
resistant cases.

114
All of the following are true regarding smallpox except:
A. Caused by variola virus
B. Transmitted by respiratory secretions
C. Virus is found in skin lesions
D. Vaccination is contraindicated in children under 5
E. Associated with thrombocytopenia
►D

Smallpox is an acute exanthematous infection caused by the variolae poxvirus. It is transmitted


primarily by respiratory secretions as well as more infrequently skin inoculation and fomite
spread. Following contact, there is an asymptomatic period of 12-13 days. Following the
asymptomatic period, patients develop a prodrome of fever, headache, back pain and vomiting
for 3 days. Then, the characteristic deep seated papules that vesicles appear. These lesions
mature into a pustule with a central umbilication. Occasionally, patients may develop a
hemorrhagic form of the disease which is associated with thrombocytopenia and poor prognosis.
Vaccination is not contraindicated in children under 5.

115
This type of syphilis has the clinical characteristic of widespread cutaneous eruptions, ham-
colored macules on the palms and soles, moth eaten alopecia and condyloma lata.
A. Primary
B. Secondary
C. Tertiary
D. Latent
E. Congenital
►B

This is an example of secondary syphilis. It involves the skin and mucous membranes. It also
lasts for 4 to 12 weeks. It involved widespread cutaneous eruptions, ham-colored macules on the
palms and soles, mucous patches, condyloma lata, split papules and moth eaten alopecia.

116
A 67 year old homeless man comes to clinic complaining of a painful lump along his lateral
jawline which has been present for many months. He has noticed that it occasionally drains a
purulent material. What is the most likely diagnosis?
A. Actinomyces israelii

49
B. Nocardia brasilensis
C. Actinomadura madurae
D. Streptomyces somaliensis
E. Mycobacteria tuberculosis
►A

Actinomycosis is a chronic suppurative infection forming nodules and sinus tracts with an
exudate containing sulfur granules. Risk factors include poor oral hygiene, penetrating foreign
bodies, and dental procedures. The causative organism is Actinomyces israelii, an anaerobic
gram-positive rod.

117
What are the three rapid grower mycobacteria and what is the current treatment of choice?
A. leprae, chelonae, abscessus; clarithromcyin
B. Tuberculum, abscessus, fortuitum; rifampin
C. Chelonae, asbcessus, fortuitum; clarithromycin
D. Lepra, fortuitum, abscessus; minocycline
E. Chelonae, absccessus, fortuitum; minocycline
►C

These three are rapid grower mycobacteria and the treatment of choice is clarithryomycin.
Minocycline can also be used, but it considered second line in WHO treatment.

118
All of the following are potential causes of a false positive RPR except:
A. Systemic Lupus Erythematosus (SLE)
B. Pregnancy
C. Malignant Melanoma
D. Lepromatous Leprosy
E. Malaria
►C

Non-treponemal tests for syphilis measure antibodies against phospholipids released from
treponemes and damaged host cells. False positive non-treponemal tests can occur in the setting
of pregnancy, autoimmune diseases, other spirochete infections, and infectious diseases such as
leprosy and malaria. It does not occur in cases of melanoma.

119
A 23 year old female patient is found to have genital warts. A referral to an gynecologist reveals
that she has cervical dysplasia. What viral proteins does HPV use to induce neoplasia?

50
A. E6 and E7
B. E1 and E2
C. L1 and L2
D. E8 and L1
E. E3 and E4
►A

Certain HPV subtypes have been shown to induce neoplasia, including cervical cancer. There are
many high-risk subtypes, including 16, 18, 31, and 33. The HPV early genes E6 and E7 bind the
tumor suppressors p53 and retinoblastoma protein, respectively, inactivating them and initiating
oncogenesis.

120
A patient is given a single dose of dimethylcarbamazine and soon develop edema, itching, fever,
arthralgias, and exacerbation of pruritus. He most likely has which underlying condition?
A. Onchocerciasis
B. Loiasis
C. Filariasis
D. Gnothostomiasis
E. Trichinosis
►A

This patient developed an acute reaction to dimethylcarbamazine indicating a Mazzotti reaction.


This reaction is seen with onchocerciasis. Appropriate treatment for onchocerciasis, caused by
Onchocerca volvulus, is ivermectin. These patients may demonstrate pruritus, dermatitis, leopard
skin, onchocercomas (nodules with microfilariae) and blindness.

121
How long is the life cycle for the mite that causes this infestation?
A. 1 day
B. 1 week
C. 1 month
D. 6 months
E. 1 year
►C

Scabies is caused by the mite Sarcoptes scabiei var. hominis. The life cycle of the mite is 30
days. A female mite will lay 60-90 eggs during her life.

122

51
Winterbottom‟s sign is a characteristic feature of which of the following diseases?
A. Leishmaniasis
B. Onchocerciasis
C. Loaiasis
D. African Trypanosomiasis
E. Strongyloidiasis
►D

Winterbottom‟s sign (posterior cervical lymphadenopathy) is a clinical feature of African


trypanosomiasis. Romaña‟s sign (eyelid edema and conjunctivitis at site of incoculation) is a
diagnostic finding in American trypanosomiasis (Chagas‟ disease).

123
A 40-year-old male with HIV (CD4 <200) presents with asymptomatic, slowly enlarging yellow-
pink papules and nodules localized to his perianal skin. A skin biopsy is performed, which
reveals foamy dermal interstitial histiocytes with characteristic basophilic laminated inclusion
bodies. The latter are noted to stain positively with von Kossa, Perls, and PAS stains. The most
likely diagnosis is:
A. eruptive xanthomas
B. malakoplakia
C. hidradenitits suppurtiva
D. granuloma inguinale
E. chancroid
►B

Malakoplakia is a multi-organ inflammatory granulomatous disease that most frequently arises in


the setting of immunosuppression. Common sites of involvement include the genitourinary, lung,
lymph nodes and bone. Cutaneous involvement is rare, and typically presents as asymptomatic
papules, nodules, or plaques. Lesions may have associated erosions or ulcerations, and sinus
tracts may be seen. While skin lesions are non-specific, histopathology reveals foamy dermal
histiocytes known as von Hansemann cells which contain basophilic granular inclusions. The
latter are known as Michaelis-Gutmann bodies and are considered pathognomonic for
malakoplakia. Michaelis-Gutmann bodies are comprised of calcium, iron, and phosphate salts,
and hence stain positively with von Kossa, Perls, and PAS, respectively. The pathophysiology of
malakoplakia is thought to arise from an acquired defect in macrophage activation in response to
bacterial infection, most commonly. E. coli. Treatment for cutaneous lesions is surgical excision,
although recurrence is not uncommon.

124

52
Carrion's disease is characterized by fevers, headaches and arthralgias and is accompanied by
severe hemolytic anemia. Superinfection with which organism is the most frequent cause of
death?
A. Bartonella bacilliformis
B. Bartonella henselae
C. Bartonella Quintana
D. Salmonella
E. Shigella
►D

Carrion's disease (Oroya fever) is characterized by fevers, headaches and arthralgias and is
accompanied by severe hemolytic anemia. Superinfection with Salmonella is the most frequent
cause of death. Protection from sandfly bites is all-important.

125
All of the following are true regarding tularemia except:
A. Caused by the gram negative Francisella tularensis
B. Ticks serve as a vector
C. No longer one of the CDC's list of reportable diseases
D. Presents as an acute febrile illness
E. Potential for use as a biologic weapon
►C

Tularemia is a zoonotic disease caused by the gram-negative coccobacillus Francisella tularensis.


It is transmitted by ticks: dog tick, wood tick, lone star tick, or from deer flies (Chrysops) or
from contact with infected mammals (usually rabbits) or from infected water. It characteristically
presents as an acute febrile illness with other variable clinical manifestations depending on the
route of inoculation. Possible symptoms could include an ulcer at the site of inoculation,
pharyngitis, ocular lesions, lymphadenopathy, and pneumonia. Diagnosis can be made by culture
or a fourfold titer change. Because of its potential use as a biologic agent, tularemia is once again
a reportable disease.

126
The characteristics of tuberculoid leprosy or TT is:
A. Lepromin test +
B. IL4, IL-10,
C. Multibacillary
D. Lepromin test -
E. No loss of sensation or sweating
►A

53
Tuberculoid leprosy has a TH1 cytokine profile of IFN gamma, IL-2, IL-12. It is paucibacillary,
Lepromin test +. There is less than 3 lesions and anesthetic and anhidrotic lesions.

127
Exanthem Subitum is caused by which of the following?
A. Group A Steptococcus
B. Coxsackie virus
C. Human Herpes Virus-6 (HHV-6)
D. Parvovirus B19
E. Epstein-Barr Virus
►C

Exanthem Subitum (Roseola Infantum, Sixth Disease) is caused by human herpesvirus 6, and 7
(HHV-6, HHV-7).

128
This is strain of HPV is associated with Butcher's warts:
A. HPV-7
B. HPV 10, 13
C. HPV 30
D. HPV 2,4
E. HPV 28, 29
►A

The butcher warts are associated with HPV-7. Laryngeal carcinoma is associated with HPV-30.
Common warts are HPV 2, 4.

129
Which porphyrin is responsible for the fluorescence under Wood's lamp examination of
erythrasma?
A. Porphobilinogen
B. Protoporphyrin IX
C. Coproporphyrin III
D. Uroporphobilinogen
E. Coproporphyrinogen
►C

54
Erythrasma is caused by corynebacterium minutissimum. The presence of coprophyrinogen III
created by the bacteria is responsible for the characteristic coral-red fluorescence under Wood's
lamp.

130
Similar lesions were also seen on this infant's face. Treatment of choice is:
A. phototherapy
B. oral steroids
C. acyclovir
D. topical retinoic acid
E. oral erythromycin
►C

This child has eczema herpeticum, a cutaneous infection with HSV which occurs in patients with
pre-existing skin disorders such as atopic dermatitis. It begins as clusters of umbilicated and
hemorrhagic vesicles in areas of abnormal skin. Lesions spread rapidly but can respond quickly
to antiviral treatment. Children with underlying AD who come into frequent contact with an
individual with orolabial HSV may have recurrent outbreaks of eczema herpeticum.

131
Erythema infectiosum is caused by a:
A. RNA virus
B. Herpes virus
C. Double stranded DNA virus
D. Single stranded DNA virus
E. None of these answers are correct
►D

Erythema infectiosum is caused by parvovirus B19 which is a single stranded DNA virus.

132
Blueberry Muffin Baby can be a feature of all of the following EXCEPT:
A. TORCH
B. Klippel-Trenaunay-Weber syndrome
C. Hemolytic disease of the newborn
D. Rhabdomyosarcoma
E. Langerhans‟ cell histiocytosis
►B

55
Blueberry muffin lesions can be seen in the setting of prenatal infections (e.g. TORCH), severe
anemia (e.g. Hemolytic Disease of the newborn) and neoplastic diseases (e.g.
rhabdomyosarcoma). It is not associated with the Klippel-Trenaunay-Weber syndrome.

133
Which of the following is the causative organism of cat-scratch disease?
A. B. quintana
B. B. henselae
C. B. bacilliformis
D. B. duttonii
E. B. burgdorferi
►B

B. henselae is the cause of cat-scratch disease. Humans are infected by a cat bite or scratch,
while the cat flea is responsible for cat to cat transmission.

134
The most common location of herpes gladiatorum is the:
A. Groin
B. Chest
C. Anterior thigh
D. Face
E. Hand
►D

The face is the most common location for Herpes Gladiatorum.

135
You are on a medical mission in the Peruvian Amazonia and are asked to examine a patient. The
patient is concerned about a skin lesion on her abdomen. What is the most likely diagnosis in
your differential?
A. Yellow fever
B. Dengue fever
C. Malaria
D. Strongyloidosis
E. Leptospirosis
►D

Strongyloides stercoralis has a very low prevalence in societies where fecal contamination of soil
or water is rare. It is a very rare infection in developed economies and most common in tropical

56
climates. Strongyloides stercoralis causes morphologically variable skin lesions. Rarely, a
petechial purpuric eruption which resembles multiple thumbprints may initially present over the
periumbilical area. Later on it may involve the whole abdomen, thighs, legs and arms. The
petechial purpuric skin lesions in disseminated strongyloidiasis are due to damage to the blood
vessels caused by larval migration.

136
Which of the following are potential forms of perinatally-acquired neonatal herpes disease?
A. Skin, eyes, mouth (SEM)
B. CNS
C. Disseminated
D. None of the above
E. All of the above
►E

Perinatally acquired neonatal herpes disease can present in three different, occasionally
overlapping, forms: 1. skin, eyes, and mouth; 2. CNS; and 3. Disseminated, causing encephalitis,
hepatitis, pneumonia, and coagulopathy.

137
Measles is caused by a:
A. Paramyxovirus
B. Togavirus
C. Picornavirus
D. Rhabdovirus
E. Adenovirus
►A

The measles virus is an RNA virus belonging to the Paramyxovirus family.

138
A patient presents with this anesthestic plaque. Which cytokines are upregulated in this disease?
A. IL-2
B. IL-4
C. IL-5
D. IL-10
E. All of these cytokines are upregulated
►A

57
Leprosy is caused by infections to Mycobacterium leprae. It is a acid fast bacilli. In tuberculoid
leprosy, Th1 cytokines (IL-2, IFN-gamma) are upregulated. Clinically, patients have few, well-
circumscribed anesthetic plaques.

139
The Mikulicz is the histologic hallmark of which of the following conditions:
A. Leishmaniasis
B. Granuloma inguinale
C. Rhinoscleroma
D. Rhinosporidiosis
E. Histoplasmosis
►C

Rhinoscleroma is caused by Klebsiella rhinoscleromatis, a gram negative rod that causes a


chronic granulomatous infection of the upper respiratory tract. It is predominantly seen in
Mexico, Central and South America. The histologic hallmark is the Mikulicz cell, a large
histiocyte that contains the bacteria in its cytoplasm. Rhinosporidiosis is not an intracellular
bacteria, but an extracellular fungi caused by Rhinosporidium seeberi mainly seen in India, Sri
Lanka and occasionally the southeastern United States. Extremely large endospores are present
in tissue. The other listed organisms are intracellular pathogens but do not have the Mikulicz cell
as a feature of infection.

140
A 24 year-old man from New York City develops fever, chills, malaise, and a severe headache.
His apartment has a mouse problem. On examination, multiple, discrete papulovesicles and a
single eschar are found. What is the most likely diagnosis?
A. Ehrlichiosis
B. Rickettsialpox
C. Q Fever
D. Meningococcemia
E. Rocky Mountain Spotted Fever
►B

The clinical description is that of rickettsialpox, which is caused by R. akari and spread by
liponyssoides sanguineus - the house mouse mite.

141
High doses of which of the following antiviral agents has been associated with thrombotic
thrombocytopenic purpura in immunosuppressed patients?
A. Valcyclovir

58
B. Acyclovir
C. Foscarnet
D. Famciclovir
E. Cidofovir
►A

Thrombotic thrombocytopenic purpura has been reported with high doses of valcyclovir in
immunosuppressed patients.

142
A 42 year-old woman presents with a large, vegetating ulcer involving her left labia majora and
groin for over 1 year. A Giemsa”s stained touch preparation reveals bipolar, safety pin-shaped
intracytoplasmic inclusions. What is the most likely diagnosis?
A. Lymphogranuloma venereum
B. Granuloma Inguinale
C. Chancroid
D. Primary herpes simplex
E. Gonorrhea
►B

Bipolar, safety pin-shaped intracytoplasmic inclusions on Giemsa stain - known as Donovan


bodies - are diagnostic of granuloma inguinale.

143
Sun exposure has been associated with the development of what type of warts?
A. Flat warts
B. Myrmecial warts
C. Common warts
D. Butcher's warts
E. Condyloma accuminatum
►A

Warts are common cutaneous lesions caused by the various human papilloma virus subtypes.
Flat warts are commonly caused by HPV subtypes 3, 10, 28, and 41. Sun exposure has been
implicated as a potential factor in the formation of flat warts.

144
The culture medium that is used for Leishmaniasis is:
A. Novy-MacNeal-Nicolle
B. Giemsa

59
C. Sodium stibogluconate
D. FTA-ABS
E. ELISA
►A

Leishmaniasis should be cultured in Novy-MacNeal Nicolle (NNN) medium. Sodium


stibogluconate is the treatment for cutaneous Leishmaniasis. FTA-ABS and ELISA are used to
diagnose syphilis.

145
Of the choices listed, which antiobiotic is the best to treat a 7-year old child with erythema
migrans?
A. Doxycycline
B. Amoxicillin
C. Azithromycin
D. Tetracycline
E. Rifampin
►B

Doxycycline is the usual treatment for erythema migrans in adults in the absence of neurologic
or cardiovascular complications. In children less than 8, all tetracyclines are relatively
contraindicated because of they can cause tooth discoloration. Therefore amoxicillin should be
used as the first line therapy for children less than 8. Note: Changes have been made to the
treatment of RMSF. Children of any age should be treated with doxycycline for that disease.

146
A patient with sparsely distributed hemorrhagic vesiculopustules with erythematous bases on
palms, soles and over joint. The causative agent is Neisseria gonorrhoeae and the treatment is:
A. Ceftriaxone IV
B. Amphicillin
C. Rifampin
D. Penicillin
E. TMP-SMX
►A

Gonococcemia is caused by Neisseria gonorrhoeae. The clinical features are fevers, chills,
arthralgias, malaise with hemorrhagic vesiculopustules with erythematous base on palms, soles
and over joints.

147

60
A 24 year old male presents with a high fever, arthralgias, and a rash characterized by
nonspecific erythematous macules in a generalized distribution. He was recently cleaning his
family's barn, and was bitten more than once by rats and mice. What is the treatment of choice
for this infection?
A. Penicillin
B. Erythromycin
C. Clindamycin
D. Ciprofloxacin
E. Amikacin
►A

Rat-bite fever is also called Haverhill Fever. It is caused by Streptobacillus moniliformis and is
acquired from infected rodents or by touching or eating contaminated food. The signs and
symptoms are nonspecific, but the diagnosis can be suspected based on history. Treatment is
with penicillin.

148
A 10 year-old girl presents to the dermatology clinic with a pruritic eruption on her trunk after
swimming at a beach in Florida. Of note, the lesions are limited to skin that was covered by her
bathing suit. The most likely diagnosis is:
A. Seabather?s eruption
B. Swimmer?s itch
C. Cercarial dermatitis
D. Swimming pool granuloma
E. Cutaneous larva migrans
►A

Seabather?s eruption characteristically involves areas covered by swimwear, as a result of


coelenterate larvae becoming trapped underneath bathing suits. The causative organisms are
larval forms of Edwardsiella lineata (sea anemone) and Linuche unquiculata (thimble jellyfish).

149
The mechanism of action of this virulence factor of Bacillus anthracis is via increasing the
activity of tumor necrosis factor alpha:
A. Lethal toxin
B. Edema toxin
C. Polyglutamate acid capsule
D. All of the above answers are correct
E. None of the above answers are correct
►A

61
Bacillus anthracis is responsible for causing Anthrax. Anthrax of the skin is characterized by a
painless edematous nodule which rapidly degenerates into an eschar. B. anthracis has 3 virulence
factors: a polyglutamate acid capsule which inhibits phagocytosis of the bacteria, edema toxin
(edema factor and protective factor) which results in edema secondary to induction of cAMP
pathway, and lethal toxin (lethal factor and protective factor) which leads to shock and death via
increasing TNFalpha activity.

150
A 62 year-old man presents with tinnitus, facial paralysis, and vesicles of the external ear. What
is the most likely diagnosis?
A. Primary Herpes Simplex infection
B. Primary Varicella Infection
C. Herpes Zoster Infection
D. Cytomegalovirus infection
E. Coxsackievirus A16 infection
►C

This patient has Ramsay Hunt Syndrome, herpes zoster infection of the geniculate ganglion.
Vesicles can be seen on the tympanic membrane and the external ear. Symptoms include
ipsilateral facial paralysis, tinnitus, or other auditory symptoms. Treatment is with acyclovir and
systemic corticosteroids.

151
Scrofuloderma is most commonly associated with an underlying infection of what organ
system?
A. Lymph nodes
B. Spine
C. Lungs
D. Gastrointestinal tract
E. No underlying systemic disease
►A

Scrofuloderma is a tuberculous or nontuberculous mycobacterial infection affecting children and


young adults, representing direct extension of tuberculosis into the skin from underlying
structures such as lymph nodes, bone, or lung. It most commonly presents as a neck abscess
secondary to a tuberculosis infection of the cervical lymph nodes. It is manifested by the
development of painless subcutaneous swellings that evolve into cold abscesses, multiple ulcers,
and draining sinus tracts. The diagnosis is based on positive Mantoux test/PPD test, histologic

62
features of caseating granulomatous inflammation, culture, and favorable response to anti-
tuberculosis therapy.

152
An 18-year old man presents to the Dermatology Clinic with a nontender penile erosion that has
been present for 2 weeks. An indurated border and nontender bilateral inguinal lymphadenopathy
are also noted. What is the most likely diagnosis?
A. Primary syphilis
B. Chancroid
C. Herpes simplex
D. Lymphogranuloma venereum
E. HIV
►A

A painless chancre with an indurated border is characteristic of primary syphilis. Associated


painless lymphadenopathy ("buboes") is also a common feature.

153
A 40-year-old male patient from South America with HIV (CD4+ <200) presents with new
purple-red papules and nodules on his face that have been progressively enlarging. A biopsy is
performed, showing lobular vascular proliferations with plump endothelial cells on H&E
staining. Immunohistochemical staining of skin tissue for human herpes virus 8 (HHV8) is
negative. Upon closer inspection of H&E-stained sections, faint purple granular interstitial
deposits are noted in the dermis. Which of the following tissue stains is most appropriate for
further work-up?
A. Fite
B. Chloroacetate esterase
C. Warthin-Starry
D. Giemsa
E. Ziehl-Neelsen
►C

Causes of angiomatous papulonodules arising in immunocompromised patients include Kaposi‟s


sarcoma, bacillary angiomatosis, and verruga peruana, as well as pyogenic granulomas and
cherry angiomas. Bartonella, a gram negative bacillus, appears as faint purple-blue granular
dermal deposits on routine H&E-stained sections. These deposits appear black upon Warthin-
Starry staining. Prominent endothelial cells may also be seen. Both B. henselae and B. quintana
have been implicated in the pathogenesis of bacillary angiomatosus, while B. bacilliformis is the
causative agent in verruga peruana (as well as Bartonellosis). Erythromycin is the first-line

63
treatment for bacillary angiomatosus, while fluoroquinolones plus a tetracycline are the first-line
agents for treatment of verruga peruana.

154
Bacillary angiomatosis is caused by which of the following organisms?
A. B. quintana
B. B. henselae
C. B. bacilliformis
D. B. quintana and B. henselae
E. All of these answers are correct
►D

B. quintana or B. henselae are both causative organisms for bacillary angiomatosis.

155
One of your colleagues returned from a vacation to India with fever, vomiting, pleural effusions,
ascites and conjunctival petechiae. She also has a diffuse macular rash with notable areas of
sparing that your attending refers to as “white islands in a sea of red”. What is your diagnosis?
A. Leishmania
B. Dengue hemorrhagic fever
C. Malaria
D. Typhoid
E. Leptospirosis
►B

Dengue fever is caused by an Arbovirus and transmitted by the mosquito, Aedes aegypti and
may cause Dengue Shock Syndrome and Dengue Hemorrhagic Fever. The infection starts with
sudden onset of high fever, backache, retroorbital pain, bone and joint pain, depression and
malaise. The disease is also called "break-bone fever." One to seven days after onset of fever,
rash presents characteristically starting on the dorsum and hands and feet spreading to limbs and
torso. The eruption may become confluent with small, round islands of sparing, the so called
"white islands in a sea of red." Treatment is generally supportive as no antivirals are effective.

156
Acrodermatitis chronica atrophicans is typically caused by:
A. Borrelia burgdorferi
B. Borrelia burgdorferi senso stricto
C. Borrelia garinii
D. Borrelia afzelii
E. Unknown

64
►D

Acrodermatitis chronica atrophicans, also known as primary diffuse atrophy, is characterized by


the appearance on the extremities of diffuse reddish or bluish red, paper-thin skin allowing the
blood vessels to easily be seen beneath the skin. It is caused by Borrelia afzelii and is tick
transmitted by Ixodes ricinus. It is seen almost exclusively in Europe.

157
Patients that are diagnosed with purpura fulminans with hemorrhagic infarction of the skin are
infected with:
A. Group A streptococcus
B. Staphylococcus aureus
C. Francisella tularensis
D. Bacillus anthracis
E. Streptomyces somaliensis
►A

Purpura fulminans is causes by group A streptococcal infection. Clinically there are hemorrhagic
infarction of the skin caused by disseminated intravascular coagulation. One can see symmetric
large ecchymotic areas with irregular ("geographic") borders on extremities, ears and nose most
commonly.

158
A 67-year-old man presents with grouped painful vesicles on an erythematous base on his lateral
sacrum. He denies any history of herpes simplex type I or II. He is angry with you for suggesting
that he may have herpes. What could explain this presentation?
A. He actually has “shingles” and your diagnosis is incorrect
B. A significant number of people are asymptomatic for HSV infection
C. He is lying
D. He has the early stages of Alzheimer‟s disease
E. This presentation represents a primary infection with Epstein Barr Virus,
(HHV4)
►B

A significant number of people are asymptomatic for HSV infection but are seropositive for anti-
HSV antibodies. Lumbosacral HSV occurs in about 10% of HSV infections and is more common
in older age groups. In some cases, this may be the “first” presentation of HSV that the patient is
aware of. Shingles would be a reasonable consideration for the differential diagnosis and a direct
immunofluorescence scraping and viral culture would confirm that this typical presentation of

65
HSV actually is. It is possible that the patient is lying or that he has early Alzheimer‟s disease,
but less likely. EBV does not present in this manner.

159
An 18 year old male presents complaining of pruritus, erythema, and edema of the hands and
feet. A serum study for parvovirus antibodies is negative. What is the next most common cause
of this syndrome?
A. Epstein-Barr virus
B. Measles virus
C. Coxsackie virus
D. Cytomegalovirus
E. Human herpes virus 6
►A

Papular-purpuric gloves and socks syndrome is characterized by pruritus, erythema, and edema
of the hands and feet, primarily in teenagers and young adults. Parvovirus B19 is classically
associated with this eruption. While all of the viruses listed as answer choices have been
associated with this syndrome; Epstein-Barr virus is the best answer choice.

160
A middle-aged female presents with purplish pruritic plaques on her dorsal wrist and whitish
plaques on her buccal mucosa. A diagnosis of lichen planus is made. Which type of lichen planus
is most strongly associated with Hepatitis C virus infection?
A. Mucosal ulcerative lichen planus
B. Hypertrophic lichen planus
C. Lichen planus pemphigoides
D. Lichen planus of the nails
E. Vulvar lichen planus
►A

Lichen planus presents a wide variety of cutaneous morphologies that share common histologic
findings of a lichenoid lymphocytic infiltrate, basal vacuolization, and dyskeratotic
keratinocytes. The form of lichen planus most strongly associated with Hepatitis C virus
infection is mucosal ulcerative lichen planus.

161
Pediculus humanus var. corporis (human body louse) is the vector in which of the following
diseases:
A. Endemic typhus (R. typhus)
B. Epidemic typhus (R. prowazekii)

66
C. Q fever (C. burnetii)
D. Rickettsialpox (R. akari)
E. Rocky Mountain Spotted Fever (R. rickettsii)
►B

The human body louse is the implicated Trench fever, epidemic typhus, and relapsing fever.
Trench fever usually affects alcoholic men and is manifested by fevers. Treatment is with
ceftriaxone, erythromycin, or doxycycline. Epidemic typhus is manifested by fevers, chills,
malaise, and a pink macular eruption beginning in the axilla and trunk. Treatment is with
tetracycline or chloramphenicol. Relapsing fever is manifested by paroxysmal fevers, headache,
lymphocytoma, and erythematous/petechial macules on trunk/extremities. Treatment is with
doxycycline.

162
Which of the following are inconsistent with the diagnosis of staphylococcal scalded skin
syndrome?
A. Epidermal changes are produced by exfoliative toxins of staphlococcus
B. Initial event is usually a localized staph infection
C. Swabs and cultures of fluid filled blisters overwhelmingly grow staph
D. Prognosis is good in children with low mortality when anitbiotics are
administered early
E. Cell necrosis does not occur in staphylococcal scalded skin syndrome as it does
in T.E.N
►C

Staphylococcal scalded skin syndrome: Lesions extend far beyond areas of actual staphylococcal
infection, by action of the epidermolytic exotoxin elaborated by the staphylococcus in remote
sites. Usually the staphylococci are present at a distant focus such as the parynx, nose ear, or
conjuctiva. If cultures are taken they should be obtained fromthe mucous membranes because the
skin erythema and desquamation is due to the distant effects of the exfoliative toxin, unlike the
sitaution in bullous impetigo, where S. aureus is present in the lesions. Epidermal changes are
produced by exfoliative toxins of Staphylococcus. Inital event is usually a localized Staph
infection. Prognosis is good in children with low mortality when anitbiotics are administered
earyl. Cell necrosis does not occur in SSS as it does in toxic epidermal necrolysis.

163
Milker‟s nodule is caused by infection with which of the following viruses?
A. Human herpesvirus-6 (HHV-6)
B. Human herpesvirus-8 (HHV-8)
C. Paravaccinia virus

67
D. Vaccinia virus
E. Orf virus
►C

Milker‟s nodule is caused by paravaccinia virus, a poxvirus of the genus Parapoxvirus. It is


transmitted to humans from infected cows.

164
In addition to Kaposi”s sarcoma, HHV-8 infection is also the causative agent in:
A. Pityriasis rosea
B. Angiosarcoma
C. Primary effusion lymphoma
D. Bacillary angiomatosis
E. Pyogenic granuloma
►C

Human herpes virus 8 is a double stranded DNA virus. It is thought to be pathogenic in Kaposi”s
sarcoma, primary effusion lymphoma, and Castleman”s disease. Primary effusion lymphoma is a
rare B-cell lymphoma seen predominantly in patients with AIDS.

165
WHO recommendations for the treatment of multibacillary leprosy include all of the following
EXCEPT:
A. Dapsone
B. Clofazimine
C. Clarithromycin
D. Rifampin
E. All of these answers are recommended for the treatment of multibacillary
leprosy.
►C

Clarithromycin is not part of the WHO recommendations for the treatment of multibacillary
leprosy.

166
Granuloma Inguinale is caused by:
A. Chlamydia trachomatis types I, II & III
B. Hemophilis ducreyi
C. Klebsiella granulomatis
D. Treponema pertenue

68
E. Treponema pallidum
►C

Granuloma inguinale is caused by an infection of Klebsiella granulomatis. It is a granulomatous


skin disease of the genitals caused by sexual transmission. It is a gram negative rod that is similar
to the Enterobacteriaceae. It is a smll, raised papule or subcutaneous nodule that rapidly
ulcerates, rarely with lymphadenopathy. It is not self healing and spreads by radial extension.
Hemophilis ducreyi is the cause of chancroid, Chlamydia trachomatis causes Lymphogranuloma
Venereum, Treponema pertenue causes Yaws and T. pallidum is the cause of syphilis.

167
A patient is diagnosed with tularemia. The clinical features seen is the ulceroglandular form that
looks like a chancre like ulcer with raised boarders and regional lymphadenopathy seen on the
finger or hand. The patient mostly likely came into contact with:
A. Rabbits
B. Horses
C. Cats
D. Dogs
E. Deer
►A

Tularemia is caused by Francisella tularensis. It is most likely caused by infected rabbits seen in
hunters. There are other forms such as glandular, chancriform, oculoglandular, pulmonary,
oropharyngeal and meningeal. The treatment is streptomycin.

168
What is the definition of disseminated herpes zoster?
A. Involvement of two or more defined dermatomes
B. Symmetric bilateral involvement
C. Involvement of one cranial and one spinal nerve.
D. More than 20 vesicles outside of the primarily affected dermatome
E. Involvement of the trunk and at least one extremity.
►D

The definition of dissemianted herpes zoster is at least 20 vesicles seen outside of the primarily
affected dermatome. The other option choices are distractors.

169
Each of the following species may be involved in necrotizing fasciitis except:
A. Pseudomonas

69
B. Clostridium
C. Streptococcus
D. Mycobacterium
E. Bacteroides
►D

Necrotizing fasciitis is a rapidly advancing acute necrotizing infection that may follow trauma,
surgery, or occur spontaneously. It is associated with systemic toxicity and high mortality rate.
Clinically, erythema, edema, and edema progresses to dusky cyanosis, blistering and necrosis.
Many virulent bacteria have been culturesd including Pseudomonas, Bacteroides, streptococci,
staphylococcus, enterococci, and clostridium. Both aerobic and anaerobic cultures should be
taken. Mycobacterium is not associated with necrotizing fasciitis.

170
The usual culture medium for Mycobacterium leprae is:
A. Chocolate agar in 10% CO2
B. Lowenstein-Jensen
C. It cannot be cultured
D. Sheep‟s blood agar
E. Agar supplemented with heme and nicotinamide
►C

M. leprae cannot be grown in media or cell culture. It has been grown in mice footpads and in
armadillos. Humans are the only natural host. It is endemic in the tropics and found worldwide.
The Lowenstein-Jensen medium is used to culture mycobacteria most commonly. Neisseria
gonorrheae is grown on chocolate agar in 10% CO2. Agar supplemented with heme (x-factor)
and nicotinamide (v-factor) is needed for Hemophilus influenzae cultures. Sheep‟s blood agar is
useful for identifying hemolytic strains of streptococcus and staphylococcus.

171
A patient presents with multiple asensate patches on the abdomen. A diagnosis of leprosy is
made. Which of the following regimens is the WHO recommendation for treatment of
paucibacillary leprosy?
A. Dapsone and rifampin
B. Dapsone and clofazimine
C. Clofazimine and rifampin
D. Minocycline and rifampin
E. Minocycline and clofazmine
►A

70
Paucibacillary leprosy is characterized by three or fewer anesthetic and anhidrotic lesions. This
form of leprosy develops when the body mounts a Th1 cytokine profile leading to rare bacilli
found in the lesions. The WHO recommends treatment of paucibacillary leprosy with dapsone
100 mg daily for 6 months and rifampin 600 mg monthly for 6 months.

172
Hutchinson”s teeth are a feature of which stage of syphilis?
A. Early congenital
B. Late congenital
C. Primary
D. Secondary
E. Tertiary
►B

Hutchinson”s teeth (widely-spaced, peg-shaped upper incisors) are a seen in Late Congenital
Syphilis (> 2 years of age).

173
A patient with HIV/AIDS develops disseminated flesh-colored papules with central
umbilication. A diagnosis of molluscum contagiosum is made. What is the CD4 cell count below
which molluscum contagiosum lesions are first seen in HIV patients?
A. 100
B. 200
C. 400
D. 800
E. 1000
►A

HIV/AIDS patients are afflicted with a wide variety of opportunistic pathogens. The spectrum of
infectious diseases can be used to predict a patient's CD4 helper T cell count. Molluscum
contagiosum is usually not seen until a patient's CD4 count drops below 100.

174
Swimmer's itch caused by the flat worms of the family Schistosomatidae have clinical features
of:
A. pruritic papules and vesicles on uncovered skin
B. pruritic erythematous papules on covered swim wear
C. Serpiginous urticarial plaques on buttocks
D. Thumbprint purpura on periumbilical skin
E. Widespread petechiae on trunk/proximal extremities

71
►A

Swimmers itch also known as Clam Digger's itch is caused by the cercarial forms flatworms of
the family Schistosomatidae. The mode of infection is penetration of skin while bathing in
infested fresh or salt water in Northern United States and Canada. The clinical characteristics are
pruritic papules and papulovesicles on uncovered skin.

175
What is the first line treatment of streptococcal toxic shock syndrome?
A. Penicillin
B. Ciprofloxacin
C. Clindamycin
D. Doxycycline
E. Vancomycin
►C

Clindamycin is considered 1st line treatment of streptococcal toxic shock syndrome, as it inhibits
secretion of the exotoxin.

176
A 12 month old boy has the sudden onset of a fever of 40 degrees Celsius. When the fever breaks
spontaneously in three days, an erythematous, morbilliform rash arises on the trunk and spreads
to the arms and legs. The rash is not itchy, and lasts 1-2 days. What is the most likely viral cause
of this syndrome?
A. HHV-6
B. Measles virus
C. Rubella virus
D. Coxsackie virus
E. Cytomegalovirus
►A

Roseola infantum is also known as exanthem subitum or sixth disease. It is seen in infants ages
6-36 months, and is characterized by the signs and symptoms as described. The syndrome is
caused by Human Herpes Virus 6.

177
Xenopsylla cheopis transmits:
A. Epidemic typhus
B. Murine typhus
C. Rickettsia prowazekii

72
D. Scrub typhus
E. Rickettsia akari
►B

Xenopsylla cheopis along with Ctenocephalides felis are fleas that transmit R. typhi, the
organism responsible for murine or endemic typhus. The body louse or Pediculus humanus
corporis transmits Rickettsia prowasekii the organism responsible for epidemic typhus. Scrub
typhus is caused by R. tsutsugamushi and is transmitted by chiggers or trombiculid mite larvae.

178
A 25-year old man with a history of a healed genital ulcer develops a diffuse papulosquamous
eruption resembling pityriasis rosea. What is the most likely time lapse between the appearance
of the genital lesion and the appearance of this eruption?
A. 3-12 weeks
B. 1-2 weeks
C. 3-5 months
D. 6-12 months
E. 1-2 years
►A

The lesions of secondary syphilis are often diffusely distributed due to spirochetemia.
Papulosquamous lesions are characteristic, and the presence of constitutional symptoms such as
fever, fatigue, headaches, and bone pain may help distinguish secondary syphilis from pityriasis
rosea. Other lesions of secondary syphilis include ˜moth-eaten” alopecia, mucous patches, split
papules, and condyloma lata. The mucocutaneous manifestations of secondary syphilis typically
occur 3-12 weeks after the appearance of the chancre and last 4-12 weeks.

179
Through which cellular protein does molluscum contagiosum evade host immunity?
A. IL-18 binding protein
B. Beta-4 integrin
C. T cell receptor
D. MHC II
E. Toll-like receptor 7
►A

Molluscum contagiosum virus, like other viral and bacterial pathogens, attempts to exploit the
host immune system to avoid destruction. In this case, molluscum contagiosum virus usesIL-18
binding protein, which blocks the host helper T (Th1) response by reducing local IFN-gamma
production.

73
180
Which of the following is the vector responsible for the transmission of Verruga Peruana?
A. Lutzomyia verrucarum
B. Tsetse fly
C. Ctenocephalides felis
D. Pediculus humanus corporis
E. Simulium slossonae
►A

Verruga Peruana is caused by infection with B. bacilliformis, which is transmitted by the


sandfly, Lutzomyia verrucarum.

181
The causative organism of epidemic typhus is:
A. Francisella tularensis
B. R. prowazekii
C. R. akari
D. R. typhi
E. Salmonella typhi
►B

Epidemic typhus is caused by infection with R. prowazekii.

182
Dracunculiasis is typically acquired from:
A. Soil
B. Salt water
C. Drinking water
D. Chiggers
E. Black flies
►C

Dracunculiasis, which is caused by Dracunculus medinensis, is acquired by ingestion of


copepod-infested water.

183
A young patient presents to you after participating in a bar brawl a few nights before with a
painful swollen erythematous right hand. The most likely organism is:
A. Staphylococus aureus

74
B. Eikenella corrodens
C. Pasturella multocida
D. Streptococcus pyogenes
E. Bacteriodes fragilis
►B

Eikenella corrodens is a member of normal oral flora, that is commonly the cause of human bite
and fist fight infections. Pasturella multocida is a common organism found in dog and cat bites.
Staph and Strep could be the right answers for a cellulitis, but with this history are not the correct
choices. Baceriodes fragilis is found in normal bowel flora.

184
Which of the following manifestations of syphilis would you expect to be present 2-6 months
after the individual was exposed?
A. Osteitis
B. Aortitis
C. Tabes dorsalis
D. Condylomata lata
E. Pseudochancre redux
►D

All of the listed options except condylomata lata are manifestations of tertiary syphilis, which
does not manifest for years after the initial infection. Secondary syphilis presents 2-6 months
following infection. Condylomata lata are moist, flat and smooth. Do not confuse them with
Condyloma accuminatum which are usually dry, cauliflower-like projections on the genitalia.

185
Which of the following forms of syphilis is characterized by ham-colored macules on the palms
and soles, condyloma lata, and "moth-eaten" alopecia?
A. Primary
B. Secondary
C. Latent
D. Tertiary
E. Congenital
►B

Secondary syphilis is characterized by ham-colored macules on the palms and soles, condyloma
lata, and "moth-eaten" alopecia, in addition to mucous patches and split papules. It typically lasts
4-12 weeks.

75
186
The treatment of choice for acyclovir-resistant herpes simplex virus infection is:
A. Valcyclovir
B. Famciclovir
C. Indinivir
D. Saquinivir
E. Foscarnet
►E

Foscarnet directly inhibits viral DNA polymerase (without requiring phosphorylation by TK) and
is therefore effective in acyclovir-resistant HSV infections. Cidofovir can also be used.

187
What is the characteristic body on histopathology for this disease?
A. Cowdry Type A
B. Cowdry Type B
C. Henderson-Patterson
D. Guarnieri
E. Dutcher
►A

Herpes simlex virus is characterized by Cowdry type A bodies on histopathology. They appear
as sharply demarcated eosinophilic structures separated by a clear halo from a basophilic rim of
the infected cell's marginated nuclear chromatin. Cowdry B bodies may be seen in polio
infections.

188
Which of the following is the treatment of choice for the disease caused by Bartonella
bacilliformis and spread by the Lutzomyia sandfly?
A. Chloramphenicol
B. Erythromycin
C. Doxycyline
D. Minocycline
E. Azithromycin
►A

Oroya Fever (Carrion's Disease) is caused by Bartonella bacilliformis. The Lutzomyia sandly is
the vector. While Bartonella infections respond to multiple classes of antibiotics, including
tetracyclines and macrolides, Oroya Fever has a high frequency of superinfection with
Salmonella. Therefore, chloramphenicol is the treatment of choice.

76
189
A 10-year-old boy develops an acute illness and rash along with marked enlargement of the
posterior cervical lymph nodes. This presentation is most consistent with:
A. West African sleeping sickness
B. East African sleeping sickness
C. Chagas disease
D. Schistosomiasis
E. Sparaganosis
►A

West African sleeping sickness is caused by Trypanosoma bruci gambiense and leads to acute
illness with rapid fatal course and pronounced posterior cervical lymphadenopathy
(Winterbottom's sign). East African sleeping sickness is due to T. brucei rhodesiense (east like
rhode island) and is more chronic . American trypanosomiasis, or Chagas disease, features
unilateral conjunctivitis and edema of the face (Romana's sign) and heart and gastrointestinal
sequelae. Schistomsomiasis causes swimmer's itch, while sparaganosis from ingestion of
Spirometra leads to painful edematous lumps.

190
A patient develops gastroenteritis after a trip to the seacoast. He reports eating raw oysters at his
favorite seafood restaurant. Which of the following organisms is the most likely cause of his
gastrointestinal problems?
A. Vibrio vulnificus
B. Vibrio parahemolyticus
C. Mycobacterium marinum
D. Erysipelothrix rhusiopathiae
E. Treponema carateum
►B

Vibrio parahemolyticus is a cause of gastroenteritis after consuming contaminated seafood or


shellfish. V. vulnificus infection occurs in cuts/wounds that contact contaminated seawater or
marine organisms. M. marinum occurs after injury in an aquatic environment such as a fish tank
or swimming pool. Lesions typically occur on hands, elbows or knees and are characterized by
painless inflammatory nodules that may ulcerate and may occur in a sporotrichoid pattern. E.
rhusiopathiae infection occurs after injuries in fishermen, butchers or those handling raw meat.
Initially there is burning pain at the inoculation site followed by a gradually enlarging purple
plaque with well defined margins. T. carateum is the cause of Pinta.

191

77
A patient with a chronic nose-bleed is treated with nasal packing. The packing is accidentally left
in place, and he develops Toxic Shock Syndrome. What is the implicated toxin?
A. Enterotoxin B
B. Toxic Shock Syndrome toxin 1
C. Exfoliative Toxin A
D. Edema toxin
E. Lethal factor
►A

Toxic Shock Syndrome is characterized by fever, hypotension, and a generalized scarlatiniform


eruption followed by desquamation. Menstrual-associated TSS is generally driven by
Staphylococcus aureus, which released Toxic Shock Syndrome toxin 1. In nonmenstrual cases,
however, Enterotoxins B>C are more commonly implicated, although TSST-1 has also been
associated. Exfoliative toxins A and B are involved in bullous impetigo, and edema toxin and
lethal factor are involved in Bacillus anthracis infections.

192
What rare hematologic disorder can accompany varicella-zoster virus infection?
A. Thrombocytopenia
B. Neutropenia
C. Lymphopenia
D. Lymphocytosis
E. Anemia
►A

Symptomatic thrombocytopenia, while rare, has been associated with acute varicella-zoster virus
infection.

193
Which anatomic locations are most commonly involved in herpes gladiatorum?
A. Lateral neck, lateral face, forearm
B. Tips of fingers
C. Feet
D. Upper thighs and buttocks
E. Scalp
►A

Herpes gladiatorum is most commonly seen on the lateral neck, lateral face, and forearm, areas
which most commonly contact a competing wrestler or the wrestling mat.

78
194
Post herpetic neuralgia can be prevented with the VZV vaccine. It is a Oka/Merck strain of VZV
and is FDA approved for adults:
A. 60 and older
B. 50 and older
C. 55 and older
D. 65 and older
E. 70 and older
►A

Herpes zoster vaccine is a live attenuated vaccine and can help reduce the incidence of herpes
zoster and decrease the chance of post-herpetic neuralgia.

195
A 35 year-old woman who works in the animal product industry presents with a painless
hemorrhagic bulla. A diagnosis of anthrax is suspected. Gram stain of the vesicle fluid should
reveal which of the following?
A. Guarnieri bodies
B. Gram-negative cocci
C. Gram-negative bacilli
D. Gram-positive cocci
E. Gram-positive bacilli
►E

Anthrax is caused by Bacillus anthracis, which is a gram-positive spore-forming rod.

196
What is the most important predictor of materal-fetal transmission of HSV-2?
A. Use of pre-natal antiviral medications
B. Length of delivery
C. Nature of maternal infection
D. Nutrition status of the mother
E. Use of peri-natal monitoring equipment.
►C

The nature of the maternal infection (primary versus recurrent) is the most important predictor of
maternal-fetal transmission of HSV-2. In mothers with a primary infection, the risk of
transmission is 25-50%. In mothers experiencing a recurrent infection, the risk is 2-5%

197

79
Contagious pustular dermatitis (Orf) is caused by a:
A. Herpesvirus
B. Poxvirus
C. Gram-positive spore-forming rod
D. Paramyxovirus
E. Papovavirus
►B

Orf is caused by orf virus (OV), a poxvirus of the genus Parapoxvirus.

198
The treatment of choice of cat-scratch disease is:
A. Erythromycin
B. Ceftriaxone
C. Trimethoprim-sulfamethoxazole
D. Itraconazole
E. Mebendazole
►A

Cat-scratch disease is caused by Bartonella henselae, which is spread among cats by fleas. The
hallmark of the disease is unilateral and regional lymphadenopathy. The treatment of choice for
cat-scratch disease is erythromycin.

199
The most likely target for exfoliative toxin A in bullous impetigo is:
A. Desmocollin
B. BPAG1
C. BPAG2
D. Desmoglein 1
E. Desmoglein 3
►D

Bullous impetigo is caused primarily by staphylococcus aureus phage type 71. This bacteria
produces exfoliative toxin A binds to desmoglein 1 and produces produces the blisters
characteristic of bullous impetigo.

200
Which Rickettsial infection has a negative Weil-Felix test?
A. Rocky Mountain Spotted Fever (R. rickettsii)
B. Mediterranean fever (R. conorii)

80
C. Epidemic typhus (R. prowazekii)
D. Endemic typhus (R. typhi)
E. Rickettsialpox (R. akari)
►E

Rickettsia are short, gram-negative rods which are strict intracellular parasites. The bacteria is
transmitted via tick which much be attached for more than 6 hours. The Weil-Felix test exploits
cross-reactivity between the Proteus vulgaris antigen and RIckettsia.

201
Most common causative organism of the attached picture is
A. Corynebacterium minutissimum
B. Corynebacterium tenuis
C. Micrococcus sedentarius
D. Staphylococcus
E. Streptococcus
►C

The image shows Pitted keratolysis. It is a non-inflammatory bacterial infection caused by


microcolonies of Micrococcus sedentarius or Corynebacterium species that produce a specific
proteolytic enzyme which digests the stratum corneum. It consists of small crater-like
depressions in the stratum corneum, and usually appear on the weight-bearing regions of the
soles of the feet, but can occur in other non weight-bearing areas as shown in image. This was
confirmed with shave biopsy of epidermis that is step-sectioned and stained with methenamine
silver as this is found to be more helpful than a punch biopsy. The disease usually goes unnoticed
by patients and is most often an incidental finding. Hyperhidrosis and malodor of the skin are
very common findings. Corynebacterium minutissimum is the causative agent for erythrasma,
and Corynebacterium tenius causes trichomycosis Axillaris.

202
All of the following are features of scarlet fever except:
A. Pharyngitis
B. Pastia‟s lines
C. S. aureus infection
D. Circumoral pallor
E. Sandpaper-like texture
►C

Scarlet fever is primarily a disease of children with most cases occurring between the ages of 1
and 10 years of age. Streptococcus pyogenes is the causative organism. It produces the

81
streptococcus pyrogenic exotoxin which elicits the cutaneous manifestations by enhancing
delayed type hypersensitivity to streptococcal products. Clinical findings include fever, sore
throat, headache, chills, sandpaper rash on the trunks, strawberry tongue, and Pastia‟s lines
(linear petechial streaks found in flexural locations).

203
What is the most common cause of erythema multiforme?
A. Herpes simplex virus
B. Mycoplasma pneumonia
C. Amoxicillin
D. Ibuprofen
E. Cytomegalovirus
►A

The most common cause of erythema multiforme (EM) is herpes simplex virus, which may not
be active at the time of the EM eruption. Patients with recurrent EM are typically treated with
acyclovir or valacyclovir. Mycoplasma pneumonia is a cause of EM, but is not the most
common. Amoxicillin, ibuprofen, and cytomegalovirus may cause EM, but are not as common.

204
The patient in this photograph has claw hands, hypoesthesia of the distal extremities, madarosis
and leonine facies. What type of leprosy does she have?
A. Lepromatous leprosy
B. Tuberculoid leprosy
C. Borderline leprosy
D. Erythema nodosum leprosum
E. Lucio phenomenon
►A

Hansen's disease (aka Leprosy) is a chronic granulomatous disease principally affecting the skin
and peripheral nervous system, caused by Mycobacterium leprae. Lepromatous leprosy is
described where there is low immune response to the bacteria. The areas most commonly
affected are the superficial peripheral nerves, skin, mucous membranes of the upper respiratory
tract, anterior chamber of the eyes, and testes. These areas tend to be cool parts of the body.
Tissue damage depends on the degree to which cell-mediated immunity is expressed, the type
and extent of bacillary spread and multiplication, the appearance of tissue-damaging
immunologic complications (ie, lepra reactions), and the development of nerve damage and its
sequelae. The late complications of lepromatous leprosy include madarosis = loss of eyebrows,
leonine facies, stockin gglove neuropathy and lagopthalmosis.

82
205
This is the term that is used from tuberculosis of the skin and is spread by hematogenous,
lymphatic, or contiguous spread. The clinical characteristic is brownish red plaque, apple jelly
color on diascopy.
A. Lupus vulgaris
B. Scrofuloderma
C. Tuberculous Gumma
D. Tuberculosis verrucosa cutis
E. Tuberculosis cutis orificialis
►A

Lupus vulgaris spreads from hematogenous, lymphatic, or contiguous spread from distant site of
tuberculous infection. It is sensitizes the host with moderate to high immunity. It is pauci-
bacillarry. It also involved the head/neck in 90% of cases.

206
Which of the following diseases is caused by a virus?
A. Rocky Mountain spotted fever
B. Human monicytic ehrlichiosis
C. Colorado tick fever
D. Boutonneuse spotted fever
E. Q fever
►C

Rocky Mountain spotted fever is caused by Rickettsia rickettsii. Human monicytic ehrlichiosis is
caused by Ehrlichia chaffeensis. Colorado tick fever is caused by Reoviiridae. Boutonneuse fever
is caused by Rickettsia conorii. Q fever is caused by Coxiella burnetti.

207
Which of the following statements best describes the pathogenesis of scrofuloderma?
A. Hematogenous spread of M. tuberculosis from a distant site of infection
B. Autoinoculation of M. tuberculosis from advanced pulmonary tuberculosis
C. Contiguous spread from an underlying focus of tuberculous lymphadenitis
D. Primary incoculation of M. tuberculosis in a sensitized host
E. An cutaneous manifestation of military tuberculosis
►C

Scrofuloderma is the result of contiguous spread onto skin from an underlying focus of
tuberculous infection. This typically occurs in a sensitized host with low immunity.

83
208
Which of the following tests is a specific test for infection with Treponema pallidum?
A. RPR
B. VDRL
C. FTA-ABS
D. CBC
E. Darkfield microscopy
►C

The FTA-ABS test measures the presence of antibodies in the patient‟s serum to the syphilis
antigens, either on spirochetes or on attached red blood cells. The RPR and VDRL are non-
specific tests that rely on cross-reactive anti-phospholipid antibodies. Darkfield microscopy is
not specific to T. pallidum, but will identify other spirochetes also. A CBC would not be specific
for T. pallidum.

209
What percentage of patients undergoing stem cell transplantation for leukemia will develop
herpes zoster within the first year?
A. 5%
B. 30%
C. 50%
D. 70%
E. 90%
►D

The risk of developing herpes zoster following stem cell transplantation for leukemia is quite
high, approaching 70% in some studies.

210
Herpes simplex virus infection and this condition has been linked to which HLA type?
A. HLA Cw6
B. HLA B15
C. HLA B27
D. HLA B51
E. HLA DR3
►B

HSV related erythema multiforme has been associated with an increased frequency of HLA-B15.

211

84
Heck‟s disease is associated with which of the following types of human papillomavirus?
A. 1
B. 3
C. 7
D. 6/11
E. 13/32
►E

Heck‟s disease (focal epithelial hyperplasia) is caused by HPV 13 and 32 infections. These
lesions are multiple circumscribed papules on the oral mucosa that may resemble oral
condyloma. They are not pre-cancerous. HPV-1 is associated with plantar warts, while HPV-3 is
associated with flat warts. HPV-7 cause “butcher‟s warts” and are believed to be acquired from
handling meat, poultry and fish.

212
What is the first-line recommended therapy for orf?
A. Penicillin
B. Erythromycin
C. Doxycycline
D. Bactrim
E. No therapy is needed since the condition is self-resolving
►E

No therapy is needed, since orf is usually a benign, self-resolving infection. Orf is caused by a
parapoxvirus, often associated with sheep and goats. The infection undergoes 6 different stages
of evolution: the papular, target, acute, regenerative, papillomatous, and regressive stages.

213
A patient with hypertophic plaques on the external nares with Mikulicz cells seen on
histopathology is infected with:
A. Klebsiella pneumoniae rhinoscleromatis
B. Streptobacillus moniformis
C. Erysipelothrix rhusiopathiae
D. Bukholderia mallei
E. Pseudomonas aeruginosa
►A

This patient has rhinoscleroma, which looks like hypertrophic plaques on external nares. The
causative organism is Klebsiella pneumoniae rhinoscleromatis. Mikulicz cells are seen on
histopathology and the treatment is ciprofloxacin.

85
214
This is the cause of bacillary angiomatosis:
A. B. burgdorferi
B. B. henselae and B. quintana
C. B. bacilliformis
D. B. dammini
E. B. pacificus
►B

Bacillary angiomatosis is caused by both B. henselae and B. quintana. The vector is unknown
and the treatment is erythromycin and doxycycline.

215
The usual culture medium for mycobacteria is:
A. Chocolate agar in 10% CO2
B. Lowenstein-Jensen
C. New York City
D. Sheep”s blood agar
E. Agar supplemented with heme and nicotinamide
►B

The Lowenstein-Jensen medium is used to culture mycobacteria most commonly. Neisseria


gonorrheae is be grown on chocolate agar in 10% CO2. Thayer “Martin, Martin-Lewis or New
York City medium also allow for N. gonorrheae growth. Agar supplemented with heme (x-
factor) and nicotinamide (v-factor) is needed for Hemophilus influenzae cultures. Sheep”s blood
agar is useful for identifying hemolytic strains of streptococcus and staphylococcus.

216
Herpangina is caused by which of the following?
A. Group A coxsackievirus
B. Epstein Barr Virus
C. Parvovirus B19
D. Human Herpesvirus 6 (HHV-6)
E. Human Herpesvirus 6 (HHV-7)
►A

Herpangina is caused by Group A coxsackievirus.

217

86
What is the most common site of infection from Streptococcus Iniae?
A. Lower leg
B. Face
C. Hand
D. Foot
E. Nails
►C

Streptococcus iniae most commonly causes bacteremic cellulitis of the hand in persons who have
recently handled fresh fish. S. iniae is a fish pathogen that causes sporadic infection in tilapia,
yellowtail, rainbow trout, and coho salmon. Treatment of choice is with penicillin antibiotics.

218
An HIV infected male, presents with a two-week history of a painless 2 cm ulcer on the penile
shaft, with associated inguinal lymphadenopathy. What is the best treatment?
A. Valacyclovir
B. Benzathine penicillin G
C. Ceftriaxone
D. Doxycycline
E. Azithromycin
►B

A painless ulcer in an HIV patient is most likely a syphilitic chancre. The treatment of choice is
benzathine penicillin G in patients without a penicillin allergy. Alternative agents in allergic
patients include doxycycline, tetracycline, erythromycin, and ceftriaxone. Valacyclovir would be
the treatment of choice for genital herpes, which is typically painful. Azithromycin in a single 1
g oral dose is effective for chancroid, which is painful.

219
All of the following statements are true of Bacillus anthracis infection except:
A. Spores remain stable for decades
B. 20% mortality rate in untreated cutaneous infections
C. Inhalation, GI and cutaneous forms exist
D. Ciprofloxacin and doxycycline are first line treatments
E. All statements are true
►E

Anthrax can present in three different clinical forms: cutaneous, inhalational, and
gastrointestinal. The cutaneous form appears as a painless vesicle that later forms an eschar

87
(malignant pustule). The cutaneous form has a mortality rate of less than 1% if treated an up to
20% if untreated.

220
This animal is a nonhuman source of infections for patients for Mycobacterium leprae:
A. Armadillos
B. Horses
C. Tic
D. Fleas
E. Pigs
►A

Leprosy is caused by mycobacterium leprae. Armadillos are a source of nonhuman source for
leprosy. They are also transmitted from human to human most likely from respiratory secretions.

221
These are all old world Leishmaniasis except for:
A. L. mexicana
B. L. major
C. L. tropica
D. L. aethiopica
E. L. infantum
►A

The organisms that are responsible for old world Leishmaniasis are L. major, L. tropica, L.
aethiopica, L. infantum, L. donovani. Those that are responsible for new world Leishmaniasis are
in south and central America which are L. mexicana, L. braziliensis, and L. amazonensis.

222
Which subtype of the human papilloma virus is most likely to have caused this infection?
A. HPV, type 1
B. HPV, type 2
C. HPV, type 3
D. HPV, type 5
E. HPV, type 6
►E

The human papilloma virus is a member of the Papovavirus family, a double-stranded DNA
virus. HPV, type 6 has been implicated in giant conduloma of Buschke and Lowenstein and
anogenital condyloma.

88
223
Streptococcus Iniae has been shown to cause:
A. Perianal dermatitis in neonates
B. Necrotizing fasciitis
C. Bullous impetigo
D. Hand cellulitis in fish handlers
E. Perineal erysipelas in postpartum women
►D

Streptococcus iniae has been demonstrated to cause hand cellulitis from puncture wounds
sustained form the dorsal fin, fish bone or knife of usually a tilapia. Treatment with PCN is
curative. Group A Streptococci are the most common cause of perianal dermatitis. Many
different bacteria have been implicated in necrotizing fasciitis. Bullous impetigo is most
frequently caused by phage type 71 S. aureus or a related group 2 phage type. Group B
streptococcus is most often responsible for perineal erysipelas in postpartum women.

224
Which of the following organisms is spread by non-sexual person-to-person contact?
A. Treponema carateum
B. Chlamydia trachomatis types I, II & III
C. Hemophilis ducreyi
D. Calymmatobacterium granulomatis
E. Treponema pallidum
►A

Treponema carateum is the cause of Pinta which is primarily transmitted by direct contact.
Occasional insect vectors have also been implicated, but sexual contact is not the primary cause
of spread. The other options (C. trachomatis, H. ducreyi, C. granulomatis and T. pallidum) are all
venereal diseases.

225
A patient has a painless, soft erosion that heals spontaneously. The patient also has secondary
inguinal adenopathy with fluctuant, tender nodes above and below Poupart's ligament. This
patient most likely has:
A. Lymphogranuloma venereum
B. Granuloma inguinale
C. Chancroid
D. Chancre
E. Tertiary syphillis

89
►A

The patient has lymphogranuloma venereum that is caused by Chlamydia trachomatis L1, L2,
L3. It is a painless soft erosion that heals spontaneously. Serologic diagnosis is by complement
fixation test. The treatment is doxycycline.

226
A 52-year old male develops a brownish-red plaque on his forehead. The lesion has an "apple
jelly" color on diascopy, and the patient has a positive PPD skin test. What is the most likely
mechanism of disease in this patient?
A. Spread from a distant site
B. Exogenous re-infection
C. Primary inoculation
D. Contiguous spread to skin from underlying infection
E. Id reaction
►A

This patient has lupus vulgaris, a pauci-bacillary manifestation of tuberculosis infection. Lesions
occur in sensitized patients with a moderate to high immune response and are due to
hematogenous, lymphatic, or contiguous spread from a distant site of infection. Lesions typically
occur on the head and neck. "Apple jelly" color on diascopy is described in lupus vulgaris, but
can also be seen in cutaneous lesions of sarcoidosis.

227
A patient has palpable purpura, arthralgias and glomerulonephritis and is diagnosed with
cryoglobulinemia. The most common infection associated with this is:
A. Hep C
B. Hep A
C. Hep B
D. Epstein Barr Virus
E. CMV
►A

This patient has cryoglobulinemia. This is associated with HCV infection. Approximately 80%
of cases of mixed cryoglobulinemia are associated with HCV infection.

228
A patient presents with painful vesicles in a dermatomal distribution on his forehead and on the
tip of his nose. He is sent by his ophthalmologist who diagnosed ophthalmic zoster. Which nerve
branch is involved?

90
A. Ophthalmic
B. Nasal
C. Infraorbital
D. Supraorbital
E. Temporal
►A

The ophthalmic branch of the trigeminal branch is involved in ophthalmic zoster. This
presentation accounts for 10-15% of all cases of VZV. The other nerve branches listed are not
involved in ophthalmic zoster.

229
Coxsackievirus A16 is implicated in the pathogenesis of which of the following dieases?
A. Exanthem subitum
B. Fifth disease
C. Papular purpuric “gloves and socks” syndrome
D. Erythema infectiousum
E. Hand-Foot-and-Mouth disease
►E

Hand-Foot-and-Mouth disease is caused by coxsackievirus A16. Group A coxsackievirus


infection is also associated with herpangina.

230
A patient is diagnosed with hepatitis C. The most common associated autoimmune disorder
associated with hepatitis C is:
A. autoimmune thyroiditis
B. aplastic anemia
C. autoimmune thrombocytopenic purpura
D. Peripheral neuropathy
E. Arthralgias
►A

The most common associated autoimmune disorder is autoimmune thyroiditis. The others can be
associated with hepatitis C extra systemic manifestation but is not as common.

231
A middle aged man of southern-eastern european heritage has red to blue-black patches on the
feet. The most likely cause of this disease is:
A. HHV-8

91
B. HHV-7
C. HHV-6
D. HHV-5
E. HHV-4
►A

This patient above has classic kaposi's sarcoma. It is caused by HHV-8 that causes the abnormal
vascular proliferation associated with this condition. Classic KS is seen in middle aged men from
southern-eastern european heritage.

232
Which of the following is a gram positive rod?
A. Meningococcemia
B. Gonorrhea
C. Glanders
D. Melioidosis
E. Anthrax
►E

Anthrax is a gram positive, spore-forming rod. Meningococcemia, gonorrhea, Glanders, and


Meloioidosis are all gram negative. Infection with anthrax initially begins after skin onoculation
or may follow after ingestion or inhalation of spores. Cutaneous anthrax initially begins as a
localized infection and subsequently evolves to toxemia. It appears as a painless papule that ay
later become hemorrhagic or nerotic. It produces two exotoxins: edema toxin and lethal toxin.
Edema toxin is comprised of edema factor and protective antigen. Lethal toxin is composed of
lethal factor and protective antigen.

233
The vector of Trench Fever is the:
A. Human body louse (Pediculus humanus corporis)
B. Cat flea (Ctenocephalides felis)
C. Sandfly (Phlebotamus perniciosus)
D. Rat flea (Xenopsylla cheopis)
E. Trombiculid mite
►A

The human body louse (Pediculus humanus corporis)is the vector of Trench Fever, Epidemic
Typhus, and Relapsing Fever.

234

92
The causative organism for Brucellosis is which of the following:
A. Gram positive rod
B. Gram positive cocci
C. Gram negative rod
D. Gram negative cocci
E. Acid fast bacilli
►C

Brucella species which cause Brucellosis are gram negative rods. Brucellosis, aka undulant
fever, is characterized by an acute febrile illness with headache and joint pain. CNS and cardiac
manifestations can also occur. It is acquired by contact with infected animals or contaminated
dairy products. Treatment is with doxycycline and rifampin.

235
What is the treatment for secondary syphillis in a penacillin allergic patient?
A. Penicillin
B. Doxcycyline
C. Rifampiin
D. Clindamycin
E. Ceftriaxone
►B

Secondary syphillis is caused by the spirochete, T. pallidum. Penicillin is the treatment of choice
for secondary syphillis. In penicillin allergic individuals, doxycyline may be used. Treatment for
neurosyphilis, congenital syphilis and syphilis in pregnant women is desensitization to penicillin
and then treatment with pcn.

236
A 27 year-old man with a 1-month history of bilateral inguinal lymphadenopathy and a positive
“groove sign”. What is the most likely diagnosis?
A. Syphilis
B. Herpes simplex
C. Chancroid
D. Lymphogranuloma venereum
E. Gonorrhea
►D

Inguinal adenopathy with fluctuant, tender nodes above and below Poupart”s ligament - referred
to as “groove sign” - is a characteristic clinical feature of lymphogranuloma venereum.

93
237
A 45 year old septic female develops symmetric, large ecchymotic areas with irregular borders
on the extremities, ears, and nose. Laboratory testing reveals the patient has disseminated
intravascular coagulation. Which of the following is the most common underlying infection
associated with this condition?
A. Group A streptococcus
B. Meningococcus
C. Staphylococcus
D. Pseudomonas
E. Mycobacterium
►B

Purpura fulminans is an oftentimes dramatic presentation of large ecchymotic areas of the skin,
favoring acral sites including the extremities, ears, and nose. The underlying pathophysiology is
hemorrhagic infarction of the skin due to disseminated intravascular coagulation. Meningococcal
infection is the leading underlying infection associated with sepsis and purpura fulminans. When
Kids develop purpura fulminans after an infection the most common cause is group A Strep.

238
What characteristic color is seen on diascopy of this lesion?
A. Apple jelly
B. Blue
C. Orange
D. Red
E. Coral red
►A

Leishmaniasis recidivans is a type of Old World leishmaniasis. Clinically, it appears as a red


papule covered with white scale. On diascopy, it has a characteristic apple jelly color.

239
Neisseria gonorrheae would be identified with:
A. Sheep”s blood agar
B. Agar supplemented with heme and nicotinamide
C. Chocolate agar with 10% CO2
D. Chocolate agar
E. Darkfield microscopy
►C

94
Neisseria gonorrheae should be grown on chocolate agar in 10% CO2. Thayer”Martin, Martin-
Lewis or New York City medium also allow for N. gonorrheae growth. Agar supplemented with
heme (x-factor) and nicotinamide (v-factor) is needed for Hemophilus influenzae cultures.
Darkfield microscopy is helpful in diagnosing infections with Treponema pallidum (syphilis).
Sheep”s blood agar is useful for identifying hemolytic strains of streptococcus and
staphylococcus.

240
A patient from South America develops a recurrent fever after living in crowded conditions with
multiple roommates having known human body louse infestation. A diagnosis of louse-born
Relapsing Fever is made. Which of the following is the implicated organism?
A. Borrelia recurrentis
B. Borrelia duttonii
C. Borrelia hermsii
D. Borrelia burgdorferi
E. Streptobacillus moniliformis
►A

Louse-born relapsing fever is caused by Borrelia recurrentis. It is seen in Africa and South
America, and is spread by the human body louse (Pediculus humanus). Symptoms include
paroxysmal fevers, headache, myalgias, and petechial macules on trunk and extremities. Borrelia
burgdorferi causes Lyme disease; Borrelia duttonii and Borrelia hermsii cause tick-born
Relapsing Fever; Streptobacillus moniliformis causes Rat-bite fever.

241
A 27 year-old woman who is 30 weeks pregnant presents with erythema migrans. The treatment
of choice for this patient is:
A. Doxycycline
B. Erythromycin
C. Chloramphenicol
D. Clindamycin
E. Amoxicillin
►E

In the setting of pregnancy, amoxicillin is the treatment of choice for Lyme Disease since
doxycycline is contraindicated.

242
Which of the following HPV types causes a ridged wart?
A. HPV 1

95
B. HPV 7
C. HPV 11
D. HPV 18
E. HPV 60
►E

HPV 60 infection leads to a particular type of plantar wart called the ridged wart. HPV 1 leads to
plantar warts and myrmecia. HPV 7 causes butcher's warts. HPV 11 along with 6 causes
anogenital condyloma and giant condyloma of Bucke and Lowenstein (verrucous carcinoma).
HPV 18 causes anogenital dysplasia and neoplasms.

243
The most common extracutaneous complications of varicella zoster virus is:
A. Lymphoreticular
B. Musculoskeletal
C. Cardiovascular
D. Central nervous system
E. Genitourinary
►D

Zoster usually resolves without sequelae in children and young adults with intact immune
systems. However, the pain, cutaneous eruption and complications of zoster become more severe
with increasing age and immune compromise. Complications of zoster include post-herpetic
neuralgia (PHN), secondary bacterial infection, scarring, ophthalmic zoster, Ramsay-Hunt
syndrome, meningoencephalitis, motor paralysis, pneumonitis and hepatitis. Reference: Stalkup
JR et al. Human Herpesviruses. In: Bolognia JL, Jorizzo JL, Rapini RP, eds. Dermatology. New
York: Mosby, 2003:1243.

244
Which of the following is a pox virus?
A. Molluscum contagiosum
B. Herpes simplex virus
C. Pediculosis capitis
D. Sarcoptes scabei
E. Human immunodeficiency virus
►A

Of the answer choices, molluscum contagiosum is the only pox virus. Pediculosis capitis, the
cause of lice, and sarcoptes scabei, the cause of scabies are not viruses. Herpes simplex virus is a
herpes virus and HIV is a lentivirus.

96
245
Where do you see Gamma-Favre bodies?
A. Granuloma inguinale
B. Lymphogranuloma venereum (LGV)
C. Syphilis
D. Chancroid
E. Herpes virus
►B

LGV is a sexually transmitted disease that is characterized by suppurative inguinal adenitis with
matted lymph nodes, inguinal bubo with secondary ulceration, and constitutional symptoms. It is
caused by Chlamydia trachomatis serotypes L1, L2, and L3. First line treatment is doxycycline
100mg bid for 3 weeks as well as treating the sexual partners. Gamma-Favre bodies are found in
histiocytes in LGV.

246
An otherwise healthy six-day-old boy is brought into clinic by his mother with the complaint of
tiny thin-roofed vesicles with a rim of inflammation and a few lesions covered with a “honey-
colored” crust. Which organism is the most likely cause in this newborn?
A. Listeria monocytogenes
B. Group B streptococcus
C. Group D streptococcus
D. Staphylococcus Aureus
E. Lancefield type O organisms
►B

Group B streptococcus is S. agalactae, commonly isolated from normal vaginal mucosa. In


neonates, impetigo caused by group B streptococci may develop. Listeria monocytogenes is a
small, gram-positive organism that may present as petechial, papular or pustular eruptions in
acutely ill infants, usually those that were meconium stained at birth. The usual cause of ecthyma
is Group A streptococcus, though other organisms can occasionally cause similar cutaneous
infections. Group D streptococcus/Lancefield type O organisms are enterococci. S. aureus can
secondarily infect a lesion of ecthyma but is not the usual cause of this condition.

247
The leading infectious cause of deafness and mental retardation in the U.S. is:
A. Congenital Rubella
B. Congenital varicella
C. Congenital Cytomegalovirus (CMV)

97
D. Congenital syphilis
E. Congenital herpes simplex
►C

Congenital CMV is the leading infectious cause of deafness and mental retardation in the U.S.

248
A patient with AIDS develops umbilicated papules consistent with molluscum contagiosum.
What anatomic sites are favored for molluscum papules in patients with HIV?
A. Face and genitalia
B. Head and neck
C. Abdomen and back
D. Upper and lower extremities
E. Chest and shoulders
►A

Molluscum contagiosum is a common opportunistic infection of HIV patients. Lesions can be


seen anywhere, but the most common locations are on the face and genitalia.

249
A 56 year old homeless man has many waxy concretions on his axilla and groin hairs. Which of
the following organisms is the likely cause?
A. Corynebacterium afermentans
B. Corynebacterium tenius
C. Corynebacterium diptheriae
D. Micrococcus sedentarius
E. Corynebacterium minutissimum
►B

This condition described is trichomycosis axillaris. Despite the “mycosis” in the name, it is not
fungal in etiology, but caused by C. tenius. Treatment is topical erythromycin or simply shaving
the infected hairs. The other listed organisms do not cause this type of infection.

250
Herpes-associated erythema multiforme is most commonly associated with which HLA type?
A. HLA B27
B. HLA DR2
C. HLA DR3
D. HLA B15
E. HLA B8

98
►D

Recurrent erythema multiforme (EM minor) is usually caused by recurrent herpes simplex, most
commonly HSV-1 orolabial disease. This is more correctly now called herpes-associated
erythema multiforme(HAEM) and has been more commonly associated with people with HLA
type B15.

251
A patient presents with fever, irritability and a scarlatiniform erythema with accentuation in the
flexural areas. Nikolsky positive blisters develop within 48 hours. Which of the following would
predict the best outcome?
A. Frequent NSAID use
B. Normal renal function
C. Normal immune system
D. Age < 5 years old
E. Age > 18 years old
►D

Staphylococcal scalded skin syndrome is described in the question. This is predominantly a


disease of young children, < 5 years old. The mortality is low in children, but in adults it exceeds
50% due to sepsis and electrolyte and fluid imbalance. NSAID use may predispose patients to a
poorer outcome. Normal renal function and immune system are helpful, but do not predict the
best outcome.

252
The most common bacterial cause of purpura fulminans is:
A. S. aureus
B. P. aeruginosa
C. H. influenzae
D. E. coli
E. Group A streptococcus
►E

Group A streptococcus and Neisseria meningitidis are the leading bacterial causes of purpura
fulminans.

253
Which of the following mycobacterial organisms is classified as a photochromogen?
A. M. Kansasii
B. M. grodonae

99
C. M. fortuitum
D. M. ulcerans
E. M. tuberculosis
►A

M. Kansasii, M. marinum, and M. simiae are photochromogens. They form pigment when
exposed to light.

254
Which of the following serologic tests for syphilis is the most specific early-on in the disease
course?
A. RPR
B. FTA-Abs
C. MHA-TP
D. ELISA - Captia (IgM) EIA
E. VDRL
►D

ELISA - Captia (IgM) EIA is 100% specific for treponemes and is the serology of choice in early
primary or congenital syphilis because it is the first to become reactive. FTA-Abs is the most
sensitive serologic test for syphilis and MTA-TP stays positive for life (but is less sensitive than
FTA-ABs during early disease). The two nontreponemal tests for syphilis include VDRL and
RPR. VDRL is used to follow response to therapy.

255
A 36 year-old homeless man presents with a tender suppurative nodule on the mandible. "Sulfur
granules" are present on microscopy. The most likely diagnosis is:
A. Actinomycetoma
B. Actinomycosis
C. Anthrax
D. Acne Conglobata
E. Aspergillosis
►B

The presentation of a suppurative nodule discharging sulfur granules on the jawline of a patient
with poor oral hygeine is characteristic of actinomycosis - caused by the anaerobic gram-positive
rod, Actinomyces israelii. This differs from Actinomycetoma, which is a slowly progressive
subcutaneous infection characterized by tumefaction, draining sinuses, and an exudate containing
grains on microscopy. Unlike Eumycetomas - caused by fungi - Actinomycetomas are bacterial
infections. The foot is the most common location, followed by the thoracic area.

100
256
A child presents with fever, cutaneous tenderness and erythema of flexural and periorifacial
areas. Within 24 hours the erythema progresses to flaccid blisters and erosions. No organisms are
cultured and Nikolsky's sign is positive. What is the most likely diagnosis:
A. Staph scalded skin syndrome
B. Bullous impetigo
C. Fogo selvagem
D. Pemphigus
E. Pemphigoid
►A

Staph scaled skin syndrome is caused by exfoliative toxins A and B (ET-A and ET-B) produced
by S. aureus (phage group II, types 71 and 55 most common). Histopathology of the skin shows
superficial intraepidermal splitting without epidermal necrosis with very few inflammatory cells.
Desmoglein 1 is the specific receptor for exfoliative toxin A cleavage. Desmoglein 1 is the
antigen targeted in Fogo selvagem, the vector for which is the black fly. Bullous impetigo is
caused by Staph and is a subgranular blister with bacteria and neutrophils on histopahtology.
Pemphigoid and pemphigus would be less likely in a child.

257
Which antibiotic is best to treat this condition caused by Bartonella hensalae.
A. Erythromycin
B. Ceftriaxone
C. Clindamycin
D. Trimethoprim/sulfamethoxasole
E. Fluconazole
►A

Bartonella hensalae is the causative agent of bacillary angiomatosis. The differential diagnosis of
bacillary angiomatosis may include pyogenic granulomas and Kaposi's sarcoma. The treatment
of choice is erythromycin.

258
Most common causative agent for the lesion shown in image for this 34-year old
immunocompetent man is
A. Staphaylococcus aureus
B. Group A Beta hemolytic streptococcus
C. Pseudomonas aeruginosa
D. Klebsiella

101
E. Proteus
►B

Ecthyma is a deep or ulcerative type of pyoderma commonly seen on the lower extremities and
buttocks and caused most often by GABHS, in addition to staphyloccous aureus which also can
be involved in the infection. Ecthyma can present as small punched-out ulcers or a deep
spreading ulcerative process. The disorder begins in the same manner as impetigo, often
following infected insect bites or minor trauma, but penetrates through the epidermis to produce
a shallow ulcer. The initial lesion is a vesiculopustule with an erythematous base and firmly
adherent crust. Removal of the crust reveals a lesion deeper than that seen in impetigo, with an
underlying saucer-shaped ulcer and raised margin. The lesions are painful and heal slowly over a
few weeks, often with scar formation. Ecthyma gangrenosum is a cutaneous finding that may be
seen in patients with Pseudomonas aeruginosa bacteremia. Most of the affected individuals have
an underlying immunodeficiency (either congenital or acquired) or a history of cancer
chemotherapy. Neutropenia may be a risk factor for ecthyma gangrenosum

259
Unilateral palpebral and periorbital edema is characteristic of which of the following infectious
diseases?
A. African trypanosomiasis
B. American trypanosomiasis
C. Amebiasis
D. Schistosomiasis
E. Onchocerciasis
►B

American trypanosomiasis (Chagas‟ disease) is caused by infection with Trypanosoma cruzi via
Triatoma arthropod vectors (reduviid bugs) and is prevalent predominantly in Central and South
America. Transmission of this protozoal infection may occur following the bite of an infected
reduviid bug, who then defecates on human skin after feeding, allowing the metacyclic
trypomastigotes entry into the bite wound. Alternatively, infected feces may gain direct entry
through mucosal surfaces, including the conjunctivae. When the latter occurs, characteristic
unilateral palpebral and periorbital edema ensues, known as “Romana‟s sign.” Entry into the skin
may result in a localized plaque of erythema and induration with regional lymphadenopathy,
termed a “chagoma.” Other systemic complications resulting from chronic infection include
cardiac arrhythmias, congestive heart failure, megacolon, and megaesophagus. Treatment is with
nifurtimox or benznidazole.

260
What is the following is the best choice for treatment of herpes zoster?

102
A. Ganciclovir
B. Foscarnet
C. Valacyclovir
D. Gabapentin
E. Cidofovir
►C

First line treatment of herpes zoster (shingles) is with valacyclovir, acyclovir, or famciclovir.
Gabapentin may be used for post-herpetic neuralgia. Cidofovir and ganciclovir are treatment
options for cytomegalovirus. Foscarnet is used to treat acyclovir resistant herpes simplex
infections.

261
Which of the following is true regarding the Gardasil vaccine?
A. It is a live vaccine
B. It is FDA approved for females aged 18 to 25
C. It protects against HPV 16 and 18 only
D. It can be administered regardless of history of abnormal pap smear
E. It does not protect against genital warts
►D

Gardasil is a quadrivalent vaccine containing HPV types 6, 11, 16 and 18. It is approved for
females ages 9 to 26. Neither HPV testing nor pap smears are necessary before vaccine
administration. It is recommended for the prevention of cervical, vaginal, and vulvar cancers, as
well as for genital warts.

262
Non tuberculosis mycobacterial infections include M. marinum treatment would include:
A. Clarithromycin
B. Cephalosporin
C. Penicillins
D. Quinolones
E. Azithromycin
►A

M. marium is also known as fish tank granuloma. Empiric therapy is suggested with M. marinum
infection and the first line treatment is minocycline, #2 is clarithromycin.

263

103
American trypanosomiasis is transmitted by humans via reduviid or kissing bugs. It is caused
by:
A. Trypanosome cruzi
B. Toxoplasma gondii
C. Pneumocystis carinii
D. Entamoeba histolytica
E. Tamandua tetradactyla
►A

African trypanosomas is also known as sleeping sickness and is caused by Trypanosoma brucei
and transmitted by the tsetse fly Glossina. It can occasionally have cutaneous manifestations
such as chancre and trypanids.

264
Streptobacillus moniliformis is the causative organism of which infectious disease?
A. Rat-bite fever (Haverhill fever)
B. Scrub typhus
C. Tularemia
D. Glanders
E. Cat scratch disease
►A

Rat-bite fever (Haverhill fever) is caused by Streptobacillus moniliformis.

265
Which of the following is the causative organism of Tularemia?
A. Burkholderia mallei
B. Streptobacillus moniliformis
C. Francisella tularensis
D. Orientia tsutsugamushi
E. Klebsiella pneumoniae
►C

Tularemia is caused by infection with Francisella tularensis - most commonly acquired after
contact with infected rabbits (e.g. hunters).

266
A woman who handles fish tanks develops a slow-growing cluster of papules on the hand. What
temperature (Celsius) is most optimal for this organism's growth?
A. 25 degrees

104
B. 30 degrees
C. 37 degrees
D. 40 degrees
E. 42 degrees
►B

This patient most likely has contract Mycobacterium marinum, otherwise known as "fish tank
granuloma." On primary isolation M. marinum grows best at 30-33 degrees C in 7-21 days.
Unlike Mycobacterium tuberculosis, most strains of M. marinum will not grow at the usual
incubation temperature of 37 degrees C. Dimorphic fungi have both yeast and fungal forms:
hyphae morphology at 25 degrees C and yeast phase at 37 degrees C.

267
A patient recently had pharyngitis and now has an eruption of guttate psoriasis. Which of the
following tests would be helpful in determining that this patient had a Streptococcal infection?
A. A complete blood count
B. FTA-ABS
C. U1-RNP
D. DNAse B
E. Serum calcium
►D

The DNAse B, hyaluronidase or streptolysin O (ASLO) antibodies in serum can be helpful for
confirming streptococcal infection in situations where streptococci cannot be isolated. The FTA-
ABS is used for detecting syphilis, and is the only test that is positive in the first 14 days of
infection. U1-RNP is an extractable nuclear antibody which is a marker for Mixed Connective
Tissue Disease.

268
At what rate of speed does this parasite migrate through the skin?
A. 0.1 cm/day
B. 1 cm/day
C. 2 cm/day
D. 10 cm/day
E. 100 cm/day
►C

The infection depicted is larva migrans or creeping eruption which is caused by Ancylostoma
braziliense. This nematode is typically a hookworm of cats and dogs. It migrates at a rate of 2
cm/day. Larva currens migrates at a rate of 10 cm/day.

105
269
A male patient experiences painful vesicles on the palate, tongue, and ipsilateral eardrum.
Additionally, he complains of vertigo and difficulty with speaking and eating. Involvement of
what anatomic structure is responsible for this constellation of symptoms?
A. Geniculate ganglion
B. Dorsal root ganglion
C. Hippocampus
D. Basal portion of pons
E. Hypoglossal nerve
►A

The question stem describes Ramsay-Hunt syndrome, which is characterized by unilateral


vesicles on palate, tongue, earlobe, and eardrum, in association with neurologic symptoms
including vertigo, difficulty speaking, difficulty eating, hearing loss, and facial droop. The
syndrome is caused by varicella-virus infection of the geniculate ganglion.

270
A 37-year-old man presents with a red, tender papule on his lateral right hand that subsequently
becomes necrotic with regional lymphadenopathy. An exanthem is also present. Which animal
was this individual hunting?
A. Rabbits
B. Grouse
C. Squirrels
D. Deer
E. Moose
►A

The presentation described above is that of Tularemia, caused by Francisella tularensis. The most
common form of Tularemia is the ulceroglandular form, which is described above. Infected
animals, most commonly rabbits, transmit the organism though occasionally tick bites or deer
flies may be the vector. Squirrels may be the vector of bubonic plague in endemic areas. The
other animals would not cause a syndrome as described.

271
Which of the following Rickettsial diseases would be rarely positive on Weil-Felix test?
A. Rocky Mountain Spotted Fever
B. Epidemic Typhus
C. Rickettsial pox
D. Endemic typhus

106
E. Scrub Typhus
►C

Rickettsial pox is diagnosed by clinical suspicion and confirmatory biopsy, the Weil-Felix test is
rarely positive. The other listed Rickettsial diseases will have positive Weil-Felix tests. The
Weil-Felix test makes use of the cross-reaction between antigens from the gram-negative Proteus
vulgaris species and Rickettsia spp. It is not specific and rising titers in the appropriate clinical
presentation is necessary for confirmation. The Proteus antigens OX-K, OX-19 and OX-2 are
employed.

272
A 10 year-old with a 3-day history of fever and headache presents to the clinic with multiple
erythematous papulopustules on his extremities, many of which have central eschars. The most
likely diagnosis is:
A. Lyme disease
B. Rickettsialpox
C. Erlichiosis
D. Rocky mountain spotted fever
E. Meningococcemia
►B

Rickettsialpox is an acute febrile illness caused by the bacteria Rickettsia akari, a member of the
spotted fever group. R. akari is transmitted by the house mouse mite, Liponyssus sangineus.
Clinically, patients have an eschar at the inoculation site with a subsequent papulovesicular rash,
fevers, headache, chills, diaphoresis, myalgia, and anorexia.

273
There are four clinical signs of granuloma inguinale and one can see Donovan bodies on
microscopy. Granuloma inguinale is caused by:
A. Klebsiella granulomatis
B. Chlamydia trachomatis
C. H. ducreyi
D. T. pallidum
E. Pseudomonas aeruginosa
►A

Granuloma inguinale is caused by Klebsiella (old name: Calymmatobacterium) granulomatis.


The primary lesions are pseudobubo or ulcer. There are four clinical forms: ulcerovegetative,
nodular, hypertrophic, and cicatricial. Donovan bodies are seen on microscopy.

107
274
The usual cause ecthyma is:
A. Group A streptococcus
B. Group B streptococcus
C. Group D streptococcus
D. Staphylococcus Aureus
E. Lancefield type O organisms
►A

The usual cause of ecthyma is Group A streptococcus, though other organisms can occasionally
cause similar cutaneous infections. Group B streptococcus is S. agalactae, commonly isolated
from normal vaginal mucosa. Infections can be problematic for neonates, but not a frequent
cause of ecthyma. Group D streptococcus/Lancefield type O organisms are enterococci. S.
aureus can secondarily infect a lesion of ecthyma but is not the usual cause of this condition.

275
Which of the following is the vector for Dengue fever?
A. Phlebotomus papatasii
B. Culex mosquito
C. Aedes aegypti
D. Lutzomyia verrucarum
E. Xenopsylla cheopis
►C

Dengue fever also known as "break-bone" fever is characterized by sudden high fever, backache,
retro-orbital pain, bone/joint pain, weakness, and malaise. It is caused by an arbovirus (RNA
virus) which is transmitted by Aedes aegypti, a species of mosquito.

276
What is the most common cause of focal epithelial hyperplasia?
A. Ebstein Bar virus(EBV)
B. Pachyonychia Congenita
C. human immunodeficiency virus(HIV)
D. human papilloma virus(HPV)
E. Dyskeratosis Congenita
►D

Focal epithelia hyperplasia, also known as Heck's disease, is caused by HPV types 13, 32 and 57.
It is most commonly found on the lower lip but also on the buccal mucosa, gums and tongue. It
is mainly a disease of native Americans and Greenlander Eskimos. EBV causes an oral hairy

108
leukoplakia in patients with AIDS. Pachyonychia Congenita patients commonly develop a
benign oral leukoplakia, while Dykeratosis Congenita patients develop a premalignant oral
leukoplakia.

277
Tricomycosis axillaris is caused by:
A. Burkholderia mallei
B. Proteus species
C. Corynebacterium tenuis
D. Micrococcus sedentarius
E. Corynebacterium minitissimum
►C

Corynebacterium tenuis is the causative organism associated with most cases. Although up to
33% of adults have colonization by this bacterium in the inguinal or axillary regions, factors such
as hyperhidrosis predispose to more extensive growth and resultant clinical manifestations.

278
What is the classic CXR finding of a patient with inhalational anthrax?
A. Alveolar infiltrates
B. Increased interstitial markings
C. Pleural effusions
D. Widened mediastinum
E. Normal x-ray
►D

Inhalation anthrax is the most lethal form of anthrax. Other forms include cutaneous anthrax and
GI anthrax. The classic radiographic finding is a widened mediastinum.

279
The treatment of choice for Oroya Fever is:
A. Penicillin
B. Doxycycline
C. Erythromycin
D. Minocycline
E. Chloramphenicol
►E

The treatment of choice for Oroya Fever is chloramphenicol because of frequent superinfection
with Salmonella.

109
280
A 21 year old female with a family history of C8 complement deficiency presents with sparsely
distributed hemorrhagic vesiculopustules on the palms, soles, and over joints. She has associated
fever, chills, arthralgias, and malaise. What is the treatment of choice for disseminated
gonococcemia?
A. Ceftriaxone IV
B. Penicillin IM
C. Piperacillin-tazobactam IV
D. Levofloxacin PO
E. Azithromycin
►A

Gonococcemia is characterized by sparsely distributed hemorrhagic vesiculopustules on the


palms, soles, and over joints with associated fever, chills, arthralgias, and malaise. Recurrent
cases associated with complement deficiencies (C5-C8). Treatment is with IV ceftriaxone.

281
What is the most common single nerve involved with herpes zoster?
A. Facial nerve
B. Trigeminal nerve
C. Spinal nerve C7
D. Spinal nerve T4
E. Spinal nerve T10
►B

While herpes zoster can be seen in the dermatome of any cranial or spinal nerve, the most
common single nerve involved is the trigeminal nerve.

282
Hebra nose deformity is characteristic of infection with which organism?
A. Klebsiella pneumoniae
B. E. coli
C. Streptococcus pyogenes
D. Haemophilus influenzae
E. Serratia marcesens
►A

Hebra nose deformity is seen in Rhinoscleroma which is caused by Klebsiella pneumoniae


subspecies rhinoscleomatis.

110
283
A 52 year-old butcher presents with a tender lesion on his left hand as shown. The treatment of
choice for this condition is:
A. Penicillin
B. Azithromycin
C. Ciprofloxacin
D. Tetracycline
E. Chloramphenicol
►A

The clinical description and lesion shown in the image suggest a diagnosis of erysipeloid, caused
by Erysipelothrix rhusiopathiae. The treatment of choice is penicillin.

284
A laboratory worker undergoes injection of vaccinia virus for vaccination against smallpox.
What type of reaction is needed to ensure adequate development of immunity?
A. Vesicle or ulcer surrounded by 4 cm of erythema
B. Lichenoid plaque
C. Development of vesicles at site separate from injection
D. Targetoid plaque
E. Systemic flu-like reaction with fever and myalgia
►A

Vaccinia virus is used to vaccinate high-risk individuals against smallpox virus. Development of
a vesicle or ulcer with 4 cm of surrounding erythema is a reaction considered to have a high rate
of adequate immunization.

285
What is the risk of transmission of HSV from active cervical lesions when a C-section is
performed?
A. 1%
B. 8%
C. 17%
D. 31%
E. 43%
►A

111
When a C-section is performed in the setting of active cervical lesions of HSV, the risk of
maternal-fetal transmission is decreased to 1%. If a child were delivered vaginally in such a
setting, the risk of transmission is approximately 8%.

286
A pool worker develops these painful erythematous nodules with ulceration on his fingers and
then his arms. The antibiotic of choice to treat this infection is:
A. Penicillin
B. Ceftriaxone
C. Minocycline
D. Trimethoprim-sulfamethoxasole
E. Cephalexin
►C

Sporotrichoid spread may be seen in cutaneous leishmaniasis, actinomycosis, atypical


mycobacterial infections, deep fungal infection, and melanoma. Mycobacterium marinum is
associated with skin injury followed by exposure to contaminated water, usually from an
aquarium, lake, or pool. The treatment of choice for M. marinum infections is minocycline.

287
A 25-year old male from Egypt complains of slowly enlarging chronic growths on his external
nares that are causing nasal obstruction. Biopsy reveals Mikulicz cells on histopathology. Which
of the following medications would be most appropriate for treatment of these lesions?
A. Ciprofloxacin
B. Penicillin
C. Erythromycin
D. Trimethoprim-Sulfamethoxazole
E. Imipenem
►A

This patient has rhinoscleroma, a chronic granulomatous infection of the nose and upper
respiratory tract by Klebsiella pneumoniae rhinoscleromatis. The disease is endemic in Africa,
Southeast Asia, Mexico, Central and South America, and Central and Eastern Europe and is
rarely reported in the United States. Presentation may vary from rhinitis to diffuse airway
obstruction. Mikulicz cells are foamy, vacuolated macrophages seen in rhinoscleroma.
Antibacterial therapeutic choices include ciprofloxacin, tetracycline, and rifampin. Surgical
treatment may be required for obstructive lesions.

288

112
A patient has a granulomatous plaque that is diagnosed as swimming pool granuloma. The
treatment for this is:
A. Minocycline
B. Ampicillin
C. Thalidomide
D. Penicillin
E. Rifampin
►A

Swimming pool granuloma or fish tank granuloma is caused by M. marinum and begins as a
small plaque at the site of inoculation and evolves to a granulomatous plaque. It occurs as a
sporotrichoid spread. The treatment is minocycline.

289
Pediculus humanus corporis can transmit:
A. Murine typhus
B. Epidemic typhus
C. Scrub typhus
D. Endemic typhus
E. Oriental typhus
►B

The body louse or Pediculus humanus corporis transmits Rickettsia prowasekii the organism
responsible for epidemic typhus. Endemic typhus or murine typhus is caused by R. typhi that is
transmited by the rat flea/Xenopsylla cheopis. Scrub typhus is caused by R. tsutsugamushi and is
transmitted by chiggers or trombiculid mite larvae.

290
The causative organism in Whitmore disease is which of the following?
A. Streptobacillus moniliformis
B. Vibrio vulnificus
C. Klebsiella pneumoniae
D. Pseudomonas aeruginosa
E. Burkholderia pseudomallei
►E

Whitmore disease also known as Melioidosis is characterized by pulmonary disease, septicemia,


and miliary abscesses. It is caused by Burkholderia pseudomallei.

291

113
All of the following are features of the Ramsay Hunt Syndrome EXCEPT:
A. Tinnitus
B. Facial paresis
C. Vesicles on the external ear
D. Herpes simplex infection
E. Infection of the geniculate ganglion
►D

Ramsay Hunt syndrome results from varicella zoster virus infection of the geniculate ganglion of
the seventh cranial nerve (CN VII). It is characterized by vesicles on the external ear or ear
canal, tinnitus and/or other auditory symptoms, and ipsilateral facial paresis.

114
Chapter -9-
Pediatric Dermatology
1
Late onset subungual keratotic tumors are associated with:
A. Incontinentia pigmenti
B. Neurofibromatosis Type 1
C. Carney complex
D. Cowden syndrome
E. Basal cell nevus syndrome
►A

A NEMO gene defect can cause subungual keratotic growths. The typical age of presentation is
early adulthood.

2
816 activating mutation in c-kit are found most often in which subset of patients with
mastocytosis?
A. Adults with systemic disease refractory to imantinib
B. Adults with systemic disease associated with eosinophilia
C. Patients with familial history of mastocytosis
D. Adults with Telangiectasia Macularis Eruptiva Perstans
E. As a mosaic mutation in children with solitary cutaneous mastocytoma
►A

Almost all sporadic adult onset mastocytosis patients demonstrate mutations in c-kit. Most of
these are activating mutations linked to the 816 codon. Unfortunately, patients with this specific
mutation tend to not respond to systemic therapy with oral imantinib, a tyrosine kinase inhibitor.

3
A 2-year old female presents with a skin eruption, and a history of joint swelling, painful
movement and mucosal lesions. What is the most likely diagnosis?
A. Pustular psoriasis
B. PAPA syndrome
C. Familial Mediterranean fever
D. DIRA
E. Subcorneal pustular dermatosis
►D

115
Deficiency of the IL-1 receptor antagonist(DIRA). These patients, early in their lives, present
with severe pustulosis and ichthyosiform skin eruptions. Also they can have joint pains and oral
mucosal lesions. DIRA is a new addition to the spectrum of autoinflammatory disorders. It will
not be encountered often, but its origin highlights the importance of IL-1 in skin function.

4
The risk of fetal death with intrauterine parvovirus infection may occur with infection in which
trimester:
A. First
B. Second
C. Third
D. First, Second and Third
E. None of these answers are correct
►D

Fetal hydrops may occur with parvovirus infection during all three trimesters although the
greatest risk is during the second trimester. Congenital anomalies are not a feature.

5
This syndrome is associated with a large nevus sebaceous associated with ocular lesions,
intracranial masses, mental retardation, seizures, and skeletal and pigmentary abnormalities:
A. Schimmelpenning's syndrome
B. Linear epidermal nevus syndrome
C. Nevus sebaceous of Jadassohn
D. Epstein's pearls
E. Pseudoverrucous syndrome
►A

Schimmelpenning's syndrome involves a large nevus sebaceous associated with ocular lesions,
intracranial masses and mental retardation. Nevus sebaceous is a congenital hairless, yellow to
orange plaque on the scalp.

6
Which of the following disorders is more likely to occur in children with chronic fecal
incontinence?
A. Langerhans cell histiocytosis
B. Perianal pseudoverrucous papules and nodules
C. Perianal streptococcal disease
D. Granuloma gluteale infantum
E. Seborrheic dermatitis

116
►B

The warty papules of perianal pseudoverrucous papules and nodules are seen more commonly in
the setting of chronic fecal incontinence.

7
A newborn has a nodule over his lumbar spine. Skin biopsy reveals a lipoma. The next
appropriate step is:
A. Observation
B. Excision of the lesion
C. Genetic testing
D. Imaging study
E. Malignancy work up
►D

The skin can provide an important clue to the presence of an underlying neural tube defect, such
as meningomyelocele and encephalocele. Cutaneous lesions along the midline of the spine
should always prompt consideration of this possibility. Although, midline neural tube defects are
uncommon, early recognition and diagnosis of a spinal dysraphism can have important
implications for early surgical correction and minimizing loss of neurologic function. Clues to
the diagnosis include a midline dimple, tuft of hair, lipoma, or vascular lesion. In these instances,
imaging studies (MRI, CT, ultrasound) should be promptly initiated.

8
An infant presents with the lesion depicted in the photo. What information do you provide the
parents regarding this condition?
A. The lesion will go through rapid growth followed by stabilization and regression
B. The lesion will respond to laser therapy
C. The lesion will persist and may grow further
D. The lesion will resolve with antifungal therapy
E. The lesion is associated with a XO karyotype
►C

The lesion represented in the photo is an arteriovenous vascular malformation. These lesions do
not typically regress.

9
Which of the following diseases with immunodeficiency has an increased risk of lymphoreticular
malignancy?
A. Wiskott-Aldrich syndrome

117
B. Chronic granulomatous disease
C. Job syndrome
D. Severe combined immunodeficiency syndrome
E. Leiner‟s disease
►A

Wiskott-Aldrich syndrome is an X-linked recessive disorder caused by mutations in WAS gene.


Patients with Wiskott-Aldrich have atopic dermatitis with increased risks for secondary
infection, thrombocytopenia, and recurrent bacterial infections. They are also at an increased risk
for lymphoreticular malignancy (20%).

10
What is the most likely diagnosis is this 16 year old patient who developed generalized eruption
2 weeks after onset of sore throat due to strep infection?
A. Psoriasis
B. Pityriasis rosea
C. Cutaneous T-cell lymphoma
D. Contact dermatitis
E. Lichen planus
►A

Although the exact mechanism is not known, there is a well-known correlation between a history
of upper respiratory infection secondary to group A beta-hemolytic streptococci and the
subsequent development of guttate psoriasis. The eruption may resolve spontaneously or with
treatment (topical steroids or ultraviolet therapy). Some portion of these patients eventually
develop chronic, plaque-type psoriasis.

11
The differential diagnosis of zinc deficiency is least likely to include:
A. Granuloma gluteale infantum
B. Biotin deficiency
C. Multiple carboxylase deficiency
D. Cystic fibrosis
E. Holocarboxylase synthetase deficiency
►A

All of the options result in an eczematous acrodermatitis enteropathica-like eruption except


granuloma gluteale infantum. As the name suggests, the lesions of granuloma gluteale infantum
are granulomatous.

118
12
Ankyloblepharon filiforme adnatum is seen with a defect in:
A. Plakophilin
B. Plakoglobin
C. Desmoglein
D. C-kit
E. p63
►E

The ectodermal-clefting syndromes are caused by a defect in the p63 gene. Specifically, AEC or
Hay-Wells syndrome is comprised of ankyloblepharon filiforme adnatum, ectodermal dysplasia
and clefting.

13
A full term newborn develops erythematous, indurated plaques on the upper back. Which of the
following tests should be performed?
A. Calcium
B. Echocardiogram
C. Blood culture
D. Thyroid function test
E. Alkaline phosphatase
►A

The most likely diagnosis is subcutaneous fat necrosis of the newborn. Hypercalcemia may result
and serum calcium levels should be monitored up to 4 weeks after resolution of the skin.

14
Rapp-Hodgkin is caused by a defect in the following gene:
A. Plakophilin
B. Desmoglein 1
C. Ectodysplasin A
D. P63
E. Connexin 30
►D

Rapp-Hodgkin, along with the other ectodermal dysplasia-clefting syndromes, is reportedly


caused by a defect in the p63 gene.

15
“Slapped cheeks” followed by a lacy eruption on extremities:

119
A. ssDNA virus
B. dsDNA virus
C. ssRNA virus
D. dsRNA virus
E. Streptococcus
►A

The erythematous eruption on the cheeks precedes the generalized lacy rash of Fifth disease. The
causative agent is parvovirus B19, a single stranded DNA virus.

16
Variants of xeroderma pigmentosum are due to all of the following defects except:
A. Nucleotide excision repair
B. Helicase
C. Endonuclease
D. Thymidine kinase
E. Postrepliction repair
►D

The photosensitivity of xeroderma pigmentosum is caused by defect in DNA repair mechanisms.


Thymidine kinase is not affected.

17
Pastia”s lines are characteristic for which eruption:
A. Measles
B. Scarlet fever
C. Rubella
D. Kawasaki”s disease
E. Mumps
►B

Pastia”s lines are defined as a linear accentuation of the erythematous sandpaper rash within the
flexures. This is classically described with scarlet fever.

18
The genetic disorder depicted in this photo is caused by a mutation in:
A. MSH2
B. STK11
C. PTEN
D. Patched

120
E. TSC1
►B

The multiple labial lentigo seen on this young boy are seen with Peutz-Jeghers syndrome. A
defect in the STK11 gene is responsible.

19
Individuals with which of the following syndromes characteristically present with
photosensitivity, mental retardation, a "wizened" appearance, "bird-headed" facies, and "Mickey
Mouse" ears?
A. Tay Syndrome
B. Rothmund-Thomson Syndrome
C. Hutchinson-Gilford Progeria Syndrome
D. Cockayne Syndrome
E. Werner Syndrome
►D

Cockayne Syndrome is AR, caused by defective excision repair, cross-complementing group 8


gene (ERCC8). It presents with photosensitivity, mental retardation, and cachectic dwarfism.
Patients have a characteristic "wizened" appearance, "bird-headed" facies, and "Mickey Mouse"
ears. Cataracts, deafness, pigmentary retinopathy, dental caries, and skeletal, GU, and endocrine
abnormalities may be seen.

20
What syndrome is the disorder shown in the photo associated with?
A. Turner syndrome
B. Down syndrome
C. Noonan syndrome
D. Griscelli syndrome
E. Bloom syndrome
►C

The photo shows keratosis pilaris atrophicans faceii and surgically corrected ptosis. Both of these
findings are associated with Noonan syndrome.

21
Juvenile self-healing papular mucinosis is associated with which of the findings?
A. Arthralgias
B. Cataracts
C. Type I diabetes

121
D. Calcinosis cutis
E. Xeropthalmia
►A

Self-healing papular mucinosis is a rare disorder characterized by small, nontender, flesh-colored


papules commonly seen on the head, neck, trunk, and periarticular areas of children. Self-healing
papular mucinosis is associated with systemic findings during acute stages, including fever,
arthralgias, weakness, and often carpal tunnel syndrome. A handful of cases have been reported
in adults, although systemic symptoms are not typically found in this population.

22
An 8 year-old boy presents with pink, flushed cheeks and a low-grade fever. One week later, a
lacy eruption appears. What is the most likely etiology?
A. Paramyxovirus
B. Togavirus
C. Group A streptococcus
D. HHV6
E. Parvovirus
►E

Fifth disease is caused by Parvovirus B19. Most cases start with prodrome of fever, malaise,
headache and rhinorrhea. Cutaneous reaction follows approximately 5-7 days later with erythema
of the cheeks ("slapped cheeks") and reticulate rash of the trunk and extremities.

23
An infant presents with yellowish-brown, crusted papules with petechiae in a seborrheic
distribution. Which of the following statements about this entity is likely to be true?
A. This presentation is consistent with Jacquet's dermatitis
B. CD1+, S100+ cells with comma-shaped nuclei should be seen on biopsy
C. This presentation occurs when the infant is weaned off of breast milk
D. An autosomal recessive defect in holocarboxylase synthetase is the cause
E. Maternal-fetal transmission most likely occured in the peripartum period
►B

This infant has Langerhans Cell Histiocytosis (Letterer-Siwe disease). Multisystem involvement
may be present. Jacquet's Erosive Dermatitis presents with severe erosive papules in the diaper
region, and is multifactorial in etiology (yeast, irritant dermatitis, moisture). Acrodermatitis
enteropathica (zinc deficiency) presents with brown, orange crusted plaques with vesicles and
bullae, especially in perineal and perioral areas and distal extremities. The inherited form occurs

122
when the infant is weaned off of breast milk. Biotin deficiency presents similarly; the neonatal
form is attributed to an AR defect in holocarboxylase synthetase.

24
The standard of care of patients with acute Kawasaki”s disease is:
A. Prednisone
B. Supportive care
C. Aspirin and IVIG
D. Penicillin
E. Acetaminophen and IVIG
►C

Kawasaki disease, also called mucocutaneous lymph node syndrome, is an acute febrile disorder
based on the clinical criteria of changes in peripheral extremities, polymorphous exanthema,
conjunctival injection without exudates, changes in the lips or oral cavity, acute cervical
lymphadenopathy. Fever must be present, lasting more than 5 days. Treatment is aimed to
prevent coronary aneurysms and myocardial infarction. Treatment for acute Kawasaki disease is
intravenous immunoglobulin 2 g/kg over 10-12 hours and aspirin therapy.

25
Which of the following is a ssDNA virus:
A. Herpesvirus
B. Parvovirus
C. Picornovirus
D. Adenovirus
E. Parapox
►B

Parvovirus is the only ssDNA virus listed.

26
Which of the following is the most common long term sequelae from congenital rubella
syndrome?
A. Macrocephaly
B. Saber shins
C. Deafness
D. Nystagmus
E. Microcephaly
►C

123
Deafness may occur in up to 50% of infants with congenital rubella syndrome.

27
Osteopathia striata is found in which disorder?
A. McCune-Albright syndrome
B. Neurofibromatosis I
C. Buschke-Ollendorff syndrome
D. Gorlin's syndrome
E. Focal dermal hypoplasia
►E

Osteopathia striata (vertical striations in the metaphysis of long bones on x-ray) is seen is greater
than 80% of cases of focal dermal hypoplasia (or Goltz syndrome). Polyostotic fibrous dysplasia
with recurrent fractures is seen in McCune-Albright syndrome. Sphenoid wing dysplasia and
thinning of long bone cortex is found in neurofibromatosis I. Osteopoikilosis is an asymptomatic
x-ray finding in patients with Buschke-Ollendorf syndrome. Osteopoikilosis reflects ectopic
calcification that does not increase risk of fracture. Bifid ribs, vertebral fusion/Sprengel
deformity of the spine, and kyphoscoliosis can be seen in basal cell nevus syndrome (Gorlin's
syndrome).

28
What is the most common cause of neonatal purpura fulminans?
A. Strep
B. Staph
C. Varicella
D. Protein C deficiency
E. Factor V Leiden deficiency
►D

In children, purpura fulminans may have several causes. It is a highly characteristic feature of
meningococcal septicemia and may occur as a sequel to a number of infections, including
streptococcal infections, varicella, and measles. In the neonate, however, its occurrence is highly
suggestive of homozygous protein C deficiency.

29
A newborn infant presents with ring of long, dark, coarse hair surrounding a midline scalp patch
of alopecia. What is associated with this finding?
A. Alopecia areata
B. Ectopic brain tissue
C. Thyroid disease

124
D. Nevus sebaceus of Jadassohn
E. Deafness
►B

The "hair collar sign" is associated with ectopic brain tissue and is thought to arise from a
congenital herniation through the skull. Caution must be used in evaluation as biopsy or needle
aspiration may lead to retrograde infection.

30
What is the most likely diagnosis?
A. Psoriasis
B. Langerhans cell histiocytosis
C. Granuloma gluteale infantum
D. Contact dermatitis
E. Perianal streptococcal disease
►E

The bright red erythema of perianal streptococcal disease can also involve the creases of the
groin and axillae.

31
The muscle that is affected in a patient with a fibromatosis colli is:
A. Sternocleidomastoid muscle
B. Masseter muscle
C. Frontalis muscle
D. Bicepts
E. Tricepts
►A

Fibromatosis colli affects the lower sternocleidomastoid muscle that my arise from birth trauma.
These occur in the pediatric patients and can spontaneous remit.

32
Which of the following clinical signs is most likely associated?
A. Clitoral hypertrophy
B. Neurofibromas
C. Axillary freckling
D. Hypertrichosis
E. Alopecia
►A

125
The “coast of Maine” café au lait macule is associated with McCune-Albright syndrome.
Endocrine abnormalities, including precocious puberty and polyostotic fibrous dysplasia are
associated features. The genetic mutation which results in this genodermatosis is due to a post-
somatic mutaion in the alpha sub-unit of stimulatory G-protein.

33
Regarding the Langerhans cell histiocytoses, which of the following statements is true:
A. Letterer-Siwe primarily affects children ages 2-4 years
B. Hand-Schuller-Christian represents the triad of diabetes mellitus, bone lesions,
and exophthalmos
C. Hashimoto-Pritzker is characterized by a solitary, self-resolving papule or
nodule
D. Approximately 50% of cells will have Birbeck granules.
E. Langerhans cell histiocytoses stain positively for CD1a, S100, HAM56, and
CD68.
►D

Letterer-Siwe primarily affects children under age 2. Hand-Schuller-Christian represents the


triad of diabetes insipidus, bone lesions, and exophthalmos. Hashimoto-Pritzker is characterized
by a self-resolving widespread eruption. Langerhans cell histiocytoses stain positively for CD1a
and S100 but not for HAM56 and CD68.

34
A 2 day-old full term neonate develops blotchy erythematous macules with small central
pustules over the upper trunk and extremities. A gram stain reveals predominantly eosinophils.
What is the most likely diagnosis?
A. Erythema toxicum neonatorum
B. Incontinentia pigmenti
C. Urticaria pigmentosa
D. Transient neonatal pustular melanosis
E. Miliaria
►A

Erythema toxicum neonatorum is a very common eruption in healthy newborns. A gram stain
reveals sterile pustules containing eosinophils.

35
Which of the following is a sign of EARLY postnatal congenital syphilis?
A. Clutton's joints

126
B. Higoumenaki's sign
C. Hutchinson's teeth
D. Saber shins
E. Wimberger's sign
►E

Wimberger's sign is a sign of early postnatal congenital syphilis (occurs < 2 years old). All other
choices are signs of late postnatal congenital syphilis. Wimberger's sign is a radiographic sign
showing a sawtooth appearance of the proximal tibia. Clutton's joints are nontender and
represent synovitis with effusions of the knees and elbows. Higoumenaki's sign is a unilateral
clavicular enlargement secondary to periostitis. Hutchinson's teeth are centrally notched, wide
spaced, peg shaped upper incisors. Saber shins are anterior bowing of the tibia.

36
Which of the following bullous disorders is due to a target antigen that is a 97 kD protein which
is a member of BPAG-2?
A. Bullous Dermolysis of the Newborn
B. Neonatal Pemphigus
C. Linear IgA Bullous Dermatosis
D. Junctional epidermolysis bullosa (Herlitz)
E. Epidermolysis bullosa simplex
►C

Linear IgA Bullous Dermatosis is due to a target antigen that is a 97 kD protein which is a
member of BPAG-2. Transient bullous dermolysis of the newborn is due to a transient defect in
intracytoplasmic packaging or in the transport of type VII collagen within basal keratinocytes.
Neonatal pemphigus is due to maternal transfer of IgG antibodies reacting to Desmoglein 3
(pemphigus vulgaris). Epidermolysis bullosa simplex is due to keratin 5/14 abnormalities and
junctional EB (Herlitz) is due to defects in Laminin 5.

37
The most common age group for papular-purpuric gloves and socks syndrome is:
A. Newborns
B. Toddlers
C. 6-10 year olds
D. Adolescents
E. Elderly
►D

127
This unique presentation of parvovirus infection typically occurs in adolescents and young
adults.

38
What the most likely diagnosis?
A. Atopic dermatitis
B. Lamellar ichthyosis
C. Rud syndrome
D. Wiskott-Aldrich syndrome
E. Keratosis pilaris
►A

Ichythosis vulgaris and hyperlinear palms are both independent minor criteria for the diagnosis
of atopic dermatitis in both children and infants.

39
In a child with zinc deficiency, yet normal or near normal zinc levels, which test could be a
valuable adjunctive test?
A. Magnesium
B. Niacin
C. Manganese
D. Alkaline phosphatase
E. Iron
►D

The diagnosis of zinc deficiency should be consideredi in at-risk individuals with acral or
periorificial dermatitis. Chronic diaper dermatitis in an infant should lead to the evaluation for
zinc deficiency. A low serum zinc level can usually confirm the diagnosis. If normal or near
normal a low serum alkaline phosphatase, a zinc-dependent enzyme, may be a valuable
adjunctive test.

40
A 6 month-old presents with orange-brown crusted plaques around the mouth and groin. Several
bullae are present on the fingers and toes. Which of the following laboratory values is likely to
be abnormal?
A. Hematocrit
B. Calcium
C. Platelet count
D. ALT
E. Alkaline phosphatase

128
►E

The most likely diagnosis is acrodermatitis enteropathica. Alkaline phosphatase is a zinc


dependant enzyme that is decreased in response to low serum zinc levels.

41
What is the best therapeutic option?
A. Oral cephalexin
B. Topical immune modulator
C. Oral acyclovir
D. Topical mupirocin
E. Oral prednisone
►C

The condition shown in an example of eczema herpeticum, also called Kaposi varicelliform
eruption. It occurs when pre-existing dermatitis becomes superinfected with a viral infection,
most commonly atopic herpes infection of atopic dermatitis. The initial treatment of choice is
antiviral medications.Oral acyclovir is accpetable if the patient is able to swallow and is not
clinically deteriorating, otherwise IV acyclovir is necessary.

42
A young boy presents with a port-wine stain, a Mongolian spot, and a nevus spilus. Which
phakomatosis pigmentovascularis does he have?
A. Type I
B. Type II
C. Type III
D. Type IV
E. Type V
►D

This patient has type IV phakomatosis pigmentovascularis characterized by a port-wine stain,


Mongolian spot, and nevus spilus. All types have a nevus flammeus. In addition, type II
demonstrates a Mongolian spot (may be associated with granular cell tumor), type III a nevus
spilus, and type V cutis marmorata telangiectasia. Type II-IV can also have a nevus anemicus.
Phakomatosis pigmentovascularis is thought to be caused by the "twin spot" phenomenon. Type
I: CM + epidermal nevus Type II: CM + dermal melanocytosis +/- nevus anemicus Type III: CM
+ nevus spilus +/- nevus anemicus Type IV: CM + dermal melanocytosis + nevus spilus +/-
nevus anemicus. (ie types II+III) Type V: CM + cutis marmorata

43

129
The association of Port-wine stains on a limb with soft tissue swelling with or without bony over
growth is:
A. Bannayan-Riley-Ruvalcaba syndrome
B. Goldenhar's syndrome
C. Sturge-Weber syndrome
D. Klippel-Trenaunay syndrome
E. Proteous syndrome
►D

The association of port-wine stain on a limb with soft tissue swelling with or without bony
overgrowth is Klippel-Trenuanay syndrome. Klippel-Trenaunay syndrome is characterized by
the Triad of port-wine malformations in association with deep venous system malformations,
superficial varicosities, and bony and soft tissue hypertrophy. Sturge-Weber syndrome has 2
essential components: Facial port-wine stain and homolateral leptomeningeal angiiomattosis.
The port wine stain most commonly involves the areas innervated by the ophthalmic(V1) and
maxillary (V2) divisions of the trigeminal nerve. Complications of leptomeningeal angiomatosis
are epilepsy, mental retardation, and occasionally, contralateral hemiplegia. Proteus Syndrome is
characterized by vascular malformations including nevus flammeus, hemihypertrophy,
macrodactyly, verrucous epidermal nevus, soft-tissue subcutaneous masses, and cerebriform
overgrowth of the plantar surface. Bannayan Riley Ruvalcaba syndrome may include multiple
cutaneous and visceral venous, capillary, and lympathtic malformations, macroephaly,
pseudopapilledema, systemic lipoangiomatosis, spotted pigmentation of the penis,
hamartomatous intestinal polyps, and rarely trichilemmonmas. (multiple subcutaneous lipomas
as well as acanthosis nigricans).

44
Rhinorrhea, condylomata lata, and mucous patches are all seen with which congenital disorder?
A. Rubella
B. Toxoplasmosis
C. Herpes simplex virus
D. Syphilis
E. Human papillomavirus infection
►D

Signs of congenital syphilis include rhinorrhea, snuffles, rhagades, condylomata lata, and
mucous patches. Condylomata lata or a generalized papulosquamous eruption of secondary
syphilis may be present in diaper area.

45
Psammomatous melanotic schwannomas are associated with:

130
A. Bloom syndrome
B. Carney complex
C. Neurofibromatosis Type 1
D. Neurofibromatosis Type 2
E. Tuberous sclerosis
►B

Psammomatous melanotic schwannomas have been described in the Carney complex, a defect in
the tumor suppressor gene, PRKAR1A.

46
Neonatal acne is associated with species of which organism:
A. Staphylococcus
B. Propionibacterium
C. Candida
D. Malassezia
E. Streptococcus
►D

Acne which develops within the first 30 days of life is termed neonatal acne. Neonatal acne has a
predilection for the face, chest, back and groin appearing as small, discrete papules at 2 to 4
weeks of age, and persisting for up to 8 months. As these lesions are self-resolving, no treatment
is necessary, though 2.5% benzoyl peroxide may hasten resolution. Neonatal acne is quite
common and is postulated to occur as a result of hyperplasia of premature sebaceous glands
coupled with transient increases in circulating androgens. More recent data suggests that
Malassezia species may be implicated at etiologic factors in neonatal acne therefore
ketoconazole is a treatment. These organisms have been cultured from the skin of affected
patients, though their exact role in unclear.

47
An infant presents with the lesion depicted in the photo. Which of the following is possible?
A. Sternal clefting
B. Supraumbilical raphe
C. Dandy-Walker malformation
D. All are possible
E. Congenital cataracts
►D

Extensive facial hemangiomas are a component of the PHACES syndrome.

131
48
What is the diagnosis?
A. Transient neonatal pustular melanosis
B. Acropustulosis of infancy
C. Neonatal cphalic pustulosis
D. Miliaria
E. Erythema toxicum neonatorum
►C

In neonatal cephalic pustulosis (neonatal acne), papules and pustules are present but comedones
are absent. Spontaneous remission occurs.

49
What is the most common tumor associated with this condition?
A. Trichoblastoma
B. Syringocystadenoma papilliferum
C. Sebaceous carcinoma
D. Basal cell carcinoma
E. Trichoadenoma
►A

Recent reviews have identified trichoblastoma as the most common tumor arising within nevus
sebaceous.

50
A 10 year old girl presents with desquamation of the fingertips. Which exam should be ordered?
A. Renal ultrasound
B. ASO titer
C. EEG
D. Eye exam
E. Chest x-ray
►B

Desquamation of the fingertips is commonly associated with infections with group A beta-
hemolytic Strep and Staph aureus, including scarlet fever, perianal Strep, Staph scalded skin
syndrome, and toxic shock syndrome. Thus ASO titer would be indicated in this case. Fingertip
desquamation is also a manifestation of Kawasaki disease.

51
Menkes kinky hair syndrome is associated most commonly with:

132
A. Trichorrhexsis invaginata
B. Trichostasis spinulosa
C. Pili multigemini
D. Pili torti
E. Plica neuropathica
►D

The X-linked recessive Menkes kinky hair syndrome is associated with multiple hair shaft
abnormalities, most characteristically, pili torti.

52
Which of the following syndromes is associated with development of giant cell epulides?
A. Cherubism
B. Gardner syndrome
C. Cowden syndrome
D. Multiple hamartoma syndrome
E. CHILD syndrome
►A

Cherubism is an autosomally-dominant inherited disorder characterized by asymptomatic fibro-


osseous hyperplasia and replacement of normal bone which commences in childhood. Typically
the mandible is affected, leading to progressive enlargement and swollen “cherub-like” cheeks.
Similar involvement of maxillary bones results in periorbital enlargement, resulting in the
tendency of affected individuals to “look up towards the sky.” Frequently the disorder is self-
limited and self-regressive. Mutations in SH3BP2 have been identified as a candidate gene
defect. Affected individuals may also present with single or multiple giant cell epulides
(singular: epulis), which is a benign, bluish-red nodule arising on the gingiva, near deciduous
incisors or bicuspids. It bears histologic similarity to giant cell tumor of the tendon sheath.

53
Which of the following is not a major criterion for Kawasaki‟s disease:
A. Fever >5 days
B. Palmoplantar erythema > desquamation
C. Cardiac aneurysm
D. Strawberry tongue/ red lips
E. Cervical adenopathy
►C

133
Cardiac aneurysm is a serious complication of Kawasaki‟s disease. However, as the
cardiovascular manifestations generally present 1 –5 months after presentation, they are not
criteria for diagnosis.

54
What is the function of the gene which is defective in ataxia-telangiectasia?
A. Gap junction protein
B. Cross-linking of structural proteins in the protein and lipid envelope of the upper
epidermis
C. Pathway of cholesterol biosynthesis
D. DNA repair protein
E. Tumor supressor protein
►D

The defective gene is ataxia-telangiectasia (Louis-Bar syndrome) is the ATM gene, which is
responsible for DNA repair, especially after ionizing radiation. Tumor suppressor genes
mutations are responsible for basal cell nevus syndrome, xeroderma pigmentosum, Muir-Torre
syndrome, dyskeratosis congenita, Gardner syndrome, Peutz-Jeghers syndrome, Cowden
syndrome, and MEN syndromes. Connexins are gap junction proteins that are responsible for
intercellular communication and signaling. Mutations in connexins are responsible for
Vohwinkel syndrome and erythrokeratoderma variabilis. Mutations in the cholesterol
biosynthesis pathways cause CHILD syndrome and Conradi-Hunermann syndrome.
Tranglutaminase 1 (TGM 1) is involved in the normal cross-linking of structural proteins in the
protein and lipid envelope of the upper epidermis. TGM 1 is mutated in lamellar ichthyosis and
congenital ichthyosiform erythroderma.

55
What is the most likely diagnosis?
A. Traction alopecia
B. Tinea capitis
C. Aplasia cutis congenita
D. Nevus sebaceous
E. Alopecia areata
►C

Aplasia cutis congenita is characterized by the absence of a portion of skin, most commonly
presenting as a solitary defect on the scalp, but sometimes it may occur as multiple lesions. The
lesions are non-inflammatory and well demarcated appear as an atrophic, membranous, ulcerated
area with alopecia. The condition may be associated with other physical anomlies.

134
56
Which of the following is the most common complication associated with cutis marmorata
telangectatica congenita?
A. Seizure disorder
B. Atrial septal defect
C. Systemic lupus erythematosus
D. Hypercalcemia
E. Limb hypertrophy or atrophy
►E

Hypertrophy or atrophy of the affected limb is the most likely consequence of cutis marmorata
telangectatica congenita. Orthopedic evaluation should be a part of the patient‟s routine
management.

57
What is the most likely diagnosis?
A. Psoriasis
B. Pityriasis rosea
C. Cutaneous T-cell lymphoma
D. Contact dermatitis
E. Lichen planus
►D

This periumbilical eruption is classic for a contact nickel allergy. The metal snaps on pants are
the cause in this case.

58
A 6-year-old boy presents with osteoma cutis on his face. Which of the following genes is most
likely defective?
A. GNAS
B. NSDHL
C. PTEN
D. SPINK5
E. SLURP1
►A

Osteoma cutis in a child is associated with Albright‟s hereditary osteodystrophy


(pseudohypoparathyroidism), which is caused by a mutation in GNAS1. Albright hereditary
osteodystrophy is characterized by including short stature, obesity, round facies, subcutaneous
ossifications, brachydactyly, and other skeletal anomalies. Some patients have mental

135
retardation. AHO is often associated with pseudohypoparathyoidism, hypocalcemia, and elevated
PTH levels (PHP Ia) (OMIM, 103580). NSDHL is defective in CHILD syndrome, PTEN in
Cowden‟s syndrome, SPINK5 in Netherton‟s syndrome and SLURP1 in Mal de Maleda.

59
Most common location of cutaneous lesions in neonatal lupus erythrematosus is
A. Cheeks
B. Nose
C. Scalp
D. Perioral
E. Periorbital
►E

neonatal lupus erythrematosus(NLE) is a form of subacute cutaneous lupus


erythrematosus(SCLE) that occur in infants whose mothers have anti-Ro autoantibodies. Unlike
SCLE in adults, lesions of NLE have a predilection for the face especially periorbital region.
Other sites include scalp, arms, legs, trunk and groin.

60
An infant who presents with this abnormality will often have the following:
A. The lesion will be on the left-side in a segmental distribution
B. Airway restriction
C. Have multiple liver hemangiomas
D. Most likely be a male infant
E. An anterior fossa malformation
►A

PHACES syndrome represents posterior fossa malformations, hemangiomas, arterial


abnormalities, cardiac abnormalities, eye abnormalities, and sternal defects. Facial hemangiomas
are most often on the left-side of the face. The majority of reported cases are female.

61
What is the best therapeutic option?
A. Oral cephalexin
B. Topical immune modulator
C. Oral acyclovir
D. Topical mupirocin
E. Oral prednisone
►C

136
The condition shown is eczema herpeticum. The most appropriate treatment is oral acyclovir.

62
Subcutaneous fat necrosis of the newborn is usually treated with:
A. Aspirin
B. Topical calcipitriol
C. Excision
D. Observation
E. Retinoids
►D

Subcutaneous fat necrosis of the newborn is rare and self-limited. It is thought to result from
perinatal stress and resolves in 3-6 months. Treatment is usually not necessary unless
hypercalcemia develops.

63
A 10 year-old male presents with new pink-red papules distributed on his bilateral cheeks,
elbows, knees, and buttocks. The lesions are not pruritic. His mother notes he has suffered from
low grade fevers over the past few days, but is otherwise healthy. Which of the following is the
most likely diagnosis?
A. rubella
B. rubeola
C. roseola
D. papular acrodermatitis of childhood
E. dermatitis herpetiformis
►D

Papular acrodermatitis of childhood (Gianotti-Crosti syndrome) typically affects children 6


months to 10 years of age, and is characterized by symmetric papules (and sometimes
papulovesicles) involving the face and buttocks (“four cheeks”) and extensor surfaces of the
upper and lower extremities. Lesions tend to be asymptomatic. Multiple infectious etiologies
have been implicated, including Epstein-Barr virus (most common in the U.S.) and hepatitis B
(most common in Europe). However, the eruption is most often self-limited, resolving in 4 to 8
weeks, and thus further work-up (e.g. testing for hepatitis) is indicated only if additional clinical
symptoms are present.

64
Hemangiomas of infancy are more likely to occur:
A. In males
B. In Asians

137
C. In children who are large for gestational age
D. In multiple gestation
E. In post term infants
►D

Hemangiomas of infancy occur more commonly in girls, in Caucasians, in babies who are
premature and low birth weight. They are more likely to occur in multiple gestation

65
Which presentation of psoriasis is more common in children:
A. Pustular psoriasis
B. Acrodermatitis continua of Hallopeau
C. Keratoderma blennorragica
D. Erythrodermic psoriasis
E. Guttate psoriasis
►E

The majority of cases of guttate psoriasis occur in persons under the age of 30.

66
A child presents with high fever, strawberry tongue, lymphadenopathy, and a polymorphous
exanthem. Approximately how many of these patients will have coronary artery aneurysms if left
untreated?
A. Two percent
B. Five percent
C. Ten percent
D. Twenty-five percent
E. Sixty percent
►D

This child has Kawasaki's disease, or mucocutaneous lymph node syndrome. Other features
include conjunctival injection and extremity erythema and edema. Twenty-five percent of
untreated patients go on to develop coronary artery aneurysms. Treatment is with IVIG and
aspirin.

67
What syndrome can accessory tragi be associated with?
A. Goldenhar syndrome
B. Turner syndrome
C. Neurofibromatosis

138
D. Ichthyosis
E. Birt Hogg Dube
►A

Accessory tragus is the most common congenital defect of the external ear and linked to
maldevelopment of the first branchial arch. It can be associated with several syndromes
including Goldenhar syndrome, in which epibulbar dermoid and vertebral defects are also
common. An accessory tragus usually it appears as a small skin-colored tag or nodule arising
near the tragus; it is composed of normal epidermis with dermal adipose tissue, pilosebaceous
units, eccrine glands, elastic fibers, and cartilage.

68
Which of the following most commonly presents as collodion baby?
A. Ichthyosis vulgaris
B. X-linked ichthyosis
C. Lamellar ichthyosis
D. Bullous congenital ichthyosiform erythroderma
E. Sjogren-Larsson syndrome
►C

The most common presentation of collodion baby is lamellar ichthyosis, followed by congenital
ichthyosiform erythroderma. Patients with ichthyosis vulgaris and x-linked ichthyosis are normal
at birth. Bullous congenital ichthyosiform erythroderma or epidermolytic hyperkeratosis presents
with widespread bullae, erthroderma, and denuded skin. Sjogren-Larsson presents with
generalized ichthyosis and erythroderma in infancy. It is important to know the at-birth
presentations of all the disorders of cornification.

69
Multiple cylindromas are associated with:
A. Myotonic dystrophy
B. Cowden syndrome
C. Carney complex
D. Trichoepitheliomas
E. Pilomatrichomas
►D

The Brooke-Spigler syndrome is defined by the presence of multiple trichoepitheliomas and


cylindromas.

70

139
What is the most likely diagnosis?
A. Psoriasis
B. Langerhans cell histiocytosis
C. Seborrheic dermatitis
D. Contact dermatitis
E. Perianal streptococcal disease
►A

The well-defined erythematous plaque is an example of psorisis in the diaper area. The moist
nature of the diaper environment results in a loss of the classic silvery scale.

71
A 12 y/o female with autoimmune hepatitis on oral prednisone presents with an eruption on the
face for 3 weeks. What is the most likely diagnosis?
A. Tinea facei
B. Acne vulgaris
C. Keratosis pilaris
D. Pityrosporum Folliculitis
E. Diagnosis not listed
►D

Pityrosporum Folliculitis is often associated with oral corticosteroid use such as prednisone,
diabetes and other immune suppressed states. It presents as 1-2mm pruritic monomorphic
papules and pustules on chest, back, upper arms, sometimes face.

72
The causative agent of Roseola is:
A. A ssDNA virus
B. A dsDNA virus
C. A ssRNA virus
D. A dsRNA virus
E. Streptococcus
►B

Roseola is caused by Human Herpesvirus 6, a double stranded DNA virus.

73
A young girl presents with enlarged tongue, exomphalmos, and organomegaly and has history of
Wilm's tumor. What cutaneous finding is most likely on physical exam?
A. Palmoplantar keratoderma

140
B. Lymphatic malformation
C. Acral edema
D. Midline capillary malformation
E. Angiokeratoma
►D

This patient has Beckwith-Wiedemann syndrome, a sporadic condition that is also known as
exomphalos-macroglossia-gigantism syndrome. It also features a midline capillary malformation,
linear earlobe creases, intestinal malrotation, and other tumors such as rhabdomyosarcoma an
hepatoblastoma.

74
A 2 week-old infant is brought to the ER with a rash on her face. She is found to have a 3rd
degree heart block. What is the risk that a second child born to this mother will have the same
diagnosis?
A. 5%
B. 10%
C. 25%
D. 50%
E. 100%
►C

The diagnosis here is neonatal lupus erythematosus. Babies are normal at birth and develop skin
lesions within the few months of life. About half of these babies will have an associated
congenital heart block, usually 3rd degree, which is permanent. Most infants with NLE are girls
and are born to mothers who are Ro/La positive. There is a 25% chance that a second child born
to the same mother will have NLE.

75
Which enanthem is most commonly seen in association with Exanthem subitum?
A. Koplik spots
B. Red strawberry tongue
C. Chapped lips; dry, red mucosa
D. Red macules and streaks on the soft palate
E. Palatal erosions
►D

Exanthem subitum (roseola or Sixth disease) is caused by HHV6, a dsDNA virus. It presents
with high fever for several days followed by an exanthem of erythematous macules and papules
on the trunk that begins as the fever ends. An associated enanthem of red macules/streaks on the

141
soft palate may be seen. Koplik spots are seen in measles; the red strawberry tongue (following
the white strawberry tongue) is seen in Scarlet fever; chapped lips and dry, red mucosa may be
seen in Kawasaki's disease; palatal erosions may be seen in Papular-purpuric gloves and socks
syndrome.

76
An infant presents with red-purple, granulomatous nodules occurring in the diaper area. The
etiology is secondary to local irritation, maceration and Candida albicans. What is the most likely
diagnosis?
A. Granuloma gluteale infantum
B. Langerhans cell histiocytosisc.
C. Seborrheic dermatitis
D. Biotin deficiency
E. Psoriasis
►A

The etiology of granuloma gluteale infantum is multifactorial, resulting from the unique
environment of the diaper area. Treatment consists of topical antifungal agents, barrier creams,
and anti-inflammatory agents as needed.

77
What is the diagnosis?
A. Unilateral laterothoracic exanthem
B. Roseola
C. Rubella
D. Urticaria
E. Contact dermatitis
►A

The child in the photo depicts the “Statue of Liberty” sign of unilateral laterothoracic exanthem.

78
An infant with failure to thrive has multiple xanthomas on skin exam and foamy histiocytes on
bone marrow biopsy. Your diagnosis is:
A. Gauchers disease
B. Tay-Sacs disease
C. Niemann-Pick disease
D. Fabris disease
E. Hunters syndrome
►C

142
Niemann-Pick disease is an autosomal recessive disease caused by mutations in sphingomyelin
phosphodiesterase-1. Patients with Type A Niemann-Pick disease have onset in infancy and die
by 3 may have xanthomas, progressive psychomotor deterioration, hepatosplenomegaly,
blindness, cherry red spots, and deafness. Type B is the late onset non-neurologic form that only
involves the viscera and patients survive into adulthood.

79
A 3 month old baby girl is brought to your office for consultation. You note an infantile
hemangioma on the tip of the nose. Imaging reveals no underlying structural anomalies in the
cerebro-vasculature. You discuss starting propranolol. What side effect do you need to monitor
for?
A. Hypertension
B. Hypercalcemia
C. Hypoglycemia
D. Tachycardia
E. Hypernatremia
►C

In the treatment of infantile hemangiomas with propranolol, the most common serious adverse
effects are bradycardia and hypotension. Other potential adverse effects include bronchospasm
and hypoglycemia. Sustained hypoglycemia in infancy has been associated with longterm
neurologic sequelae. Infants with very large hemangiomas or miliary hemangiomatosis are also
at risk for high-output cardiac compromise.

80
Pachyonychia congenita type 2 is most commonly associated with which of the following:
A. Increased risk of malignancy
B. Poikiloderma
C. Natal teeth
D. Aplastic nails
E. Deafness
►C

Pachyonychia congenita type 2 may be associated with natal teeth and steatocystoma.
Pachyonychia congenita type 1 is associated with benign leukoplakia.

81
POEMS syndrome is associated with which of the following:
A. Premature aging

143
B. Odontogenic cysts
C. Eye abnormalities
D. M protein
E. Saddle nose deformity
►D

The acronym POEMS stands for polyneuropathy, organomegaly, endocrinopathy, M-protein,


and skin changes.

82
A newborn presents with a well-defined, shiny patch with complete alopecia on the vertex of the
scalp along the suture lines. Which of the following is the first step in diagnosis?
A. MRI
B. Calcium
C. Skin biopsy
D. Skull x-ray
E. Fungal culture
►A

The newborn most likely has aplasia cutis congenita. A MRI would be the best means of imaging
to evaluate for any underlying bony abnormality. The infant does not need unnecessary radiation.

83
A boy is noted at birth to have coarse scales over his trunk and extremities. The face, palms,
soles and flexures are spared. What is the least likely association?
A. Corneal opacities
B. Cryptorchidism
C. Ectropion
D. Prolonged maternal labor
E. Neurologic abnormality
►C

X-linked ichthyosis is characterized by small, dark, firmly adherent scales accentuated on the
sides of the neck and trunk. The face, palms, soles, antecubital and popliteal flexures are
generally spared. Associated extracutaneous findings include corneal opacities (50%),
undescended testes (20%), and prolonged maternal labor (usual). Neurological or mental
retardation are rare but documented associations as XLI can be associated with a contiguous
gene syndrome with Kallman syndrome, mental retardation and X linked recessive
chrondrodysplasia punctata.

144
84
A 2 year-old has a high fever for three days, as the fever breaks, a generalized rash appears on
the trunk. The most likely diagnosis is:
A. Measles
B. Mumps
C. Rubella
D. Erythema infectiosum
E. Roseola
►E

Roseola typically appears in toddlers. A high fever followed by a generalized maculopapular


rash is characteristic for this HHV6 infection.

85
What is the best test to confirm a diagnosis?
A. DsDNA antibody
B. Anti Ro antibody
C. Biopsy
D. KOH
E. Gram stain
►D

The picture depicts tinea faceii. A KOH exam should be performed to look for hyphae.

86
This syndrome has severe aplasia cutis congenita, cutis marmorata telangiectatica congenita,
limb defects and atrial septal defect:
A. Adams Oliver syndrome
B. Gianotti Crosti cynsrome
C. Castleman's disease
D. Carrion's disease
E. McCune Albright syndrome
►A

Adams Oliver syndrome is characterized by severe aplasia cutis congenita, cutis marmorata
telangiectatica congenita, limb defects and atrial septal defect. Aplasia cutis congenital is
characterized by an absence of skin and subcutaneous tissue.

87
The lesion depicted is most often associated with:

145
A. Polyostotic fibrous dysplasia
B. Chondrodysplasia punctata
C. Sphenoid wing hypoplasia
D. Cleft palate
E. Osteopoikilosis
►A

The “coast of Maine” café au lait macule is associated with McCune-Albright syndrome and
polyostotic fibrous dysplasia.

88
Which of the following may be associated?
A. Paronychia
B. Cleft palate
C. AVM
D. Seizure disorder
E. Atrial septal defect
►D

Nevus sebaceus can very rarely be associated with multiple anomalies. Schimmelpenning
syndrome can include seizure disorder, mental retardation, coloboma, as well as skeletal, cardiac
and genitourinary abnormalities.

89
Trichothiodystrophy classically includes which of the following findings:
A. Trichostasis
B. Trichoschisis
C. Pili torti
D. Trichorrhexis invaginata
E. Trichorrhexis nodosa
►B

Trichothiodystrophy or PIBIDS is associated with the “tiger-tail” abnormality of trichoschisis.

90
Which of the following is most likely associated?
A. No additional abnormality
B. Deafness
C. Coarctation of the aorta
D. Bifid rib

146
E. Alopecia
►A

The majority of patients with accessory tragii do not have any associated abnormalities. Multiple
anomalies, including deafness and midline defects, have been reported but are rare.

91
What is the most likely diagnosis?
A. Papular acrodermatitis of childhood
B. Mucocutaneous lymph node syndrome
C. German measles
D. Letterer-Siwe disease
E. Exanthem subitum
►A

Gianotti-Crosti syndrome is also known as papular acrodermatitis of childhood. The eruption is


characterized by lichenoid papules in an acral distribution.

92
A healthy, full-term infant develops a pustular, erythematous eruption on her face and trunk on
the third day of life. A smear taken from one of these pustules would show:
A. Gram-positive bacteria
B. Predominantly neutrophils
C. Multi-nucleated giant cells
D. Predominantly eosinophils
E. Hyphae
►D

This baby has developed erythema toxicum neonatorum, a benign, self-limited eruption that
occurs in the majority of healthy, full-term infants. Erythema toxicum usually develops on the
second or third day of life and resolves by day 10. A smear taken from one of the pustules will
demonstrate eosinophils, which is sufficient to make the diagnosis. Viral infections of the skin
may demonstrate multi-nucleated giant cells on Tzanck smear. Transient neonatal pustular
melanosis is present at birth; neonates present with small, superficial pustules that rupture easily.
Some may have ruptured in utero, leaving pigmented macules. The pigmentation may persist for
weeks to months in darkly-complexed infants. A smear of the intracorneal/subcorneal pustule
will show mostly neutrophils, but eosinophils may also be present.

93
What systemic abnormalities are associated with neonatal lupus?

147
A. Thrombocytopenia, transaminitis
B. Thrombocythemia, transaminitis
C. Anemia, hypocomplementemia
D. Thrombocytopenia, hypocomplementemia
E. Transaminitis, hypocomplementemia
►A

Thrombocytopenia and transaminitis are associated with neonatal lupus and may indicate a
worse prognosis.

94
What is the most likely diagnosis?
A. Incontinentia pigmenti
B. Bullous pemphigoid of infancy
C. Herpes zoster
D. Disseminated herpes simplex
E. Epidermolysis bullosa simplex
►A

The vesicular lesions following the lines of Blaschko are typical for incontinentia pigmenti.

95
All 5 subtypes of Phakomatosis Pigmentovascularis have which feature in common:
A. Nevus spilus
B. Epidermal nevi
C. Nevus flammeus
D. Nevus anemicus
E. Dermal Melanocytosis.
►C

Phakomatosis Pigmentovascularis: Patients with a combination of vascular malformations and


melanocytic or epidermal nevi are grouped into 4 subtypes of this disorder. All have nevus
flammeus/capillary malformation (CM). Type I: CM + epidermal nevus Type II: CM + dermal
melanocytosis +/- nevus anemicus Type III: CM + nevus spilus +/- nevus anemicus Type IV:
CM + dermal melanocytosis + nevus spilus +/- nevus anemicus. (ie types II+III) Type V: CM +
cutis marmorata

96
Koplik spots typically appear:
A. Before the exanthem

148
B. At the same time as the exanthem
C. 1 week after the exanthem
D. 6 weeks after the exanthem
E. 8 weeks after the exanthem
►A

The enanthem of measles precedes the morbilliform eruption.

97
Which of the following should be the next step in the management of this patient?
A. Barium swallow
B. spine x-ray
C. Cardiac evaluation
D. Administration of oral antibiotics
E. Pulmonary function studies
►C

This patient must be evaluated thoroughly for PHACES syndrome. PHACES is an acronym for
Posterior fossa malformations (Dandy-Walker malformation is most common), Hemangiomas,
Arterial anomalies, Coarctation of the aorta, Eye abnormalities, and Sternal cleft defects. This
patient should have a complete cardiac evaluation, neuroimaging, and ophthalmologic exam. If
the facial hemangioma involves the beard area, this may indicate laryngeal involvement and
appropriate imaging and evaluation is mandated. Propranolol is the treatment of choice however,
caution must be taken if there is coarctation of the aorta and cardiology is often involved.
Systemic steroids at high doses (5 mg/kg) can be considered if propranolol fails (it usually
doesn't). If caught early, the sequelae of PHACES syndrome can be minimized.

98
A newborn presents with a pustular rash, joint swelling, oral mucosal lesions, and pain with
movement. Over time, cutaneous pustulosis, ranging from discrete crops of pustules to
generalized severe pustulosis and ichthyosiform lesions develop. In addition the newborn
develops sterile multifocal osteomyelitis and periostitis. What is this condition?
A. Muckle-Wells syndrome
B. Deficiency of the interleukin-1-receptor antagonist (DIRA)
C. Familial Mediterranean fever
D. Hyper-IgD syndrome
E. PAPA syndrome
►B

149
This condition is deficiency of the interleukin-1-receptor antagonist (DIRA) and due to a
mutation in the IL1RN gene. Muckle-Wells syndrome is due to a mutation in the CIAS1 gene
and presents with sensorineural deafness, recurrent hives, and amyloidosis. Familial
Mediterranean fever is most often due to a mutation in the MEFV gene. Hyper-IgD syndrome is
a rare periodic fever syndrome characterised by attacks of fever, arthralgia, skin lesions
including cyclical mouth ulcers, and diarrhea due to mutations in the mevalonate kinase gene.
PAPA syndrome presents with pyogenic arthritis, pyoderma gangrenosum and acne and is
associated with mutations in the CD2 binding protein 1 (CD2BP1).

99
Hemangiomas of infancy are more likely to occur:
A. a. In males
B. b. In Asians
C. c. In children who are large for gestational age
D. d. In multiple gestation
E. e. In post term infants
►D

D. is Correct. Hemangiomas of infancy occur more commonly in girls, in Caucasians, in babies


who are premature and low birth weight. They are more likely to occur in multiple gestation

100
A patient presents with an epidermal nevus of the lower abdomen. You take a skin biopsy to
confirm the diagnosis. The pathology report indicates that there were findings of epidermolytic
hyperkeratosis. Which of the following defects is her offspring at risk for based on these
findings?
A. Transglutaminase
B. Keratin 6a/16
C. Keratin 6b/17
D. Keratin 1/10
E. Keratin 2e
►D

Women with epidermal nevi with epidermolytic hyperkeratosis histologically are at risk for
offspring with the “full-blown” epidermolytic hyperkeratosis syndrome, which is due to Keratin
1/10 defects. Transglutaminase defects are seen in lamellar ichthyosis, Keratin 6a/16 and 6b/17
in Pachyonychia congenita types I and II and Keratin 2e in Ichthyosis bullosa of Siemens.

101
Posterior auricular adenopathy is a common feature of which exanthem:

150
A. Measles
B. Mumps
C. Scarlet fever
D. Rubella
E. Erythema infectiosum
►D

Lymphadenopathy with 1-7 days of malaise is commonly described with rubella infection. A
rose-pink macular rash follows the prodrome.

102
Which of followings syndromes characterized by Seborrheic-like or exfoliative dermatitis
A. Leiner's disease
B. Hyperimmunoglobulinemia E syndrome
C. Wiskott-Aldrich syndrome
D. Chronic granulomatous disease
E. X-linked agammaglobulinemia
►A

Immunodeficiency disorders may be associated with a variety of cutaneous abnormalities, and


recognition of these clinical features may allow an early diagnosis of primary immunodeficiency.
Cutaneous abnormalities may include cutaneous infections, atopic- or seborrheic-like dermatitis,
macular erythemas, alopecia, poor wound healing, purpura, petechiae, telangiectasias,
pigmentary dilution, cutaneous granulomas, extensive warts, angioedema, and lupus-like
changes. Leiner's disease or erythroderma desquamativum is a complication of seborrheic
dermatitis in infants (dermatitis seborrhoides infantum) There is usually a sudden confluence of
lesions, leading to a universal scaling redness of the skin (erythroderma). The young patients are
severely ill with anemia, diarrhea, and vomiting.The disease is both a familial and a nonfamilial
form. The former is noted for having a functional deficiency of C5 complement, resulting in
defective opsonization. These patients respond to antibiotics and infusions of fresh frozen plasma
or whole blood. The other choices in the questions can all be associated with atopic-like
dermatitis.

103
This condition may be associated with which of the following:
A. Hypercalcemia
B. Hyperlipidemia
C. Vitiligo
D. Epidermolysis bullosa dystrophica
E. Perinatal trauma

151
►D

Aplasia cutis congenita may be associated with dystrophic epidermolysis bullosa (Bart
syndrome.)

104
A 24 month-old infant presents with yellowish-brown, crusted papules with petechiae in a
seborrheic distribution. A biopsy is done to confirm a diagnosis. Which histologic picture is most
likely?
A. CD1-, S100- cells with reniform nuclei
B. Foamy histiocytes with Touton giant cells
C. CD1+, S100+ cells with reniform nuclei
D. Mixed cellular infiltrate in a “ball and claw” pattern
E. Superficial perivascular infiltrate with mild spongiosis and neutrophil containing
scale crust
►C

Langerhans cells are CD1 and S100 positive. The nuclei are described as kidney shaped, or
reniform.

105
A 4 month-old with diffuse blisters and erosions has a skin biopsy diagnostic of generalized
mastocytosis. Which topical dressing should be avoided in this patient?
A. Mupirocin ointment
B. Polymyxin B ointment
C. Petrolatum
D. clobetasol ointment
E. Silver sulfadiazine
►B

Mastocytosis comprises a group of diseases characterized by increased number of mast cells in


the skin and other organs. Seventy-five percent of cases occur before the age of 2. Patients with
mastocytosis should avoid potential mast degranulators including aspirin, codeine, opiates,
procaine, spicy foods, cheese, alcohol, polymyxin B.

106
What is the most common tumor associated with this condition?
A. Trichoblastoma
B. Squamous Cell carcinoma
C. Sebaceous carcinoma

152
D. Basal cell carcinoma
E. Trichoadenoma
►A

Recent reviews have identified trichoblastoma as the most common tumor arising within nevus
sebaceous, even more frequent than syringocystadenoma papilliferum.

107
Which of the following is a potential complication of subcutaneous fat necrosis of the newborn?
A. Cholestrol clefts in fat cells
B. Hypercalcemia
C. Acute renal failure
D. Hepatitis
E. Elevated uric acid levels
►B

Subcutaneous fat necrosis of the newborn is a rare condition characterized by firm, reddish or
purple nodules which appear on the arms, thighs, buttocks, back and cheeks. This condition is
believed to result from cold injury. Crystallization occurs within the lipocytes, and this can be
seen on histopathology with an associated granulomatous reaction. This is thought to occur due
to the higher melting point of neonatal fat, which contains more saturated fatty acids. Onset of
this condition occurs within the first two weeks of life and usually resolved over a period of
weeks to months. Occasionally, lesions heal with atrophy. Hypercalcemia is an infrequent
complication of subcutaneous fat necrosis of the newborn. Associated symptoms can include
irritability, weight loss, vomiting and failure to thrive. Repeated serum calcium tests are advised
until one month after all the cutaneous lesions have resolved.

108
The disorder caused by a defect in intestinal zinc-specific transporter SLC39A4 is most likely to
present:
A. At birth
B. Upon weaning from breast milk
C. Upon weaning from formula
D. In childhood
E. In adulthood
►B

In acrodermatitis enteropathica, there is decreased absorption of zinc from the infant”s


gastrointestinal tract. The zinc in breast milk has greater availability than nonmaternal sources
thus protecting the child from disease expression until weaning.

153
109
What deficiency is responsible for this condition?
A. Thiamine
B. Niacin
C. Zinc
D. Vitamin C
E. Vitamin A
►C

Acrodermatitis enteropathica is a rare inherited disorder transmitted in an autosomal recessive


fashion. The disorder is caused by an inability to absorb zinc. the clinical syndrome is
characterized by acral dermatitis, alopecia, and diarrhea. Zinc deficiency from other causes
(including chronic wasting, poor oral intake, lack of supplementation in total parenteral nutrition)
can cause similar clinical changes.

110
The most likely etiology of Jacquet”s diaper dermatitis is:
A. Candida
B. Trichophyton rubrum
C. Group A beta-hemolytic streptococcus
D. Herpes simplex virus, Type 2
E. Multifactorial
►E

Jacquet”s diaper dermatitis is a multifactorial process. Yeast, irritants and moisture all contribute
to the occurrence of this eruption.

111
The Carney complex is associated with a defect in:
A. PRKAR1A
B. LYST
C. PTEN
D. MASH2
E. MLH1
►A

A PRKAR1A gene defect is associated with the Carney complex.

112

154
Which of the following is not true about Gianotti-Crosti?
A. It is associated with viral infections like enterocirus, EBV, and CMV
B. It typically affects children between the age of 3 months and 15 years
C. It is characterized by monomorphic symmetric flat topped papules in acral areas
D. It may have associated fevers, lymphadenopathy, and diarrhea
E. Corticosteroids should be given to alleviate the pruritus of the lesions
►E

Gianotti-Crosti, or papular acrodermatitis of childhood, is typically nonpruritic and


corticosteroids should be avoided as they may have adverse effect. All the other statements are
true. The condition typically resolves after 2-3 weeks.

113
What disorder is associated with a defect in LMX1B?
A. Nail patella syndrome
B. Steatocystoma
C. Monilithrix
D. Lhermitte-Duclos syndrome
E. Chediak-Higashi syndrome
►A

A LMX1B gene defect is associated with nail patella syndrome.

114
A full term neonate is noted to have small pustules with no underlying erythema present at
delivery. The pustules are easily removed with clearing of the vernix and a collarette appears. A
gram stain is done showing predominately neutrophils without bacteria. What is the most likely
diagnosis?
A. Miliaria
B. Erythema toxicum neonatorum
C. Transient neonatal pustular melanosis
D. Congenital candidiasis
E. Urticaria pigmentosa
►C

Transient neonatal pustular melanosis typically begins with sterile pustules that leave a
characteristic collarette when ruptured. The lesions heal with hyperpigmented macules.

115
Which finding is associated with Pachydermoperiostosis:

155
A. Cutis verticis gyrata
B. Palmoplantar hyperkeratosis
C. Mucosal keratoses
D. Osteopathia striata
E. Cutis marmorata
►A

The scalp change seen in pachydermoperiostosis is cutis verticis gyrata.

116
What is the best treatment option?
A. Oral cephalexin
B. Oral acyclovir
C. Observation
D. Topical tretinoin
E. Topical ketoconazole
►D

The lesions shown in the photo are papules of infantile acne which occurs between the ages of
6mo to three years. The infants get comedones, papules and pustules. Topical tretinoin would be
the best treatment option.

117
This patient had significantly elevated serum CPK. The likely diagnosis is:
A. Dermatomyositis
B. Lupus erythematosus
C. Psoriasis
D. Atopic dermatitis
E. Lichen planus
►A

The answer is dermatomyositis with the heliotrope color and distribution of erythema or
violaceous color. The skin over the metacarpal and proximal interphalangeal joints can become
inflamed and erythematous forming Gottron‟s papules.

118
Most common malignancy associated with multiple lesions similar to the attached image is:
A. Acute myelogenous leukemia
B. Chronic myelogenous leukemia
C. Acute lymphocytic leukemia

156
D. Chronic lymphocytic leukemia
E. Melanoma
►B

The image shown is juvenile xanthogranuloma. The most commonly associated malignancy is
chronic myelogenous leukemia. Other associations include Neurofibromatosis type 1 and
aquagenic pruritus. Reference: Zvulunov A, Barak Y, Metzker A. Juvenile xanthogranuloma,
neurofibromatosis, and juvenile chronic myelogenous leukemia. World statistical analysis. Arch
Dermatol 1995;131:904-908.

119
A neonate presents with a large segmental hemangioma of the V1 distribution. What are features
may be associated with this finding?
A. Tram-track calcifications
B. Anterior fossa defect
C. Posterior fossa defect
D. M-paraproteinemia
E. Muscular dystrophy
►C

PHACES syndrome consists of posterior fossa defect, hemangiomas (often segmental and large),
arterial defects, cardiac defects, eye abnormaliteis, and sternal clefting. Posterior fossa defects
include the Dandy-Walker malformtion.

120
Schimmelpenning-Feuerstein-Mims syndrome may be associated with which of the following:
A. Osteopokilosis
B. Polyostotic fibrous dysplasia
C. Osteopathia striata
D. Chondrodysplasia punctata
E. Hypophosphatemic rickets
►E

Linear nevus sebaceous syndrome, also known as Schimmelpenning-Feuerstein-Mims syndrome,


may be associated with hypophospatemic, vitamin D resistant rickets.

121
GLUT-1 stains placental tissue. What other tissue shows positive GLUT-1 staining?
A. capillary vascular malformation
B. salmon patch

157
C. glomus tumor
D. infantile hemangioma
E. angel's kiss
►D

The correct answer is infantile hemangioma, which reliably displays GLUT-1 positivity. GLUT-
1 stains infantile hemangioma with high sensitivity and specificity. Glomus tumor, capillary
vascular malformation (also called port-wine stain), salmon patch, and angel's kiss are other
vascular lesions that do not stain with GLUT-1.

122
Hemangiomas of infancy are more likely to be:
A. GLUT-1 negative
B. Merosin positive but GLUT-1 negative
C. Lewis Y antigen positive, merosin negative
D. Glut-1 positive, merosin positive, Lewis Y antigen negative
E. Glut-1 positive, merosin positive, Lewis Y antigen positive
►E

Hemangiomas show GLUT-1 positivity and stain positively with placental markers.

123
A 3 month-old girl with multiple hemangiomas along her right jaw is at increased risk for:
A. Bleeding complications
B. Underlying bone abnormalities
C. Subglottic hemangioma
D. Oral obstruction
E. Hearing defecits
►C

Hemangiomas are benign vascular tumors which have proliferating phase and then a spontaneous
involution phase. Depending upon the location, some hemangiomas may have more long term
sequelae such as scarring or structural malformation, and rarely, consumptive coagulopathy.
Infants with hemangiomas of the “beard” distribution should be evaluated for subglottic
hemangiomas which may lead to airway obstruction.

124
A deficiency of the surface glycoprotein sialophorin is seen in which immunedeficient disease?
A. Wiskott-Aldrich syndrome
B. Chronic granulomatous disease

158
C. Job syndrome
D. Severe combined immunodeficiency syndrome
E. Leiner”s disease
►A

Wiskott-Aldrich syndrome is an X-linked recessive disorder caused by mutations in WAS gene.


Patients with Wiskott-Aldrich may have unstable sialoglycoprotein CD 43 on the surface of
lymphocytes. Patients with Wiskott-Aldrich have atopic dermatitis with increased risks for
secondary infection, thrombocytopenia, and recurrent bacterial infections. They are also at an
increased risk for lymphoreticular malignancy (20%).

125
Which treatment choice would be contraindicated in a one-year old child who presents with
monomorphous, nonpruritic flat-topped papules on the face, buttocks, extremities, palms and
soles?
A. Advil
B. Acetaminophen
C. Hydration
D. Corticosteroids
E. Observation
►D

Gianotti-Crosti or papular acrodermatitis of childhood is associated with a variety of viral


infections. Patients have a typical cutaneous manifestation, low-grade fever, mild
lymphadenopathy and diarrhea. Corticosteroids should be avoided as they may have an adverse
effect.

126
An eight year-old boy presents with pink, flushed cheeks and a low-grade fever. Eruptions then
appeared. What is the most likely etiology?
A. Paramyxovirus
B. Togavirus
C. Group A streptococcus
D. HHV6
E. Parvovirus
►E

Fifths Disease or erythema infectiosum, is caused by parvovirus B19, a single-stranded DNA


virus. There are a few clinical presentations including, diffuse lacy rash on the trunk that spreads

159
gradually toward the distal extremities, papular-pruritic "gloves-and-socks" syndrome and
aplastic crisis which does not have concomitant rash.

127
A patient presents with multiple juvenile xanthogranulomas, axillary freckling, multiple café-au-
lait macules, three neurofibromas and a family history of NF-1. What other condition is this
patient at increased risk for?
A. AML
B. JMML
C. CLL
D. Medulloblastoma
E. Pancreatic carcinoma
►B

A patient with Neurofibromatosis type I and JXG‟s present is at increased risk for JMML.
Medulloblastomas are increased in patients with Basal Cell Nevus syndrome. Pancreatic
carcinoma occurs at an increased risk in patients with mutations in CDKN2A in the familial
melanoma syndrome.

128
What is the most likely diagnosis:
A. Epidermolysis bullosa simplex
B. Epidermolysis bullosa dystrophica
C. Pemphigus vulgaris
D. Linear bullous IgA disease
E. Facticial dermatosis
►A

The bullous lesions represented are nonscarring and are associated with obvious nail dystrophy.
Epidermolysis bullosa simplex, Dowling-Meara type, is the best option.

160
Chapter -10-
Cutaneous Manifestations of Systemic Disease
1
Hyperkeratotic follicular nasal papules have been described as a paraneoplastic sign in the
setting of which neoplasm?
A. Multiple myeloma
B. Castleman's tumor
C. AML
D. Adenocarcinoma of the lung
E. Renal cell carcinoma
►A

Hyperkeratotic follicular nasal papules have been described as a paraneoplastic phenomenon in


the setting of multiple myeloma. AML is associated with Sweet's syndrome and Castleman's
tumor is associated with paraneoplastic pemphigus.

2
Which of the following may be associated with Graves‟ disease?
A. Dermatitis herpetiformis
B. Geographic tongue
C. Hypohidrosis
D. Madarosis
E. Thick, pale lips
►A

Graves‟ disease is a thyrotoxic condition that results from the production of thyroid-stimulating
immunoglobulins (TSI) by stimulated B lymphocytes. The TSI bind to the thyroid-stimulating
hormone (TSH) receptor and mimic TSH thereby stimulating thyroid growth and thyroid
hormone overproduction. Signs and symptoms of Graves‟ disease include goiter, tachycardia,
exophthalmos, tremor, sweating, palpitations, smooth moist skin, diarrhea, sleeplessness,
irritability, and weight loss. Autoimmune cutaneous disease may also be associated with Graves‟
disease including vitiligo, dermatitis herpetiformis, herpes gestationis, and pemphigus vulgaris.
Cutaneous manifestations of hypothyroidism include xerosis, hyperhidrosis, yellowish hue,
myxedema, and purpura. The hair may be dry, brittle and coarse; alopecia may be diffuse and/or
involve the lateral eyebrow (madarosis).

3
A patient with gluten-sensitive enteropathy presents with vesicles on the extensor surfaces of the
extremities. What findings are most likely on a perilesional biopsy?

161
A. Granular IgA at the dermoepidermal junction on direct immunofluorescence
B. Linear C3 and IgG at the dermoepidermal junction on direct
immunofluorescence
C. Linear IgA surrounding vessels on direct immunofluorescence
D. Pautrier”s micro-abscesses in the epidermis on H & E
E. Leukocytoclastic vasculitis
►A

This patient has dermatitis herpetiformis (DH). DH is characterized by itchy papulovesicles on


extensor surfaces of the extremities. Neutrophilic infiltrates at the dermal papillae with vesicle
formation are found on histopathologic examination. On immunofluorescence, granular IgA
(directed against transglutaminase) deposits are found in perilesional skin. Over 90% of patients
have gluten-sensitive enteropathy of varying severity. Dapsone is almost universally therapeutic
for the skin disease, but not the enteropathy. A gluten free diet, although difficult to maintain,
treats the enteropathy. The majority of DH patients have the HLA class II DQ2 genotype. On
indirect immunofluorescence, one may find antigliadin, antiendomyseal or antireticulin
antibodies. Autoimmune thyroid disease (especially Hashimoto”s thyroiditis), enteropathy-
associated T-cell non-Hodgkin”s lymphoma and insulin-dependent diabetes are the most
common autoimmune associations. Pautrier”s micro-abscesses are found in mycosis fungoides.
Leukocytoclastic vasculitis is not a feature of DH.

4
A 64-year-old woman presents with lichenified keratotic plaques on the bilateral lower
extremities. Biopsy reveals suppurative inflammation and collagen extending from the dermis
through the epidermis. Which is the most likely associated systemic disease:
A. Congestive Heart Failure
B. Hepatitis C Infection
C. Hypothyroidism
D. Chronic kidney disease
E. Castleman‟s disease
►D

Chronic kidney disease is associated with perforating disorders. These disorder are characterized
by pruritus and a biopsy revealing collagen and inflammatory cells broaching the epidermis. The
other disorders are not closely associated with perforating diseases.

5
A patient presents with mild mental retardation, infertility, joint contractures, short stature,
ichthyosis, and sparse hair with trichoschisis. All of the following are true regarding this patient's
condition EXCEPT:

162
A. This syndrome is inherited in an autosomal recessive manner
B. If photosensitivity is a feature, gonad size is likely normal
C. Patients may have associated cataracts
D. The syndrome is caused by impaired nucleotide excision repair
E. Perifoveal glistening white dots are a feature
►E

The patient described has IBIDS syndrome (Ichthyosis, Brittle hair, Intellectual impairment,
Decreased fertility, and Short stature), or Tay's syndrome. This syndrome is autosomal recessive,
caused by mutations in the ERCC2/XPD or ERCC3/XPB genes, resulting in impaired nucleotide
excision repair. Cataracts may be a feature. If photosensitivity is a feature (PIBIDS), gonal size is
likely normal. Perifoveal glistening white dots are a feature of Sjorgen-Larsson Syndrome, not
IBIDS.

6
A patient with congenital hypertrophy of retinal epithelium is most likely to have:
A. An autosomal dominant mutation in the MSH2 gene
B. Pheochromocytoma
C. Adenomatous polyposis
D. Tram-track calcifications on head radiograph
E. Peg-shaped teeth
►C

Congenital hyperpigmentation of the retinal pigment (CHRPE) is an early feature of Gardner


syndrome (GS). It is found in approximately 60% of patients with GS. GS is an autosomal
dominant disorder characterized by precancerous intestinal polyposis and subsequent
adenocarcinoma of the gastrointestinal tract. Cutaneous manifestations include epidermoid cysts,
osteomas, desmoids and fibrous tumors. A mutation in the adenomatous polyposis coli (APC)
gene, a tumor suppressor gene, is responsible for the disease. Most patients develop colon
carcinoma by the 2nd or 3rd decade. Therefore, prophylactic colectomy is warranted. Mutations
in the MSH2 gene are found in Muir-Torre syndrome. Pheochromocytomas are found in multiple
endocrine neoplasia (MEN) syndromes IIa and IIb. Tram track calcifications are found in Sturge-
Weber syndrome. Peg-shaped teeth are found in multiple syndromes including ectodermal
dysplasia.

7
A 14-year-old female presents with gastrointestinal pain and seizures. Laboratory studies reveal
hyponatremia. Which of the following medications was most likely to induce her systemic
symptoms:
A. Phenytoin

163
B. Diphenhydramine
C. Griseofulvin
D. Morphine
E. Acetaminophen
►C

Acute intermittent porphyria (AIP) is associated with colicky abdominal pain, neurologic
symptoms, and SIADH. This disorder is caused by deficiency of porphobilinogen deaminase.
AIP may be exacerbated by drugs including barbiturates, sulfonamides, and griseofulvin, among
others.

8
Which of the following is not a feature of Cronkhite-Canada syndrome?
A. Diarrhea
B. Alopecia
C. Lung carcinoma
D. Dystrophic nails
E. Hyperpigmented macules
►C

Cronkhite-Canada syndrome is a rare, non-familial disease characterized by patchy alopecia, nail


changes or loss, lentigines, inflammatory polyps, abdominal pain, and a protein losing
enteropathy.

9
Which of the following is a paraneoplastic disease most often associated with lung carcinoma?
A. Hypertrichosis lanuginosa acquisita
B. Dermatomyositis
C. Acanthosis nigricans
D. Paraneoplastic pemphigus
E. Erythroderma
►A

Hypertrichosis lanuginosa acquisita is the abrupt onset of downy, soft, non-pigmented hair of the
face, trunk, and extremities. It may have an associated glossitis. It is associated with underlying
lung carcinoma and may resolve with treatment of the underlying malignancy.

10
Which of the following is true regarding piebaldism?
A. It is caused by a mutation in the GJB2 gene

164
B. It is caused by defective metabolism of phytanic acid
C. It is caused by a defect in a protein subunit of a kinase that activates NFkappaB
D. It is caused by a deficiency of fatty aldehyde dehydrogenase
E. It is caused by a mutation in the proto-oncogene c-KIT
►E

Piebaldism is caused by an autosomal dominant mutation of the proto-oncogene c-KIT which


encodes tyrosine a tyrosine kinase receptor on melanocytes, preventing activation by steel factor.

11
A middle-aged gentleman who avoided healthcare dies suddenly from gastrointestinal
hemorrhage. Post-mortem examination reveals multiple soft blue compressible tumors on the
trunk, arms, and tongue. Blue rubber bleb nevus syndrome is caused by a mutation in what
gene?
A. VMCM1
B. ENG
C. VEGF
D. PTEN
E. RET
►A

Blue rubber bleb nevus syndrome is a rare sporadic or autosomal dominant disorder with soft
compressible blue tumors on the trunk and arms. Nocturnal pain is characteristic.
Gastrointestinal hemangiomas can cause hemorrhage. Mutations in the VMCM1=TEK=Tie-2
gene are reported. It is also knkow n as VMCM syndrome.

12
Which of the following statements regarding porphyrias is TRUE?
A. Elevated uroporphyrins are found in the red blood cells of hepatoerythropoietic
porphyria
B. Delta aminolevulenic acid is the only oxidized porphyrin
C. Acute intermittent porphyria is the most common form of porphyria
D. Griseofulvin is safe for those with variegate porphyria
E. Plasma fluoresces at 410 nm in patients with variegate porphyria
►B

Acute intermittent porphyria (AIP) is the second most common porphyria and is caused by a
deficiency in porphobilinogen (PBG) deaminase, which is located in the cytosol. Patients suffer
from colicky pain, paralysis and psychiatric disorders. There are no specific skin manifestations.
PBG and aminolevulenic acid (ALA) are elevated in the urine. Attacks are precipitated by

165
medications such as barbiturates, estrogen, griseofulvin, and sulfonamides as well as starvation,
fever and infection. Treatment includes glucose loading and hematin infusion.
Congenital erythropoietic porphyria (CEP) or Günter‟s disease is caused by a defect in
uroporphyrinogen III synthase, which is found in the cytosol. Patients are extremely
photosensitive and erythema, blistering and scarring result. Patients present with red urine early
in life along with hypertrichosis and red-stained teeth that fluoresce. Uroporphyrins (URO) are
much high than coproporphyrins (COPRO) in the urine. URO is found in the red blood cells
(rbcs) and COPRO is found in the stool. The rbcs display stable fluorescence.
Porphyria cutanea tarda (PCT) is the most common porphyria and is caused by a deficiency
(usually sporadic) in uroporphyrinogen decarboxylase, which is found in the cytosol. Patients
present with photosensitivity and blistering of sun-exposed areas, especially the dorsal hands.
Hypertrichosis and sclerodermoid changes may occur as well. Liver disease (hepatitis C or
alcoholic cirrhosis) is often present and hemochromatosis may be associated. Urine may
fluoresce pink or coral-red with Wood‟s lamp. URO>COPRO in the urine and low levels of
COPRO are found in the stool. Treatments include phlebotomy, antimalarials, and therapy for
liver disease if appropriate.
Hereditary coproporphyria (HCP) is caused by a deficiency in coproporphyrinogen oxidase,
which is found in the mitochondria. One-third of patients are photosensitive, and patients suffer
gastrointestinal and neurological symptoms similar to AIP. Urine COPRO is elevated only with
attacks, and COPRO is present in the stool.
Variegate porphyria is the result of decreased activity of protoporphyrinogen oxidase, which is
present in the mitochondria. It combines the skin lesions of PCT with the systemic
manifestations of AIP. Urine COPRO:URO is 1:1 or COPRO> URO to distinguish it from PCT,
and PROTO is found in the stool. The plasma fluoresces at 626nm. Precipitators and treatments
are similar to AIP.
Erythropoietic protoporphyria (EPP) is caused by ferrochetalase deficiency, which is present
in the mitochondria. Patients experience immediate burning of the skin with sun exposure.
Protoporphyrin IX, the only oxidized porphyrin in the heme pathway and absorbs in the Soret
band (400-410nm). Patients have erythematous plaques in a photo-distribution. Urine porphyrins
are normal. PROTO is found in the rbcs and the stool. Excessive porphyrins deposited in the
liver lead to gallstones and cirrhosis. Beta carotene may helpful.
Hepatoerythropoietic porphyria (HEP) is essentially a homozygous form of PCT, with
deficiency in uroporphyrinogen decarboxylase. It is clinically similar to CEP with red urine and
hypertrichosis, vesicles and scarring of sun-exposed skin. URO is present in the urine and
COPRO in the stool. PROTO is present in rbcs which distinguishes it from CEP, which was
URO in rbcs.

13
The expected histology of a biopsy take from the lesion shown in the image would
A. Increased mucin

166
B. Schumann bodies
C. Granulomatous infiltrate localized to the papillary dermis
D. Elastic fiber degeneration
E. Collagen degeneration
►E

The histologic findings of necrobiosis lipodica diabeticorum are: dermal granulomatous


inflammation in a horizontal pattern, collagen degeneration, and normal or atrophic epidermis.

14
Squamous cell carcinoma is seen in which syndrome?
A. Gorlin syndrome
B. Rombo syndrome
C. Nicolau-Balus syndrome
D. Rasmussen syndrome
E. Bazex syndrome (Acrokeratosis paraneoplastica)
►E

Basex syndrome, also Acrokeratosis paraneoplastica, presents with symmetric erythematous,


nearly violaceous, psoriasiform dermatoses of the hands, feet, ears and nose. The syndrome is
nearly associated with an underlying malignancy, usually squamous cell carcinoma of the upper
aerodigestive tract.

15
All of the following statements are true regarding this condition EXCEPT:
A. Diabetes or glucose intolerance is found in 20% of these patients
B. This condition may be associated with cutaneous anesthesia, hypohidrosis, and
partial alopecia
C. There is no impact of tight glucose control on the likelihood of developing this
condition
D. 0.3-3% of diabetics have this skin condition
E. This condition is associated with increased dermal mucin
►E

Necrobiosis Lipoidica Diabeticorum (NLD) is found in 0.3-3% of diabetics. Approximately 20%


of NLD patients have diabetes or glucose intolerance. It presents with single or multiple red-
brown papules which progress to sharply demarcated yellow-brown atrophic, telangiectatic
plaques with violaceous, irregular borders; common sites include the shins. Cutaneous
anesthesia, hypohidrosis, and partial alopecia can be found. Pathology shows palisading

167
granulomas containing degenerating collagen (necrobiosis); with NO increase in dermal mucin.
There is no impact of tight glucose control on the likelihood of developing NLD.

16
Which of the following is NOT associated with Hepatitis C disease?
A. Mixed cryoglobulinemia
B. PCT
C. Single-stranded RNA viridae
D. Leukocytoclastic vasculitis
E. Single-stranded DNA viridae
►E

Hepatitis C virus (HCV) is a single-stranded RNA virus that is a member of the flaviviridae
family. Approximately 20-30% of patients develop symptoms with acute infection and 70% will
progress to chronic disease. Porphyria cutanea tarda or PCT is associated with HCV in a
substantial percentage of patients; in one study, antibodies to HCV were found in 82% of PCT
patients. Up to 80% of mixed cryoglobulinemia (MC) cases are associated with HCV. The
incidence of lichen planus in HCV patients varies from region to region (0.1-35%). The
histopathology of MC lesions is leukocytoclastic vasculitis. Another relatively common
association is polyarteritis nodosa (PAN) which is also related to hepatitis B infection. Finally,
generalized pruritis is a common complaint of HCV patients.

17
All of the following are seen more commonly in Crohn's disease than in ulcerative colitis
EXCEPT:
A. Oral cobblestoning
B. Polyarteritis nodosa
C. Pyostomatitis vegetans
D. Perineal fistulas
E. Perineal fissures
►C

Pyoderma vegetans consists of vegetating plaques and vesicopustules of intertriginous areas than
heal with hyperpigmentation. When the process involves mucosal surfaces it is called
pyostomatitis vegetans. These processes are both associated with ulcerative colitis, not Crohn's
disease.

18
Patients with plexiform neuroma and NF I who also have JXG are at increased risk for
developing:

168
A. Juvenile chronic myelogenous leukemia
B. Non-Hodgkin's Lymphoma
C. Esophageal cancer
D. Breast cancer
E. Medullary carcinoma of the thyroid gland
►A

Patient with NF 1 who have multiple juvenile xanthogranulomas are at 20 times greater risk for
developing juvenile chronic myelogenous leukemia.

19
The peak sensitivity to prophyrins occurs at which wavelengths?
A. 220-290nm
B. 290-320nm
C. 320-400nm
D. 400-410nm
E. 410-450nm
►D

The Soret band (400-410 nm) is the portion of ultraviolet wavelengths at which most porphyrins
are most sensitive.

20
A patient presents with plane/palmar xanthomas. The most likely genetic disorder would be:
A. familial lipoprotein lipase deficiency
B. Familial hypertriglyceridemia
C. Familial hypercholesterolemia
D. Familial dysbetalipoproteinemia
E. Cerebrotendinous xantomatosis
►D

Patients with Type III hyperlipidemia have both elevated triglyceride levels and cholesterol
levels in the plasma. A genetic basis for the primary disorder, familial dysbetalipoproteinemia,
has been well established. These patients present as adults with premature atherosclerosis and
xanthomas, particularly plane (palmar) xanthomas.

21
A 70-year old female develops erythema with fine adherent scale on acral skin that progresses to
keratoderma and eventually a more generalized psoriasiform dermatitis. What is the most likely
underlying malignancy?

169
A. Squamous cell carcinoma of the larynx
B. Adenocarcinoma of the colon
C. Adenocarcinoma of the breast
D. Squamous cell carcinoma of the vagina
E. Medullary thyroid carcinoma
►A

Acrokeratosis Paraneoplastica (aka Bazex) is always associated with malignancy. Most


commonly, the associated malignancy is a squamous cell carcinoma of the upper aerodigestive
tract. The skin disease usually follows the course of the malignancy.

22
Patients with Werner‟s syndrome typically experience which of the following types of cardiac
disease?
A. Hypertrophic cardiomyopathy
B. Aortic aneurysms
C. Premature atherosclerosis
D. Cardiomegaly
E. Mitral valve prolapse
►C

Werner‟s syndrome or Adult Progeria is caused by autosomal recessive mutations in WRN


(Recql2) gene, which encodes DNA helicase. This defect leads to defects in DNA repair and
replication. Patients prematurely age and essentially experience many diseases of aging early in
childhood and teen years. They exhibit tight atrophic skin, relatively large heads for body size,
leg ulcers and cannities. Patients experience early, accelerated atherosclerosis leading to death by
myocardial infarction. In addition, type II diabetes, cataracts, osteoarthritis, osteoporosis and
hypogonadism are features.

23
A patient with end stage renal disease undergoes an MRI examination with contrast. He
subsequently develops woody indurated plaques on the extremities. Fatalities seen in
nephrogenic systemic fibrosis are due to what underlying process?
A. Fibrosis of respiratory muscles
B. Fibrosis of cardiac smooth muscle
C. Fibrosis of the gastrointestinal tract
D. Fibrosis of cerebral arteries
E. Fibrosis of the liver
►A

170
Nephrogenic systemic fibrosis is a chronic fibrosing disorder seen most commonly in the setting
of renal failure. There is a reported association also with gadolinium as contrast in an MRI.
Patients present with woody indurated plaques or nodules primarily of the extremities, with
occasional involvement of the trunk. Fulminant and fatal disease is rare, occurring in about 5%
of cases. Death is due to impaired ventilation secondary to fibrosis of respiratory muscles.

24
Which of the following skin findings is most closely linked to hepatocellular carcinoma as a
paraneoplastic syndrome?
A. Pityriasis lichenoides
B. Pityriasis alba
C. Pityriasis amiantacea
D. Pityriasis rotunda
E. Pityriasis rosea
►D

Pityriasis rotunda is an dermatosis that features characteristic discrete, circular, scaly, brown
patches on the trunk and extremities. Pityriasis rotunda may be associated with systemic diseases
in certain racially predisposed groups (blacks), and has been linked to hepatocellular carcinoma.

25
A 20-year-old male develops an eruption of 100's of red-brown yellowish papules with
involvement of the mucous membranes. He has no lymphadenopathy. The most likely diagnosis
is:
A. Montgomery's syndrome
B. Benign cephalic histiocytosis
C. Rosai-Dorfman disease
D. Necrobiotic xanthogranuloma
E. Multicentric reticulohistiocytosis
►A

This patient has Montgomery's syndrome, or xanthoma disseminatum, which is characterized as


a benign normolipidemic but disfiguring condition that demonstrates an eruption of 100's of red-
brown yellowish papules and plaques that may involve the mucous membranes. These patients
should be evaluated for diabetes insipidus. Benign cephalic histiocytosis shows brownish-yellow
papules on the upper face. Rosai-Dorfman, or sinus histiocytosis with massive lymphadenopathy,
features polymorphic papules and plaques in the first two decades with fever, increased ESR, and
cervical lymphadenopathy. Necrobiotic xanthogranuloma is characterized by red-orange plaques
that may ulcerate. Finally, multicentric reticulohistiocytosis features coral beading around the
fingers and is associated with arthritis mutilans and malignancy.

171
26
Which of the following porphyria cutanea tarda associations has a direct relationship to the level
of urine uroporphyrins?
A. Dystrophic calcifications
B. Estrogen levels
C. Sclerodermoid changes
D. RBC fluorescence
E. Hypertrichosis
►C

Connective tissue dystrophic calcifications, increased estrogen levels, hypertrichosis and


sclerodermoid changes are all associated with porphyria cutanea tarda, but sclerodermoid
changes are the only finding with a direct relationship to urine uroporphyrin levels. Urine
fluorescence occurs in PCT, not RBC fluorescence.

27
A 55 year old patient presents with new onset brown macules on arms, legs, face and palms. She
gives a 3 month history of diarrhea, abdominal cramps, weight loss and protein-losing
enteropathy. The most likely diagnosis is:
A. Peutz-Jeghers syndrome
B. Cowden disease
C. Ulcerative colitis
D. Cronkhite-Canada syndrome
E. Plummer-Vinson syndrome
►D

Cronkhite-Canada is an aquired disease characterized by the development of polyps throughout


the GI tract. Patients can present with hyperpigmented macules as well as the sequella of GI
malabsorption. The malignant transformation of polyps can occur.

28
Which of the following diseases is caused by an enzymatic defect that occurs in the
mitochondria?
A. Porphyria Cutanea Tarda
B. Acute Intermittent Porphyria
C. Congenital Erythropoeitic Porphyria
D. Erythropoeitic Protoporphyria
E. Hepatoerythropoetic Porphyria
►D

172
Defects in many of the enzymes involved in heme synthesis are responsible for porphyrias. The
first and last three steps of the heme synthesis pathway occur in the mitochondria.

29
Which one of the following clinical findings suggests the diagnosis of multiple myeloma?
A. Sudden eruption of seborrheic keratoses
B. Hyperkeratotic follicular papules on the nose
C. Erythema, vesicles, and erosions in periorifical and acral areas
D. Concentric erythematous rings with trailing scale on trunk and extremities
E. Tripe palms
►B

Hyperkeratotic follicular nasal papules are associated with multiple myeloma. A sudden increase
in number and size of seborrheic keratoses is called the sign of Lesser-Trelat and can be seen in
gastric or colon carcinoma. Erythema and vesiculation in periorificial and acral areas is
characteristic of necrolytic migratory erythema most often associated with pancreatic cancer.
Concentric erythematous rings with trailing scale is a feature of erythema gyratum repens most
commonly seen in lung cancer. Tripe palms with AN more likely gastric whereas without AN
more likely lung.

30
All of the following disorders have an increased risk of systemic malignancy except
A. Rothmund Thompson syndrome
B. Cockayne syndrome
C. Bloom syndrome
D. Werner syndrome
E. Xeroderma pigmentosum
►B

Patients with Rothmund thompson have early photosensitivity and poikilodermatous skin
changes, juvenile cataracts, skeletal dysplasias, and a predisposition to osteosarcoma and skin
cancer. Patients with Bloom syndrome have an increased risk for GI malignancies and
lymphoma. Patients with xeroderma pigmentosum can develop ocular melanoma. Patients with
Werner syndrome can develop thyroid and hematologic malignancies, sarcomas, and
meningiomas. Cockayne syndrome patients have photosensitivity, short stature, premature aging,
visual problems, neurologic deficits, but no internal malignancies.

31

173
A 6 month-old has a verrucous plaque on the mucosal surface of the lower lip. Skin biopsy is
consistent with Riga-Fede disease. You should refer the patient to:
A. An ophthalmologist
B. A gastroenterologist
C. A neurologist
D. A hematologist
E. An otolarngologist
►C

Riga-Fede disease is a benign ulcerative granulomatous process that occurs in reaction to


chronic, repetitive trauma of the oral mucosa by the teeth. Clinically, it appears as firm,
verrucous plaques. It may be associated with an underlying developmental anomaly or
underlying neurologic disorder.

32
A 64-year old woman presents with scalloped erosions in the dermatome of the first branch of
the trigeminal nerve. Biopsy would reveal which of the following histologic findings:
A. Asteroid Bodies
B. Caterpillar Bodies
C. Cowdry Type A Bodies
D. Donovan Bodies
E. Dutcher Bodies
►C

Varicella zoster virus is the causative agent in chicken pox and shingles, where recrudescence
from latency in the dorsal root ganglion causes a dermatomal distribution of painful vesicles and
erosions. Biopsy of herpes viruses reveals cells with multinucleation, margination of the
chromatin, and Cowdry Type A bodies, which are intranuclear eosinophilic amorphous bodies
surrounded by a clear halo.

33
The most common autoimmune disease associated with hepatitis C is:
A. Autoimmune thyroiditis
B. Myasthenia gravis
C. Aplastic anemia
D. Lymphocytic sialadenitis
E. Rheumatoid arthritis
►A

174
The most common autoimmune disease seen in association with Hepatitis C is autoimmune
thyroiditis.

34
All of the following are true regarding Cockayne Syndrome EXCEPT:
A. Inheritance is autosomal recessive
B. It is caused by an inability to repair cyclobutane dimers
C. It is associated with basal ganglia calcifications
D. It is associated with retinal pigment degeneration with a "salt and pepper"
appearance
E. None of these answers are incorrect
►E

Cockayne Syndrome is an autosomal recessive disorder caused by a mutation of an unknown


gene that results in the inability to repair cyclobutane dimers induced by UV exposure.
Cutaneous findings include photosensitivity, "bird-headed" facies, and "Mickey Mouse" ears.
Statements A-D are true regarding the syndrome.

35
With regards to patients with diabetes mellitus and skin, which of the following is most
accurate?
A. Diabetic dermopathy is most commonly expressed on the forearms and feet
B. Scleredema most often presents on the tibial surfaces
C. Scleredema is the most common skin manifestation of diabetes mellitus
D. Less than 5% of patients with diabetes mellitus have necrobiosis lipoidica
E. Less than 5% of patients with necrobiosis lipoidica have diabetes mellitus
►D

Patients with diabetes mellitus (DM) may develop thickened skin and tightened joints.
Scleredema is occasionally seen in type II diabetics and presents with a peau d‟orange texture
and appearance on the upper back. Perhaps as many as 20% of those with necrobiosis lipoidica
diabeticorum(NLD) have diabetes, but only 0.3% to 3% of diabetics have NLD. The most
common skin manifestation of DM is diabetic dermopathy, or skin hyperpigmentation, normally
on tibial surfaces. Eruptive xanthomas, which are associated with high triglycerides are
occasionally seen as are generalized or perforating granuloma annulare.

36
An 8-year-old boy presents with immediate burning of his skin on sun exposure. Biopsy of his
dorsal hand is notable only for PAS-positive hyalinization of the vessel walls in the superficial
papillary dermis. What is the most likely diagnosis?

175
A. Systemic lupus erythematosis
B. Atrophe blanche
C. Erythropoietic Protoporphyria
D. Actinic prurigo
E. Hydroa vacciniforme
►C

Erythropoietic Protoporphyria (EPP) is characterized by a burning sensation experienced


immediately upon photoexposure. Biopsy reveals PAS-positive hyalinization of the superficial
dermal vessel walls. Atrophe blance appears similar histologically but is not characterized by
photosensitivity. The other disorders exhibit different histopathologic findings than described.

37
Erythema gyratum repens is known to be associated with all of the following malignancies
except:
A. Lung carcinoma
B. Breast carcinoma
C. Cervical carcinoma
D. Bladder carcinoma
E. Bone carcinoma
►E

Erythema gyratum repens presents more commonly in men than in women, and appears
clinically as concentric erythematous rings with trailing scale on the trunk and proximal
extremities. The skin is described as having a "wood grain" appearance. Skin findings often
precede the diagnosis of an associated malignancy. Lung carcinoma is the most commonly
associated malignancy, but it has also been reported in association with breast, cervical, bowel,
prostate, gastric/bowel and bladder cancer.

38
A patient presents with a complaint of facial flushing that spreads to the neck and upper trunk.
Review of systems reveals that the patient has occassional bouts of diarrhea, and intermittent
bronchospasm. Laboratory testing reveals an elevated urine 5-hydroxyindole-acetic acid level.
Which of the statements regarding this condition is NOT correct?
A. Chest and abdominal/pelvic CT scanning should be the next step in this patient's
evaluation
B. The patient may have an associated sclerodermoid-like eruption on examination
C. The patient likely has a neoplasm originating in the endocrine argentaffin cells
D. Treatment with cyproheptadine would be contraindicated
E. The patient likely has a neoplasm located in the GI tract

176
►D

This patient has carcinoid sydrome, which presents with facial flushing, diarrhea, and
intermittent bronchospasm. Patients can also develop telangiectasia, pellagra-like, or
sclerodermoid-like cutaneous findings. This syndrome is caused by a neoplasm originating in the
endocrine argentaffin cells. 80-85% are found in the GI tract. Treatment is surgical removal of
the tumor, or medical treatment with medications including somatostatin, methylsergide,
cyproheptadine, beta-blockers, and phenothiazine derivatives.

39
A patient comes to your office with extensive alopecia and melanocytic macules on the fingers.
In the review of systems he states he has diarrhea and was told by his primary doctor that he has
malabsorption symptoms. A previous colonoscopy revealed a few polyps which were removed.
What kind of mutation does this patient have?
A. Autosomal recessive
B. Autosomal dominant
C. X-linked recessive
D. Sporadic
E. Mosaicism
►D

Patient has Cronkhite-Canada syndrome. This is a syndrome with a sporadic mutation. Patients
develop GI symptoms(diarrhea, malabsorption, polyposis), weight loss and weakness. Later on
they develop cutaneous changes such as melanocytic macules on the fingers, generalized
hyperpigmentation and alopecia

40
Which of the following is TRUE about diabetic-related skin disease?
A. Bullae are common on the thighs
B. Yellow skin affects the majority of diabetic patients
C. Diabetic dermopathy affects the upper back
D. Necrobiosis lipoidica affects approximately 20% of diabetics
E. Direct immunofluorenscence is usually negative in bullous diabeticorum
►E

Approximately 30% of diabetic patients have cutaneous manifestations of their disease. Diabetic
dermopathy, or “shin spots” is the most common cutaneous association with diabetes. Patients
are generally long-standing diabetics, and are associated with symmetrical brownish, atrophic
plaques on the shins. Acanthosis nigricans is more common in black and Hispanic diabetics, and
may be a component of the HAIRAN (hyperandrogen, insulin resistance, acanthosis nigricans)

177
syndrome. Hyperpigmented velvety plaques are most often found in the flexures. Waxy, thick
skin and limited joint mobility are both related to poor glucose control. Scleredema
diabeticorum, which most often causes thickened plaques of the upper back and neck most often
affects type II diabetics. Necrobiosis lipoidica diabeticorum (NLD) is present in only 0.3 to 3%
of diabetics, but perhaps 20% of patients with NLD have diabetes or glucose intolerance.
Sharply-demarcated yellow-brown, telangiectatic plaques are found on the tibial surfaces.
Ulceration may occur. Yellow skin may affect approximately 10% of diabetics and presents
diffusely. Serum carotene levels may be elevated. Perforating disorders may be related,
especially in the setting of renal disease. In the setting of hypertriglyceridemia, eruptive
xanthomata may be present. The association between diabetes and granuloma annulare (GA) is
controversial. If truly an association, the generalized and perforating forms of GA are most
commonly associated. Infectious associations include candidiasis, which may present an angular
cheilitis, chronic paronychia, interdigital or intertriginous disease. Cutaneous bacterial infections,
including group A and B streptococci, pseudomonas aerugenosa and corynebacterium are more
common. Dermatophyte infections are not more common, although tinea pedis may increase the
risk for cellulitis in diabetic patients. Finally, rhinocerebral mucormycosis is a potential
disasterous complication of uncontrolled hyperglycemia with ketoacidosis. Mortality rates can
approach 35%.

41
Hypothyroidism may result in all of the following cutaneous findings except:
A. Increase in the percentage of telogen hairs
B. Yellowish hue
C. Purpura
D. Madarosis
E. All of these answers are correct
►E

All of the above are non-specific cutaneous manifestations of hypothyroidism: an increase in


percentage of telogen hairs, yellowish hue caused by carotenemia, purpura secondary to impaired
wound healing, and loss of the lateral third of the eyeborw (madarosis).

42
Regarding carcinoid syndrome, which of the following is TRUE?
A. Symptoms are caused by metastases to the skin
B. The most common location for the tumor is the rectum
C. Somatostatin is a preferred treatment
D. VMA is elevated in the urine
E. The tumors should not be removed
►C

178
Carcinoid syndrome is caused by a tumor originating in the endocrine argentaffin cells. In order
of descending frequency, the most common locations are appendix > small bowel > rectum.
Patients experience facial flushing that descends anatomically, diarrhea, hyperhidrosis and
bronchial reactivity. A pellagra-like eruption may occur as well as telangiectasia and
sclerodermoid changes. Patients experience symptoms once the disease metastasizes to the liver
or if it did not involve the GI tract initially. Diagnosis can be made finding elevated 5-
hydroxyindolacetic acid (5-HIAA) in the urine. Nitrosonaphthol turns urine purple if 5-HIAA
levels are significantly elevated, and can be used as a screening test. Bananas, tomatoes, plums,
avocadoes and eggplant can increase levels of 5-HIAA. Treatment involves removal of the tumor
along with medical therapy if appropriate. Agents used include somatostatin, methylsergide,
cyproheptadine, and beta blockers. Vanillylmandelic acid (VMA) is an important urinary
metabolic product of epinephrine and norepinephrine and is detected in the diagnosis of
pheochromocytoma.

43
All of the following are true regarding Henoch-Schonlein Purpura EXCEPT:
A. It is an IgG mediated small vessel vasculitis
B. Direct immunofluorescence of lesional and peri-lesional skin will demonstrate
C3 and fibrin deposits in small vessel wall
C. It is often preceded by an upper respiratory infection
D. It may be complicated by intussusception
E. It is self-resolving
►A

Henoch-Schonlein Purpura (HSP) is an IgA mediated small vessel vasculitis. Clinically, patients
present with palpable purpura of the lower extremities and buttocks, GI vasculitis, and
glomerulonephritis.

44
Which of the following statements regarding porphyrias is FALSE?
A. Fecal protoporphyrin is increased in variegate porphyria
B. Hepatoerythropoietic porphyria is the homozygous form of porphyria cutanea
tarda
C. Acute intermittent porphyria has no skin findings
D. Coproporphyrinogen is elevated more than uroporphyrinogen in 24 hour urine
samples in porphyria cutanea tarda
E. In erythropoietic protoporphyria, protoporphyrin IX absorbs in the Soret band
►D

179
Acute intermittent porphyria (AIP) is the second most common porphyria and is caused by a
deficiency in porphobilinogen (PBG) deaminase, which is located in the cytosol. Patients suffer
from colicky pain, paralysis and psychiatric disorders. There are no specific skin manifestations.
PBG and aminolevulenic acid (ALA) are elevated in the urine. Attacks are precipitated by
medications such as barbiturates, estrogen, griseofulvin, and sulfonamides as well as starvation,
fever and infection. Treatment includes glucose loading and hematin infusion.
Congenital erythropoietic porphyria (CEP) or Günter‟s disease is caused by a defect in
uroporphyrinogen III synthase, which is found in the cytosol. Patients are extremely
photosensitive and erythema, blistering and scarring result. Patients present with red urine early
in life along with hypertrichosis and red-stained teeth that fluoresce. Uroporphyrins (URO) are
much high than coproporphyrins (COPRO) in the urine. URO is found in the red blood cells
(rbcs) and COPRO is found in the stool. The rbcs display stable fluorescence.
Porphyria cutanea tarda (PCT) is the most common porphyria and is caused by a deficiency
(usually sporadic) in uroporphyrinogen decarboxylase, which is found in the cytosol. Patients
present with photosensitivity and blistering of sun-exposed areas, especially the dorsal hands.
Hypertrichosis and sclerodermoid changes may occur as well. Liver disease (hepatitis C or
alcoholic cirrhosis) is often present and hemochromatosis may be associated. Urine may
fluoresce pink or coral-red with Wood‟s lamp. URO>COPRO in the urine and low levels of
COPRO are found in the stool. Treatments include phlebotomy, antimalarials, and therapy for
liver disease if appropriate.
Hereditary coproporphyria (HCP) is caused by a deficiency in coproporphyrinogen oxidase,
which is found in the mitochondria. One-third of patients are photosensitive, and patients suffer
gastrointestinal and neurological symptoms similar to AIP. Urine COPRO is elevated only with
attacks, and COPRO is present in the stool. Variegate porphyria is the result of decreased activity
of protoporphyrinogen oxidase, which is present in the mitochondria. It combines the skin
lesions of PCT with the systemic manifestations of AIP. Urine COPRO:URO is 1:1 or COPRO>
URO to distinguish it from PCT, and PROTO is found in the stool. The plasma fluoresces at
626nm. Precipitators and treatments are similar to AIP.
Erythropoietic protoporphyria (EPP) is caused by ferrochetalase deficiency, which is present
in the mitochondria. Patients experience immediate burning of the skin with sun exposure.
Protoporphyrin IX, the only oxidized porphyrin in the heme pathway and absorbs in the Soret
band (400-410nm). Patients have erythematous plaques in a photo-distribution. Urine porphyrins
are normal. PROTO is found in the rbcs and the stool. Excessive porphyrins deposited in the
liver lead to gallstones and cirrhosis. Beta carotene may helpful.
Hepatoerythropoietic porphyria (HEP) is essentially a homozygous form of PCT, with
deficiency in uroporphyrinogen decarboxylase. It is clinically similar to CEP with red urine and
hypertrichosis, vesicles and scarring of sun-exposed skin. URO is present in the urine and
COPRO in the stool. PROTO is present in rbcs which distinguishes it from CEP, which was
URO in rbcs.

180
45
Which of the following is FALSE regarding Fabry‟s disease?
A. It is inherited in an X-linked recessive fashion
B. It is associated with acral parasthesias
C. It is inherited in an X-linked dominant fashion
D. It may be associated with renal failure
E. The etiology is a defect in alpha-galactosidase A
►C

Fabry‟s disease is an X-linked lysosomal disorder that leads to excessive deposition of neutral
glycosphingolipids in the vascular endothelium. The disorder is caused by a deficiency of alpha-
galactosidase-A leading to progressive endothelial accumulation of glycosphingolipids. This
accumulation accounts for the associated clinical abnormalities of skin, eye, kidney, heart, brain,
and peripheral nervous system. Acroparesthesias are a frequent presenting symptom of Fabry‟s
disease. The primary cutaneous manifestation are angiokeratomata (angiokeratoma corporis
diffusum), which are most common in a “bathing-trunk” distribution. Lens opacities and retinal
and conjunctival vascular malformations may be found in the eyes. Patients may experience
cardiac disease, stroke or renal failure.

46
An end stage AIDS patient with tuberculosis presents with diffuse hyperpigmentation of both
sun-exposed and unexposed areas. The palmar creases are markedly hyperpigmented and the
patient is very ill. The most likely diagnosis is:
A. Argyria
B. Lymphoma
C. Tinea versicolor
D. Kaposi's sarcoma
E. Addison disease
►E

Addison disease is caused by destruction of the adrenal glands by any cause. Tuberculosis used
to be the primary cause, now the most common cause is auto-immune destruction. Other causes
of adrenal gland destruction include; coccidiomycosis, cryptococcosis, histoplasmosis,
sarcoidosis, metastatic tumor and amyloidosis.

47
Which of the following is not a cause of a saddle nose deformity?
A. Relapsing polychondritis
B. Hypohidrotic ectodermal dysplasia
C. Congenital syphilis

181
D. Trichorhinophalangeal syndrome
E. Wegener's granulomatosis
►D

d. Trichorhinopharyngeal syndrome is the only condition from the mentioned conditions that is
not associated with saddle nose deformity.It is associated with sparse hair, a bulbous (pear-
shaped) nose and cone-shaped epiphyses.

48
A patient infected with the hepatitis C virus is at highest risk for which of the following
cutaneous manifestations?
A. Increased hyaluronic acid in dermis of lower and upper extremities
B. Lindsay‟s nails
C. Congenital hyperpigmentation of the retinal pigment (CHRPE)
D. Periorificial lentigines
E. Positive rheumatoid factor and decreased complement (C3)
►E

Patients with hepatitis C virus (HCV) may experience numerous cutaneous manifestations of
their internal disease. Cryoglobulinemia (CG) may result in small vessel leukocytoclastic
vasculitis. Types of CG include: type I, which is monoclonal IgM; type II, which is polyclonal
IgG and monoclonal IgM; type III, which is polyclonal IgG and polyclonal IgM. Mixed CG is
the most commonly associated type of CG with HCV. Patients often have a positive rhematoid
factor and have C3 complement decreased. Other diseases associated with HCV include
porphyria cutanea tarda, lichen planus and polyarteritis nodosa. Thyroid-related dermopathy
results in increased hyaluronic acid in the dermis. Lindsay's nails, or half and half nails, are
associated with renal disease. Congenital hyperpigmentation of the retinal pigment (CHRPE) is
seen in Gardner's syndrome. Peutz-Jeghers, among other conditions presents with periorificial
lentigines.

49
A 24-year-old woman presents with painful sores involving the bilateral cheeks. Biopsy reveals a
dense lichenoid infiltrate with Civatte body formation. What is the most likely associated
systemic disease:
A. Diabetes mellitus
B. Hepatitis C Infection
C. Hypothyroidism
D. Chronic kidney disease
E. Castleman‟s disease
►B

182
Lichen planus of the oral mucosa is most highly associated with Hepatitis C Infection. Biopsies
of lichen planus on the oral mucosa may exhibit plasma cells and parakeratosis, unlike cutaneous
lichen planus. The other disorders are not as closely related to oral lichen planus.

50
Regarding eruptive xanthomas, which of the following is true?
A. They occur in the setting of familial hyperlipidemia types I, IV, and V
B. They occur in the setting of familial hyperlipidemia types II and III
C. Triglyceride levels are usually below 500mg/dl
D. They are most commonly found on the eyelids
E. They are not related to alcohol consumption
►A

Eruptive xanthomas generally occur in patients with triglyceride levels of 2000mg/dl or greater.
Associations include poorly-controlled diabetes mellitus, retinoids, estrogens, excessive alcohol
consumption (leading to pancreatitis) and familial hyperlipidemias types I, IV and V. Clinically,
they appear as crops of firm, non-tender yellowish papules with an erythematous border. Most
commonly, they occur on the extensor surfaces, but they can be diffuse. A reduction in
triglycerides and/or tight glucose control usually results in a reduction in the number of lesions.

51
Patients with this syndrome are at increased risk for developing Lhermite-Duclos disease:
A. Bourneville's Disease
B. Nail-Patella Syndrome
C. MEN Type IIA
D. Fabry's Disease
E. Cowden's Syndrome
►E

Patients with Cowden's Syndrome (multiple hamartoma syndrome) are at increased risk for
Lhermite Duclos disease (dysplastic gangliocytoma of the cerebellum).

52
The patient demonstrated in figure 6 would have:
A. LDL receptor deficiency
B. Decreased chylomicrons
C. Elevated triglycerides
D. Decreased LDL
E. Decreased triglycerides

183
►C

This patient has eruptive xanthomas, a condition which is classically associated with elevated
serum triglycerides.

53
Which of the following statements about necrolytic migratory erythema is TRUE?
A. There is a low incidence of metastasis of the offending tumor at the time of
diagnosis
B. Acanthosis and parakeratosis are found on routine histology
C. Serum glucagon levels are usually normal
D. Vacuolar changes are normally found on routine histology
E. The offending tumor originates from endocrine argentaffin cells
►B

Necrolytic migratory erythema or glucagonoma syndrome is clinically characterized by


periorificial and acral erythema, vesicles, pustules and erosions. A circinate pattern is often seen.
Glossitis and cheilitis are features as well. The underlying tumor is an alpha-2 glucagon
producing islet cell pancreatic carcinoma, which is metastatic at the time of diagnosis in the
majority of cases. Histopathological findings include dyskeratotic keratinocytes in the stratum
granulosum, acanthosis and parakeratosis. Serum glucagon levels are elevated. The tumors must
be resected if feasible, and intravenous somatostatin and amino acids have been used for
treatment.

54
Which of the following is NOT associated with Cronkhite-Canada syndrome?
A. Lentigines
B. Adenomatous gastrointestinal polyps
C. Fibrocystic breast disease
D. Onycholysis
E. Weight loss
►C

Cronkhite-Canada syndrome (CCS) is a rare, sporadically occurring, non-inherited disorder


characterized by generalized gastrointestinal polyps, cutaneous pigmentation, alopecia, and
onychodystrophy. The pathogenesis is unknown. The GI polyps are hamartomatous and
malignant degeneration is unusual. Cutaneous signs include circumscribed lentiginous
hyperpigmentation, alopecia and nail dystrophy. Often, symptoms appear in the sequence of
gastrointestinal symptoms, weight loss, weakness, edema, and then cutaneous changes after
weeks or months. CCS is a progressive disease, and often carriers a poor prognosis primarily

184
because of difficulties with fluid and electrolyte management. Fibrocystic breast disease is
associated with Cowden”s disease (multiple hamartoma syndrome).

55
A 34-year-old woman presents with dell-shaped papules of the bilateral cheeks. Biopsy of a
representative lesion reveals clear cell syringoma. Which is the most likely associated systemic
disease:
A. Diabetes mellitus
B. Hepatitis C Infection
C. Hypothyroidism
D. Chronic kidney disease
E. Castleman‟s disease
►A

Clear cell syringomas are associated with diabetes mellitus. Eruptive syringomas may arise in
patients with skin type V and VI and in patients with Down‟s syndrome. The other disorders are
not associated with syringomas.

56
A patient with end stage renal disease complains of some itchy spots on his back. Examination
reveals flesh-colored papules with a central keratotic core. Acquired perforating dermatosis is
more commonly seen in end stage renal disease due to what underlying disorder?
A. Diabetic nephropathy
B. Lupus nephritis
C. Mesangial glomerulonephropathy
D. IgA nephropathy
E. Medication-related renal failure
►A

Acquired perforating dermatosis is the transepidermal elimination of altered dermal substances.


The etiology is unknown, but may be related to micro-deposits of calcium with subsequent
evacuation through the epidermis. It is more commonly seen in darker skin types and in patients
with ESRD secondary to diabetic nephropathy.

57
Which of the following is NOT normally associated with dermatomyositis?
A. Ovarian cancer in women
B. Psoriasiform dermatitis of the scalp
C. Elevated aldolase levels
D. Cuticular dystrophy

185
E. Testicular cancer in men
►E

Dermatomyositis is characterized by proximal muscle weakness and a constellation of cutaneous


findings. Skin findings include periorbital edema and poikiloderma (heliotrope), Gottron‟s sign
and papules, nail fold telangiectasia, cuticular dystrophy, poikiloderma in a shawl distribution
and photosensitivity. Psoriasiform dermatitis and scaling of the palms and soles may occur--
NOT on the scalp! Children more often display calcinosis cutis and vasculitis. Within the first 3
years of diagnosis there is a higher likelihood of malignancy. Women are at risk for ovarian and
breast carcinoma, and men are at risk for gastric carcinoma and lymphoma. Diagnostic findings
include elevated creatine kinase (CK) and aldolase, inflammatory muscle biopsy changes or
abnormal muscle MRI, and abnormal EMG. Treatment includes corticosteroids, methotrexate
and other immunosuppressive medications.

58
Which of the following is NOT true regarding Cushing's Syndrome?
A. Urine cortisol levels are elevated
B. Corticotropin is suppressed with the administration of dexamethasone
C. It may be associated with an underlying oat cell lung carcinoma
D. It may be associated with hypertension and hypokalemia
E. May present with hyperpigmentation and facial plethora
►B

In Cushing's Syndrome, corticotropin is NOT suppressed with the administration of


dexamethasone. The other statements are true.

59
The shoulder pad sign has been described in which disease?
A. Dermatomyositis
B. Cushings disease
C. Systemic amyloidosis
D. Nephrogenic fibosing dermopathy
E. Systemic lupus erythematosis
►C

The shoulder pad sign has been described in the setting of late systemic amyloidosis and is due
to the direct deposition of amyloid in the deltoid muscles.

60

186
Paraneoplastic pemphigus is associated with all of the following underlying malignancies
EXCEPT:
A. Non-Hodgkins lymphoma
B. Lung carcinoma
C. Chronic lymphocytic leukemia
D. Thymoma
E. Castleman's tumor
►B

Paraneoplastic pemphigus has been associated with Non-Hodgkins lymphoma, chronic


lymphocytic leukemia, thymoma, Castleman's tumor, and sarcoma. It has not been associated
with lung carcinoma. Treatment includes management of the underlying malignancy, as well as
prednisone or other immunosuppressive agents.

61
Which of the following is true regarding cutaneous associations with hepatitis C virus (HCV)
infection?
A. Erosive mucosal lichen planus has a weaker association with HCV than does
cutaneous lichen planus
B. Pruritus in the setting of chronic HCV infection is generally correlated with
elevated bile salt levels in the setting of liver failure
C. Polyarteritis nodosa is associated with HCV infection but not hepatitis B virus
infection
D. HCV-related porphyria cutanea tarda may be caused by decompartmentalization
of iron stores and resultant oxidation of uroporphyrinogen decarboxylase
E. Cutaneous reactions to HCV treatment are less common with interferon/ribavirin
combination treatment than with treatment with interferon alone
►D

Erosive mucosal variant of lichen planus has the strongest association with HCV. The
pathogenesis of pruritus in the setting of chronic HCV infection may be related to elevated bile
salt levels in the setting of liver failure, but there is not always a direct correlation between serum
bile salt level and degree of pruritus. Polyarteritis nodosa is associated with both HCV infection
and hepatitis B virus infection. Statement D is correct. Cutaneous reactions to HCV treatment are
more common with interferon/ribavirin combination treatment than with treatment with
interferon alone.

62
A patient presents with diffuse waxy keratoderma of the palms and soles as well as oral hairy
leukoplakia. Which of the following statements regarding this condition is NOT true?

187
A. This condition may be associated with esophageal carcinoma
B. This condition is autosomal recessive
C. Family members should be advised to undergo cancer screening
D. Features may include squamous cell carcinomas arising from keratodermic skin
E. None of these answers are correct (all are true)
►B

Howel-Evans syndrome is an autosomal dominant disorder that presents with diffuse waxy
keratoderma of the palms and soles, as well as oral leukoplakia and squamous cell carcinoma
arising from keratodermic skin. It is associated with esophageal carcinoma; cancer screening in
family members is advised.

63
Which of the following laboratory abnormalities is most common in patients with cholesterol
emboli?
A. Neutrophilia
B. Elevated amylase
C. Hypercalcemia
D. Eosinophilia
E. Anemia
►D

Eosinophilia develops within 3 days of embolization in 70-80% of patients and may remain
elevated for up to 1 month. Patients with cholesterol emboli typically have an elevated ESR and
C-reactive protein as well. Leukocytosis can also be seen in up to half of patients.

64
Regarding paraneoplastic pemphigus, which of the following is TRUE?
A. Desmoplakin, one of the molecular antigens, has a molecular weight of 190kd
B. Metastatic squamous cell carcinoma of the skin is a common cause
C. Granular C3 deposition is found at the dermoepidermal junction on direct
immunofluorescence
D. Monkey esophagus is the preferred substrate for indirect immunofluorescence
E. Non-Hodgkins lymphoma is rarely associated
►C

Paraneoplastic pemphigus (PNP) is characterized by painful stomatitis, which is extremely


resistant to therapy. Cutaneous lesions are variable and can include flaccid or tense bullae,
targetoid lesions or lichenoid eruptions. PNP is associated with the following neoplasms (in
descending order of frequency): non-Hodgkins lymphoma, chronic lymphocytic leukemia,

188
Castleman‟s disease and malignant and benign thymus tumors. Histopathologic findings include
suprabasal acantholysis, dyskeratosis and vacuolar interface with a lichenoid infiltrate. Direct
immunofluorescence of perilesional skin demonstrates intercellular and granular IgG and
granular C3 at the dermoepidermal junction. The preferred substrate for indirect
immunofluorescence is rat bladder and intercellular IgG is found. Target antigens include
desmoplakin (250kD), envoplakin (210kD), bullous pemphigoid antigen-1 (230kD), periplakin
(190kD), and desmogleins 1 and 3. Many patients succumb to the underlying cancer. Treatment
requires management of the malignancy and immunosuppressive agents.

65
Which of the following is an autosomal recessive disorder characterized by pili torti and
deafness?
A. Waardenburg Syndrome
B. Sjorgen-Larsson Syndrome
C. Refsum Disease
D. Bjornstad's Syndrome
E. Cockayne Syndrome
►D

Waardenburg Syndrome is an autosomal dominant disorder. Sjorgen-Larsson Syndrome is an


autosomal recessive disorder characterized by a triad of ichthyosis, di- or tetraplegia, mental
retardation. Refsum Disease is an autosomal recessive disorder whose features include deafness,
mild ichthyosis in adulthood, and accentuated palmar creases. Bjornstad's Syndrome is an
autosomal recessive disorder characterized by pili torti and deafness. Cockayne Syndrome is an
autosomal recessive disorder whose features include photosensitivity, bird-headed facies, and
"Mickey-Mouse" ears, as well as deafness.

66
Which of the following is characteristic of diabetic skin?
A. Approximately 20% of diabetics have necrobiosis lipoidica diabeticorum (NLD)
B. The level of cleavage in bullous diabeticorum is subcorneal
C. Candida tropicalis is the most common cause of angular cheilitis
D. There is a well-established association between deep granuloma annulare (GA)
and diabetes
E. Yellow skin may occur in up to 10% of diabetics
►E

Perhaps 0.3 to 3% of diabetics have NLD, whereas approximately 20% of NLD patients have
diabetes or glucose intolerance. The split in bullous diabeticorum is normally either
intraepidermal or subepidermal. C. albicans is the most common cause of diabetes-related yeast

189
infections. The association between GA and diabetes is controversial, but if the two are related,
generalized and perforating GA have been implicated. Yellow skin may occur in up to 10% of
diabetic patients and is characterized by diffuse yellow-orange skin. Half of these patients have
elevated serum carotene levels. The suggested is cause is elevated consumption of yellow fruits
and vegetables in the setting of impaired hepatic metabolism of carotene and subsequent non-
enzymatic glycosylation of dermal collagen.

67
A patient develops cold-exacerbated dusky acral plaques consistent with pernio. The systemic
condition most likely to manifest such lesions is:
A. Lupus Erythematosus
B. Dermatomyositis
C. Acquired Immunodeficiency Syndrome
D. Hepatitis C
E. Diabetes Mellitus
►A

Chilblain lupus is a rare manifestation of lupus erythematosus. Lesions resemble common pernio
clinically, but should have histological findings consistent with lupus, such as interface changes.
These lesions may not resolve upon warming and often respond poorly to other lupus treatments
such as antimalarials.

68
The most common location of the lesions in nephrogenic fibrosing dermopathy is:
A. Face
B. Palms and soles
C. Lower extremities
D. Back
E. Chest
►C

The most common location for lesions of nephrogenic fibrosis dermopathy is the lower
extremities.

69
Which of the following syndromes necessitates a work-up for colon cancer?
A. Turcot
B. Birt-Hogg-Dube
C. LAMB
D. Muckle Wells

190
E. Gardner
►A

Turcot syndrome is a variant of HNPCC-hereditary nonpolyposis colorectal cancer, Lynch


syndrome with the homozygous mutations in MSH2, MSH6, MLH1, PMS2 as opposed to
heterozygous mutaitons in HNPCC. It is characterized by polyposis and brain cancer. Patients
may also exhibit other clinical features of HNPCC syndrome. Both patients with Lynch
Syndrome and Turcot Syndrome necessitate a colon cancer work-up.

70
Which step is rate-limiting in the synthesis of protoporphyrinogen?
A. Aminolevulinic acid synthase
B. Aminolevulinic acid dehydratase
C. Porphobillinogen deaminase
D. Coproporphyrinogen oxidase
E. Ferrochelatase
►A

The rate-limiting step in the heme synthesis pathway is the synthesis of delta aminolevulanic
acid from glycine and succinyl CoA via aminolevulinic acid synthase. This step takes place in
the mitochondria. Porphobillinogen deaminase is defective in acute intermittent porphyria,
coproporphyrinogen oxidase in in hereditary coproporphyria and protoporphyrinogen oxidase in
variegate porphyria.

71
Carotenemia can be a manifestation of:
A. Porphyria
B. Hypothyroidism
C. Pretibial myxedema
D. Grave”s disease
E. Amyloidosis
►B

Reduced matabolism of beta-carotene in the diet, can result in yellowing of the skin in
hypothyroidism. Treatment of porphyria with beta-carotene can result in carotenemia. Pretibial
myxedema, Grave”s disease and amyloidosis do not result in carotenemia.

72
Which of the following is (are) characteristic of basal cell nevus syndrome?
A. Autosomal recessive inheritance

191
B. NEMO gene mutation
C. Tram-track calcifications
D. Colobomas
E. Telangiectasias
►D

Basal cell nevus syndrome (Gorlin syndrome) is caused by an autosomal dominant mutation in
PTCH gene that encodes PTC protein involved in sonic hedge hog pathway. This mutation leads
to loss of inhibition of smoothened (SMO) leading to an increased expression of other genes.
Cutaneous findings include nevoid basal cell carcinomas, milial cysts, epidermoid cysts and
palmar-plantar pits. Neurological findings include calcifications of the falx cerebri, agenesis of
the corpus callosum, mental retardation, medulloblastoma and spina bifida. Ocular findings
include blindness, cataracts, colobomas and strabismus. Other findings include odontogenic cysts
of the jaw, frontal bossing, bifid ribs, and pectus deformity. NEMO gene mutations are found in
incontinentia pigmenti. Tram-track calfications are seen Sturge-Weber syndrome.
Telangiectasias are seen in ataxia-telangiectasia among others.

73
A patient with gastric cancer develops acanthosis nigricans and a sudden eruption of numerous
warty stuck-on papules on the trunk. What other finding may be seen?
A. Numerous wart-like lesions on the dorsal hands and wrists
B. Follicular spicules on the nose
C. Carpal tunnel syndrome
D. Periorbital pupura
E. Migratory thrombophlebitis
►A

Florid cutaneous papillomatosis is the rapid onset of numerous wart-like lesions on the backs of
the hands and wrists that is associated with pruritus, acanthosis nigricans, and the sign of Leser-
Trelat. All reported cases are associated with internal malignancy, most commonly gastric
carcinoma. Follicular papules on the nose are seen in multiple myeloma. Carpal tunnel syndrome
and periorbital purpura can be seen in primary systemic amyloidosis with multiple myeloma.
migratory thrombophlebitis is associated with pancreatic cancer.

74
An uncommon complication of treatment with potassium iodide is:
A. Acute generalized exanthematous pustulosis
B. Erythema nodosum
C. Wolff-Chiakoff effect
D. Exacerbation of lichen planus

192
E. Hyperhidrosis
►C

An uncommon complication of treatment with potassium iodide is the Wolff-Chiakoff effect.

75
Which of the following is true about blue rubber bleb nevus syndrome?
A. typically appears in adulthood
B. the venous malformations regress with time
C. characterized by compressible blue nodules 0.1 to 5 cm in size.
D. the nodules are pruritic
E. the gastrointestinal lesions are typically located in the stomach
►C

Blue rubber bleb nevus is associated with soft and compressible blue nodules 0.1 to 5 cm in size.
The cutaneous lesions typically appear in childhood, and they do not regress with time. The
venous malformations are mainly asymptomatic but can be painful when thrombosis occurs. The
venous malformations also involve the gastrointestinal tract, most commonly the small bowel,
where they are friable and can bleed.

76
A 14-year-old girl presents with a 20-pound weight loss along with painful oral erosions and
severe stomatitis. Which is the most likely associated systemic disorder:
A. Small cell lung cancer
B. Hepatitis C Infection
C. Hypothyroidism
D. Chronic kidney disease
E. Castleman‟s disease
►E

Castleman‟s tumor is the most common cause of paraneoplastic pemphigus in children.


Paraneoplastic pemphigus presents with painful stomatitis and varied skin lesions. Indirect
immunofluorescence with rat bladder substrate shows intercellular IgG.

77
All of the following are true regarding the condition pictured EXCEPT:
A. Perilesional direct immunofluroescence shows granular IgA in the dermal
papillae and at the dermoepidermal junction
B. Only 20% of patients have a gluten-sensitive enteropathy
C. It is associated with HLA-DQ2, HLA-DR3, and HLA-B8

193
D. It is associated with Hashimoto's thyroiditis
E. Cutaneous findings are due to autoantibodies to epidermal transglutaminase
►B

All of the statements regarding dermatitis herpetiforms, or "Duhring's Disease," are true except
for statement B. Virtually all DH patients have gluten-sensitive enteropathy, although only 20%
of them have symptoms (such as diarrhea, steatorrhea, weight loss, bloating, and malabsorption).

78
A 64-year old man develops yellowish periorbital plaques that occasionally ulcerate and heal
with scarring. What is the most likely associated lab finding?
A. Monoclonal gammopathy
B. Positive antinuclear antibodies
C. Elevated creatinine
D. Elevated triglycerides
E. Increased thyroid stimulating hormone
►A

The patient has necrobiotic xanthogranuloma. Lesions develop periorbitally and in flexural areas,
often extend deeply into the dermis or subcutis, and heal with scarring. These lesions are
typically associated with a monoclonal gammopathy, usually IgG. Patients with necrobiotic
xanthogranuloma rarely develop myeloma. Leukopenia and hepatosplenomegaly are also often
seen.

79
Which of the following cutaneous findings are characteristic for Vohwinkles Syndrome?
A. Honeycombed diffuse palmoplantar keratoderma, pseudoainhum with
autoamputation, star-shaped keratosis over knuckles, nail dystrophy, and
alopecia
B. Dense depigmented lusterless hair, pili torti, doughy skin, diffuse cutaneous
hypopigmentation
C. Transient erythroderma at birth, palmoplantar keratoderma, follicular
hyperkeratosis, scarring alopecia, dystrophic nails
D. Rapidly progressive alopecia of all hair-bearing areas, onycholysis,
onychoschizia, onychomadesis, hyperpigmented macules on extremities
E. Oral papillomatosis, palmoplantar keratoses, acral keratoses, lipomas,
hemangiomas, scrotal tongue
►A

194
Vohwinkles Syndrome is an autosomal dominant disorder caused by a mutation of the GJB2
gene which encodes Connexin 26. Clinical features include deafness, as well as the cutaneous
findings described in choice A.

80
Which of the following is NOT true regarding calciphylaxis?
A. Patients with proximally-located lesions have a better prognosis than those with
acral lesions
B. May be treated with parathyroidectomy
C. May be present with retiform purpura
D. Histologic findings include medial calcification and intimal hyperplasia of small
arteries and arterioles
E. Has an associated mortality of 60-80%
►A

Distribution of lesions in calciphylaxis may predict prognosis; those with acral lesions have a
better outcome than those with proximally located lesions.

81
Features of Cushing disease include all of the following EXCEPT?
A. Facial plethora
B. Striae
C. Hypertension
D. Suppression of corticotropin occurs with administration of dexamethasone
E. Hirsutism
►D

Cushing syndrome is caused by excess levels of either exogenously administrated


glucocorticoids or endogenous overproduction of cortisol. The syndrome is most commonly
caused by the therapeutic administration of exogenous glucocorticoids. The term "Cushing's
disease" is reserved for Cushing's syndrome that is caused by excessive secretion of
adrenocorticotropin hormone (ACTH) by a pituitary tumor, usually an adenoma. Other
endogenous sources include an adrenal tumor and other malignancies (i.e. oat cell carcinoma of
the lung). Patients develop moon facies, facial plethora, supraclavicular fat pads, buffalo hump,
truncal obesity, and striae. In addition, proximal muscle weakness, easy bruising, weight gain,
hirsutism, and, in children, growth retardation may occur. Systemic sequelae include
hypertension, osteopenia, diabetes mellitus, and impaired immune function. Excess endogenous
or exogenous glucocorticoids can result in a leukocytosis and hypokalemic metabolic acidosis.
Urine cortisol levels 3 to 4 times normal are diagnostic. The dexamethasone suppression test is a
valuable diagnostic tool. For this study, 1mg is given at 11pm and the 8am cortisol level is

195
tested. A level of >10 ìg/dL is diagnostic. In patients with Cushing‟s disease, the dexamethasone
does not suppress corticotropin. Corticotropin-releasing hormone (CRH) stimulation test is also
used to distinguish patients with pituitary adenomas and those with ectopic ACTH syndrome or
cortisol-secreting adrenal tumors.

82
What is the treatment of choice for porphyria cutanea tarda?
A. Phlebotomy
B. Antimalarials
C. Erythropoietin
D. Oral iron supplementation
E. Naproxen
►A

Porphyria cutanea tarda (PCT) is caused by a deficiency in the enzyme uroporphyrinogen


decarboxylase. Phlebotomy is the treatment of choice and may improve enzymatic activity by
removing iron, an inhibitor of the enzyme. Antimalarials and erythropoietin are alternative
therapies. Oral iron supplementation may worsen PCT. Naproxen is a common cause of
pseudoporphyria.

83
Which of the following statements about multiple endocrine neoplasia (MEN) syndromes is
FALSE?
A. MEN I is associated with angiofibromas and collagenomas
B. MEN IIa is related to a defect in RET proto-oncogene
C. MEN IIa and IIb are both associated with medullary thyroid carcinoma
D. MEN I, IIa and IIb are all inherited in an autosomal dominant fashion
E. MEN IIa is associated with multiple mucosal neuromas
►E

Multiple endocrine neoplasia (MEN) syndromes are divided into types I, IIa and IIb. All are
autosomal dominantly inherited. Type I is associated with mutations in MENI; type IIa and IIb
involve mutations of RET, which encodes a tyrosine kinase receptor. Type I has the following
cutaneous features: facial angiofibromas, collagenomas, lipomas, hypopigmented macules and
café-au-lait macules. Type IIa displays macular amyloidosis, while IIb features multiple mucosal
neuromas. Systemic features of type I include peptic ulcer disease (as part of Zollinger-Ellison
syndrome), parathyroid hyperplasia or adenoma, pancreatic tumors and pituitary neoplasia. Type
IIa is characterized by Zollinger-Ellison syndrome, medullary thyroid carcinoma,
pheochromocytoma, and parathyroid neoplasia. Type IIb displays marfanoid habitus,

196
Hirschsprung disease, medulloblastoma, medullary thyroid carcinoma, pheochromocytoma and
ocular neuromas.

84
Tripe palms are a cutaneous manifestation associated with which of the following malignancies?
A. Renal carcinoma
B. Lung carcinoma
C. Prostate carcinoma
D. Colon carcinoma
E. Pancreatic carcinoma
►B

Tripe palms are rugose thickening of the palms which are nearly always associated with internal
malignancy. When tripe palms are present without other cutaneous findings, the most likely
internal malignancy is lung cancer. Tripe palms in association with acanthosis nigricans is most
likely to be associated with gastric cancer.

85
The clinical findings in figure 3 are associated with which disorder?
A. Grave”s disease
B. Myasthenia gravis
C. Dermatomyositis
D. Sarcoidosis
E. Rheumatoid Arthritis
►A

This slide shows pretibial myxedema in a patient with Grave‟s disease.

86
A 7 year old boy is seen for "acne". Hypopigmented patches are seen on the trunk and flesh-
colored papules are seen around the nails. Biopsy of one of the papules of the face reveals an
angiofibroma. A review of his chart reveals that he takes medications for seizures. What is the
most common neoplasm associated with the most likely disorder?
A. Renal angiomyolipoma
B. Renal cell carcinoma
C. Gastric carcinoma
D. Transitional cell carcinoma
E. Prostate carcinoma
►A

197
Tuberous sclerosis is an autosomal dominant disorder with seizures, mental retardation, and
characteristic skin findings, including hypopigmented macules, facial angiofibromas, periungual
fibromas, and collagenomas. The most common associated tumor is renal angiomyolipoma.
Rarely, renal cell carcinoma can develop.

87
What is the most common internal cause of intractable pruritus?
A. Hepatitis C
B. Hypothyroidism
C. Hyperthyroidism
D. Chronic renal failure
E. Internal malignancy
►D

Chronic renal failure is the most common internal systemic cause of pruritus. Up to 49% of
patients with chronic renal failure have pruritus. Other systemic causes of pruritus include liver
disease, hepatitis C, hypo and hyperthryoidism, iron deficiency anemia, polycyhtemia vera,
Hodkin's lymphoma, leukemia, carcinoid, internal malignancy, AIDS, and internal parasites.

88
Which of the following may be an early cutaneous manifestation of Sipple syndrome (multiple
endocrine neoplasia type 2A)?
A. mucosal neuromas
B. Koenen tumor
C. mandibular osteomas
D. multiple pilomatricomas
E. cutaneous amyloidosis
►E

Sipple syndrome (multiple endocrine neoplasia type 2A) is an autosomal dominant disorder
characterized by the development of pheochromocytomas, medullary thyroid carcinomas, and
hyperparathyroidism. The associated mutation is found in the RET proto-oncogene. An early
cutaneous manifestation includes the development of lichen or macular amyloidosis (both
keratin-derived cutaneous amyloidoses) often localized to interscapular patches, which may be
pruritic (similar to notalgia paresthetica). The latter arises during teenage years and may be the
earliest presenting sign of this disease.

89
Mutations in the STK11 gene encoding a serine threonine kinase are seen in:
A. Muir-Torre Syndrome

198
B. Bannayan-Riley-Ruvalcaba Syndrome
C. Birt-Hogg-Dubbe Syndrome
D. Cronkhite-Canada Syndrome
E. Peutz-Jeghers Syndrome
►E

Peutz-Jeghers Syndrome is an autosomal dominant syndrome. 50% of patients have mutations in


the STK11 gene which encodes a serine threonine kinase. It presents with periorificial and
mucosal lentigines beginning in infancy, as well as hamartomatous intestinal polyps with
intussusception and bleeding; gallbladder, pancreatic, breast, ovarian, and testicular cancer.

90
Which of the following is true regarding treatment of inflammatory dermatoses with potassium
iodide?
A. The Wolff-Chaikoff effect must be considered
B. Binding of excess organic iodide in the thryoid gland may occur
C. Thyroid hormone synthesis may be inhibited
D. None of the answers are correct
E. All of these answers are correct
►E

The Wolff-Chaikoff effect is described as the binding of excess organic iodide in the thyroid
gland with resultant inhibition of thyroid hormone synthesis. This can occur in the setting of
patients with erythema nodosum (or other inflammatory dermatoses) being treated with
potassium iodide.

91
A 36-year-old woman presents with infiltrative, verrucous plaques of the bilateral lower
extremities in association with a 20-pound weight loss. Biopsy reveals increased mucin
deposition in the dermis without increased cellularity. Which is the most likely associated
systemic disease:
A. Diabetes mellitus
B. Hepatitis C Infection
C. Hyperthyroidism
D. Chronic kidney disease
E. Castleman‟s disease
►C

199
Grave‟s disease is associated with pretibial myxedema. Pathology reveals diffuse increased
mucin in the dermis, without increased fibroblasts as seen in scleromyxedema and nephrogenic
systemic fibrosis. The other disorder are not associated with pretibial myxedema.

92
A 2 year old child is evaluated for suspected diagnosis of neurofibromatosis. Which of the
following diagnostic findings is typically absent on exam in this age group?
A. neurofibromas
B. macrocephaly
C. pigmented iris hamartomas
D. seizures
E. cafe au lait macules
►A

Cutaneous neurofibromas typically appear after puberty. The absence of neurofibromas in young
children does not rule out this diagnosis. Pigmented iris hamartomas are more likely finding in
young children than neurofibromas.
93
A 56-year-old female presents with longitudinal grooving of the nail plate of her index finger.
Examination reveals a 4mm spongy subcutaneous nodule just distal to the DIP joint. Biopsy
reveals a cyst with no true lining. What is the most common associated systemic disease:
A. Hypothyroidism
B. Bronchiolitis Obliterans Pneumonia (BOOP)
C. Hypertrophic Cardiomyopathy
D. Sporotrichosis
E. Osteoarthritis
►E

Digital myxoid cysts are composed of mucin contained in a cystic structure without a true lining.
They may have connections to the joint space, especially in patient‟s with osteoarthritis, who are
predisposed to developing myxoid cysts. Historically, tuberculosis infection has also been
associated with myxoid cysts but there is little current support for this relationship.

94
A patient that has congenital erythropoietic porphyria and has red urine with severe
photosensitivity. The patient also has redness, swelling, blistering in the sun-exposed areas with
resultant scarring. The defect is in the:
A. Uroporphyrinogen III synthase homozygous defect
B. Protoporphyrinogen oxidase deficiency
C. Decreased fecal coproporphyrin

200
D. Elevated urinary porphobilinogen
E. Coproporphyrinogen oxidase deficiency
►A

The defect in congenital erythropoietic porphyria is in the uroporphyrinogen III synthase


homozygous defect. It is present after birth with erythrodontia, hyperthrichosis, growth
retardation, hemolytic anemia, and increased bone features.

95
Each of the following demonstrates a vasculitis except:
A. Granuloma faciale
B. Henoch-Schoenlein purpura
C. Type 1 cryoglobulinemia
D. Wegener‟s granulomatosis
E. Leukocytoclastic vasculitis
►C

In type I cryoglobulinemia, monoclonal IgG or IgM cryoglobulins are found often in association
with lymphoma, leukemia, Waldenstrom‟s macroglobulinemia, or multiple myeloma. On
histopathology, type I cryoglobulinemia is characterized by the deposition of precipitated
amorphous cryoglobulins on the endothelium and throughout the vessel wall. The precipitates
stain with PAS stain. An inflammatory infiltrate is typically lacking in contrast to mixed
cryoglobulinemia (which shows a leukocytoclastic vascultitis)

96
A patient on hemodialysis presents with indurated plaques having a peau d‟orange texture on the
bilateral lower legs. Which of the following statements is TRUE?
A. The diagnostic histopathological findings include acanthosis and hyperkeratosis
B. Serum protein electrophoresis should be performed
C. Dapsone will likely be effective treatment
D. The face is usually is affected
E. The palms and soles are usually affected
►B

This patient likely has nephrogenic fibrosing dermopathy (NFD). NFD is an acquired, idiopathic
disorder that occurs in renal disease patients. It resembles scleroderma or eosinophilic fasciitis
clinically and scleromyxedema histopathologically. Large areas of indurated skin with fibrotic
nodules and plaques develop. The extremities are most commonly involved, followed by the
trunk. The face, palms and soles are almost never involved. Histopathologically, NFD displays a
proliferation of dermal fibroblasts and dendritic cells, thickened collagen bundles, increased

201
elastic fibers, and mucin deposition. Serum protein electrophoresis and immunoelectrophoresis
results are negative unlike scleromyxedema, and may be helpful to distinguish the two diseases.
NFD is usually a chronic, progressive condition, and favorable responses to medical intervention
are anecdotal.

97
All of the following are true regarding incontinentia pigmenti EXCEPT:
A. It is caused by a mutation in the NEMO gene
B. It is an X-linked recessive disorder
C. Inflammation and blistering may be followed by hyperkeratotic, verrucous
lesions
D. It is associated with cerebellar ataxia
E. It is associated with coloboma and retinal detachment
►B

Incontinentia pigmenti is an X-linked dominant disorder caused by a mutation in the NEMO


gene. Females only present at birth with linear lesions of inflammation and blistering (stage 1),
followed by hyperkeratotic verrucous areas (stage 2), hyperpigmentation (stage 3), and then
atrophy (stage 4). Systemic findings include psychomotor retardation, microcephaly, seizures,
cerebellar ataxia, coloboma, and retinal detachment.

98
Hepatitis C infection is associated with:
A. Pityriasi rubra pilaris
B. Mixed cryoglobulinemia
C. Pityriasis rosea
D. Kaposi's sarcoma
E. Oral hairy leukoplakia
►B

Cutaneous manifestation associated with hepatitis C include necrolytic acral erythema, porphyria
cutanea tarda, lichen planus, polyarteritis nodosa and mixed cryoglobulinemia. Mixed
cryoglobulinemias is a systemic vasculitis with variable manifestations including palpable
purpura, arthralgias and weakness.

99
What is the unique laboratory finding in stool in porphyria cutanea tarda (PCT) and
hepatoerythropoietic porphyria (HEP)?
A. elevated coproporphyrins
B. elevated isocoproporphyrins

202
C. decreased isocoproporphyrins
D. elevated protoporphyrins
E. elevated uroporphyrins
►B

PCT and HEP are associated with elevated isocoproporphyrins in the stool. Congenital
erythropoietic porphyria, hereditary coproporphyria is associated with increased coproporphyria
in the stool, and elevated stool proporphyria is seen in variegate porphyria.

100
A 12 year old boy is referred for evaluation of a rash in his groin. It has become progressively
worse over the preceding three years. Discrete erythematous papules are seen on the lower
anterior abdomen, upper thighs, and buttocks. Biopsy of a representative papule reveals an
angiokeratoma. An evaluation by a nephrologist reveals proteinura. A diagnosis of Fabry\'s
disease is made. Renal failure in this syndrome is related to accumulation of what substance in
the kidney?
A. Ceramide trihexoside
B. Galabiosylceramide
C. Hyaluornic acid
D. Mucopolysaccharide
E. Alpha-galactosidase
►A

Fabry's disease is an x-linked recessive disorder characterized by angiokeratomas in a bathing


trunk distribution, hypohidrosis, corneal opacities, and acral paresthesias. The diagnosis is
confirmed by decreased levels of alph-galactosidase in white blood cells, serum, and fibroblasts.
Renal failure is due to accumulation of ceramide trihexoside aka Globotriaosylceramide.

101
All of the following are true regarding the condition pictured EXCEPT:
A. Men with this condition may be at increased risk for lymphoma
B. Associated calcinosis cutis may be seen in pediatric patients
C. Age-appropriate cancer screening is recommended
D. May be associated with a psoriasiform scalp dermatitis
E. Elevated aldolase is more specific than CPK
►E

All of the statements regarding dermatomyositis are true except for statement E. Elevated muscle
enzymes are found in dermatomyositis, but elevated CPK is more specific than aldolase.

203
Diagnosis is also established with muscle biopsy showing evidence of inflammation and
abnormal EMG.

102
In patients with mixed cryoglobulinema associated with hepatitis C, the most likely laboratory
abnormality is:
A. Elevated rheumatoid factor
B. + ANA
C. + p-ANCA
D. Decreased cryoglobulins
E. Elevated hematocrit
►A

In patients with mixed cryoglobulinemia, the most likely laboratory abnormality among the
options listed is an elevated rheumatoid factor.

103
Regarding eruptive xanthomata, which of the following is TRUE?
A. They are associated with type II and III hyperlipidemias
B. They are associated with calcium channel blockers
C. There is no association with ethanol consumption
D. They are associated with type I, IV, and V hyperlipidemias
E. They favor the flexor surfaces of the extremities
►D

Eruptive xanthomata appear as erythematous to yellow papule, from1 to 4mm in diameter,


generally distributed over the extensor arms, hands, and buttocks. They are present in response to
either primary or secondary hypertriglyceridemia. Primary causes are discussed below.
Secondary causes include obesity, diabetes, excessive alcohol consumption, estrogens, and
systemic retinoids. Type I hyperlipidemia is caused by lipoprotein lipase deficiency and patients
have elevated levels of triglycerides (TG) and chylomicrons (chylo). Type I may be associated
with eruptive xanthomata, lipemia retinalis, abdominal pain, pancreatitis, and
hepatosplenomegaly. Type IIA is heterozygous for apolipoprotein B deficiency, and results in
increased low-density lipoproteins (LDL). Patients may show tendinous or tuberous xanthomata
and are at high risk for coronary artery disease (CAD) and stroke. Type IIB is homozygous for
apolipoprotein B deficiency and patients have elevated LDL, very low-density lipoprotein
(VLDL) and TG. Intertriginous and tuberous xanthomata in addition to advanced atherosclerosis,
CAD and stroke at an early age are associated with type IIB. Type III is caused by apolipoprotein
E deficiency resulting in elevated intermediate-density lipoproteins (IDL) and TG, and displays
palmar/plantar, tendinous and tuberous xanthomata. This disease is associated with diabetes,

204
gout, CAD and stroke. Type IV disease is characterized by increased TG and VLDL and patients
may have eruptive xanthomata along with CAD, diabetes and stroke. Type V is caused by an
apolipoprotein C2 defect resulting in increased TG, chylo, and VLDL. Eruptive xanthomata,
diabetes, hepatosplenomegaly, lipemia retinalis and pancreatitis are associated with type V.

104
All of the following statements regarding cryoglobulinemia are true EXCEPT:
A. Type I is composed of monoclonal IgG and polyclonal IgM
B. Type II is composed of polyclonal IgG and monoclonal IgM
C. Type III is composed of polyclonal IgG and polyclonal IgM
D. 80% of cases of mixed cryoglobulinemia are associated with Hepatitis C
infection
E. None of these answers are correct (all statements are true)
►A

Type I is composed of monoclonal immunoglobulins.

105
Which of the following statements regarding multiple endocrine neoplasia syndromes is true?
A. MEN Type IIB is also known as Sipple's Syndrome
B. MEN Type IIA is also known as Wermer's Syndrome
C. Multiple mucosal neuromas are seen in association with MEN Type IIA
D. Lichen or macular amyloidosis is seen in association with MEN Type IIA
E. Patients with MEN Type IIB are at increased risk for developing follicular
thyroid carcinoma
►D

MEN Type I is also known as Wermer's Syndrome. MEN Type IIA is also known as Sipple's
Syndrome. Mucosal neuromas are seen in association with MEN Type IIB. Statement D is
correct. Patients with MEN Type IIB are at increased risk for developing medullary thyroid
carcinoma.

106
A 64-year-old woman presents with lichenified keratotic plaques on the bilateral lower
extremities. Biopsy reveals suppurative inflammation and collagen extending from the dermis
through the epidermis. Which is the most likely associated systemic disease:
A. Congestive Heart Failure
B. Hepatitis C Infection
C. Hypothyroidism
D. Chronic kidney disease

205
E. Castleman‟s disease
►D

Chronic kidney disease is associated with perforating disorders. These disorder are characterized
by pruritus and a biopsy revealing collagen and inflammatory cells broaching the epidermis. The
other disorders are not closely associated with perforating diseases.

107
A patient presents with episodic flushing of the face and neck, abdominal pain, wheezing, cough,
and diarrhea. Carcinoid syndrome is suspected. Which of the following tests would confirm the
diagnosis?
A. Change of urine to green color upon addition of nitrosonaphthol
B. Elevated urinary 5-hydroxyindolacetic acid
C. Elevated 24-hour urine norepinephrine
D. Decreased 24-hour urine vanilylmandelic acid
E. Elevated serum tryptase
►B

The urine in a patient with carcinoid syndrome features high levels of 5-hydroxyindolacetic acid
and changes color to purple upon addition of nitrosonaphthol. Elevated 24-hour urine
norepinephrine or vanilylmandelic acid are seen in pheochromocytoma. Mastocytosis features
high levels of tryptase.

108
What is the most common malignancy associated with this condition at this location?
A. Breast cancer
B. Gastric cancer
C. Thyroid cancer
D. Pancreatic cancer
E. Melanoma
►B

Malignant acanthosis nigricans usually presents with sudden onset and is rapidly progressive. It
may be associated with diffuse keratodermas of the palms and soles or eruptive seborrheic
keratoses.

109
This autosomal dominant condition is characterized by trichodiscomas, fibroepithelial polyps,
and fibrofolliculomas:
A. POEMS syndrome

206
B. Rombo syndrome
C. Birt-Hogg-Dube syndrome
D. Proteus syndrome
E. Cowden's disease
►C

Birt-Hogg-Dube is an autosomal dominant condition characterized by trichodiscomas,


fibroepilethial polyps and fibrofolliculomas.

110
Bilateral diagonal earlobe creases may be associated with which of the following systemic
diseases?
A. Gout
B. Chronic renal insufficiency
C. Interstitial pulmonary fibrosis
D. Hepatobiliary cirrhosis
E. Atherosclerotic coronary artery disease
►E

Several studies have indicated that bilateral diagonal earlobe creases (known as “Frank‟s sign”)
may serve as a clinical marker for underlying coronary artery disease independent of age,
particularly in persons younger than 40 years old.

111
A 58 year old female with lifelong Type 1 diabetes mellitus has end-stage renal failure. What is
the most common cutaneous manifestation of end stage renal disease?
A. Pruritus
B. Nephrogenic systemic fibrosis
C. Hyperpigmentation
D. Acquired perforating dermatosis
E. Skin pallor
►A

There are may cutaneous manifestations of end-stage renal disease. Chronic anemia can cause
skin pallor. Deposition of carotenoids can give the skin a yellowish hue. Photo-distributed
hyperpigmentation and ecchymoses are also commonly seen. The most common cutaneous
manifestation, however, is pruritus. Other cutaneous findings include metastatic calcification,
acquired perforating dermatosis, and nephrogenic systemic fibrosis.

112

207
One might see all of the following laboratory and clinical abnormalities in cryoglobulinemia
associated with hepatitis C virus infection EXCEPT:
A. Elevated liver function tests
B. Positive rheumatoid factor
C. Elevated C3 levels
D. Acrocyanosis
E. Urticarial plaques
►C

Laboratory abnormalities of HCV include an elevation of liver enzymes, positive rheumatoid


factor (70-90%), and DEPRESSED C3 levels. Classical clinical presentation includes palpable
purpura, arthralgias, and glomerulonephritis. Livedo reticularis, hemorrhagic bullae,
acrocyanosis, and urticarial plaques can also be found.

113
When metastases to the skin occur from a thyroid malignancy, they are usually due to:
A. Medullary carcinoma
B. Papillary adenocarcinoma
C. Follicular carcinoma
D. Anaplastic carcinoma
E. Cutaneous metastases have not been reported in association with thyroid
malignancies
►B

Metastases to the skin from a thyroid malignancy are rare, but most reported cases occur with
papillary adenocarcinoma.

114
The presence of antibodies to c-ANCA is characteristically seen in patients with which disease?
A. Ulverative colitis
B. Churg-Strauss disease
C. Polyarteritis nodosa
D. Wegener”s granulornatosis
E. Nodular vasculitis
►D

Wegener”s granulomatosis is a necrotizing granulomatous disorder that most commonly affects


the upper and lower respiratory tracts, kidneys, and eye. Cytoplasmic pattern antineutrophil
cytoplasmic autoantibody (anti -proteinase-3) is often positive in this disease. Churg-Strauss is
associated with a positive p-ANCA.

208
115
Which type of porphyria is the autosomal recessive form of porphyria cutanea tarda?
A. Erythropoietic protoporphyria (EPP)
B. Congenital erythropoietic porphyria (CEP)
C. Hepatoerythropoietic porphyria (HEP)
D. Variegate Porphyria (VP)
E. Acute Intermitent Porphyria (AIP)
►C

HEP results from deficient, but not absent, activity of uroporphyrinogen decarboxylase (UROD).
HEP is the recessive form of familial PCT. It manifests during infancy or early childhood as
photosensitivity, skin fragility in sun-exposed areas, pink urine, erythrodontia, and
hypertrichosis.

116
A 70-year old male develops hyperpigmented velvety plaques on his lips, dorsal hands and feet,
and in his axilla. What underlying malignancy is most commonly associated with this finding?
A. Gastric Carcinoma
B. Lung Carcinoma
C. Ovarian Carcinoma
D. Lymphoma
E. Breast Carcinoma
►A

Acanthosis Nigricans can be associated with obesity, insulin resistance, Crouzon”s syndrome,
congenital lipodystrophy, and internal malignancy. 90% of related malignancies are tumors
within the abdominal cavity. Adenocarcinoma of the stomach is the most common. Malignancy
related acanthosis nigricans often occurs in the setting of weight loss, helping distinguish it from
other associated diseases.

117
All of the following are true regarding calciphylaxis except:
A. Seen in end-stage renal disease
B. Vascular mural calcification occurs late in the process
C. High morality rate is due to sepsis
D. Association with hyperparathyroidism
E. Association with an elevated calcium: phosphate product
►B

209
Calciphylaxis is a rare, life threatening disorder that is associated with end stage renal disease.
Clinically, patients develop a livedoid purpura and enlarging, tender, indurated subcutaneous
plaque typically on the legs or abdomen. These plaques are subject to ischemia, skin necrosis,
and secondary infection, sepsis, and death. Histologic examination of calciphylaxis is
characterized by the triad of small vessel mural calcification, extravascular calcification, and
vascular thrombosis. It appears that vascular mural calcification is an early and essential process
in the development of calciphylaxis.

118
A complication seen in women of childbearing age with the condition in the figure is:
A. Atrial septal defect
B. Pulmonary lymphangioleiomyoma
C. Gastrointestinal bleeding
D. Lisch nodules
E. Pulmonary emboli
►B

This picture demonstrates a shagreen patch in a patient with Tuberous sclerosis. A complication
occurring in female patients of child-bearing age is pulmonary lymphangioleiomyoma.

119
Porphyria cutanea tarda may be associated with all of the following except:
A. Hepatitis C virus infection
B. Alcohol
C. Estrogens
D. Polyhalogenated hydrocarbons
E. Inherited deficiency of uroporphyrinogen III synthase
►E

A homozygous defect in uroporphyrinogen III synthase in a cause of congenital erythropoietic


porphyria, not porphyria cutanea tarda (PCT). PCT may be caused by sporadic or familial
deficiency in uroporphyrinogen decarboxylase, or by any of the other causes listed above.

120
Which of the following is true regarding nephrogenic fibrosing dermopathy?
A. Has a rapidly progressive but reversible course
B. Is associated with a paraproteinemia
C. Is associated with peripheral eosinophilia
D. May be associated with antiphospholipid antibodies
E. Is associated with a dramatic increase in dermal mucin

210
►D

Nephrogenic fibrosing dermopathy has an indolent course and treatment is usually not
satisfactory. It is not associated with a paraproteinemia or peripheral eosinophilia. Some patients
have been reported to have antiphospholipid antibodies. Histopathology demonstrates a minimal
to slight increase in dermal mucin.

121
A patient is diagnosed with acute intermittent porphyria with periodic attacks of colic, paralyses
and psychiatric disorders. All of the following can precipitate attacks except:
A. Barbiturates
B. Estrogen
C. Griseofulvin
D. Sulfonamides
E. Tetracyclines
►E

Patients with acute intermittent porphyria have a deficiency in porphobilinogen deaminase.


Attacks are precipitated by barbiturates, estrogen, griseofulvin, and sulfonamides. Only 10% of
those with genetic defect develop the disease but all at risk for primary liver cancer.

122
A 50-year old woman with a history of spontaneous pneumothorax develops multiple firm, skin
colored lesions on her face and neck over a period of several years. This patient should have
periodic surveillance for the development of:
A. Renal Cell Carcinoma
B. Gastric Carcinoma
C. Breast Carcinoma
D. Ovarian Carcinoma
E. Lung Carcinoma
►A

This patient has Birt-Hogg-Dube syndrome, an autosomal dominant defect of the BHD gene that
encodes folliculin. This syndrome is characterized by the development of trichodiscomas,
fibrofolliculomas, and acrochordons starting around age 30. The patients are also at risk for the
development of spontaneous pneumothorax and renal cell carcinoma.

123
The organism that causes this infection shown in the figure is:
A. T. tonsuran

211
B. C. immitis
C. C. albicans
D. P. aeruginoas
E. T. verrucosum
►C

The organism that causes erosio interdigitalis blastomycetica is candida albicans.

124
What is the most common primary site of a carcinoid tumor?
A. Liver
B. Stomach
C. Appendix
D. Duodenum
E. Ileum
►E

The most common site of carcinoid tumor is the ileum. Clinical symptoms of episodic flushing,
abdominal pain, diarrhea, wheezing, and a pellagra-like dermatosis (due to shunting of
tryptophan to serotonin) etc present with metastases to the liver. In a report from the SEER
database of 11,427 carcinoid cases treated between 1973 and 1997, the majority were located in
the gastrointestinal (GI) tract (55 percent) and bronchopulmonary system (30 percent). Within
the GI tract, most carcinoids arose in the small intestine (45 percent, most commonly in the
ileum), followed by rectum (20 percent), appendix (16 percent), colon (11 percent), and stomach
(7 percent)

125
Which of the following is more commonly associated with ulcerative colitis as compared to
Crohn‟s disease?
A. Pyoderma gangrenosum
B. Oral lesions
C. Polyarteritis nodosa
D. Fistulae
E. Metastatic lesions
►A

Crohn‟s disease (CD) and ulcerative colitis (UC) may both have gastrointestinal tract (GIT) and
cutaneous manifestations. CD lesions can involve any area of the GIT (from the lips to the anus),
with patchy disease normally present, whereas UC is limited to the colon and rectum and tends
to be continuous. Oral disease, fissures and fistulae, metastatic disease to the skin, and

212
polyarteritis nodosa are more commonly associated with CD. Conversely, erythema nodosum,
pyoderma gangrenosum and pyoderma vegetans tend to be more commonly associated with UC.

126
A 63-year-old male develops small, non-tender, violaceous papules on his dorsal hands, face,
ears, and trunk. The patient also develops a destructive arthropathy with finger deformities. What
percentage of patients with this condition has an associated malignancy?
A. 20-25%
B. 5-10%
C. 65-70%
D. 50-55%
E. 85-90%
►A

This patient has developed multicentric reticulohistiocytosis. Histopathology of the skin lesions
will display multinucleated oncocytic giant cells with eosinophilic ground glass cytoplasm. Half
of these patients will develop a mutilating arthritis. 20-25% of patients with multicentric
reticulohistiocytosis will develop an associated malignancy. The malignancy is usually a
carcinoma, but there is no predominant type.

127
A young African American patient presents with anemia and spontaneously appearing leg ulcers
over both lateral and medial malleoli. The most likely diagnosis is:
A. Atherosclerotic disease
B. Sickle cell anemia
C. Factitial dermatitis
D. Lupus erythematosus
E. Trauma
►B

Sickle cell anemia should be considered most likely in any young African American with
spontaneous leg ulcers. The ulcers are more common in people with severe anemia.

128
A 6-year-old girl presents with immediate burning of his skin on sun exposure and is ultimately
diagnosed with Erythropoietic Protoporphyria (EPP). Which internal organ is most likely to be
affected?
A. Heart
B. Lungs
C. Liver

213
D. Kidneys
E. Pancreas
►C

Erythropoietic Protoporphyria (EPP) is characterized by a burning sensation experienced


immediately upon photoexposure. Biopsy reveals PAS-positive hyalinization of the superficial
dermal vessel walls. This disorder is due to ferrochelatase deficiency. Excess porphyrins are
deposited in the liver, which may become cirrhotic.

214
Chapter -11-
Disorders of the Hair and Nails
1
A 34-year-old man undergoing chemotherapy for acute myelogenous leukemia presents with
smooth white horizontal bands involving the nail plates of all digits. What is the most likely
disorder:
A. Beau‟s lines
B. Mee‟s lines
C. Muehrcke‟s lines
D. Onychophagia
E. Dolichonychia
►B

Beau‟s lines are transverse indented nail plate furrows caused by temporary growth arrest of the
nail matrix, often due to chemotherapy or other stressful events and illnesses. Mee‟s lines are
transverse white lines affecting all nails and growing out with the nail; there is no associated
indentation. Mee‟s lines may be caused by chemotherapy, arsenic poisoning, rheumatic fever, or
other systemic diseases. Muehrcke‟s lines disappear with squeezing of the nail and are due to
disorders of low albumin. Oncyhophagia refers to nail biting. Dolichonychia exhibits long,
slender nails, and is associated with Ehlers-Danlos and Marfans.

2
The distal nail matrix is visible at the following portion of the nail unit:
A. Onychodermal band
B. Hyponychium
C. Lunula
D. Cuticle
E. Lateral nailfold
►C

The distal nail matrix is located at the lunula, which is visible as the half-moon structure at the
base of the nail. The proximal matrix is located under the proximal nailfold. Damage to the
proximal nail matrix produces defects at the dorsal nailplate, and damage to the distal nail matrix
produces produces defects at the ventral nail plate.

3
The "ragged cuticle" seen in dermatomyositis is also known as:
A. Samitz sign
B. Ventral pterygium

215
C. Plummers sign
D. Candy-cane nails
E. Emperipolesis
►A

Cuticular fraying from dermatomyositis is called the Samitz sign or "ragged cuticle". The other
options are not associated with dermatomyositis. A ventral pterygium can be seen in systemic
sclerosis. There is no such sign as Plummers sign. Candy-cane nails are seen in Darier's disease.
Emperipolesis is seen in Rosai-Dorfman disease and is not a nail finding.

4
What is the common cause of the “one hand-two-foot” syndrome?
A. Trichophyton rubrum
B. Trichophyton mentagrophytes ( var. Interdigitale)
C. Candida albicans
D. Scytalidium dimidiatum
E. Scopulariopsis brevicaulis
►A

This condition is due to T. rubrum in 90% of the cases , T. interdigitale and Scytalidium
dimidiatum share the 10% left.

5
Pseudo-pili annulati with light reflection off periodic flattening of the hair shaft is seen in:
A. Blond hair
B. Red hair
C. Brown hair
D. Black hair
E. White hair
►A

Pseudo pili annulati is seen in blond hair. Pili annulati is the alternating of light and dark bands
seen in normal light as opposed to tiger tail hair in thrichothiodystrophy seen in polarized light.

6
The term used to describe the thinning of hair shaft forming constrictions along hair with
disruption at the anagen cycle is:
A. Pohl Pinkus
B. Trichorrhexis invaginata
C. Trichoschisis

216
D. Monilethrix
E. Pili annulati
►A

Pohl pinkus constriction is due to a toxic insult. This causes the disruption of the anagen cycle
with thinning of hair shaft forming constrictions along hair. This is analogous to Beau's lines in
nails.

7
Mee's lines are:
A. Double white transverse lines from abnormal vascular bed
B. Brownish macules beneath the nail plate
C. Local or diffuse hyperkeratotic tissue that develops on the lateral or proximal
nail folds
D. Transverse white lines that affect all nails, grow out with nail growth
E. Vertical black lines on a single or multiple nails
►D

Mee's lines are transverse white lines that affect all nails, grow out with nail growth. They are
seen in arsenic poisoning, rheumatic fever, congenital heart failure, leprosy and with significant
systemic disease. Brownish macules beneath the nail plate are oil spots, seen in psoriasis.
Onychophosis is the local or diffuse hyperkeratotic tissue that develops on the lateral or proximal
nail folds. The double white transverse lines from abnormal vascular bed are Muehrcke's lines
which are caused by a nephrotic syndrome, low albumin, liver disease or malnutrition.

8
A patient with sparse hair, a pear-shaped broad nose and cone-shaped epiphyses likely suffers
which of the following conditions?
A. Focal Dermal Hypoplasia (Goltz syndrome)
B. Tricho-rhino-phalangeal Syndrome
C. Hay-Wells Syndrome
D. Ectrodactyly-Ectodermal dysplasia-Clefting (EEC)
E. Trichodentoosseous Syndrome
►B

Tricho-rhino-phalangeal Syndrome is characterized by sparse hair, a pear-shaped broad nose and


cone-shaped epiphyses.

217
The is a term used to describe the thin epithelium immediately beneath the nail plate with an
absent granular layer:
A. Nail bed
B. Hyponychium
C. Nail matrix
D. Proximal nail fold
E. Nail plate
►A

The nail bed is responsible for the thin epithelium immediate beneath the nail plate and does not
have a granular layer.

10
This hair disorder and mutations in ATP7A (MNK) gene are found in what genodermatosis?
A. Bjornstad syndrome
B. Menke's kinky hair syndrome
C. Crandall's syndrome
D. Bazex's follicular atrophoderma
E. Trichothiodystrophy
►B

Menke's kinky hair syndrome is characterized by pili torti and mutations in APT7A (MNK)
gene. Bjornstad syndrome, Crandall‟s syndrome and Bazex‟s follicular atrophoderma are
associated with pili torti but are not associated with this mutation.

11
Fraying of the cuticles is a sign of which of the following conditions?
A. Dermatomyositis
B. Systemic sclerosis
C. Lupus erythematosus
D. Hypothyroidism
E. Lead poisoning
►A

Dermatomyositis can be associated with fraying of the cuticles (Samitz sign).

12
Which part of the matrix gives rise to the dorsal nail plate?
A. Distal matrix
B. Proximal matrix

218
C. Ventral matrix
D. Dorsal matrix
E. Lateral matrix
►B

The proximal matrix gives rise to the dorsal nail plate, while the distal matrix gives rise to the
ventral nail plate.

13
A 36-year-old woman presents with arthralgias and three fingernails exhibiting adherence of the
ventral surface of the distal nail plate to the hyponychium. What is the most likely associated
disease:
A. Psoriasis
B. Lichen Planus
C. Systemic Sclerosis
D. Yellow Nail Syndrome
E. Multicentric Reticulohistiocytosis
►C

The most common cause of ventral pterygium is systemic sclerosis, which often first presents
with arthralgias. Lichen planus is most commonly associated with dorsal, rather than ventral,
pterygium. Multicentric reticulohistiocytosis is associated with deforming arthritis and “coral
bead” nodules around the proximal nail fold, but not with pterygium.

14
A 52-year-old man presents to clinic with erythematous papules and pustules in the region of the
cheeks, chin, and neck that are worsened by shaving. Examination reveals pseudofolliculitis
barbae. With what disease may this be associated:
A. Cirrhosis
B. Heart Failure
C. Chronic Renal Failure
D. Myeloma
E. Emphysema
►C

Pseudofolliculitis barbae has been associated with renal failure requiring hemodialysis,
particularly in Caucasian patients. Most commonly, this syndrome arises in patients with darker
skin types, whose hair shafts are elliptical in cross section, rather than round. This elliptical
nature predisposes the hairs to kinking and invagination into the skin.

219
15
The syndrome including mental and physical retardation, convulsions, episodic unconsciousness,
liver enlargement, skin lesions, and dry and brittle hair showing trichorrhexis nodosa
microscopically and fluorescing red. is:
A. Bazex Follicular Atrophoderma
B. Crandall's syndrome
C. Citrullinemia
D. Argininosuccinic aciduria
E. Bjornstad's syndrome
►D

Argininosuccinic aciduria has the features listed including trichorrhexis nodosa and red
fluorescence of the hair. Bazex's follicular atrophoderma and Crandall syndrome both have
findings of pili torti and citrullinemia has trichorrhexis nodosa. None of the other options have
red fluorescence as a feature.

16
A commonly known cause of splinter hemorrhages in the nail is endocarditis. Which of the
following would be unlikely to cause splinter hemorrhages?
A. Trichinosis
B. Trauma
C. Psoriasis
D. Vasculitis
E. Rheumatoid arthritis
►E

Rheumatoid arthritis is not a cause of splinter hemorrhages of the nail. Endocarditis is the most
commonly thought of cause of splinter hemorrhages. Trichinosis, trauma, psoriasis and vasculitis
also can be causes of this also and should be considered when splinter hemorrhages are seen.

17
Papular atrichia is caused by mutations in which gene?
A. Wingless
B. Patched
C. Hairless
D. Fox forehead
E. Distal-less homebobox
►C

Hairless gene mutations cause papular atrichia.

220
18
Ventral pterygium pictured below is characteristically associated with what disorder?
A. Lichen planus
B. Systemic sclerosis
C. Cicatricial pemphigoid
D. SJS/TEN
E. Chronic GVHD
►B

Ventral pterygium is most characteristically associated with systemic sclerosis. All other choices
are associated with dorsal pterygium.

19
A patient with end stage renal disease and on dialysis has an opaque white appearance of half of
the nail. This is called:
A. Lindsay's nails
B. Terry's nails
C. Birt Hogg nails
D. Riley's nails
E. Hutchinson's nails
►A

This finding associated with end stage renal disease where half of the nail is opaque is called
Lindsay's nail. Patients with cirrhosis or hypoalbuminemia that have whitening of the proximal
two third of the nail with a 1/3 that is red is called Terry's nails.

20
A patient is diagnosed having Menke's kinky's hair syndrome with a deficiency in:
A. ATP7A gene
B. ATP2A2 gene
C. Argininosuccinic aciduria
D. BCS1L gene
E. ABCC2A gene
►A

Patients with Menke's Kinky Hair syndrome have a X-linked recessive disorder in the ATP7A
gene. This gene is responsible for the copper translocating membrane ATPase. They have brittle
"steel wool" hair with a cupid's bow lips, pale and lax skin.

221
21
Primary total dystrophic onychomycosis is due to:
A. Candida spp
B. Fusarium
C. Trichophyton rubrum
D. Scytalidium dimidiatum
E. Scopulariopsis brevicaulis
►A

The main characteristic of this immunologic disorder the simultaneous involvement of all the
tissues of the nail unit.

22
Congenital generalized hypertrichosis is associated with which of the following symptoms?
A. Androgen-secreting ovarian tumors
B. Pituitary insufficiency
C. Distichiasis
D. Methimazole teratogenicity
E. Gingival fibromatosis
►E

Gingival fibromatosis. Congenital Generalized Hypertrichosis with Gingival Fibromatosis is a


genetic condition with hypertrichosis on the face and upper body and gingival hyperplasia and
fibromatosis.

23
A 4 year-old boy has a single circular alopecic atrophic patch on the vertex of the scalp. This
most likely is due to which of the following conditions?
A. Aplasia cutis congenita
B. Congenital syphilis
C. Keratosis Follicularis Spinulosa Decalvans
D. Morphea en coup de sabre
E. Triangular Alopecia
►A

Aplasia cutis congenita often affects the vertex of the scalp and heals with a circular atrophic
alopecic scar.

24

222
A patient with curly hair that straightened after puberty, enamel hypoplasia, dental pits, and
increased bone density likely has a mutation in which of the following genes?
A. Distal-less homeobox-3 gene (DLX3)
B. Vascular-endothelial growth factor receptor 3
C. Bone morphogenetic protein type 2
D. SLURP 1
E. SPINK 5
►A

Distal-less homeobox-3 gene (DLX3) mutations cause trichodentoosseous syndrome, which is


characterized by curly hair that straightens in the 2nd-3rd decades, enamel hypoplasia, dental
pits, and increased bone density.

25
A rapid onset of hair growth with or without accompanying virilization can occur in all of the
following conditions EXCEPT:
A. Adrenal adenoma
B. Adrenal carcinoma
C. Arrhenoblastoma
D. Krukenburg tumor of the ovary
E. Ovarian cyst
►E

All of these options except an ovarian cyst can cause a rapid onset of hair growth with or without
accompanying virilization. An Arrhenoblastoma is a tumor of the ovary that secretes
testosterone.

26
Disorders associated with cortisol excess can cause hirsutism. All of the following clinical
findings are associated with cortisol excess except:
A. Plethora
B. Centripetal obesity
C. Striae
D. Dorsocerical/supraclavicular fat pads
E. Enlarged cystic ovaries
►E

Plethora, centripetal obesity, striae and dorsocervical/supraclavicular fat pads are associated with
signs of cortisol excess. Patients that have enlarged cystic ovaries are associated with polycystic
ovarian disease.

223
27
A 24 year-old over-weight woman presents with the complaint of hirsutism and asks about
therapy. She has a virilization pattern of hair growth. On questioning, she relates having irregular
menstrual periods and acne. Which of the following is the most likely diagnosis?
A. Pheochromocytoma
B. Polycystic ovarian disease
C. Congenital adrenal hyperplasia
D. Ovarian carcinoma
E. Normal variant
►B

This patients diagnosis is most reasonably Polycystic Ovarian disease and includes the features
listed above in the question in addition to enlarged cystic ovaries, obesity and amenorrhea.
Congenital adrenal hyperplasia can present with hirsutism and menstrual abnormalities, but
usually not obesity. Ovarian carcinoma, pheochromocytoma and normal variant are not likely
causes of this presentation.

28
This characteristic nail finding is seen in what disorder?
A. Lichen planus
B. Pachyonychia congenita
C. Darier's disease
D. COIF syndrome
E. Coffin-Siris syndrome
►C

Alternating red and white longitudinal bands on the nail are characteristic of Darier’s
disease. Other nail findings include subungual hyperkeratosis and distal v-shaped nicking.

29
Which of the following is not a feature of Bazex-Dupre-Christol Syndrome?
A. peripheral vasodilation with cyanosis
B. BCC
C. hypohidrosis
D. milia
E. epidermoid cysts
►A

224
Peripheral vasodilation with cyanosis is not found in BDC Syndrome. This is a feature of Rombo
Syndrom. Bazex-Dupre-Christol Syndrome includes BCC, follicular atrophoderma,
hypohidrosis, milia, epidermoid cysts, and hypotrichosis.

30
A 6-year old girl with atopic dermatitis presents with patchy non-scarring alopecia and nail
pitting. You make the diagnosis of alopecia areata. What portion of the nail unit is responsible
for her nail plate pitting:
A. Hyponychium
B. Proximal nail plate
C. Proximal nail matrix
D. Distal nail matrix
E. Eponychium
►C

The proximal nail matrix produce the dorsal nail plate. Defective keratinization in this region is
therefore responsible for the pitting seen in alopecia areata, psoriasis, eczema, and other
cutaneous diseases. The distal nail matrix produces the ventral nail plate and is visible as the
lunula.

31
Which of the following syndromes does NOT have the finding shown in the image as part of the
spectrum?
A. Crandall
B. Argininosuccinic aciduria
C. Citrullinemia
D. Menke's Kinky Hair
E. Netherton
►A

Crandall syndrome has pili torti as its hair finding. The other options all have trichorrhexis
nodosa as one of the hair findings in the syndrome. Trichorrhexis nodosa is the fraying of hair
ends so that the resemble a "broom-stick" or "paint brush".

32
Transverse white bands on one or two nails is caused by:
A. Trauma to the matrix
B. Alopecia areata
C. Psoriasis
D. Systemic lupus erythematosis

225
E. Lichen planus
►A

Longitudinal leukonychia or transverse white bands are caused by trauma to the nail matrix. AA,
psoriasis, SLE and LP all cause spotted red lunulae, not transverse white bands.

33
What condition most likely describes a hair defect with ruffled cuticle in 4-year old female with
blond hair?
A. Pili torti
B. Pili annulati
C. Wooly hair
D. Loose anagen hair syndrome
E. Monilethrix
►D

Loose anagen hair syndrom occurs primarily in young, females with blond hair. The defect
results in early keratinization of the inner root sheath which leads to poor anchoring.
Examination of hair shaft reveals ruffled proximal cuticle.

34
A 24-year-old man reports that he has developed white patches of scalp hair that seem to migrate
across his scalp. Examination reveals three 1cm foci of poliosis with surrounding normally
pigmented hair. He has a history of hypothyroidism, but no other systemic diseases. What is the
most likely etiology:
A. Form fruste of alopecia areata
B. Vogt-Koyanagi-Harada syndrome
C. Tinea capitis
D. Allezandrini syndrome
E. Lichen planopilaris
►A

Forme fruste, or limited, alopecia areata, may present with migratory poliosis due to destruction
of follicular melanocytes but survival of the hair-producing bulb. Vogt-Koyanagi-Harada
syndrome presents as meningoencephalitis followed by visual and hearing deficits and poliosis
of the periorbital region. The other answer choices are not typically associated with migratory
poliosis.

35
The most common cause for superficial type onychomycosis is:

226
A. Trichophyton rubrum
B. Trichophyton mentagrophytes
C. Epidermophyton floccosum
D. Fusarium oxysporum
E. Scopulariopsis brevicalis
►B

T. mentagrophytes is the most common cause of superficial type onychomycosis. Other causes
include: Fusarium, Acremonium and Aspergillus spp.

36
Terry's nails are seen in which of the following conditions?
A. Renal failure
B. Pulmonary fibrosis
C. Diabetes
D. Bladder carcinoma
E. Aerodigestive carcinoma
►C

Terry's nails are described as all but the distal 2mm of each nail evenly white due to a defect in
the nail bed. It can be seen in cirrhosis, congestive heart failure and diabetes.

37
Which of the following nutritional deficiencies is associated with perifollicular hemorrhage?
A. Vitamin C
B. Niacin
C. Vitamin B12
D. Folate
E. Zinc
►A

Vitamin C deficiency, or scurvy, may lead to perifollicular hemorrhage. Niacin deficiency may
result in a photo-distributed eruption, diarrhea, and psychiatric symptoms (diarrhea, dermatitis,
and dementia). Vitamin B12 may lead to glossitis with a shiny, painful tongue. Zinc deficiency
may result in a periorificial dermatitis.

38
The visible portion of the nail matrix is called the:
A. Eponychium
B. Lunula

227
C. Cuticle
D. Hyponychium
E. Nail plate
►B

The nail unit is comprised of six major components including the nail matrix, nail plate, cuticle,
nail bed, anchoring portion, and the framing portion (proximal/lateral/distal folds). The lunula is
a component of the distal matrix. It is grayish white because of the nature of the keratinization of
its epithelium.

39
Endonyx onychomycosis is due to:
A. Trichophyton rubrum
B. Trichophyton mentagrophytes
C. Trichophyton soudanense
D. Scytalidium dimidiatum
E. Scopuliaropsis
►C

Endonyx onychomycosis may also be observed with T. violaceum. This is the equivalent of
endothrix infection of the hair associated with tinea capitis.

40
A patient with diffuse severe sudden hair loss developing 3 months after hospitalization for
septicemia likely has which of the following conditions?
A. Anagen effluvium
B. Catagen effluvium
C. Telogen effluvium
D. Loose anagen syndrome
E. Uncombable hair syndrome
►C

Telogen effluvium typically ensues stressful events after a 2-3 month period.

41
A patient with yellow nail syndrome should be evaluated for systemic disease involving which
organ system?
A. Hepatic
B. Renal
C. Pulmonary

228
D. Cardiovascular
E. Hematologic
►C

Yellow nail syndrome is characterized by slowly growing, yellow nails that are excessively
curved. The lunulae and cuticles are absent. It has been associated with a array of pulmonary
diseases including tuberculosis, asthma, pleural effusion, bronchiectasis, sinusitis, bronchitis, and
COPD.

42
Triangular lunulae are:
A. Part of the syndrome caused by a LMX1b mutation
B. Associated with hyperplastic patellae
C. Associated with a renal cell carcinoma
D. Associated with a mutation of ATP2C1
E. Associated with a mutation of SERCA2
►A

Triangular lunulae are part of the Nail-Patella syndrome. Findings in this syndrome include
LMX1b mutation, hypoplastic or absent patellae, nephropathy, a "Lester Iris" -
hyperpigmentation of the pupillary margin, cataracts, heterochromia irides and glaucoma.
ATP2C1 is associated with Hailey hailey which encodes a golgi ATPase. ATP2A2 mutation
causes Darier-White disease which encodes SERCA2 sarcoplasmic/endoplasmic reticulum
calcium-pumping ATPase.

43
Plummer's nails are associated with which of the following disorders?
A. Dermatomyositis
B. Systemic sclerosis
C. Lupus erythematosus
D. Hyperthyroidism
E. Lead poisoning
►D

Plummer's nails are onycholysis due to hyperthyroidism.

44
This hair disorder is used to describe end stage scarring alopecia with areas of hair growth that
look like "footprints in the snow":
A. Pseudopelade of Brocq

229
B. Lichen planopilaris
C. Aplasia cutis congenita
D. Lupus erthematosus
E. Central centrifugal cicatricial alopecia
►A

Pseudopelade of Brocq is an end stage scarring alopecia and describes a oval or irregularly
shaped atrophic patches with areas of hair growth that look like footprints in the snow.

45
Pathognomonic nails changes in HOOD syndrome or Hereditary osteoonychodysplasia are:
A. Red and white longitudinal banding
B. Pterygium inversum
C. Yellow nails
D. Pincer nails
E. Triangular lunulae
►E

In the HOOD syndrome (Hereditary osteoonychodysplasia) or the Nail-Patella syndrome,


patients have different degrees of nail dysplasia. In general, the thumbs are most severely
involved and nail disease decreases in severity from the second to the fifth digits. Characteristic
nail changes include longitudinal ridging, koilonychia, and splitting. Toenails are rarely
involved. Pathognomonic nail change is the V shaped triangular lunulae with the distal peak in
the midline.

46
Yellow nails can be seen in each of the following syndromes except:
A. Bronchiectasis
B. Hyperlipidemia
C. Tobacco use
D. Thyroid disease
E. Penicillamine
►B

Yellow discoloration of the nail plate can be associated with bronchiectasis or underlying
pulmonary disease such as COPD. Other associated conditions include lymphedema, rheumatoid
arthritis, thyroid disease, malignancies, penicillamine, and chronic sinusitis or bronchitis.
Hyperlipidemia is not associated with nail changes.

47

230
A 26-year-old woman presents with hair loss. Examination of a single shaft reveals division into
two shafts in short segments, with each individually covered with a complete cuticle. What is the
most likely diagnosis:
A. Pili multigemini
B. Pili bifurcati
C. Trichoptilosis
D. Trichoschisis
E. Trichothiodystrophy
►B

Pili multigemini refers to a rare disorder of multiple divided hair matrices, with multiple hair
shafts each with their own cuticle all arising from one follicle. Pili bifucarti exhibits hair shaft
bifurcation into short segments along hair shafts, with each segment covered with its own cuticle.
Trichoptilosis refers to “split ends” of the distal portion of the hair shaft. Trichoschisis is a clean
fracture of the hair shaft. Trichothiodystrophy is a disorder of beaded hair when viewed with
polarizing light.

48
A 44-year-old man undergoing chemotherapy for Hodgkin‟s lymphoma presents with horizontal
rough bands involving the nail plates of all digits. What is the most likely disorder:
A. Beau‟s lines
B. Mee‟s lines
C. Muehrcke‟s lines
D. Onychophagia
E. Dolichonychia
►A

Beau‟s lines are transverse indented nail plate furrows caused by temporary growth arrest of the
nail matrix, often due to chemotherapy or other stressful events and illnesses. Mee‟s lines are
transverse white lines affecting all nails and growing out with the nail; there is no associated
indentation. Mee‟s lines may be caused by arsenic poisoning, rheumatic fever, or other systemic
diseases. Muehrcke‟s lines disappear with squeezing of the nail and are due to disorders of low
albumin. Oncyhophagia refers to nail biting. Dolichonychia exhibits long, slender nails, and is
associated with Ehlers-Danlos and Marfans.

49
A patient that has Costello syndrome has thin anterior hair, redundant and thickened skin around
neck. The patient has congenital myopathy, coarse facies, cardiac and developmental disability.
The mutation is in:
A. HRAS gene

231
B. KRAS gene
C. BRAF gene
D. MEK1 gene
E. MEK2 gene
►A

Patient with Costello syndrome has a mutation in the HRAS gene. It is also known as
faciocutaneoskeletal syndrome.

50
Acquired progressive kinking of hair is a complication of which of the following medications?
A. Antimetabolites
B. Antimalarials
C. Retinoids
D. Slfonamides
E. Dapsone
►C

Retinoids can cause acquired progressive kinking of hair.

51
This nail finding is characteristically associated with what disorder?
A. Congenital heart failure
B. Chronic renal failure
C. Anemia
D. Hyperthyroidism
E. Cirrhosis
►B

Lindsay‟s nail is characteristically associated with chronic renal failure.

52
This term is used to describe the dorsal part of the digit that lies adjacent to the nail:
A. Proximal nail fold
B. Nail plate
C. Nail matrix
D. Nail bed
E. Hyponychium
►A

232
The proximal nail fold is the dorsal part of the digit that lies just adjacent to the nail.

53
This medication is a source of anagen effluvium and can cause abrupt shedding of hair:
A. Colchincine
B. Hydrochlorothiazide
C. Beta blockers
D. Calcium channel blockers
E. Procainamide
►A

Colchichine, mercury intoxication, thallium poisoning and severe protein deficiency can cause
anagen effluvium. Hairs are usually broken and not shed.

54
An 84-year-old man presents to clinic with atypical fingernails and is diagnosed with
hapalonychia. What defect does this term represent?
A. Long nails
B. White nail plates
C. Edema of the nail bed
D. Broad nails
E. Soft nails
►E

Hapalonychia refers to soft nails and is caused by a defect in the nail matrix. Dolichonychia
describes abnormally long, narrow nails, while brachyonchia describes abnormally short, broad
nails. Edema of the nail bed may present as Meurhcke‟s lines.

55
The hair finding characterized by an invagination of the distal hair shaft into the cup formed by
the proximal hair shaft is:
A. Trichoschisis
B. Pili torti
C. Pili triangulati
D. Trichorrhexis nodosa
E. Trichorrhexis invaginata
►E

233
The above description is that of trichorrhexis invaginata, which is characteristic of Netherton
syndrome. Other findings include: ichthyosis linearis circumflexa and atopy. It is caused by the
mutation of the serine protease inhibitor, SPINK5.

56
The combination of ankyloblepharon, ectodermal dysplasia, and cleft palate with wiry sparse
hair is characteristic of which of the following syndromes?
A. Focal Dermal Hypoplasia (Goltz syndrome)
B. Tricho-rhino-phalangeal Syndrome
C. Hay-Wells Syndrome
D. Ectrodactyly-Ectodermal dysplasia-Clefting (EEC)
E. Trichodentoosseous Syndrome
►C

Hay-Wells Syndrome, also known as AEC (Ankyloblepharon-Ectodermal dysplasia-Clefting) is


characterized by wiry, sparse hair or alopecia, ankyloblepharon, PPK, partial anhidrosis, cleft lip,
palate, absent, and dystrophic nails.

57
Frontal bossing, saddle nose, hypoplastic midface, peg shaped or conical teeth and
hypopigmented short sparse scalp and body hair are prominent features of which of the
following
A. Christ Siemens Touraine syndrome
B. Hidrotic ectodermal dysplasia
C. Arginosuccinic aciduria
D. Monilethrix
E. Pachyonychia congenita
►A

Christ Siemens Touraine syndrome also known as Hypohidrotic ectodermal dysphasia or


anhidrotic ectodermal dysplasia is X-linked recessive disorder that presents in infancy to early
childhood. Features include hypo-anhidrosis with increased body temperature, hypopigmented
sparse scalp and body hair, frontal bossing, saddle nose, hypoplastic midface, abnormal ears,
hypo-anodontia, and increased bronchopulmonary inections. Cloustons syndrome or hidrotic
ectodermal dysplasia is AD and associated with palmoplantar keratoderma, nail dystrophy, short
sparse scalp hair, and tufting of the terminal phalanges. Arginosuccinic aciduria is associated
with seizures, trichorrhexis nodosa, failure to thrive, and hyperammonemia. Monelothrix is
associated with beaded hairs and keratosis pilaris. Pachyonychia congenita is associated with nail
dystrophy and focal palmoplantar keratoderma.

234
58
The combination of poliosis, uveitis, deafness, and vitiligo are characteristic of which of the
following conditions?
A. Piebaldism
B. Waardernburg syndrome
C. Cronkhite-Canada syndrome
D. Vogt-Koyanagi-Harada syndrome
E. Cornelia de Lange Syndrome
►D

Vogt-Koyanagi-Harada syndrome is characterized by poliosis, uveitis, deafness, and Vitiligo.

59
Telogen hairs are in the resting phase and last for:
A. 3 months
B. 1 month
C. 2 months
D. 4 months
E. 5 months
►A

Telogen hairs are at the resting phase and make up 10-15% of hairs and last for 3 months.
Anagen is the growing phase and last for 2-6 years and catagen hairs last for 2 weeks.

60
Which of the following medication is a cause of hirsutism without virilization?
A. Dapsone
B. Diazoxide
C. Diazepam
D. Dinitrochlorobenzene
E. Dantrolene
►B

Phenytoin, diazoxide, cyclosporine and hexachlorobenzene all can cause increased hair growth
in patients. In addition, oral (and topical) minoxidil can cause hirsutism. The other listed
medications do not cause hirsutism.

61
A defect in which part of the nail cause a true leukonychia?
A. Cuticle

235
B. Hyponychium
C. Lunula
D. Nail bed
E. Nail plate
►E

True leukonychia is caused by an acute stress to the nail and manifests in the nail plate.
Examples of true leukonychia are Mee's lines and punctate leukonychia. Apparent leukonychia,
are a result of problems in the nail bed and are indicative of overall health compromise.
Examples of apparent leukonychia include Lindsay's, Terrys and Muehrcke's nail.

62
A patient is diagnosed with generalized congenital hypertrichosis lanuginosa has silvery, blond
and gray hair over entire body. The acquired form is associated with:
A. Fetal alcohol syndrome
B. Ambras syndrome
C. Cushings syndrome
D. Congenital adrenal hyperplasia
E. Arrhenoblastoma
►A

Patients with generalized congenital hypertrichosis lanuginosa with silvery, blond, and gray hair
over entire body. It is autosomal dominant with dental anomalies and the acquired form is
associated with fetal hydantoin and fetal alcohol syndrome.

63
A 42 year-old woman presents with the complaint of excess hair growth on her face. She has
normal menses and has recently had her "annual" exam and the note relates normal sized ovaries.
What is the most logical next step?
A. Check plasma levels of androstenedione and testosterone
B. Send a 21-hydroxylase enzyme deficiency test
C. Biopsy from the most affected area
D. Refer her to endocrinology
E. Order a CT of the abdomen
►A

Women with idiopathic hirsutism will have evidence of androgen excess but with normal
menses, normal-sized ovaries and no evidence of tumors of adrenal or ovary and normal adrenal
function. They will often have slight elevations of plasma androstenedione and testosterone.
Check the blood levels of the plasma steroids would be a logical first step.

236
64
All of the following are associated with anonychia or absence of the nail except:
A. Nail-patella syndrome
B. Congenital onychodysplasia of the index finger
C. Coffin-Siris syndrome
D. Onychodystrophy
E. Proteus syndrome
►E

All of the following except Proteus syndrome is associated with anonychia or absence of the nail.

65
This term describes transverse white lines that affect all the nails and can be from arsenic
poisoning, leprosy and systemic disease. The nail defect is called:
A. Mee's lines
B. Micronychia
C. Muehrchke's lines
D. Onychalgia
E. Onychauxis
►A

Transverse white lines that affect all the nails and grows out with the nail growth and is called
Mee's lines. It is a defect in the nail plate. This is associated with arsenic poisoning, rheumatic
fever, congenital heart failure, leprosy, and systemic disease.

66
A patient with synophrys, dystopia canthorum, and heterochromia iridum irides likely has which
of the following hair abnormalities?
A. Diffuse thinning
B. Scarring alopecia
C. Trichorrhexis invaginata
D. Trichorrhexis nodosa
E. White forelock
►E

White forelock is a common feature of Waardenburg syndrome.

67
Tay‟s syndrome is characterized by this hair condition pictured below – identify the condition:

237
A. Trichorrhexis nodosa
B. Trichorrhexis invaginata
C. Trichoschisis
D. Monilethrix
E. Pili trianguli et canaliculi
►C

Tay‟s syndrome is also known as trichothiodystrophy which is characterized by trichoschisis or


clean fracture of the hair shaft.

68
A 28-year-old male presents with three coin-shaped areas of scalp alopecia without scaling or
erythema. Which of the following is most likely to be associated with alopecia areata?
A. Hypothyroidism
B. Seborrheic dermatitis
C. Tinea capitis
D. Congestive heart failure
E. Chronic kidney disease
►A

Alopecia area is associated with autoimmune diseases, especially thyroid disease, hypo- or
hyperthyroidism. It presents with well-defined patches of alopecia without scale or erythema.
Seborrheic dermatitis may result in temporary hair thinning, but does not result in coin-shaped
patches of alopecia. Tinea capitis presents with itchy scale on the scalp and possibly temporary
poorly defined alopecia. Congestive heart failure and chronic kidney disease are not associated
with alopecia areata.

69
Anonychia is not a feature of which of the following syndromes?
A. Hay Wells syndrome
B. Coffin-Siris syndrome
C. DOOR syndrome
D. COIF syndrome
E. Hidrotic ectodermal dysplasia
►E

Hidrotic ectodermal dysplasia features hyperconvex nails, micronychia and nail dystrophy. All
of the other listed options can cause anonychia. COIF syndrome is Congenital Onychodysplasia
of the Index Finger and DOOR syndrome is Deafness, Congenital Onychodystrophy Recessive
Form. Coffin-Siris syndrome is characterized by: nail onychodysplasia or aplasia (usually of the

238
fifth finger or toe), coarse facies including bushy eyebrows, scant scalp hair, full lips, and
microcephaly, mental/growth deficiency, short distal phalanges, and other abnormalities.Hay
wells syndrome is ankyloblepharon ectyodermal defects cleft lip/palate syndrome and anonychia
has been reported.

70
A 32-year-old male presents with progressive curliness of the temporal scalp hairs that seem to
precede hair loss. What is the most likely associated etiology?
A. Secondary syphilis
B. Systemic tuberculosis
C. Lichen planopilaris
D. Hodgkin‟s lymphoma
E. HIV
►E

HIV and retinoids are associated with acquired progressive kinking. This syndrome presents with
gradual curling and darkening of hair in the frontal and temporal regions of the scalp, followed
by androgenetic alopecia. Secondary syphilis may present as a non-scarring alopecia, typically in
a diffuse pattern.

71
Follicular atrophoderma is a feature of which of the following conditions?
A. Bazex christol dupre syndrome
B. Menkes kinky hair syndrome
C. Papular atrichia
D. Tricho-dento-osseous syndrome
E. Tricho-rhino-phalangeal syndrome
►A

Bazex christol dupre syndrome is characterized by follicular atrophoderma, hypotrichosis,


hypohidrosis, and mutiple basal cell carcinomas of face.

72
Pili Annulati is associated with which of the following syndromes?
A. Netherton
B. PIBIDS
C. Citrullinemia
D. Hidrotic ectodermal dysplasia
E. Alopecia areata
►E

239
Pili Annulati is the alternating light and dark bands secondary to air-spaces seen in normal light
and has been associated with Alopecia areata. The "tiger-tail" pattern seen in trichothiodystrophy
(PIBIDS) is only seen with polarized light. Netherton and Citrullinemia do not have this hair
finding associated.

73
A patient is diagnosed with netherton's syndrome and has invagination of the distal hair shaft
into the cup formed by the proximal hair shaft also known as:
A. Trichorrhexis invaginata
B. Trichorrhexis nodosum
C. Trichoschisis
D. Trichothiodystrophy
E. Pili torti
►A

Patients with Netherton's syndrome have trichorrhexis invaginata. This syndrome also includes
ichthyosis linearis circumflexa, atopy, trichorrhexis nodosa. They have a mutation in the
SPINK5 gene with mutation in the serine protease inhibitor.

74
This term is used to describe "angel wing" deformity with a gradual shortening of the proximal
nail groove and leading to progressive thinning of the nail:
A. Pterygium
B. Beau's lines
C. Onychorrhexis
D. Brachyonychia
E. Anonychia
►A

Pterygium is angel wing deformity with gradual shortening of the proximal nail groove, leading
to progressive thinning of the nail plate and secondary fissuring caused by the fusion of the
proximal nail fold to the matrix and then to the nail bed.

75
The defect associated with the hair finding shown in the image is:
A. hHB6/hHB1
B. hHB7/hHB2
C. hHB8/hHB3
D. hHB9/hHB4

240
E. hHB10/hHB5
►A

hHB6/hHB1 keratins are associated with Monilethrix. Other findings include: short, brittle hair,
extensor keratosis pilaris, mental retardation, syndactyly, cataracts, and nail/teeth abnormalities.

76
This is used to describe the epithelium which starts at the mid distal phalanx which generates the
nail plate that determines the thickness of the nail plate and keratinizes without a granular layer:
A. Nail matrix
B. Nail plate
C. Proximal nail fold
D. Nail bed
E. Hyponychium
►A

The nail matrix generates the nail plate and is responsible for the thickness of the nail plate.

77
Ferritin levels need to be, at minimum, above which of the serum levels to treat iron-deficiency
related telogen effluvium?
A. 20ng/dL
B. 40ng/dL
C. 60ng/dL
D. 30ng/dL
E. 10ng/dL
►B

The therapeutic target for iron deficiency related telogen effluvium is greater than 40mg/dL.
Other causes of telogen effluvium include thyroid dysfunction, as well as such medications as
beta-blockers, antihyperlipidemic drugs, and NSAIDs.

78
Patients with horizontal parallel ridges often accompanied by absent cuticle due to trauma is
known as:
A. Habit tic
B. Elkonyxis
C. Dolichonyshia
D. Heller's median
E. Hapalonychia

241
►A

Patient with horizontal parallel ridges often accompanied by absent cuticle due to trauma to the
cuticle is called habit tic deformity. Patients can have anxiety and obsessive compulsive disorder
with this finding.

79
Yellow nail syndrome is associated with which of the following?
A. Multiple myeloma
B. Panhypopituitarism
C. Diabetes mellitus
D. Dermatophyte infection
E. Lymphedema and bronchiectasis
►E

Lymphedema and bronchiectasis are associated with yellow nail syndrome.

80
A child presents with sparse, fine hair, thin nails and loose skin in infancy. He has a pear-shaped,
broad nose and on X-ray has cone-shaped epiphyses. Which of the following genes is mutated in
this syndrome?
A. P63
B. DLX3
C. RMRP
D. TRPS1
E. ABCC6
►D

TRPS1 is the gene mutated in Tricho-rhino-phalangeal syndrome. TRPS1 is a putative zinc


finger transcription factor that is passed in an autosomal dominant fashion. P63 is mutated in
Hay-Wells syndrome and Ectrodactyly-ectodermal dysplasia-clefting (EEC) dyndrome. DLX3
(distal-less homeobox-3) gene is associated with Trichodentoosseous syndrome. RMRP (RNA
component of RNase MRP) is defective in Cartilage-Hair hypoplasia syndrome. ABCC6 is
defective in pseudoxanthoma elasticum.

81
Hirsutism is the development of terminal body hair in women in areas where this type of hair is
not normally found. All of the following drugs are associated with hirsutism except:
A. Minocycline
B. Phenytoin

242
C. Minoxidil
D. Diazoxide
E. Cyclosporine
►A

All of the above are associated with hirsutism except minocycline. They cause hirsutism without
virilization or defeminization.

82
A patient with psoriasis has pitting of the nails. This finding is due to involvement of which part
of the nail unit?
A. Proximal matrix
B. Nail bed
C. Hyponychium
D. Proximal nail fold
E. Distal matrix
►A

Disease in the proximal matrix is responsible for producing pitting, onychorrhexis, and Beau‟s
lines. Changes in the intermediate matrix can cause leukonychia, and the distal matrix may be
responsible for focal onycholysis, thinned nail plate, and erythema of the lunula.

83
Blue lunulae are characteristic of which disease?
A. Yellow nail syndrome
B. Wilson's disease
C. Rubenstein-Taybi syndrome
D. Hypertrophic pulmonary osteoarthropathy
E. Clubbing
►B

Wilson's disease is associated with characteristic blue lunulae.

84
A patient with cirrhosis has noticed that the proximal aspect of his nails have become white. This
characteristic finding is called:
A. Half-and-half nails
B. Mees' lines
C. Beau's lines
D. Terry's lines

243
E. Muehrcke's lines
►D

Terry‟s lines are described as an abnormal white appearance of the nail except for the most distal
portion. It has been described in association with cirrhosis, congestive heart failure, and diabetes
mellitus.

85
A subungual hematoma covering 50% of the nailbed should be managed by which of the
following?
A. No treatment
B. Removal of the nail plate
C. Trephination of the nail plate
D. Needle aspiration of the hematoma
E. Pressure dressing
►B

Removal of the nail plate is necessary for management of hematomas covering more than 25%
of the nailbed.

86
Distal subungual onychomycosis is most often caused by:
A. Trichophyton mentagrophytes
B. Epidermophyton floccosum
C. Trichophyton schoenleinii
D. Trichophyton rubrum
E. Trichophyton megninii
►D

Trichophyton rubrum is the most common organism involved in distal subungual


onychomycosis.

87
Cutaneous signs of cortisol excess include all of the following EXCEPT:
A. Plethora
B. Atrophic/fragile skin
C. Striae distensae
D. Dorsocervical/Supraclavicular fat pads
E. Peripheral obesity
►E

244
All of the listed options are signs of cortisol excess except peripheral obesity. The most common
pattern of obesity in patients with cortisol excess is centripetal or central obesity.

88
Best diagnosis of this type of cicatricial alopecia shown in attached image is
A. Chronic cutaneous lupus erythematosus
B. Lichen planopilaris
C. Central centrifugal cicatricial alopecia
D. Pseudopelade of Brocq
E. Alopecia mucinosa
►C

Central centrifugal cicatricial alopecia is a new term adopted by the NAHRS consensus group to
encompass the terms hot comb alopecia, follicular degeneration syndrome, pseudopelade in
African Americans, and central elliptical pseudopelade in Caucasians.In general, it is a condition
that presents with flesh-colored, noninflammatory cicatricial alopecia of the central scalp that,
over time, enlarges centrifugally. Pseudopelade of Brocq can mimic alopecia areata, lichen
planopilaris, and DLE. In addition, central centrifugal cicatricial alopecia (CCCA), a
noninflammatory condition of the central scalp that is seen primarily in black women. CCCA
usually presents as a symmetric patch, unlike the irregularly bordered, typically atrophic plaques
seen in classic pseudopelade of Brocq

89
A 52 year old male with history of diabetes mellitus with a >3 month history of recurrent
warmth, erythema, and slight pain surrounding the first toenail of the right foot. What is the most
likely diagnosis?
A. Acute Paronychia
B. Orf
C. Contact Dermatitis
D. Chronic Paronychia
E. Herpetic Whitlow
►D

Paronychia is an inflammatory reaction involving the folds of skin surrounding the nail. It can be
characterized as ACUTE or CHRONIC -Acute paronychia usually due to direct or indirect
trauma followed by separation of eponychium from the nail plate, the causative bacteria usually
Staph aureus and Strep pyogenes. The treatment includes culturing the skin, systemic antibiotics,
and draining of abscess if necessary. -Chronic paranochya is most likely a contact reaction to
irritants or allergens. It is characterized by inflammation of the proximal nail fold with erythema,

245
edema and absence of the cuticle. The most frequently recovered organism is CANDIDA. It was
originally thought to be result of Candida infection and treatment was based upon antifungal
therapy with minimal-moderate response. Newer studies suggest that it is actually dermatitis of
the nail fold COLONIZED by Candida. Treatment now is based on topical anti-inflammatory
steroids

90
All of the following are associated with increased fragility of hair except:
A. Trichorrhexis invaginata
B. Monilethrix
C. Pili torti
D. Trichorrhexis nodosa
E. Pili multigemini
►E

All of the following are associated with increased fragility except for pili multigemini. It is a
malformation characterized by the presence of bifurcated or multiple divided hair matrices and
papillae, giving rise to the formation of multiple hair shafts within the individual follicles.

91
The term used to describe absent cuticles/lunulae and slow growth with transverse ridging is:
A. Yellow nail syndrome
B. Unguis incartus
C. Transverse white bands
D. Trachyonychia
E. Terry's nails
►A

Yellow nail syndrome is absent cuticles/lunulae, slow growth, dystrophic shape and transverse
ridging. This is associated with lower extremity with lymphedema and bronchiectasis and pleural
effusion. It is also associated with D-penicillamine and bucillamine use in patients with
rheumatoid arthritis

92
Lindsay's nails (distal nail normal, proximal nail white) is characteristic of:
A. Chronic renal failure
B. Plummer-Vinson syndrome
C. Hemochromatosis
D. Ectodermal dysplasia
E. LEOPARD syndrome

246
►A

Lindsay's nails are characteristic of chronic renal failure and indicate nailbed edema. The
remaining options are causes of koilonychia, a spooning defect of the nail.

93
Which autoantibody is associated with cuticular changes, shawl sign, and overall favorable
prognosis?
A. Mi-2
B. Jo-1
C. High titered ANA with speckled pattern
D. Scl-70
E. Ro
►A

Dermatomyositis has many different cutaneous manifestations including heliotrope rash,


Gottron's papules and sign, mechanic's hand and poikiloderma atrophicans vasculare. Antibodies
to Mi-2 in dermatomyositis are associated with cuticular changes, shawl sign and a overall
favorable prognosis.

94
A patient with a history of gastrointestinal polyposis presents with alopecia, generalized
pigmentation, and nail dystrophy. Which of the following is the most likely diagnosis?
A. Peutz-Jegher syndrome
B. Gardner syndrome
C. Cronkhite-Canada Syndrome
D. Cowden syndrome
E. Dyskeratosis congenita
►C

Cronkhite-Canada Syndrome is a sporadic gastrointestinal polyposis syndrome characterized by


alopecia, generalized pigmentation, and nail dystrophy.

95
A 68-year old man presents with double white transverse lines on all nails that disappear with
compression of the nail plate. What is the most likely associated systemic disease:
A. Ehlers-Danlos Syndrome
B. Arsenic poisoning
C. Rheumatic Fever
D. Neurofibromatosis

247
E. Nephrotic Syndrome
►E

Muehrcke‟s lines are white transverse lines from an abnormal vascular bed. They disappear with
pressure on the nail plate. Muehrcke‟s lines are associated with disorders causing low albumin,
such as nephrotic syndrome, liver disease, and malnutrition.

96
This term is used to describe a "tiger tail" or alternating bright and dark bands from a reduced
sulfur content seen in Tay's syndrome
A. Trichothiodystrophy
B. Trichoschisis
C. Pili torti
D. Trichorrhexis nodosa
E. Bamboo hair
►A

A patient with Tay's syndrome can have trichothiodystrophy and looks like "tiger tail" with
alterations of bright and dark bands. Other clinical findings in Tay's syndrome have ichthyosis,
intellectual impairment, decreased fertility, and short stature.

97
A thick scale with matted hair is seen on a patients head. The most likely diagnosis is:
A. Pityriasis amiantacea
B. Traction alopecia
C. Triangular alopecia
D. Loose anagen syndrome
E. Telogen effluvium
►A

Pityriasis amiantacea is a condition seen in pediatric patients that improves with age. There is
thick scale, matted hair that may mimic seborrheic dermatitis or psoriasis. Treatment is
keratolytics.

98
Regarding androgens in women, which of the following statements is NOT correct?
A. Testosterone binds the androgen receptor
B. Eyebrows, eyelashes and vellus hairs are androgen-dependent
C. The hair follicle requires conversion of testosterone to dihydrotestosterone for
expression of androgen action

248
D. Dihydrotestosterone binds the androgen receptor
E. There are no differences in eyelashes, eyebrows and vellus hair-bearing areas in
men and women
►B

Only testosterone and dihydrotestosterone bind the androgen receptor, Eyebrows, eyelashes and
vellus hairs are NOT androgen-dependent, thus there are no differences between these areas in
men and women, and the hair follicle requires conversion of testosterone to dihydrotestosterone
for expression of androgen action.

99
Which part of the hair follicle is the first to cornify?
A. Huxley‟s layer of inner root sheath
B. Henley‟s layer of inner root sheath
C. Outer root sheath
D. Medulla
E. Cuticle
►B

Henley‟s layer of inner root sheath. Henleys layer is the outer layer of the inner root sheath. The
inner root sheath is keratinized with trichohyaline granules and is shed with the growing hair
shaft at the level of the isthmus

100
A patient has hyperkeratotic tissue on the lateral and proximal nails folds. What is the
diagnosis?
A. Onychophosis
B. Onychomadesis
C. Hapalonychia
D. Onychocryptosis
E. Onychophagia
►A

Onychophosis describes hyperkeratosis tissue on the lateral and proximal nails folds.
Onychomadesis is shedding of the nail starting at the proximal end. Hapalonychia refers to soft,
bendable nails. Onychocryptosis refers to an ingrown toenails. Onychophagia means nail biting.

101
Mutations in hair keratins hHB6 and hHB1 cause this characteristic finding pictured below –
name the condition:

249
A. Trichoptilosis
B. Hair casts
C. Tiger-tail hair
D. Trichoschisis
E. Monilethrix
►E

Mutations in the hair keratins, hHB6 and hHB 1 cause monilethrix.

102
A 7-year old girl presents to clinic with onychomadesis afflicting all nails. Her mother reports
that she has been healthy other than a self-limited illness two months prior. What is the most
likely causative etiology:
A. Rheumatic fever
B. Kawasaki disease
C. Systemic T cell lymphoma
D. Yellow nail syndrome
E. Hand, Foot, and Mouth Disease
►E

Coxsackievirus, the virus most associated with hand, foot, and mouth disease, is a well-known
cause of onychomadesis in children. Rheumatic fever has been associated with Mee‟s lines of the
nail. Yellow nail syndrome is associated with yellow nail plates, edema, and lung disease. The
other answer choices are not commonly associated with nail disease.

103
A 16-year-old woman presents with hair breakage. Examination of a single shaft reveals division
into two shafts at the distal-most portion of the hair. What is the most likely diagnosis:
A. Pili multigemini
B. Pili bifurcati
C. Trichoptilosis
D. Trichoschisis
E. Trichothiodystrophy
►C

Pili multigemini refers to a rare disorder of multiple divided hair matrices, with multiple hair
shafts each with their own cuticle all arising from one follicle. Pili bifucarti exhibits hair shaft
bifurcation into short segments along hair shafts, with each segment covered with it‟s own
cuticle. Trichoptilosis refers to “split ends” of the distal portion of the hair shaft. Trichoschisis is

250
a clean fracture of the hair shaft. Trichothiodystrophy is a disorder of beaded hair when viewed
with polarizing light.

104
Black nail is caused by:
A. Trichophyton mentagrophytes
B. Proteus mirabilis
C. Pseudomonas spp.
D. Staphylococcus aureus
E. Hortaea werneckii
►B

Nail infections with Proteus mirabilis. T. mentag causes white discoloration of the nail and
Pseudomonas will cause a green or yellow/green discoloration. Hortaea werneckii causes tinea
nigra. S. aureus does not cause black nails.

105
Acute paronychia is most commonly caused by which of the following organisms?
A. Candida albicans
B. Candida tropicalis
C. Pseudomonas aeruginosa
D. Staphylococcus aureus
E. Trichophyton rubrum
►D

Staphylococcus aureus is the most common cause of acute paronychia.

106
A woman with hypopigmented in lines of Blaschko and scarring alopecia likely suffers which of
the following conditions?
A. Chondrodysplasis punctata
B. Anhidrotic Ectodermal Dysplasia
C. Focal Dermal Hypoplasia
D. Rothmund-Thomson Syndrome
E. Bloch-Sulzberger Syndrome
►E

Bloch-Sulzberger Syndrome (incontinentia pigmenti).

107

251
A six year old girl presents after being sent home from school for "lice". On examination, there
are structures on the hair which are freely slide along the hair shaft. What is the most likely
diagnosis?
A. Pediculosis
B. Hair casts
C. Trichoptilosis
D. Monilithrix
E. Pili annulati
►B

Hair casts present most commonly in young girls between the age of 2 and 8 years old. These
casts are freely sliding along the hair shaft and are not adherent. They must be distinguished
from nits from pediculosis capitis which are adherent to the hair and do not slide. Trichoptilosis
are "spit ends" that form from chemical or physical damage to the hair.

108
Which of the following is the predominant dermatoscopic finding seen in alopecia areata?
A. Diffuse white knots and a brush-pattern
B. Perifollicular arborizing vessels
C. Yellow dots
D. Reduction of follicular ostia
E. Small oval nodes
►C

Dermatoscopic findings of alopecia areata include yellow dots, dystrophic hair shafts features,
and hypopigmented vellus hairs. Diffuse white knots and a brush pattern is seen in trichorrhexis
nodosa. In lichen planopilaris, a reduction to absence of follicular ostia, perifollicular scale and
arborizing vessels, pigmented networks, and white to blue-gray dots can be observed. Small oval
nodes are seen in monilethrix.

109
A 24-year-old man with Lesch-Nyhan syndrome is most likely to present with what nail
disorder:
A. Beau‟s lines
B. Mee‟s lines
C. Muehrcke‟s lines
D. Onychophagia
E. Dolichonychia
►D

252
Oncyhophagia refers to nail biting. Patient‟s with Lesch-Nyhan syndrome, caused by defective
HGPRT enzyme, are prone to self-mutilating behaviors, including onychophagia. Beau‟s lines
are transverse indented nail plate furrows caused by temporary growth arrest of the nail matrix,
often due to chemotherapy or other stressful events and illnesses. Mee‟s lines are transverse
white lines affecting all nails and growing out with the nail; there is no associated indentation.
Mee‟s lines may be caused by arsenic poisoning, rheumatic fever, or other systemic diseases.
Muehrcke‟s lines disappear with squeezing of the nail and are due to disorders of low albumin.
Dolichonychia exhibits long, slender nails, and is associated with Ehlers-Danlos and Marfans.

110
The epithelium that lies on the volar surface of the digit is the:
A. Nail bed
B. Nail matrix
C. Nail plate
D. Hyponychium
E. Eponychium
►D

The epithelium that lies on the most distal volar surface of the digit is the hyponychium. The nail
plate is the actual nail made up of tightly packed onychocytes. The nail matrix is the epithelium
which starts mid distal phalanx which generated the nail plate. It keratinizes without a granular
layer and determines the thickness of the nail plate. The nail bed is the thin epithelium
immediately beneath the nail plate.

111
Scalp biopsy of which of the following types of hair loss will reveal increased telogen hairs?
A. Lichen planopilaris
B. Discoid lupus erythematosis
C. Central centrifugal alopecia
D. Dissecting cellulitis
E. Androgenetic alopecia
►E

Androgenetic alopecia, telogen effluvium, and alopecia areata all have an increased fraction of
telogen hairs. Lichen planopilaris and discoid lupus typically reveal a chronic inflammatory
alopecia on biopsy. Dissecting cellulitis typically exhibits chronic inflammation with
hypertrophic scar tissue intervening between suppurative follicles.

112
Muehrcke's nails are most often associated with:

253
A. Hyperthyroidism
B. Hypothyroidism
C. Hypoalbuminemia
D. Hyperalbuminemia
E. None of these answers are correct
►C

Muehrcke's nails are most associated with hypoalbuminemia. They represent transverse double
white lines that are the abnormality of the vascular bed, probably a localized edematous state
secondary to the hypoalbuminemia.

113
Yellow nail syndrome can be associated with:
A. D-penicillamine use in patients with rheumatoid arthritis
B. Wilson”s disease
C. Alopecia areata
D. Doxorubicin
E. Minocycline use in acne patients
►A

D-penicillamine use in patients with rheumatoid arthritis. Yellow nail syndrome has been
described in patients with rheumatoid arthritis receiving thiol compounds including D-
penicillamine and bucillamine. Reversal of the condition has been described after discontinuation
of the drug.

114
A significantly increased number of "club hairs" on a pull test is indicative of:
A. Anagen effluvium
B. Telogen effluvium
C. Normal hair anatomy
D. Alopecia areata
E. Angrogenetic alopecia
►B

Telogen hairs are characteristically club shaped. An increased number of telogen hairs on hair
pull would be suggestive of telogen effluvium. Anagen hairs have a curled appearance at the
root. While a small number of telogen hairs are normal, a significantly increased number would
not be normal hair anatomy.

115

254
Which of the following syndromes can present with woolly or curly hair?
A. Bjornstad syndrome
B. Citrullinemia
C. Menkes syndrome
D. Noonan syndrome
E. Netherton syndrome
►D

Noonan syndrome, Naxos syndrome, Carvajal syndrome, Costello syndrome, and


Trichodentoosseous syndrome can present with curly or woolly hair. Bjornstad syndrome and
Menkes syndrome present with pili torti. Citrullinemia presents with trichorrhexis nodosa.

116
Patients that have a normal distal nail and a proximal nail that is white is associated with:
A. Chronic renal failure
B. Liver failure
C. Pulmonary stenosis
D. Gastrointestinal malignancy
E. Tuberous sclerosis
►A

Patients that have a normal distal nail and a proximal white nail have a condition called
Lindsay's nails or half and half nail. The proximal nail that is white is from nail bed edema. This
is from chronic renal failure.

117
Which part of the nail complex results in nail pits?
A. Proximal nail fold
B. Proximal nail matrix
C. Distal nail matrix
D. Nail plate
E. Nail bed
►B

Pitting of the nails are punctate depressions that migrate distally and result from defect in the
proximal nail matrix. It may be seen in conditions such as psoriasis, eczema and alopecia areata.

118
Psoriatic onycholysis is caused by psoriasis in which of the following nail subunits?
A. Nail matrix

255
B. Nail plate
C. Nail bed
D. Proximal nail fold
E. Hyponychium
►C

Nail bed involvement by psoriasis often causes onycholysis.

119
Patients that experience hypertrichosis lanuginosa acquisita can be due to all of the following
except:
A. Tetracyclines
B. Thyrotoxicosis
C. Corticosteroids
D. Phenytoin
E. Spironolactone
►A

Patients with hypertrichosis lanuginosa aquisita could be due from thyrotoxicosis,


corticosteroids, phenytoin, and spironolactone. It is abrupt onset of downy, soft non-pigmented
hair on the face and can be associated with glossistis and lung carcinoma.

120
Which genodermatosis characterized by broad thumbs and this nail disorder pictured below?
A. Rothmund-Thompson
B. Tuberous sclerosis
C. Pachyonychia congenita
D. Proteus syndrome
E. Rubenstein-Taybi syndrome
►E

Rubensteom-Taybi syndrome is characterized by broad thumbs and broad nails or


brachyonychia. This syndrome is characterized by mutations in CREB-binding protein.

121
This condition is used to describe an impulse control disorder with the repeated plucking or
pulling of hairs:
A. Trichotillomania
B. Anagen effluvium
C. Telogen effluvium

256
D. Loose anagen syndrome
E. Triangular alopecia
►A

Patients with trichotillomania have an impulse control disorder with repeated plucking or pulling
of hair. The confluence of short sparse hairs within an otherwise normal area of the scalp is seen.

122
A patient with wooly hair, diffuse palmoplantar keratoderma, and right ventricular
arrhythmogenic cardiomyopathy likely has a mutation in which of the following genes?
A. HHB6
B. Plakoglobin
C. Plakophilin
D. SPINK 5
E. SLURP 1
►B

Naxos syndrome is caused by mutations in the plakoglobin gene. Its clinical features include
wooly hair, diffuse palmoplantar keratoderma, and right ventricular arrhythmogenic
cardiomyopathy.

123
A patient is diagnosed with aplasia cutis congenita with a focal absence of epidermis on the
scalp. This is also associated with:
A. Adam Oliver syndrome
B. Parry Romberg syndrome
C. Cicatricial pemiphigoid
D. Mastocytosis
E. Epidermal nevi
►A

Patients that are diagnosed with Adam Oliver syndrome have aplaisa cutis congenita over the
posterior scalp and have terminal transverse defects of the limbs and small toenails. Most aplasia
cutis congenita are on the scalp and most are single lesions. ACC can also be associated with
Trisomy 13.

124
Scarring of the proximal nail fold is called dorsal pterygium and is associated with all of the
following except:
A. Lichen planus

257
B. Acrosclerosis
C. Onychotillomania
D. Lesch-Nyhan syndrome
E. Osteoarthritis
►E

Patients with dorsal pterygiums are associated with all of the following except osteoarthritis. It is
defined as scarring of the proximal nail fold.

125
Anagen hairs make up nearly this percentage of the total hair population:
A. 90%
B. 80%
C. 99%
D. 85%
E. 95%
►A

Anagens hairs make up nearly 90% of the hair population and is the growing phase of the hair.
Anagen hairs lasts from 2-6 years.

126
Deposition of mucin in the hair follicle can be associated with which disease?
A. Keratosis Follicularis Spinulosa Decalvans
B. Adams-Oliver Syndrome
C. Mycosis fungoides
D. Acne Keloidalis Nuchae
E. Perifolliculitis Capitis Abscedens et Suffodiens
►C

Mycosis fungoides can be associated with follicular mucinosis.

127
How long should the last wash-out period before performing mycologic investigations in a
clinically recurrent onychomycosis treated by terbinafine, itraconazole or fluconazole:
A. One month
B. Two months
C. Three months
D. Four months
E. Six months

258
►C

This is an interesting point, because we know that these systemic antifungals are retained in the
keratin of the toenails up to six months after the treatment has been stopped. Therefore this
answer could have been debatable were it not for the presence of fungi in the nail bed (which is
the primary location of the disease) and where the scraping should be properly performed for
mycologic testing after removal of a piece of nail as proximal as possible.

128
Air spaces in the hair shaft lead to this condition pictured below – name the condition:
A. Pili trianguli et canaliculi
B. Pili annulati
C. Trichoschisis
D. Pohl Pinkus constriction
E. Wooly hair
►B

Pili anulati is caused by air spaces in the hair shaft.

129
Pili trianguli et canaliculi is the characteristic hair finding in which of the following disorders?
A. Monilethrix
B. Pili annulati
C. Naxos syndrome
D. Anhidrotic ectodermal dysplasia
E. Uncombable hair syndrome
►E

Pili trianguli et canaliculi, characterized by a triangular hair shaft with a central longitudinal
groove, is the finding seen in uncombable hair syndrome.

130
Which of the following are characteristics of Trichorhinophalangeal syndrome?
A. Kinky hair, frontal bossing, small widely spaced teeth with poor enamel
B. Palmoplantar keratoderma, trichorrhexis nodosa, sinus infections
C. Shortened phalanges, sparse hair, bulbous nose
D. Brittle hair, short stature, cerebellar ataxia
E. Sparse fine hair, Short extremities, Immunodeficiency
►C

259
Characteristic features of trichorhinophalangeal syndrome are shortened phalanges, sparse hair,
and bulbous nose.

131
The most common cultured agent in chronic paronychia associated with a black hue of the lateral
edge of the nail plate is:
A. Trichophyton rubrum
B. gram-negative bacteria
C. Candida spp
D. Scytalidium dimidiatum
E. Scopulariopsis brevicaulis
►D

Chronic paronychia is likely a condition that represents a contact reaction to allergens or


irritants. It often is superinfected. Scytalidium dimidiatum causes a black discolored nail

132
What determines the thickness of the nail plate?
A. Hyponychium
B. Nail matrix
C. Nail bed
D. Proximal nail fold
E. Nail plate
►B

The nail matrix determines the thickness of the nail plate.

133
A patient with diffuse hair loss developing after a thallium scan likely has which of the following
conditions?
A. Anagen effluvium
B. Catagen effluvium
C. Telogen effluvium
D. Loose anagen syndrome
E. Uncombable hair syndrome
►A

Anagen effluvium can result from infusions of thallium.

134

260
Choose the correct statement regarding white piedra.
A. Commonly occurs in the tropics
B. Consists of cream-colored colonies attached to hairs
C. Treatment consists of cutting the hair
D. Is caused by piedra hortae
E. Both Consists of cream-colored colonies attached to hairs and Treatment
consists of cutting the hair
►E

White piedra: caused by Trichosporon beiglii, occurs commonly in temperate climates. The
culture shows cream colored, colonies attached to hairs. Treatment consists of cutting hair. Black
piedra is caused by piedra hortae.

135
Familial alopecia areata is commonly associated with what laboratory abnormality?
A. Hypocalcemia
B. Hypereosinophilia
C. Hyperkalemia
D. Leukopenia
E. Thrombocytopenia
►E

Familial alopecia areata has been associated with persistent thrombocytopenia. Alopecia areata
has recently been linked to the ULBP3 gene. Alopecia areata is associated with other
autoimmune disorders such as thyroid disease and vitiligo.

136
Trichobezoar is is a potentially life-threatening complication of:
A. Alopecia areata
B. Anhidrotic ectodermal dysplasia
C. Trichotillomania
D. Discoid lupus
E. Lichen planopilaris
►C

Trichophagia is more common in individuals who have richotillomania. This chewing behavior
frequently can lead to the formation of trichobezoars in the stomach or small intestines.
Trichobezoars can result in anemia, abdominal pain, hematemesis, nausea and/or vomiting,
bowel obstruction, perforation, gastrointestinal (GI) bleeding, acute pancreatitis, and obstructive
jaundice.

261
137
A 66-year-old woman presents with “black heads” on the nose. Examination reveals multiple
small hairs protruding from the same follicle. What is the most likely associated diagnosis:
A. Pili multigemini
B. Pili bifurcati
C. Trichostasis spinulosa
D. Trichoschisis
E. Trichothiodystrophy
►C

Pili multigemini refers to a rare disorder of multiple divided hair matrices, with multiple hair
shafts each with their own cuticle all arising from one follicle. Pili bifucarti exhibits hair shaft
bifurcation into short segments along hair shafts, with each segment covered with its own cuticle.
Trichostasis spinulosa refers to follicles filled with bundles of vellus hairs. This disorder may be
associated with renal failure. Trichoschisis is a clean fracture of the hair shaft.
Trichothiodystrophy is a disorder of beaded hair when viewed with polarizing light.

138
Which of the following are findings associated with Nail-Patella Syndrome?
A. Glomerulonephritis
B. Alopecia
C. Bladder diverticuli
D. Photosensitivity
E. Hip dysplasia
►A

Nail patella syndrome is a rare, AD syndrome associated with nail hypoplasia, absent or
hypoplastic patella, posterior iliac horns, glaucoma, hyperpigmentation of the pupillary margin
or Lester iris, and glomerulonephritis and nephrotic syndrome. Glomerulonephritis and renal
failure are features seen in nail patella syndrome. This syndrome is also known as HOOD
syndrome or hereditary osteoonychodysplasia.

139
Signs of virilization include all of the following EXCEPT:
A. Centripetal obesity
B. Temporal balding
C. Clitoromegaly
D. Increased muscle mass in the limb girdles
E. Acne

262
►A

Centripetal obesity is a sign of cortisol excess not of virilization. In addition to the those listed
above, irregular or absent menses and deepening of the voice are signs of virilization.

140
The combination of a low-set hairline and synophrys is seen in which of the following
conditions?
A. Werner syndrome
B. Turner Syndrome
C. Noonan Syndrome
D. Cornelia de Lange Syndrome
E. Waardenburg's syndrome
►D

Cornelia de Lange Syndrome is a sporadic disorder that may be caused by mutations in nipped-
Beta-like gene. Clinically, patients have cutis marmorata, hirsutism, synophrys, mental
retardation, short stature, GU abnormalities, hearing loss, and congenital heart defects.

141
Which of the following is an example of true leukonychia?
A. Terry's lines
B. Mee's lines
C. Muehrcke's lines
D. Hutchinson's lines
E. Onychoschizia
►B

Apparent leukonychia disappears with pressure and is a problem with nail bed, not nail plate.
Apparent leukonychia may be indicative of overall health compromise. Muehrcke's lines are
paired white bands and are a type of apparent leukonychia. Terry's lines are apparent
leukonychia are associated with cirrhosis, the proximal 2/3rd of the nail is white. True
leukonychia is due to a nail plate problem, the distal matrix is disordered. Mee's lines are due to
arsenic poisoning.

142
The common culprit of proximal white subungual onychomycosis is :
A. Candida albicans
B. Trichophyton rubrum
C. Trichophyton interdigitale

263
D. Scydalidium dimidiatum
E. Fusarium spp
►B

T. rubrum is responsible for more than 95% of these cases.

143
This term is to describe hair loss mostly on the vertex of the scalp in black females associated
with chemical relaxers:
A. Follicular degeneration syndrome
B. Pseudopelade of Broq
C. Folliculitis decalvans
D. Dissecting folliculitis
E. Traction alopecia
►A

Hair loss on the vertex of the scalp in black females also known as hot comb alopecia is
follicular degeneration syndrome. It can occur with chemical relaxers and premature
desquamation of the inner root sheath with fibrosis.

144
An 18 year-old girl who was hospitalized last month after a serious car accident is noted to have
white transverse grooves on her fingernails and toenails. The most likely diagnosis is:
A. Twenty-nail dystrophy
B. Beau”s lines
C. Half-and-half nails
D. Mees” lines
E. Terry”s nails
►B

Beau”s lines are transverse grooves in the nails. They are caused by a generalized systemic
condition which disrupts nail formation. Conditions may include infection, myocardial infection,
neurologic events and cytotoxic medications.

145
This is a small reddish-blue tender subungual tumor that can affect the nails:
A. Glomus tumor
B. Myxoid cyst
C. Acquired digital fibrokeratom
D. Accessory digit

264
E. Wart
►A

A glomus tumor is a reddish subungual tumor that can affect the nail. On pathology, one can see
small uniform round cells with eosinophilic cytoplasm.

146
A 36-year-old woman presents with hair loss. Examination of a single follicle reveals multiple
hair shafts protruding, each surrounded by its own cuticle. What is the most likely diagnosis:
A. Pili multigemini
B. Pili bifurcati
C. Trichoptilosis
D. Trichoschisis
E. Trichothiodystrophy
►A

Pili multigemini refers to a rare disorder of multiple divided hair matrices, with multiple hair
shafts each with their own cuticle all arising from one follicle. Pili bifucarti exhibits hair shaft
bifurcation into short segments along hair shafts, with each segment covered with its own cuticle.
Trichoptilosis refers to “split ends” of the distal portion of the hair shaft. Trichoschisis is a clean
fracture of the hair shaft. Trichothiodystrophy is a disorder of beaded hair when viewed with
polarizing light.

147
What is the gene defect that causes red hair?
A. Tyrosinase
B. P protein
C. Melanocortin 1 receptor
D. Tyrosine aminotransferase
E. Tyrosinase-related protein 1
►C

Defects in melanocortin 1 receptor lead to the phenotypic appearance of red hair. Individuals
with red hair have increased ratio of pheomelanin to eumelanin and are at increased risk for
melanoma.

148
Which of the following conditions with hair abnormalities is caused by mutations in a DNA
helicases?
A. Anhidrotic Ectodermal Dysplasia

265
B. Hidrotic Ectodermal Dysplasia
C. KID Syndrome
D. Rothmund-Thomson Syndrome
E. Incontinentia Pigmenti
►D

Rothmund-Thopmson syndrome is casued by mutations in RecQL4. Key clinical features include


poikiloderma, photosensitivity, dystrophic nails, hypoplastic thumbs, cataracts, and reports of
osteosarcoma. Anhidrotic ectodermal dysplasia is associated with mutations in ectodysplasin.
Hidrotic ectodermal dysplasia is associated with mutations in connexin 30. Kid syndrome is
associated with mutations in connexin 26 and IP with NEMO.

149
Which of the following syndromes does not have Dorsal pterygium as a feature:
A. Lichen planus
B. Lesch-Nyhan syndrome
C. Cicatricial pemphigoid
D. Chronic GVHD
E. Lamellar ichthyosis
►E

Dorsal pterygium (scarring of the proximal nail fold) is caused by lichen planus, acrosclerosis,
onychotillomania, Lesch-Nyhan syndrome, chronic GVHD, SJS/TEN and cicatricial
pemphigoid. All of the listed items are correct except lamellar ichthyosis. It does not have this
nail finding.

150
Blue lunulae are associated with each of the following except:
A. Argyria
B. Cardiac failure
C. Quinacrine
D. Wilson‟s disease
E. Phenolphthalein
►B

Blue lunulae have been reported in association with argyria, Wilson‟s disease, hereditary
acrolabial telangectasia, paronychia, phenolphthalein, quinacrine, topical bichloride, and
mercury exposure.

151

266
This is used to describe a follicular degeneration syndrome mainly on the crown of the scalp also
known as hot-comb alopecia:
A. Central centrifual cicatricial alopecia
B. Alopecia mucinosa
C. Traction alopecia
D. Pseudopelade of Brocq
E. Lichen planopilaris
►A

Central centrifugal cicatricial alopecia is a follicular degeneration syndrome with loss mainly on
the crown of the scalp and secondary to tight braids, ponytails, hair straighteners or curlers.

152
The best test for Cushing syndrome is:
A. Plasma LH
B. Plasma testosterone
C. Plasma prolactin
D. Plasma DHEA
E. Overnight dexamethasone suppresion test
►E

The overnight dexamethasone suppression test is the test of choice for diagnosis of Cushing
syndrome. In this test, a dose of dexamethasone is given at 11pm and the plasma cortisol level is
drawn the following morning. If the Cushing syndrome is from an adrenal tumor or an ectopic
ACTH producing tumor, there is no change in cortisol secretion. A normal response is
suppression of the cortisol level by the extra dexamethasone. If the Cushings is from a pituitary
tumor, at low doses of dexamethasone there are no changes in cortisol levels while at high doses
of dexamethasone there will be a normal suppression. The other listed items are not used for
diagnosis of Cushing syndrome.

153
Steatocystoma multiplex and natal teeth are associated with which of the following conditions?
A. Jadassohn-Lewandowsky pachyonychia congenita (Type 1)
B. Jackson-Lawler pachyonychia congenita (Type 2)
C. Nail-patella syndrome
D. Coffin-Siris syndrome
E. Rubenstein-Taybi syndrome
►B

267
Jackson-Lawler pachyonychia congenita (Type 2) is characterized by thickening of the nailbed
and plate, steatocystoma multiplex, and natal teeth.

154
Which of the following is NOT a cause of red lunulae?
A. Endocarditis
B. Psoriasis
C. Carbon monoxide poisoning
D. Imuran
E. Rheumatoid arthritis
►A

Endocarditis is not a cause of red lunulae. Alopecia areata, psoriasis, prednisone for connective
tissue diseases, carbon monoxide poisoning, cardiac failure, SLE, RA, COPD, CO2, cirrhosis,
imuran and radiation are the causes of this condition.

155
What is the mode of inheritance of this disorder of the hair shaft?
A. Autosomal dominant
B. Autosomal recessive
C. X-linked dominant
D. X-linked recessive
E. Sporadic
►A

Monilethrix is an autosomal dominant condition with variable penetrance. The hair shaft has an
elliptical or beaded hair appearance.

156
Highest graft survival in hair transplantation is achieved through the use of:
A. 4-5 mm plugs
B. Minigrafts
C. Single hairs
D. Follicular units
E. Micrografts
►D

Seager et. al examined the survival rate of single hair grafts and compared them to follicular unit
grafts in a single patient. They found that there was a much higher survival rate in the follicular

268
unit grafts. They hypothesized that the extra tissue surrounding the follicular unit grafts protected
them from crush injury.

157
A patient presents with 20 nails with absent cuticles and lunulae, slow growth, dystrophic shape
and a yellow hue. Which of the following findings is/are associated?
A. Upper extremity edema
B. Cirrhosis
C. Pleural effusions
D. Chest pain
E. Pulmonary fibrosis
►C

Yellow Nail syndrome has been associated with lower extremity edema, bronchiectasis and
pleural effusions. The other listed findings are not part of Yellow Nail syndrome.

158
This describes a patch of vellus hairs or complete hair loss in the frontal-temporal region with no
treatment available and usually persistent. This condition is called:
A. Triangular alopecia
B. Loose anagen syndrome
C. Telogen effluvium
D. Anagen effluvium
E. Trichotillomania
►A

Triangular alopecia is also known as temporal alopecia and describes a triangular patch of vellus
hairs or complete hair loss in the frontal-temporal region. On histology there is vellus hairs seen
and there is usually no treatment and is persistent.

159
The best diagnosis for this scarring form of alopecia is
A. Chronic cutaneous lupus erythrematosus
B. Lichen planopilaris
C. Pseudopelade of Broq
D. Perifolliculitis capitis abscedens et suffodiens
E. Central centrifugal cicatricial alopecia
►D

269
Perifolliculitis capitis abscedens et suffodiens is also known as dissecting cellulitis of scalp is
uncommon chronic suppurative disorder that most often affects young black men. Together with
acne conglobata, hidradenitis suppurativa and pilonidal cysts, it is a component of the 'follicular
occlusion tetrad'. Patients present with multiple painful inflammatory nodules and fluctuant
abscesses of the scalp, which are often interconnected via sinus tracts. The vertex and occiput are
the sites of predilection. As the disease progresses, scarring alopecia may develop. All other
choices can cause scarring alopecia but with different clinical picture.

160
Koilonychia is most commonly seen in association with:
A. Renal disease
B. Progressive systemic sclerosis
C. Alopecia areata
D. Impaired iron metabolism
E. Pulmonary disease
►D

In koilonychia (spoon nails), the nail is a concave shape with raised lateral edges. Iron-deficiency
anemia and Plummer-Vinson syndrome have been associated with koilonychia. Involvement of
only the first three fingernails is suggestive of an occupational cause. In early infancy,
koilonychia is a normal finding. Koilonychia is also seen in hemochromatosis, an autosomal
recessive condition associated with increased intestinal iron absorption and increased iron
deposition leading to metallic-grey hyperpigmentation of the skin, hepatomegaly, diabetes,
hypogonadism, polyarthritis, and cardiac abnormalities.

161
This hair finding is caused by mutations in what gene?
A. SPINK 5
B. NEMO
C. SERCA2
D. Dyskerin
E. Keratin 16
►A

Mutations in SPINK 5, a serine protease inhibitor leads to Netherton‟s syndrome which is


characterized by trichorrhexis invaginata or bamboo hair.

162
Argininosuccinic aciduria is characteristically associated with this hair finding –name the
condition pictured below:

270
A. Pili trianguli et canaliculi
B. Trichoschisis
C. Monilethrix
D. Trichorrhexis nodosa
E. Trichorrhexis invaginata
►D

Trichorrhexis nodosa is a finding in argininosuccinic aciduria.

163
This is to describe a mutation in the hairless gene and is characterized by universal alopecia right
after birth. Patients with this condition have normal eyelash, cornified cysts over face and neck.
This describes:
A. Papular atrichia
B. Acquired progressive kinking
C. Trichotillomania
D. Woolly hair nevus
E. Ichthyosis follicularis, atrichia, photophobia
►A

Patients with papular atrichia have an autosomal recessive mutation in the hairless gene. They
have universal alopecia right after birth. They have normal eyelash, cornified cysts over face and
neck.

164
The most common enzyme abnormality in congenital adrenal hyperplasia is:
A. 3-beta-hydroxysteroid dehydrogenase isomerase
B. 11-beta-hydroxylase
C. 21-hydroxylase
D. 15-hydroxylase
E. 17-hydroxylase
►C

21-hydroxylase deficiency is present in 95% of cases of congenital adrenal hyperplasia. This


defect in adrenal steroidogenesis can occur at any point in life, but affected girls will generally
present around puberty with hirsutism and mentsrual irregularity/primary amenorrhea. The key
feature is excess androgen production.

165

271
Atrophy of the isthmus is a characteristic histologic finding of which of the following
conditions?
A. Discoid lupus erythematosus
B. Lichen planopilaris
C. Parry-Romberg syndrome
D. Folliculitis Decalvans
E. Acne Keloidalis Nuchae
►B

Lichen planopilaris is characterized by perifollicular lymphocytic infiltration with atrophy of the


isthmus.

166
Triangular lunulae are seen in what disorder?
A. Dyskeratosis congentia
B. Neurofibromatosis II
C. Gorlin”s syndrome
D. Nail-patella syndrome
E. Papillon-Lefevre syndrome
►D

Triangular lunulae are seen in nail-patella syndrome which is characterized by mutation in


LMX1b.

167
A 73-year-old man presents with a “red streak” involving the nail plate of the first digit. You
make the diagnosis of longitudinal erythronychia. What is the most common underlying cause:
A. Melanoma
B. Myxoid cyst
C. Subungual keratoacanthoma
D. Carbon Monoxide toxicity
E. Onychopapilloma
►E

Onychopapilloma represents a benign tumor of the nail unit and is the most common etiology of
longitudinal erythronychia. Myxcoid cysts are mucin-filled pseudocysts that may be connected to
the joint space and often present as longitudinal grooving of the nail plate. Carbon Monoxide
toxicity may present with a red lunula.

272
Keratosis follicularis has nail findings of v-shaped nicking of the distal nail, subungual
hyperkeratosis and red/white alternating longitudinal bands. In addition to those listed above,
psoriasis can also cause spotted red lunulae.

168
Spotted red lunulae are absent in which of the following conditions?
A. Alopecia areata
B. Keratosis follicularis
C. Systemic lupus erythematosus
D. Rheumatoid arthritis
E. Lichen planus
►B

Iron deficiency can lead to spoon nail deformity or koilonychias.

169
What nutritional deficiency can lead to this nail finding?
A. Vitamin B12
B. Vitamin C
C. Iron
D. Magnesium
E. Thiamine
►C

D-penicillamine has been associated with the development of yellow nail syndrome in patiens
with rheumatoid arthritis. The other options are used to treat RA, but have not been associated
with development of this syndrome.

170
Yellow nail syndrome has been associated with use of which of the following treatments for
Rheumatoid Arthritis?
A. D-penicillamine
B. Methotrexate
C. Infliximab
D. Etanercept
E. Adalimumab
►A

273
D-penicillamine has been associated with the development of yellow nail syndrome in patiens
with rheumatoid arthritis. The other options are used to treat RA, but have not been associated
with development of this syndrome.

171
Tricholemmoma is which type of hair follicle tumor?
A. Tumor of the hair germ cells
B. Tumors of matrical differentiation
C. Tumors of the external root sheath
D. Tumor of isthmus differentiation
E. Tumor of the internal root sheath
►C

Tricholemmoma is a tumor of the external root sheath. Tumors of the hair germ cells include
trichoblastoma and trichoepithelioma. Tumors of matrical differentiation include pilomatricoma.
Tumors of the external root sheath include pilar cyst, proliferating tricholemmal cyst,
tricholemmoma, tricholemmal carcinoma. Tumor of the isthmus differentiation include tumor of
the follicular infundibulum.

172
Distal subungual onycholysis associated with paronychia is due to:
A. Candida spp
B. Fusarium spp
C. Scytalidium dimidiatum
D. Trichophyton rubrum
E. Trichophyton mentagrophytes
►C

This mold is the only one responsible for paronychia in DLSO and is more often observed in
finger than in toenails.

173
Catagen hairs make up for less than 1% of the hair and lasts for:
A. 2-3 weeks
B. 4-6 weeks
C. 8-10 weeks
D. 10-12 weeks
E. 1 week
►A

274
Catagen hairs last up to 2-3 weeks and represents transition from anagen to telogen. It makes up
less than 1% of the hair population.

174
A 14-year-old man with a history of extensive striae, upward lens dislocation, and pectus
excavatum would be most likely to present with what nail abnormality:
A. Beau‟s lines
B. Mee‟s lines
C. Muehrcke‟s lines
D. Onychophagia
E. Dolichonychia
►E

Beau‟s lines are transverse indented nail plate furrows caused by temporary growth arrest of the
nail matrix, often due to chemotherapy or other stressful events and illnesses. Mee‟s lines are
transverse white lines affecting all nails and growing out with the nail; there is no associated
indentation. Mee‟s lines may be caused by arsenic poisoning, rheumatic fever, or other systemic
diseases. Muehrcke‟s lines disappear with squeezing of the nail and are due to disorders of low
albumin. Oncyhophagia refers to nail biting. Dolichonychia exhibits long, slender nails, and is
associated with Ehlers-Danlos and Marfans.

175
Mutations in c-kit are associated with which of the following conditions?
A. Waardenburg syndrome
B. Piebaldism
C. Tuberous sclerosis
D. Vogt-koyanagi-harada
E. Cornelia de Lange Syndrome
►B

Piebaldism is caused by mutations in c-kit.

176
A 34-year-old man presents with a jet-black nail plate involving the right index finger.
Hutchinson‟s sign is negative and the patient denies antecedent trauma. What is the most likely
etiology of the nail plate discoloration:
A. Multiple Myeloma
B. Wilson‟s disease
C. Congenital Oncychodysplasia of the Index Figner (COIF)
D. Pseudomonas infection

275
E. Proteus Infection
►E

Proteus infection of the nail is associated with black discoloration of the nail plate. Multiple
myeloma may be associated with anonychia. Wilson‟s disease classically exhibits blue lunulae.
COIF presents with absence of the nail plate. Pseudomonal infections of the subunguium may
produce a green discoloration due to production of pyocyanin pigment.

177
Arsenical poisoning is associated with what nail finding?
A. Muehrcke's lines
B. Mee's lines
C. Hapalonychia
D. Onycholysis
E. Oil spots
►B

Arsenical poisoning is characteristically associated with Mee's lines.

178
Patients with thickened, hypertrophied nail plates and subungual hyperatosis have pachyonychia.
The type that is associated with keratin 6 and 16 mutation is:
A. Type I
B. TYpe II
C. Type III
D. Type IV
E. Type V
►A

Pachyonychia congenita is subungal hyperkeratosis, pincer nail, and paronychia. Type I:


Jadassohn-Lewandowsky is associated with keratin 6 and 16 mutation, focal PPK, follicular
hyperkeratosis, and oral leukokeratoses that is non premalignant.

179
How long do hairs stay in telogen before they are released?
A. 3-4 weeks
B. 3-4 months
C. 3-4 years
D. 3-4 days
E. 3-4 hours

276
►B

Hairs remain in telogen phase for 3-4 months before they are released. The length of anagen
phase varies for hairs of different parts of the body and determines the ultimate length of the hair.
Scalp hair may remain in anagen for 3-5 years. The duration of catagen phase is typically 3-4
weeks.

180
This is a term used to describe transverse excessive curvature from widened base of the terminal
phalanx. It can be hereditary or from shoes usually the 1st toenails and subungual exostoses. It is
called:
A. Pincer nails
B. Platyonychia
C. Plicatured nails
D. Plummer's nails
E. Dorsal pterygium
►A

Transverse excessive curvature from widened base of terminal phalanx is called pincer nails. It
can be from hereditary or from shoes, and is from the 1st toenails. It is associated with subungual
exostoses, osteoarthritis and psoriasis

181
Splinter hemorrhage of the nail can be seen with which of the following parasitic infections?
A. Scabies
B. Trichinosis
C. Sparganosis
D. Dracunculiasis
E. Gnathostomiasis
►B

Trichinosis can cause splinter hemorrhage of the nails.

182
All of the following are associated with trichorrhexis nodosa except:
A. Mechanical or chemical damage associated with trichoptilosis
B. Absent argininosuccinase
C. Citrullinemia
D. Menke's kinky hair syndrome
E. Bjornstad syndrome

277
►E

Patients that have trichorrhexis nodosa have frayed ends resembing "broom-stick" or "pain
brush" hair. The following are associated with this condition including mechanical or chemical
damage, argininosuccinic aciduria, citrullinemia, Menke's kinky hair syndrome,
Trichothiodystrophy, and Netherton's syndrome. Bjornstad syndrome is associated with pili torti.

183
Which is the most effective for the treatment of onychomycosis?
A. Oral terbinafine
B. Terbinafine cream
C. Econazole cream
D. Clobetasol ointment
E. Oral ketoconazole
►A

Terbinafine, a squalene epoxidase inhibitor, is the treatment of choice for onychomycosis. The
treatment is with 250 mg by mouth daily for 12 weeks for toenails and 6 weeks for fingernails.
Other treatment options include oral itraconazole. Topicals such as terbinafine cream and
econazole cream are good choices for tinea pedis, but are not successful in treating
onychomycosis. Topical corticosteroids such as clobetasol ointment are not effective in treating
fungal infections. Oral ketoconazole is ineffective for onychomycosis.

184
Björnstad syndrome is the combination of pili torti and which of the following features?
A. Lentigines
B. White forelock
C. Hypogonadism
D. Deafness
E. Seizures
►D

Deafness and pili torti are the two cardinal features of Björnstad syndrome.

185
Which of the following is caused by mutations in gap junction proteins?
A. Rothmund-Thompson syndrome
B. Hidrotic ectodermal dysplasia
C. Anhidrotic ectodermal dysplasia
D. Netherton syndrome

278
E. Naxos syndrome
►B

Hidrotic ectodermal dysplasia is caused by mutations in connexin 30, which is a gap junction
protein.

186
Which palmoplantar keratoderma is characterized by painful hyperkeratosis?
A. Richner-Hanhart
B. Howell-Evans
C. Vohwinkel
D. Unna-thost
E. Mal de Meleda
►A

While most palmoplantar keratodermas are characterized by thick plaques of the palms and soles
that are usually painless, Richner Hanhart is classically associated with painful lesions, rendering
it distinctive. Richner Hanhart is also associated with a painful keratitis and pigment dilution of
the hair.

187
Proximal white subungual onychomycosis with polydactylous involvement needs at least one
laboratory test:
A. Blood count
B. Sedimentation rate
C. T-helper CD4 lymphocytes
D. Liver enzymes
E. HIV
►E

This type of onychomycosis has a prognostic value: It means that the CD4 lymphocytes are
below 450.

188
Fungal finger onycholysis usually results from:
A. Trichophyton rubrum
B. Trichophyton interdigitale
C. Candida albicans
D. Scopulariopsis brevicaulis
E. Fusarium spp

279
►C

It is widely accepted that C. albicans acts as colonizer that has found an ideal environment.

189
Chronic paronychia is most commonly caused by which of the following organisms?
A. Candida albicans
B. Candida tropicalis
C. Pseudomonas aeruginosa
D. Staphylococcus aureus
E. Trichophyton rubrum
►A

Candida albicans is the most common pathogen associated with chronic paronychia.

190
A 64-year-old man admitted for an exacerbation of his severe congestive heart failure exhibits
blanchable white horizontal lines beneath the nail plates of all digits. What is the most likely
disorder:
A. Beau‟s lines
B. Mee‟s lines
C. Muehrcke‟s lines
D. Onychophagia
E. Dolichonychia
►C

Beau‟s lines are transverse indented nail plate furrows caused by temporary growth arrest of the
nail matrix, often due to chemotherapy or other stressful events and illnesses. Mee‟s lines are
transverse white lines affecting all nails and growing out with the nail; there is no associated
indentation. Mee‟s lines may be caused by arsenic poisoning, rheumatic fever, or other systemic
diseases. Muehrcke‟s lines disappear with squeezing of the nail and are due to disorders of low
albumin. Oncyhophagia refers to nail biting. Dolichonychia exhibits long, slender nails, and is
associated with Ehlers-Danlos and Marfans.

191
Thin hair with premature graying is characteristic of which of the following syndromes?
A. Werner syndrome
B. Turner Syndrome
C. Noonan Syndrome
D. Cornelia de Lange Syndrome

280
E. Waardenburg's syndrome
►A

Werner syndrome is characterized by premature degeneration of major organs that loosely


mimics premature aging. It is associated with sclerodermoid changes, leg ulcerations, premature
graying, cataracts, premature atherosclerosis,a dn diabetes. It is associated with defects in the
helicase RecQL2.

192
The term to describe hair shafts that are abruptly thinned and break off at the skin surface seen in
anagen effluvium is:
A. Pohl-Pinkus
B. Epilated hair
C. Trichotillomania
D. Traction alopecia
E. Telogen effluvium
►A

Pohl-pinkus is used to describe the abruptly thinned hair shafts. This is seen in anagen effluvium.
The shedding of anagen hairs are always abnormal.

281
Chapter -12-
Bullous and Vesicular Dermatoses
1
Drugs that are associated with the exacerbation of pemphigus foliaceus include :
A. Captopril
B. Metoprolol
C. Fluconazole
D. Calcium channel blockers
E. Statins
►A

Pemphigus foliaceus is a form of pemphigus with superficial blisters. The autoantibodies are
against Dsg1. Medications associated with the exacerbation of pemphigus foliaceus is sulfhydryl
groups such as captopril, penicillamine and piroxicam. Metroprolol can exacerbate psoriasis.

2
An elderly gentleman with a history of hypertension and a thymoma, presents to the Emergency
Room with oral and conjunctival erosions and hemorrhagic bullae on his hands and feet. These
finds are seen in:
A. Cicatricial pemphigoid
B. Sweet”s syndrome
C. Toxic epidermal necrolysis
D. Paraneoplastic pemphigus
E. Erythema multiforme
►D

Paraneoplastic pemphigus is usually associated with cancers of lymphoid origin, including


lymphoma, leukemia, thymomas, Castleman”s tumor. Clinically, this condition may present with
intractable staomatitis, severe painful oral and conjunctival erosions.

3
A child develops an allergic reaction at the site of an influenza vaccine. To which of the
following substances may she be allergic to?
A. Lanolin
B. Thimerosol
C. Ethylenediamine dichloride
D. Triclosan
E. Gluteraldehyde
►B

282
Thimerosol is a preservatives in vaccines such as the influenza, tetanus, and diphtheria vaccines.
It is also found in antitoxins and immunoglobulins. Thimerosol is a mercury-containing organic
compound. Lanolin is from the sebum of sheep. Ethylenediamine dichloride is a stabilizer in
topical creams, medicines, dyes, insecticides, and fungicides and was previously found in
nystatin cream. Triclosan is an antibacterial agent found in soap, shampoo and mouthwash.
Gluteraldehyde is a cold sterilizing solution used for medical and dental equipment.

4
Gallstones are associated with:
A. Erythropoietic protoporphyria
B. Hepatoerythropoietic porphyria
C. Variegate porphyria
D. Coproporphyria
E. Harderoporphyria
►A

Gallstones and liver failure can complicate erythropoietic protoporphyria.

5
Antibodies against type VII collagen are seen in:
A. Epidermolysis bullosa simplex
B. Pemphigus erythematosus
C. Cicatricial pemphigoid
D. Epidermolysis bullosa acquisita
E. Bullous pemphigoid
►D

Type VII collagen is present in the basement membrane of stratified squamous epithelia in the
anchoring fibrils. It is the target antigen in several blistering disease including epidermolysis
bullosa acquista, bullous lupus erythematosus. Mutated collagen 7 occurs in dominant and
recessive dystrophic epidermolysis bullosa.

6
The dimethylglyoxime test is used to detect the presence of:
A. Gold
B. Silver
C. Mercury
D. Nickel
E. Latex

283
►D

1% dimethylglyoxime is added to to the metal. If a red precipitate forms, there is >1:10000


nickel content.

7
Characteristic eosinophilic abscesses are seen in:
A. Bullous drug
B. Pemphigus vegetans
C. Herpes gestationis
D. Incontinentia pigmenti
E. Paraneoplastic pemphigus
►B

Eosinophilic abscesses with minimal to no spongiosis in a hyperplastic epidermis are


characteristic of pemphigus vegetans.

8
Circulating autoantibodies to type XVII collagen are most characteristic of which disease?
A. Epidermolysis bullosa accquisita
B. Herpes gestationis
C. Pemphigus vulgaris
D. Pemphigus foliacious
E. Paraneoplastic pemphigus
►B

Type XVII collagen or BPAG2 is a transmembrane protein found in the hemidesmosome.


Autoantibodies to type XVII collagen are important in the pathogenesis of cicatricial
pemphigoid, bullous pemphigoid, and herpes gestationis.

9
Patients with this disorder may develop exaggerated reactions to insect bites:
A. Chronic lymphocytic leukemia
B. Bullous pemphigoid
C. Lupus erythematosus
D. Atopic dermatitis
E. Incontinentia pigmenti
►A

284
Patients with chronic lymphocytic leukemia may develop exaggerated reactions to insect bites,
including bullous reactions.

10
What is the enzyme defect in Gunther's disease?
A. Porphobilinogen deaminase
B. Uroporphyrinogen synthetase III
C. Uroporphyrinogen decarboxylase
D. Ferrochetalase
E. Protoporphyrinogen oxidase
►B

Gunther's disease (also known as congenital erythropoietic porphyhria) is caused by a defect in


uroporphyrinogen synthetase III, the enzyme which responsible for the conversion of
hydroxymethylbilane to uroporphyriogen III. Patients present with red teeth and immediate
photosensitivy during infancy. There is severe photophobia and late skin manifestations include
mutilating scarswith scarring alopecia, and sclerodermoid changes.

11
Which pair is incorrect?
A. Desmoglein 1 : 160 kDa
B. Desmoplakin I : 250 kDa
C. Type VII collagen : 290 kDa
D. Type XVII collagen: 180 kDa
E. Envoplakin : 190 kDa
►E

Envoplakin is 210 kDa. Periplakin is 190 kDa.

12
A 20-year-old female presents with an allergic contact dermatitis to a perfume containing Lily of
the valley. What is the causative allergen?
A. Cinnamic acid
B. Vanillin
C. Hydroxycitronellal
D. Atranorin
E. Evernic acid
►C

285
The main allergen in Lily of the valley, which is found in perfumes, soaps, cosmetics, eye cream
and aftershaves, is hydroxycitronellal (synthetic). Cinnamic acid and vanillin are present in
Balsam of Peru, while atranorin and evernic acid are allergens in oak moss absolute.

13
Bullous pemphigoid antigen 1 (BPAg1) is a member of this family:
A. Cadherin
B. Integrin
C. Intermediate filament
D. Plakin
E. Selectin
►D

The plakin family includes desmoplakin I and II, BPAg1, envoplakin, periplakin.

14
In the liver, the rate limiting step in the porphyria pathway is mediated by the enzyme:
A. Ferrochetalase
B. ALA dehydratase
C. ALA synthase
D. Uroporphyrinogen decarboxylase
E. Porphobilinogen deaminase
►C

ALA synthase mediates the first and rate-limiting step in the heme synthesis pathway. This step
occurs in the mitochondria and allows for the conversion of Succinyl coA + glycine to delta
aminolevilinic acid.

15
Which steriod screening agent should be used when an allergic contact dermaititis is suspected to
hydrocortisone?
A. Glutaral
B. Tixocortol-21-pivalate
C. Budesonide
D. Hydrocortisone-17-butyrate
E. Benzalkonium chloride
►B

286
Tixocortol-21-pivalate is a group A steroid and screens for allergies to hydrocortisone,
prednisone, and methylprednisolone. Budesonide screens for groups B and D steroids, and
hydrocortisone-17-butyrate screens for group D steroids.

16
Common causes of drug-induced linear IgA:
A. Vancomycin
B. Penicillin
C. Cephalosporins
D. Captopril
E. All of these answers are correct
►E

Linear IgA disease that is induced by drugs is most commonly secondary to vancomycin. Other
implicated drugs include other antibiotics and captopril.

17
A patient has desquamative lesions around the mouth and is diagnosed with antiepiligrin
cicatricial pemphigoid. The autoantibodies are:
A. Anti-L-332 IgG autoantibodies
B. Anti-32 autoantibodies
C. Anti-IgM autoantibodies
D. Anti-332 autoantibodies
E. Anti-45 IgG autoantibodies
►A

Cicatricial pemphigoid affects the mucous membrane and genital areas. Anti-L-332 IgG
autoantibodies are very specific for antiepiligrin cicatricial pemphigoid.

18
The vector of fogo selvagem may be:
A. Triatoma
B. Simulium
C. Cimex
D. Ornithodorus
E. Mus
►B

287
Fogo selvagem is considered to be an endemic form of pemphigus foliaceus, first described in
Brazil, in rural areas where Simulium (black fly) is often found. Subsequently, other rural South
American areas have had clusters of affected patients.

19
Patients with latex allergy are LEAST LIKELY to develop a reaction to:
A. Cashew
B. Avocado
C. Chestnut
D. Banana
E. Kiwi
►A

Although rare, cross-reactivity in latex-allergic patients has been demonstrated to banana, kiwi,
avocado, passion fruit, and chestnut. Cross-reaction to cashew has NOT been reported.

20
Which pair is incorrect?
A. Wrinkle-resistance : formaldehyde
B. Chewing gum : colophony
C. Parabens : artichokes
D. Glyceryl thioglycolate : acid permanent wave
E. Permethrin : chrysanthemum
►C

Wrinkle-resistant clothing is treated with formaldehyde. Chewing gum contains rosin


(colophony). Parabens are preservatives. Artichokes are a member of the family Asteraceae
(formerly Compositae). Glyceryl thioglycolate and methylchloroisothiazolinone are found in
acid permanent wave products. Permethrins are synthetic pyrethroids that are similar to
pyrethrins derived from the Asteraceae family. Permethrins can cross-react with allergy to
chrysanthemums.

21
Gap junctions consist of:
A. Laminin
B. Plectin
C. Uncein
D. Filagrin
E. Connexins
►E

288
Gap junctions are protein channels that allow for communcation between cells. A connexon of
one cell is joined to that of an adjacent cell to form an intercellular channel consisting of 12
connexin subunits. Each intercellular channel provides a channel that interconnects the
cytoplasm of the apposed cell directly and permits the passage of ion and other small molecules
between adjacent cells. These protein channels that make up the gap junctions consist of two
hemi-channels or connexons. One connexon resides in the membrane of one cell and it aligns
and joins the connexon of the neighboring cell, forming a continuous aquaeous pathway by
which these ions and small molecules can freely pass from one cell to the other. Each hemi-
channel or connexon consist of six proteins ( hexamer) called connexins.

22
This is derived from Myroxylon pereirae:
A. Thimerosol
B. Aniline dyes
C. Balsam of Peru
D. Colophony
E. Thiuram
►C

Balsam of Peru is derived from the Myroxylon pereriae tree. Thimerosal is a questionable cause
of allergic contact dermatitis; it is found in vaccines and contact lens solutions.

23
Which pair is incorrect?
A. Dermatitis herpetiformis : granular IgA
B. Lichen planus : linear C3
C. Bullous pemphigoid : linear C3
D. Systemic lupus erythematosus : linear IgG
E. IgA pemphigus: intercellular IgA
►B

The colloid bodies in lichen planus can trap IgM and C3; this can be seen on DIF as “globs.”
There is no linear pattern.

24
Patients with epidermolysis bullosa acquisita have:
A. A defect in plectin
B. A defect in collagen XVII
C. Antibodies to collagen XVII

289
D. Antibodies to collagen VII
E. A defect in laminin 5
►D

Epidermolysis bullosa acquisita (EBA) classically presents in adulthood as noninflammatory


trauma-induced bullae that heal with scarring, especially on the hands and feet. Histology shows
a subepidermal blister (usually pauci-inflammatory); DIF shows linear IgG at the basememt
membrane; salt-split skin shows antibodies at the "floor". Patients have antibodies to type VII
collagen.

25
Antibodies to which antigen are most likely responsible for this disease?
A. Bullous pemphigoid antigen II
B. Desmoglein I
C. Desmocollin
D. Keratin 5
E. Plakoglobin
►A

The clinical appearance of tense bullae and urticarial plaques is suggestive of bullous
pemphigoid. Bullous pemphigoid antigen II is a transmembrane protein in the hemidesmosome.
Antibodies to this antigen are pathogenic in bullous pemphigoid.

26
Patients with Duhring's disease are most likely to have:
A. Mutations in plectin
B. Mutations in laminin 5
C. Mutations in transglutaminase I
D. Antibodies to transglutaminase 3
E. Antibodies to BPAg2
►D

Duhring's disease is also known as dermatitis herpetiformis. Antibodies are found to


transglutaminase 3, and the direct immunofluorescent studies show granular IgA and C3 in the
dermal papillae. Mutations in plectin are found in EBS with muscular dystrophy. Mutations in
laminin 5 are found in patients with JEB,Herlitz type. Mutations in transglutaminase I are found
in pateints with lamellar ichthyosis and non bullous congenital ichthyosiform erythroderma.

27

290
Direct immunofluorescent studies in chronic bullous disease of childhood is most likely to
show:
A. Iga depostion in the superficial blood vessels
B. Linear IgG at the basement membrane
C. Linear IgA at the basement membrane
D. Granular IgG
E. Linear C3 at the basement membrane
►C

Chronic bullouse disease of childhood is believed to be a variant of linear IgA bullous disease,
since disease are characterized by antibodies to the 97kDa antigen that is a part of BPAg2.
Children present with extremely pruritic blisters in circular arrangements. Pathology shows a
subepidermal bullae with neutrophils at the dermo-epidermal junction; the DIF shows IgA
deposition at the basement membrane in a linear fashionl.

28
The primary autoantigen in pemphigoid gestationis is
A. Desmoplakin
B. BPAG1
C. BPAG2
D. Plakoglobin
E. Anchoring fibrils
►C

Herpes gestationsis is an autoimmune blistering disorder of pregnancy. It is characterized by


urticarial plaques and tense bullae. Antibodies to BPAG2 are thought to form as a result of an
aberreant response to MHC antigens on the placenta. The maternal health is generally not
affected, but infants are more commonly born premature. Remission is common following
delivery, but it may recur with OCP's, menstruation, and subsequent pregnancies.

29
Papillary dermal deposits of IgA and a papillary dermal infiltrate of neutrophils is diagnostic of:
A. Sweet's syndrome
B. Leukocytoclastic vasculitis
C. Dermatitis herpetiformis
D. Linear IgA dermatosis
E. Bullous pemphigoid
►C

291
Dermatitis herpetiformis is an uncommon chronic, pruritic papulovesicular dermatitis occurring
most commonly in young to middle-aged adults. Common sites of predilection include the
buttocks, elbows, knees, scapula and scalp. Typical histologic features include accumulation of
neutrophils at the tips of dermal papillae, sometimes admixed with eosinophils. Direct
immunoflorescence (DIF) reveals granular deposits of IgA within the dermal papillae. As the
name implies, linear IgA disease is characterized by linear IgA deposition along the basement
membrane zone with DIF in 100% of cases. The pattern of direct immunoflorescence in bullous
pemphigoid is linear C3 deposition at the dermoepidermal junction in nearly 100% of cases and
IgG in 65-95% of cases. Sweet's syndrome is not characterized by a pattern with DIF. Under
light microscopy, a dense perivascular infiltrate of neutrophils is seen assuming a bandlike
pattern throughout the papillary dermis associated with prominent dermal edema.
Leukocytoclastic vasculitides are characterized by a perivascular neutrophilic infiltration with
karyorrhexis, leukocytoclasis, fibrinoid degeneration, and endothelial swelling.

30
Bullous lesions are most often seen in:
A. Congenital syphilis
B. Primary syphilis
C. Secondary syphilis
D. Tertiary syphilis
E. All of these answers are correct
►A

Congenital syphilis, can occasionally present with bullae. There are rare case reports of
secondary syphilis having bullae however, this is much less common than congenital syphillis.

31
Which association is incorrect?
A. Epidermolysis bullosa acquisita : inflammatory bowel disease
B. Dermatitis herpetiformis : small bowel lymphoma
C. Paraneoplastic pemphigus : Castleman‟s
D. Herpes gestationis : menopause
E. Porphyria cutanea tarda : hemochromatosis
►D

Herpes gestationis is an immunoblistering disease of pregnancy. Unlike pruritic urticarial


papules and plaques of pregnancy, it recurs with successive pregnancies. It is associated with
Grave's disease and choriocarcinoma.

32

292
A 35 year-old dentist presents with tingling in his fingertips. What allergen is most likely to
cause this dermatitis?
A. Methyl methacrylate
B. Paraphenylenediamine
C. Colophony
D. Thiuram mix
E. Ethylenediamine dihydrochloride
►A

Methyl methacrylate is found in synthetic resins, dentures, artificial, nail adhesives, and acrylic
bone cement. This allergen may penetrate through the gloves to the fingertips and most often
affects the first three fingers. In addition to dermatitis, it also causes a peripheral neuropathy.

33
Which of the following agent(s) has been most effective in treating severe ocular cicatricial
pemphigoid?
A. Cyclophosphamide
B. Mycophenolate mofetil
C. Cyclophosphamide + corticosteroids
D. Cyclosporin
E. Azathioprine
►C

Cicatricial pemphigoid is a heterogeneous group of subepithelial blistering diseases involving the


mucous membranes and skin. The treatment of cictracial pemphigoid is predicated upon the
extent, severity, and location of disease. Most regimens used are empiric and are based on
clinical experience. With severe ocular involvement, most experts recommend aggressive
treatment with the combination of cyclophosphamide and steroids.

34
What is the antigen found in the variant of cicatrical pemphigoid that is associated with internal
malignancies?
A. BPAg1
B. BPAg2
C. Type VII collagen
D. Epiligrin
E. Keratins 5 & 14
►D

293
Individuals with anti-epiligrin cicatricial pemphigoid are thought to be at an increased risk for
solid internal malignancies.

35
Dermatitis herpetiformis presents as grouped papules and vesicles symmetrically and is
associated with:
A. HLA-DQ2
B. HLA-Dw2
C. HLA-3 and HLA-4
D. HLA-DR3
E. HLA23Q
►A

Dermatitis herpetiformis is associated with HLA-DQ2 and has a positive IgA granular pattern in
the dermal papilla on direct IF and a negative indirect IF.

36
Which of the following concerning Degos' Disease is true?
A. After undergoing multiple stages, it resolves without scarring
B. It affects women more than men
C. Gatrointestinal involvement is a poor prognosis
D. Glucocorticoids are standard of treatment
E. Lab results indicate a low plasma fibrinogen level and decreased platelet
aggregation
►C

Dego's Disease (Malignant Atrophic Papulosis): It most frequently affects men and is a
potentially fatal obliterative arteritis. After undergoing multiple stages, the patient is left with
varicelliform scars. Later, anemic infarcts involve the intestines to produce acute abdominal
symptoms of epigastric pain, fever, and hematemesis. Death is usually due to fulminating
peritonitis caused by multiple perforations of the intestines. GI involvement is a poor prognosis.
Lab results indicate a high plasma fibrinogen level and increased platelet aggregation.
Administration of corticosteroids has not been beneficial.

37
Paraneoplastic pemphigus:
A. Is characterized by a pathognomonic 250 kDa antigen
B. Is most often seen in association with lung cancer
C. Remits with excision of the neoplasm
D. May be caused by a benign neoplasm

294
E. All of these answers are correct
►D

Paraneoplastic pemphigus can be caused by lymphoma as well as other malignancies. It can also
be secondary to benign thymomas and Castleman's disease. Numerous antigens have been found,
including desmoplakin I and II. Desmoplakin I and II may be antigenic in Stevens-Johnson
syndrome. It may not remit after treatment of the neoplasm.

38
Which neoplasm is the most common cause of paraneoplastic pemphigus?
A. Thymoma
B. CLL
C. Castleman's disease
D. Retroperitoneal sarcoma
E. Non-Hodgkin's lymphoma
►E

All of the above have been associated with paraneoplastic pemphigus with non-Hodgkin's
lymphoma being the most common. Castlemans's disease is most common in children with
paraneoplastic pemphigus.

39
Which of the following is true of herpes gestationis?
A. Demonstrates antibodies to the C-terminal domain of BPAg2
B. Has higher frequency in females with HLA-DQ2
C. Is associated with Grave's disease
D. Usually occurs in the first trimester of pregnancy
E. Shows subepidermal bullae mostly with neutrophils
►C

Herpes gestationis is associated with Grave's disease. It is more common with patients with
HLA-DR3 and DR4 and most often occurs between 4-7 months of pregnancy. The antibodies are
against the NC 16-A portion of BPAg2. Histology shows subepidermal bullae with mostly
eosinophils.

40
In penicillamine-induced pemphigus, the split is most often:
A. Subcorneal
B. Intraspinous
C. Suprabasal

295
D. Intraepidermal and subepidermal
E. Subepidermal
►A

Penicillamine is the most common cause of drug-induced pemphigus, and the split is more often
subcorneal (pemphigus foliaceus-like) than suprabasal (pemphigus vulgaris-like).

41
Which of the following is LEAST LIKELY to cross react with Toxicodendron plant dermatitis?
A. Japanese lacquer tree
B. Cashew nut
C. Mango rind
D. Gingko tree
E. Kiwi
►E

The Toxicodendron group of plants is the #1 cause of contact dermatitis in North America. This
group of plants includes poison ivy, poison oak, and poison sumac. Because the Toxicodendron
plants belong to the Anacardiaceae family, cross reactions can occur with related plants and
substances such as braqzilian pepper, cashew nut, cashew oil, gingkgo tree, Indian marking nut,
Japanese lacquer tree, mango, and Rengas tree. Kiwi can cross react with patients who are latex-
allergic.

42
Herpes gestationis is most commonly associated with:
A. Grave's
B. Hashimoto's
C. Diabetes
D. Lupus
E. Rheumatoid arthritis
►A

Autoimmune diseases are associated with herpes gestationis; Grave's disease is the most
common association.

43
The lamina densa is primarily composed of?
A. Connexins
B. Plectin
C. Collagen XVII

296
D. Collagen VII
E. Collagen IV
►E

Collagen IV is located in the dermal matrix and forms the anchoring plaque. The anchoring
plaque serves to anchor the lamina densa to the dermis via collagens 1,3 & 5.

44
IgA pemphigus is a pustular acantholytic dermatosis with intracellular IgA deposition. The most
common target on keratinocytes with patients with IgA pemphigus is:
A. Desmocollin1
B. Dsg2
C. IgG
D. C3
E. IgM
►A

IgA pemphigus is a pustular acantholytic dermatosis that targets desmocollin 1. Other targets can
include Dsg1 and Dsg3. The pathogenesis of IgA pemphigus is unknown but can be associated
with gammopathy.

45
The C-terminal domain of BPAg2 is targeted in:
A. Bullous pemphigoid
B. Cicatricial pemphigoid
C. IgA pemphigus
D. Duhring”s disease
E. Pemphigoid gestationis
►B

The C-terminal domain of BPAg2 is targeted in cicatricial pemphigoid. The NC16A domain of
BPAg2 is targeted in bullous pemphigoid.

46
Which of the associated antigens for this condition has been associated with colon cancer?
A. Anti-epiligrin
B. 97 kD linear IgA dermatosis antigen
C. Desmoplakin
D. Bullous pemphigoid antigen 2
E. Beta-4 integrin

297
►A

When cicatricial pemphigoid is associated with cancer it is due to anti-epiligrin (laminin 5)


antibodies. While the oral mucosa is the most common site for cicatricial pemphigoid, ocular
involvement is very common. Clinical manifestations may include shortened fornices,
symblephera, ankyloblephera. The majority of of ocular antigens are thought to reside in the
cytoplasmic domain of beta-4 integrin, therefore, the occular type of CP (not malignancy
associated) is due to ab to b4 integrin.

47
The defect in junctional epidermolysis bullosa occurs at the:
A. Granular layer
B. Spinous layer
C. Lamina lucida
D. Lamina densa
E. Sublamina densa
►C

Junctional epidermolysis bullosa is an autosomal recessive disorder characterized by a defect in


the lamina lucida. The protein uncein, a molecule which binds the gamma chain of laminin and
a6b4/BPAg1, is absent in all forms of junctional EB. Uncein is normally found in the lamina
lucida, along with lamin 1, laminin 5, laminin 6m, heparan sulfate proteoglycan, and fibronectin.

48
Chloracne may be secondary to exposure to:
A. Chloroacetophenone
B. Hydrochloric acid
C. Pyrethrin
D. Dioxin
E. Methylchloroisothiazolinone
►D

Chlorinated compounds can cause irritant dermatitis and sometimes chloracne. Chlorinated
compounds include chloronaphthalene, chlorodiphenyl, dichlorobenzonitrile,
tetrachloroazooxybenzene; dioxin (exposure in the Vietnam war); and cutting oils.
Chloroacetophenone is an irritant in tear gas.

49
All of the following are a form of acute porphyria except:
A. Acute intermittent porphyria

298
B. ALA-D deficient porphyria
C. Hereditary coproporphyria
D. Variegate porphyria
E. Congenital erythropoietic porphyria
►E

All of the following are acute porphyria except for congenital erythropoietic porphyria.

50
Which of the following is known as "prickly heat"?
A. Miliaria crystallina
B. Miliaria rubra
C. Miliaria profunda
D. Grover's disease
E. Fox-Fordyce disease
►B

Miliaria are caused by occusion of ecrine sweat glands. Miliaria crystalina are clear minute
subcorneal vesicles with no inflammation. Miliaria rubra are red papules subepidermal vesicles
with inflammation. Miliaria rubra is known as "prickly heat" as it presents after sweating, heat
exposure, fever or occlusion. Miliaria profunda or pustulosa are red nodules or pustules with
deeper inflammation. Fox-Fordyce disease is apocrine miliaria that is very pruritic and presents
in axilla and anogenital area of women. Grover's disease is a papulovesicular eruption of the
trunk that is pruritic and can occur after sun exposure or in bedridden patients with occluded
backs. Histologically Grover's disease presents with small foci of acantholytic dyskeratotic cells
in the suprabasal region.

51
The ocular form of cicatricial pemphigoid is most likely to be associated with antibodies to:
A. Beta-4-integrin
B. Laminin 5
C. BPAg1
D. BPAg2-NC16A
E. Plectin
►A

Antibodies to beta-4-integrin are associated with ocular cicatricial pemphigoid. Laminin 5 is


associated a variant of cicatricial pemphigoid that is associated with malignancy. BPAg1 is the
antigen in bullous pemphigoid, while BPAg2-NC16A is associated with both bullous

299
pemphigoid and herepes gestationis. Mutations in plectin are associated with epidermolysis
bullosa simplex with muscular dystrophy.

52
Which of the following adhesion molecules are important for maintaining adhesion between
keratinocytes?
A. Selectins
B. Integrins
C. Cadherins
D. Glycans
E. Immunoglobulin superfamily
►C

Cadherins mediate cell adhesion and play a fundamental role in normal development. Calicum is
required for the normal function of these adhesion molecules. There are two main subclasses,
classic (E-, P-, N-) and desmosomal (desmoglein and desmocollin).

53
Which of the following proteins is typically NOT found to be antigenic in paraneoplastic
pemphigus?
A. BPAg1
B. BPAg2
C. Desmoplakin
D. Envoplakin
E. Periplakin
►B

BPAg2, a 180kDa protein, is typically not found to be antigenic in paraneoplastic pemphigus.


The proteins typically involved are: desmplakin 1 (250kDa) BPAg1 (230 kDa) Envoplakin
(210kDa) Desmoplakin 2(210kDa) Periplakin (190 kDa) Unknown protein (170 kDa)
Desmogleins 1& 3 (160 and 130 kDa)

54
The agent most likely to cause pseudoporphyria is:
A. NSAID's
B. Penicillin
C. Beta-blockers
D. Vancomycin
E. Ace-inhibitors
►A

300
NSAIDS,especially naprosyn, are the most common group of drugs implicated in
pseudoporphyria. Other common offenders include lasix, tetracycline, accutane, amiodarone and
tanning beds.

55
Which of the following is the most common cause of contact dermatitis due to a formaldehyde
releasing preservative?
A. Paraben mix
B. Thimerosal
C. Quaternium 15
D. Imidazolidinyl urea
E. Paraphenylenediamine
►C

Quaternium 15 is the primary cause of contact dermatitis among formaldehy-releasing


preservatives. Other formaldehyde-releasing preservatives include imidazolidinyl urea,
diazolidinyl urea, DMDM hydantoin, Bronopol, Cyanobutane, Ethyleneurea melamine
formaldehy resin (found in permanent press clothing), and dimethylol dihydroxyethyleneurea.
Paraben mix (#1 preservative in topicals), thimerosal (a type of mercurial antiseptic), and
paraphenylenediamine (hair dye) are non-formaldehyde releasing preservatives.

56
This is found in both desmosomes and adherens junctions:
A. Alpha-catenin
B. Desmoglein 3
C. Desmoglein 1
D. Plakoglobin
E. Lamin
►D

Plakoglobin is a plaque protein of the armadillo family that is found in desmosomes; it can also
substitute for beta-catenin in adherens junctions. Plakoglobin co-precipitates with desmogleins. It
is 85 kDa.

57
The croton plant is irritating secondary to:
A. Calcium oxalate
B. Thiocyanate
C. Protoanemonin

301
D. Phorbol esters
E. Capsaicin
►D

Irritant contact dermatitis secondary to the croton plant is due to phorbol esters. Calcium oxalate
is also an irritant; it is found in the "dumb caine" plant, a common house plant (Dieffenbachia) as
well as in daffodils. Thiocyanates are irritants found in garlic. Protoanemonins are irritants found
in buttercups.

58
Latex allergy can cross-react with all except:
A. Avocado
B. Banana
C. Kiwi
D. Chestnut
E. Artichoke
►E

Latex allergy can cross-react with avocados, bananas, kiwis, chestnuts, and other foods.
Artichokes are a member of the Asteraceae (formerly Compositae) family.

59
Hereditary coproporphyria is characterized with photosensitivity and the defect is:
A. Coproporphyrinogen oxidase
B. Porphobilinogen deaminase
C. Uroporphyrinogen III synthase
D. Uroporphyrinogen decarboxylase
E. Uroporphyrinogen decarboxylase
►A

Hereditary coproporphyria is characterized by photosensitivity and can also present with


neuropsychiatric issues. They also have skin fragility and blistering is common.

60
Which of the following contains the same allergen as poison sumac?
A. Mango fruit
B. Brazilian Pepper Tree
C. Balsam of Peru
D. Ragweed
E. Artichoke

302
►B

The rhus family (poison ivy, poison oak, poison sumac) contain urushiol (a pentadecacatechol).
This allergen is also associated with contact allergy to Cashew nut (shell), Japanese laquer tree,
Mango rind (not fruit), Gingko, Brazilian pepper tree, Pepeo tree of Venezuela, El litre tree of
Chile and the Rengas (black varnish) tree.

61
Which of the following proteins is a 160kDa cadherin?
A. Desmoglein 3
B. Desmoglein 1
C. Desmoplakin I
D. Envoplakin
E. Plectin
►B

Desmogleins and desmocollins belong to the cadherin family of proteins, which are calcium-
dependent adhesion molecules. Desmoglein I is a 160kDa cadherin, while Desmoglein 3 is a 130
kDa cadherin. Desmoplakin, envoplakin, and plectin belong to the plakin family of adhesion
proteins.

62
Nickel can be detected in jewelry by applying:
A. Dimethylglyoxime
B. Methylchloroisothiazolinone
C. Dowicil
D. Quinone
E. Thiocyanate
►A

Dimethylglyoxime application to metals containing nickel will show a pink color.


Methylchloroisothiazolinone is Kathon CG, a preservative. Dowicil is a formaldehyde-releasing
preservative. Primin is a type of quinone, and it is the allergen in primrose (Primula obconica).
Thiocyanates are the irritating substances in garlic.

63
Patients that are diagnosed with erythropoietic protoporphyria do not suffer form acute attacks.
They have a defect in the:
A. Ferrochelatase
B. Protoporphyrinogen oxidase

303
C. Uroporphyrinogen decarboxylase
D. Coproporphyrinogen oxidase
E. Prophobilinogen deaminase
►A

Patients with erythropoietic protoporphyria have a defect in the ferrochelatase. These patients do
not have acute attacks characterized by abdominal pain and neurologic problems.

64
The best substrate for paraneoplastic pemphigus for indirect immunoflourescent is:
A. Rat bladder
B. Rat liver
C. Rat stomach
D. Monkey esophagus
E. Guinea pig esophagus
►A

Paraneoplastic pemphigus has on direct immunofluorescence deposits of IgG and C3 in the


basement membrane and also in the intracellular spaces of the epidermis. On IIF, the best
substrate to use is rat bladder.

65
What is the most common allergen causing allergic contact dermatitis?
A. Nickel
B. Bacitracin
C. Formaldehyde
D. Quaternium-15
E. Neomycin
►A

Nickel is the most common contact allergen, found in “costume” jewelry, alloys, pigments
orthopedic appliances, scissors, razors and many other metal coated objects. Bacitracin and
neomycin are commonly used topical antibiotic agents that many are allergic to. Neomycin is the
most common topical antibiotic allergen found in testing. Quaternium-15 is a formaldehyde-
releasing preservative found in many topical products. It can cross react with formaldehyde, but
not all patients will react to formaldehyde on testing. While it is a common allergen, it is not the
most common cause of ACD.

66
Formaldehyde releasers include all except:

304
A. Methylchoroisothiazolinone
B. Germall
C. Quaternium – 15
D. Dowicil
E. DMDM hydantoin
►A

Methylchloroisothiazolinone is also known as Kathon CG and it is a preservative. Germall,


quaternium 15 (dowicil 200), DMDM hydantoin, and others are formaldehyde-releasing
preservatives.

67
Transient bullous disorder of childhood has a defect in:
A. Alpha6-integrin
B. Plectin
C. BPAg2
D. Epiligrin
E. Collagen VII
►E

Transient bullous dermolysis of the newborn may be a forme fruste/variant of dominant


dystrophic EB. Mutations have been found in type VII collagen.

68
Which of the following is true of a phototoxic reaction?
A. Requires prior sensitization of photoallergen in susceptible individuals
B. Photoallergen must bind to carrier protein
C. Develops after repeated exposures
D. Produces an eczematous reaction
E. Results in direct tissue injury
►E

Phototoxic reactions result in direct tissue injury when the phototoxic agent interacts with light
energy to form reactive oxygen species. It is characterized by an eruption similar to a sunburn.

69
The epidermolysis bullosa acquisita antigen is:
A. A transmembrane protein
B. An intracellular protein
C. A desmosomal protein

305
D. Located in the lamina lucida
E. An anchoring fibril
►E

Epidermolysis bullosa acquisita is a rare autoimmune disorder with an estimated incidence of


0.25 per million. In the classic form, it is characterized by non-inflammatory bulla with a
prediliction for traumatized sites. The autoantigen is type VII collagen, which is an anchoring
fibril located in the sublamina densa. Following salt split skin, the type VII collagen can be
localized at the base.

70
Which of the following chemicals is the most common cause of shoe contact dermatitis?
A. Thiuram mix
B. 2-Mercaptobenzothiazole
C. Carba mix
D. Mixed dialkyl thioureas
E. Formaldehyde
►B

2-Mercaptobenzothiazole is found in the soles of shoes, and is the #1 cause of shoe contact
dermatitis. It is also found in rubber accelerators, adhesives/tape, coolants, rubber, flea/tick
powder, and support hose. Thiuram mix, carba mix, and mixed dialkyl thioureas are also rubber
sensitizers. Thiuram is the second most frequent source of shoe contact dermatitis. Formaldehyde
is a preservative and not found in shoes.

71
What is the most common site of involvement of this autoimmune blistering disease?
A. Oral mucosa
B. Eyes
C. Skin
D. Genitalia
E. Nasopharyngeal mucosa
►A

Cicatricial pemphigoid is a heterogeneous group of diseases with characteristic involvement of


the oral mucosa, eyes, skin. Rarely, it can also involve the nasopharyngeal mucosa, external
genitalia, esophagus, and anus. The oral mucosa is the most common site of involvement.

72

306
A patient who works in a photography developing lab complains of eczematous dermatitis on his
hands. Which one of the following chemicals is he most likely allergic to?
A. Mercaptobenzothiazole
B. Lanolin
C. Epoxy resin
D. Parapheylenediamine
E. Colophony
►D

Photography solutions often contain parapheneylenediamine, or PPD. Mercaptobenzothiazole is


a rubber accelerator found in shoes, rodenticides, and insecticides, and is the most common
allergen of health care workers. Lanolin is a moisturizer found in cosmetics including aquaphor
and eucerin. Epoxy resin is found in glues and marine cement and, if positive, has a high degree
of clinical relevance. Colophony has the active ingredient of abietic acid and is found in pine
rosin, violins, soaps, and adhesives.

73
A patient develops erythema and vesiculation on his dorsal foot that spares the toewebs and soles
after buying a new pair of shoes. What is the most likely allergen?
A. Thiuram mix
B. Carbamates
C. Latex
D. Mercaptobenzothiazole
E. Formaldehydge
►D

This patient has allergic shoe dermatitis. The number one cause is mercaptobenzothiazole, a
rubber accelerator. Other rubber accelerators include carbamates--found in bleached fabrics like
waistbands and consumer rubber products such as condoms, swimwear etc--and thiuram mix,
found in almost all rubber. Of note thiuram is also present in antiscabies medication and
disulfiram.

74
At what level is the blister separation plane in linear IgA dermatosis?
A. Granular layer
B. Suprabasal layer
C. Basal layer
D. Lamina lucida
E. Lamina densa
►D

307
The cleavage in linear IgA dermatosis (LAD) typically occurs in the lamina lucida as the target
antigen is the 97 kD cleaved portion of bullous pemphigus antigen II (BPAgII). A variant of
LAD may separate in the sublamina densa, with a target antigen of the type VII collagen in the
anchoring fibrils. This condition may also be induced by ingestion of certain medications
including vancomycin, lithium and diclofenac.

75
Pinus palustris is the source of:
A. Urea
B. D-limonene
C. Quinones
D. Lactones
E. Abietic acid
►E

Abietic acid is the allergenic component of rosin (colophony). D-limonene is found in tea tree
oil. Primin is a quinone; it is found in the primrose (Primula obconica). Abietic acid is found in
rosin.

76
Which of the following allergens is a non-formadelhyde releasing preservative with a low
sensitization potential?
A. Methylisothiazolone
B. Paraben
C. Bronopol
D. Quternium-15
E. DMDM hydantoin
►B

Paraben mix is a non-mercurial, non-formaldehydge releasing preservative with low sensitization


potential. Methylisothiazolone is a non-formaldehydge releasing preservative found in wash-
on/wash-off products. The others are formaldehyde-releasing preservatives.

77
Which of the following immunosuppressive agents has been most effective in cicatricial
pemphigoid?
A. Cyclophosphamide
B. Azathioprine
C. Mycophenolate mofetil

308
D. Cyclosporine
E. Methotrexate
►A

Cicatricial pemphigoid is a heterogeneous group of subepithelial blistering diseases involving the


mucous membranes and skin. The treatment of cictracial pemphigoid is predicated upon the
extent, severity, and location of disease. Most regimens used are empiric and are based on
clinical experience. With severe ocular involvement, most experts recommend aggressive
treatment with the combination of cyclophosphamide and steroids.

78
What is the antigen associated with dermatitis herpetiformis?
A. Tissue transglutaminase
B. Epilegrin
C. Desmocollin
D. Desmoglein 3
E. Periplakin
►A

Dermatitis herpetiformis is an autoimmune blistering disease associated with intense pruritus.


Classically, it involves the extensor aspects of the body. Antibodies to tissue transglutaminase
(anti-endomesial, anti-gliadin, anti-reticulin) can be found. It is associated with celiac sprue. 50%
have thyroid disease.

79
Epidermolysis bullosa simplex is caused by mutations in:
A. Keratins 1&10
B. Keratins 1&9
C. Keratins 5&14
D. Keratins 6a &16
E. Keratins K2e&10
►C

EB Simplex is caused by mutations in keratins 5 &14. Mutations in Keratins 1&10 are associated
with epidermolytic hyperkeratosis and Unna-thost. Mutations in keratins 1&9 are associated with
epidermolytic palmoplantar keratoderma. Mutations in Keratins 6a and 16 are associated with
Pachyonychia congenital. Mutations in K2e and 10 are associated with Icthyosis bullosa of
Siemens.

80

309
Which pair is incorrect?
A. Eyedrops : thimerosal
B. Ear drops : neomycin
C. Theophylline : ethylenediamine
D. Antabuse : thiuram
E. Cobalamin : balsam
►E

Cobalamin is Vitamin B12; cobalt is a component of Vitamin B12. Balsam of Peru is found in
fragrances.

81
Which type of porphyria manifests with peripheral neuropathy, colicky abdominal pain, but NO
cutaneous symptoms?
A. Acute Intermittent Porphyria
B. Gunther's disease
C. Variegate porphyria
D. Hereditary Copropophyria
E. Erythropoietic protopophyria
►A

Acute intermittent pophyria is caused by a defect in the enzyme porphobilinogen deaminase.


Patients present with peripheral neuropathy and occasionally seizures, abdominal pain, and
tachycardia. There are no skin changes as no dermal porphyrins are found.

82
The best substrate for indirect immunofluorescent for pemphigus foliaceous is:
A. Guinea pig eophagus
B. Rat Bladder
C. Money esophagus
D. Salt-split skin
E. Rat liver
►A

The best indirect immunofluorescent substrate for pemphigus foliaceous is guinea pig esophagus.
Pemphigus foliaceous has antigens against Dsg 1 and presents as erosions on the chest.

83
The most important group of disease linked with the pathogenesis of paraneoplastic pemphigus
in adults is

310
A. Lymphoproliferative disorders
B. Colerectal adenocarcinoma
C. Breast Cancer
D. Sarcomas
E. Melanoma
►A

The most important group of disease linked witht eh pathogenesis of pareneoplastic pemphigus
is Lymphoproliferative disorders. Most commonly non-Hodgkin's lymphoma, CLL, Castleman's
tumor, sarcoma, and thymoma.

84
"Oriental" tiger balm cross-reacts with:
A. Balsam of Peru
B. Ylang-Ylang oil
C. Neomycin
D. Lanolin
E. Rosin
►A

"Oriental" tiger balm is a popular Chinese proprietary ointment used to relieve aches and pains.
Patients who develop allergic contact dermatitis to this agent have also been shown to have
cross-reactions with balsam of Peru. A similar substance known as "Bavarian" tiger balm has
been reported to cross react with rosin.

85
Cicatricial pemphigoid can be induced by:
A. Aminoglycosides
B. Benzene
C. Clonidine
D. Vancomycin
E. Ciprofloxacin
►C

Clonidine is a cause of drug-induced cicatricial pemphigoid. Other common causes include sulfa
drugs and thiol-containing drugs.

86
Which pair is not associated with allergic cross-reactivity?
A. Thimerosal : piroxicam

311
B. Ethylenediamine : hydroxyzine
C. PABA : procaine
D. Balsam of Peru : cinnamon
E. Thiuram : sulfa drugs
►E

Thiurams may cross-react with Antabuse (disulfiram).

87
A 59-year old female has erythema and vesiculation in an area of application of tincture of
benzoin. Which allergen is she also most likely allergic to?
A. Ammonium persulfate
B. Black rubber mix
C. Balsam of Peru
D. Eugenol
E. Epoxy resin
►C

This patient who has a contact dermatitis to tincture of benzoin is probably reacting to its
allergenic ingredient Balsalm of Peru which has healing and antibacterial properties. Ammonium
persulfate is a bleaching agent for hair and flour, not to be confused with ammonium
thioglycolate, a perming agent. Black rubber mix can produce unique lichenoid and purpuric
reactions. Eugenol is found in cloves. Epoxy resin is found in glues and plastics.

88
Fiberglass dermatitis can be prevented by:
A. Water
B. Acetic acid 5%
C. Sodium chloride
D. Alkali
E. Talcum powder
►E

Talcum powder application causes fiberglass spicules to slide off skin.

89
Herpes gestationis is exacerbated by:
A. Oral contraceptives
B. Menstruation
C. Third trimester

312
D. Postpartum state
E. All of these answers are correct
►E

Herpes gestationis has also been associated with choriocarcinoma in several case reports.

90
Which of the following is not a formaldehyde-releasing preservative?
A. Bronopol
B. Methylchloroisothiazinolone
C. Quaternium-15
D. Imidazolidinyl urea
E. DMDM hydantoin
►B

Methylchloroisothiazinolone, also known as Kathon CG, is the only non-formaldehyde releasing


preservative. It is found in cosmetics, skin/hair products, "acid" permanent waves, soaps, latex
emulsions, and biocides. All the others are formaldehyde-releasing preservatives.

91
Ethylenediamine dihydrochloride is known to cross-react with all the following substances
except?
A. Neomycin
B. Aminophylline
C. Promethazine
D. Meclizine
E. Hydroxyzine
►A

Ethylenediamine dihydrochloride is a stabilizer found in aminophylline, theophylline,


antihistamine creams, and Mycolog cream (nystatin/triamcinolone). It is also used in synthetic
coolants, rubber accelerators, insecticides, and fungicides. It is not known to cross react with
neomycin.

92
Hepatoerythropoietic porphyria is one of the acute porphyrias. The defect in this disease is:
A. Uroporphyrinogen decarboxylase
B. Porphobilinogen deaminase
C. Uroporphyrinogen III synthase
D. Uroporphyrinogen decarboxylase

313
E. Protoporphyrinogen oxidase
►A

Patients with hepatoerythropoietic porphyria have a defect in uroporphyrinogen decarboxylase.


This is an acute prophyria characterized by attacks.

93
Irritation of the hand produced by capsaicin can be relieved by application of
A. Water
B. Acetic acid 5%
C. Sodium chloride
D. Alkali
E. Talcum powder
►B

Burning secondary to capsaicin can be relieved by vinegar (acetic acid 5%) as the capsaicin is
soluble in vinegar (but not water).

94
Epidermolysis bullosa simplex with muscular dystrophy is associated with a mutation in:
A. Laminin 5
B. Plectin
C. collagen VII
D. uncein
E. alpha 6 beta integrin
►B

EB Simplex is an autosomal dominant disorder caused by mutations in keratins 5 &14, leading to


intraepidermal bullae. A subtype of EBS patients have muscular dystrophy due to a defect in
plectin. Laminin 5 mutations are associated with junctional EB. Collagen VII mutations are
associated with dystrophic EB. Uncein is a protein in the lamina lucida which is found to be
absent in junctional EB. Mutations in alpha6-beta4 integrins are associated with JEB with pyloric
atresia.

95
A 15-year-old boy presents with keratotic follicular papules involving the bilateral dorsal arms,
thighs, and cheeks. Analysis of an unaffected region of skin would likely exhibit which of the
following findings:
A. Diminished filaggrin
B. Diminished loricrin

314
C. Diminished involucrin
D. Diminished lamellar bodies
E. Decreased Transglutaminase I activity
►A

Filaggrin mutations are most commonly associated with atopic dermatitis, but have also been
implicated in keratosis pilaris as described in this question. Lamellar bodies are diminished in
Flegel‟s disease and absent in Harlequin icthyosis. Diminished tissue transglutaminase I activity
is associated with lamellar icthyosis and non-bullous congenital icthyosiform erythroderma.

96
Each of the following is true about anti-p200 pemphigoid except:
A. Responsive to dapsone
B. Subepidermal bullae
C. 200-kd antigen
D. features of linear IgA disease
E. mucosal involvement
►E

Anti-p200 pemphigoid is an autoimmune blistering disorder. These autoantibodies bind to a 200-


kD antigen found at the lamina lucida-lamina densa interface, thereby causing a subepidermal
bullae. Clinical features of this disease are similar to bullous pemphigoid, dermatitis
herpetiformis and linear IgA disease. In the case reported by Egan, et.al., the patient responded
well to steroids and dapsone and did not have mucosal lesions.

97
Which type of porphyria disease is most likely to be associated with cholelithiasis?
A. Acute intermittent porphyria
B. Congenital erythropoeitic porphyria
C. Hereditary coproporphyria
D. Erythropoetic protoporphyria
E. Variegate porphyria
►D

Erythropoeitic protoporphyria is caused by a defect in the enzyme ferrrochetalase. Patients


present with immediate photosensitivity and subsequently develop waxy scarring on the face,
hands and knuckles. In addition to abdominal pain, cholelithiasis and liver disease may also
occur. The porphyria diseases associated with abdominal pain are acute intermittent porphyria,
hereditary coproporphyria and variegate porphyria. Congenital erythropoeitc porphyria is
associated with splenomegaly.

315
98
Extension of this blister by application of perpendicular pressure is an example of what?
A. Fitzpatrick sign
B. Nikolsky sign
C. Asboe-Hansen's sign
D. Hutchinson's sign
E. Gorlin's sign
►C

Toxic epidermal necrolysis is a serious and potentially life threatening reaction to medications.
The most commonly implicated medications include penicillins, NSAIDS, and anti-convulsants.
Asboe-Hansen's sign results in extension of a blister with perpendicular pressure. Nikolsky sign
is the separation of epidermis from the dermis by application of tangential mechanical pressure.
Both are associated with TEN.

99
Patients that have congenital erythropoietic porphyria have a defect in:
A. Uroporphyrinogen III sythase
B. Porphobilinogen deaminase
C. Uroporphyrinogen decarboxylase
D. Coproporphyrinogen oxidase
E. Uroporphyrinogen decarboxylase
►A

Patients with congenital erythropoietic porphyria have a defect in uroporphyrinogen III


decarboxylase.

100
Direct immunofluorescent studies in a patient with bullous pemphigoid is most likely to show:
A. Linear IgA at the basement membrane
B. Linear C3 at the basemement membrane
C. Granular IgA in dermal papillae
D. Intercellular IgG4 throughout the epidermis
E. C3 in the dermal papillae
►B

In bullous pemphigoid, the antigenic targets are believed to be BPAg1 and BPAg2-NC16A
domain. These proteins are located in the hemidesmosome. Direct immunofluorescent studies
reveal linear basement membrane C3in approximately 95% of patients, and IgG4 in

316
approximately 80%. Linear IgA is found in linear IgA and chronic bullous deiseasaeof
childhood. Granular IgA and C3 in the dermal papillae is found in dermatitits herpetiformis.
Intercellular IgG4 throughout the epidermis is found in pemphigus vulgaris.

101
The antigen in linear IgA is:
A. BPAg1
B. Plectin
C. 97kDa part of BPAg2
D. Laminin 5
E. Alpha-6, beta integrin
►C

The antigen in linear IgA and its variant, chronic bullous disease of childhood, is the 97kDa part
of BPAg2. BPAg1 is the antigen in bullous pemphigoid. Plectin is defective in EB simplex with
muscular dystrophy. Laminin 5 is defective in junctional EB and alpha 6, beta integrin is
defective in junctional EB with pyloric atresia.

102
In Brunsting-Perry pemphigoid, the recrurrent crops of blisters are most likely to appear on:
A. Genitals
B. Palms and soles
C. Umbilicus
D. Head and neck
E. Buttocks
►D

Brunsting-Perry pemphigoid is a variant of cicatricial pemphigoid in which there are no mucosal


lesions. The most common site of blisters is the head and neck, resulting in scarring alopecia.
The antigens in cicatricial pemphigoid include BPAg2 and epiligrin.

103
Allergic contact dermatitis is characterized histologically by:
A. Psoriasiform dermatitis
B. Lichenoid infiltrate
C. Spongiosis
D. Parakeratosis
E. Granuloma
►C

317
Spongiosis refers to the accumulation of edema fluid between keratinocytes, in some cases
progressing to vesicle formation. The epidermis acquires a "spongy" appearance. This
histopathologic pattern of inflammation is usually accompanied by an infiltrate of lymphocytes
within the epidermis and around superficial vessels. In addition to spongiosis, allergic contact
dermatitis is typically accompanied by eosinophils.

104
Clumped tonofilaments are seen on electron microscopy in:
A. Dowling-Meara
B. Weber-Cockayne
C. Gunther”s
D. Naxos disease
E. Vohwinkel”s syndrome
►A

Clumped tonofilaments (keratin intermediate filaments) are seen in EB simplex subtype


Dowling-Meara. Weber-Cockayne is a subtype of EB simplex with trauma-induced blistering on
the soles of the feet and occasionally palms.

105
The direct IF and indirect IF for linear IgA bullous dermatosis is:
A. DIF: linear IgA at the basement membrane zone IIF: linear IgA at the basement
membrane zone
B. DIF: granular IgA in dermal papillae IIF: negative
C. DIF: negative IIF: negative
D. DIF: granular IgG in dermal papillae IIF:negative
E. DIF: linear IgG at the basement membrane zone IIF: linear IgG at the basement
membrane zone
►A

Linear IgAhas IgA at the basement membrane zone on DIF and on IIF has linear IgA at the
basement membrane zone.

106
Which of the following is associated with dermatitis herpetiformis?
A. Gluten-sensitive enteropathy
B. Inflammatory bowel disease
C. Autoimmune hepatitis
D. Herpes labialis
E. Rheumatoid arthritis

318
►A

Gluten-sensitive enteropathy or Celiac disease is demonstrated on small bowel biopsy of all


patients with dermatitis herpetiformis. However, most of these patients do not demonstrate
symptoms of gastrointestinal disease. Dermatitis herpetiformis (DH) is not associated with
inflammatory bowel disease, autoimmune hepatitis, herpes labialis, or rheumatoid arthritis.

107
Common cause(s) of drug-induced pemphigus foliaceus:
A. Captopril
B. Penicillamine
C. Captopril and penicillamine
D. Methotrexate
E. Dilantin
►C

Penicillamine and captopril are common causes of drug-induced pemphigus.

108
This is associated with deafness:
A. Claudin
B. Cadherin
C. Connexin
D. Desmin
E. Desmoplakin
►C

Connexin 26 is associated with PPK and deafness. Mutations in this same connexin is associated
with Vohwinkel‟s and KID syndromes.

109
Leiner's disease (erythroderma desquamativum) is associated with:
A. C5-9 deficiency
B. Numerous infections
C. Diarrhea
D. Numerous infections and diarrhea
E. All of these answers are correct
►D

319
Leiner's disease is associated with deficient C5 and possibly C3. Babies with this disease are
prone to diarrhea, infections (sepsis), anemia, and a generalized seborrheic dermatitis-like rash.

110
The most common cause of contact dermatitis in the United States is:
A. Hair dyes
B. Nickel
C. Nail lacquers
D. Toxicodendrons
E. Rubber
►D

The most common cause of contact dermatitis in the USA are (in order of most to least
common): toxicodendrons (poison ivy, oak, or sumac), fragrance, thimerosal, quarternium-15,
neomycin, formaldehyde and the formaldehyde-releasing preservatives, bacitracin, and rubber
compounds.

111
Antibodies in some forms of Stevens-Johnson bind to:
A. Desmoplakin I
B. Desmoglein 1
C. Desmoglein 3
D. Plakoglobin
E. Desmoplakin I and desmoglein 3
►A

Antibodies against desmoplakin I have been found in Stevens-Johnson.

112
A child has a defect in type VII collagen and presents with acral blisters which heal with milia
and scarring. What is the diagnosis?
A. Albopapuloid/Pasini variant of dystrophic epidermolysis bullosa
B. Bart's syndrome
C. Cockayne-Touraine disease
D. Epidermolysis bullosa acquisita
E. Weber-Cockayne
►C

This patient has Cockayne-Touraine disease which is characterized by a defect in type VII
collagen and presents with acral blisters which heal with milia and scarring. The albopapuloid

320
variant of dystrophic epidermolysis bullosa presents with flesh-colored, scar-like papules and
dystrohpic nails. Bart's syndrome is associated with aplasia cutis congenita usually on the
extremities. Weber-Cockayne does have acral lesions but it is associated with a defect in keratins
5 and 14. The pathogenesis of epidermolysis bullosa acquisita involves autoimmunity to collagen
VII, not a defect in it.

113
In staphylococcal scalded skin syndrome, the exfoliative toxin cleaves:
A. Desmoglein 1
B. Desmoglein 3
C. Desmocollin 1
D. Desmocollin 3
E. BpAg2
►A

Staphylococcal scalded skin syndrome is caused by Staphylococci group 2, phage typ 71, which
produces an exofliative toxin that cleaves desmoglein 1. Clinically, this results in a subcorneal
separation of keratinocytes, leading to skin tenderness, erythema, and superficial desquamatioin
of skin.

114
What percentage of dermatitis herpetiformis patients are asymptomatic but have findings
consistent with celiac sprue on gastrointestinal biopsy?
A. 5%
B. 15%
C. 40%
D. 70%
E. 95%
►E

Most patients with dermatitis herpetiformis have findings consistent with celiac disease on
biopsy, but not all of them are symptomatic.

115
Drugs that are associated with the exacerbation of pemphigus foliaceus include :
A. Captopril
B. Metoprolol
C. Fluconazole
D. Calcium channel blockers
E. Statins

321
►A

Pemphigus foliaceus is a form of pemphigus with superficial blisters. The autoantibodies are
against Dsg1. Medications associated with the exacerbation of pemphigus foliaceus is sulfhydryl
groups such as captopril, penicillamine and piroxicam. Metroprolol can exacerbate psoriasis.

116
Which of the following can be responsible for contact dermatitis to K-Y Jelly?
A. Lanolin alcohol
B. Budesonide
C. Alpha tocopherol
D. Propylene glycol
E. Triclosan
►D

Propylene glycol is a widely used solvent and humectant found in a variety of products such as
cosmetics, lotions, corticosteroids, antiperspirants, and K-Y jelly. Lanolin is an emollient which
comes from wool wax and is found in adhesives, cosmetics, and topical emollients such as
Aquaphor. Budesonide is a steroid, alpha tocopherol is topical Vitamin E, and Triclosan is a
topical antibiotic.

117
Cleavage of this adhesion molecule is seen in Staphylococcal Scalded Skin Syndrome:
A. Desmoplakin 1
B. Desmoplakin 2
C. Desmoglein 1
D. Desmoglein 3
E. Desmoplakin 1 and desmoplakin 2
►C

Desmoglein 1 is cleaved by exfoliative toxin in Staphylococcal scalded skin syndrome.

118
The genetic predisposition for patients with pemphigus vulgaris is:
A. HLA-DRQ402
B. HLA-DR3
C. HLA-DR4
D. HLA-0505
E. HLA-Bw3
►A

322
Patients with HLA-DRQ402 can be predisposed to the development of pemphigus vulgaris.
Pemphigus vulgaris is a severe potentially fatal disease with blister formation on the skin and the
mucosa

119
You are consulted on this patient who has just undergone an orthopedic spinal procedure. What
is true of this patient?
A. She should avoid bananas
B. She is allergic to betadine
C. She is most likely having an allergic response to an antibiotic
D. She has miliaria crystallina
E. This is not related to her procedure
►A

This patient had an allergic contact dermatitis to latex products used during the procedure. Latex
cross-reacts with bananas, avocados, kiwis and chestnuts. The linear and geometric distribution
is a clue to the diagnosis.

120
The Asboe-Hansen sign as applies to blister formation refers to
A. Lateral slipping of the epidermis when perilesional is rubbed
B. Lateral dissection of the blister when pressure is applied directly to a blister
C. Blister formation following stroking of the lesion
D. Blister induction with incised trauma to skin
E. Blister formation after cryosurgery
►B

The Asboe-Hansen sign as applies to blister formation refers to lateral dissection of the blister
when pressure is applied directly to the blister. This is also called, "bulla-spread phenomenon",
because pressure on an inact bulla, genlty forces fluid to wander under the skin away from the
pressure site.

121
Bullous lupus erythematosus is most commonly associated with antibodies to:
A. Type XII collagen
B. Type VII collagen
C. Plectin
D. Desmoglein I
E. Laminin 5

323
►B

Type VII collagen is found in the sublamina densa and patients with bullous lupus erythematosus
have been found to have antibodies to this protein. The histology can resemble that of dermatitis
herpetiformis and DIF shows linear IgG, IgM, IgA and C3.

122
Th2-related cytokines include:
A. IL10
B. IFNgamma
C. IFNalpha
D. IL10 and IFNgamma
E. All of these answers are correct
►A

Atopic dermatitis is considered a disease in which a Th2 pattern predominates, especially early
on. Th2 cytokines are IL4, IL5, IL6, IL10, IL13.

123
Which autoantibodies would most likely be found in an individual with eye findings without any
cutaneous involvement?
A. Alpha 6 beta 4 integrin
B. Bullous pemphigoid antigen 1
C. Bullous pemphigoid antigen 2
D. Desmocollin
E. Laminin 5
►A

Cicatricial pemphigoid is a rare, heterogeneous group of blistering diseases with predilection for
mucous membranes. It may involve the eyes, oral mucosa, genital/anal mucosa, and skin. The
ocular form of cicatricial pemphigoid without cutaneous findings is usually associated with
circulating autoantibodes to the Beta subunit of alpha 6 beta 4 integrin.

124
The best substrate for indirect immunoflurescent testing in bullous pemphigoid is:
A. Salt-split skin
B. Monkey esophagus
C. Rat bladder
D. Guinea pig esophagus
E. Rat liver

324
►A

For bullous pemphigoid the indirect immunofluorescent that should be used is salt split skin.

125
A 29 year old female patient is diagnosed with fogo selvagem. The likely possibility that this
person with die within two years is :
A. 90%
B. 70%
C. 40%
D. 20%
E. 10%
►C

Patients with fogo selvagem have a 40% of dying within 2 years with they do not receive
treatment. These patient have burning lesions that is denuded with roll of pushed back upper
epidermis at periphery. Patients can also develop erythroderma

126
Common drugs causing SJS/TEN include all except:
A. Sulfonamides
B. Carbamazepime
C. Allopurinol
D. Oxicam
E. Colchicine
►E

NSAIDs, anticonvulsants, allopurinol, and sulfa drugs are common inciting agents of SJS/TEN.

127
Which is incorrect?
A. Tulipalin : alstroemeria
B. Urushiol : anacardia
C. Sesquiterpene lactones : artichokes
D. Usnic acid : lichen
E. Diallyl disulfide : fig
►E

Tulipalin A is a byproduct of tuliposide A metabolism, and it is the allergen in alstroemeria, the


Peruvian lily, and tulips. Urushiol is the allergen in Toxicodendron plants as well as several other

325
cross-reactors including cashew nut shells (Anacardium occidentale). Artichokes are in the
family Asteraceae (formerly Compositae); this family is allergenic secondary to sesquiterpene
lactones. Lichens contain usnic acid. The fig tree is in the family Moraceae, which is one of the
plant families that can cause phytophotodermatitis.

128
Which form of epidermolysis bullosa presents with generalized bullae, absent nails, dysplastic
teeth (due to enamel defects), nonhealing granulation tissue periorally, and is often fatal by age
3-4?
A. Weber-cockayne
B. Herlitz
C. Non-Herlitz
D. Hyperplastic cockayne-touraine
E. Albapapuloid Pasini variant
►B

Junctional EB is an autosomal recessive disorder with a defect in the lamina lujcida; uncein is
absent in all forms. The Herlitz variant is due to mutations in laminin 5 and clinically presents as
generalized bullae (which are non-scarring), nonhealing granulation tissue periorally, absent
nails, dysplastic teeth and enamel defects, anemia, growth retardation and is often fatal by age 3-
4 due to low protein, anemia and infection. The non-Herlitz variant of Junctional EB presents
with bullae especially on the extremities which heal with atrophic scarring; pts improve with age
and typically have a normal life span. Weber-cockayne is a type of EB simplex and presents with
palmoplantar bullae and hyperhidrosis. Hyperplastic cockayne-touraine is a form of dominant
dystrophic EB which presents with bullae localized to extremities. Albapapuloid Pasini variant
of dystropohic EB presents with widespread bullae, atrophy, hypopigmented scar-like white
papules on trunk, no miliaar or scarring, and mild oral involvement.

129
Dermatitis herpetiformis is associated with:
A. HLA-Cw6
B. HLA-B27
C. HLA-DR4
D. HLA-DP1
E. HLA-DR3
►E

HLA DR3 and DQ2 are associated with dermatitis herpetiformis. HLA B27 is associated with
psoriasis with spondylarthropathy, while HLA Cw6 is associated with psoriasis.

326
130
Which of the following substances is known to cause a delayed positive patch test reaction?
A. Gold
B. Nickel
C. Bacitracin
D. Fragrance mix
E. Rosin
►A

Gold is known to cause a delayed patch test reading. Patients allergic to gold often also react to
nickel and cobalt. Bacitracin is a frequent contact allergen that often coexists with an allergy to
neomycin. Fragrance mix contains cinnamic alcohol and aldehyde, hydroxycitronellel,
isoeugenol, eugenol, oak moss absolute, alpha-amyl cinnamic aldehyde, geraniol. Rosin
(colophony) is found in adhesive tape, cosmetics, glossy papers, violin bows, and chewing gum.

131
Perioral exuberant granulation tissue is sometimes seen in:
A. Cicatricial pemphigoid
B. Herlitz
C. Duhring's
D. Weber-Cockayne
E. Stevens-Johnson
►B

The Herlitz form of junctional EB sometimes presents with excessive granulation tissue
periorally, in the axillary/neck area.

132
The picture shown is considered to be diagnostic for what type of epidermolysis bullosa
A. Junctional epidermolysis bullosa- Non Herlitz type
B. Junctional epidermolysis bullosa- Herlitz type
C. Dowling-Meara form of Epidermolysis Bullous simplex
D. Dominant dystrophic epidermolysis bullosa
E. Recessive dystrophic epidermolysis bullosa
►E

The picture is shown is known as "mitten hands" and is diagnostic for recesseive type of
dystrophic epidermolysis bullosa. RDEB have the Hallopeau-Siemens type, a severe life-altering
bullous disease characterized by widespread dystrophic scarring and deformity and by severe
involvement of mucous membranes. Much less commonly, RDEB may manifest as the non-

327
Hallopeau-Siemens types, a mild localized form (mitis), and an inverse variant (recessive
epidermolysis inversa). In the Hallopeau-Siemens form of RDEB, erosions and blisters are
usually manifested at or shortly after birth. Blisters may appear to occur spontaneously, but most
seem to arise at sites of pressure or trauma. Nikolsky sign is often positive. The hands and lower
aspects of the legs are particularly susceptible to severe blistering and scarring. The fingers and
toes may become fused, with resultant pseudosyndactyly, in which the digits become bound
together by a glove-like epidermal sac, with resulting claw-like clubbing or mitten-like
deformities as shown in the image.

133
Patients with pemphigus vulgaris have lesions that start in the mouth in approximately :
A. 70% of patients
B. 50% of patients
C. 30% of patients
D. 20% of patients
E. 10% of patients
►A

In 70% of patients, pemphigus vulgaris starts in the mouth and other mucosal surfaces can also
be involved.

134
Atopic dermatitis is associated with all except:
A. Ichthyosis hystrix
B. Central facial pallor
C. Pityriasis alba
D. Nipple eczema
E. Hyperlinear palms
►A

Icthyosis vulgaris (not hystrix) is associated with atopic dermatitis as one of the minor criteria of
Hanifin.

135
Pemphigus erythematosus:
A. Is also called Hallopeau syndrome
B. May be an abortive form of subcorneal pustulosis
C. Is often in a malar/seborrheic distribution
D. Does not have linear IgG and C3 at the basement membrane zone
E. All of these answers are correct

328
►C

Pemphigus erythematosus, or Senear Usher syndrome, presents like pemphigus foliaceus and
lupus erythematosus with very superficial erosions and erythematous/hyperkeratotic facial/chest
lesions. DIF reveals intercellular IgG and linear IgG at the dermoepidermal junction.

136
This disease is inherited in an X-linked recessive manner:
A. Chronic granulomatous disorder
B. Job‟s syndrome
C. Dowling-Meara
D. Mongomery‟s syndrome
E. Treacher-Collins
►A

Chronic granulomatous disorder is inherited in an X-linked recessive manner, as is Wiskott-


Aldrich. Job‟s syndrome is autosomal dominant. Dowling-Meara is a subtype of EB simplex,
with autosomal dominant inheritance. Mongomery‟s syndrome is xanthoma disseminatum.
Treacher Collins syndrome is also known as mandibulo-facial-dysostosis.

137
Ocular cicatricial pemphigoid has antibodies against:
A. Gamma-catenin
B. Peripherin
C. Beta4-integrin
D. Kalinin
E. Vinculin
►C

Ocular cicatricial pemphigoid (CP) has autoantibodies against beta4-integrin. CP associated with
malignancy has autoantibodies against epiligrin (laminin 5). CP with mucous membrane and skin
findings often has autoantibodies against BPAg2.

138
The most common malignancy associated with paraneoplastic pemphigus is:
A. Non-Hodgkin's lymphoma
B. Chronic lymphocytic leukemia
C. Multiple myeloma
D. Acute myelocytic leukemia
E. Hodgkin's lymphoma

329
►A

Paraneoplastic pemphigus is associated with various benign as well as malignant internal tumors,
with the most common being non-Hodgkin's lymphoma.

139
What is the most common cause of this entity?
A. Drugs
B. Infections
C. Malignancies
D. Idiopathic
E. UV exposure
►A

Stevens-Johnson syndrome (SJS) is an immune-complex mediated hypersensitivity complex that


is a severe expression of erythema multiforme. Most authors and experts consider SJS and toxic
epidermal necrolysis (TEN) different manifestations of the same disease. The most common
cause of recurrent episodes are erythema multiforme (AKA herpes associated erythema
multiforme (HAEM)), whereas drugs are the most common cause of SJS/TEN.

140
Bullous diabeticorum typically presents on the:
A. Face
B. Chest
C. Arms
D. Legs
E. Groin
►D

Bullous diabetocorum presents as tense blisters on the legs. They are typically non-inflammatory
and appear suddenly.

141
A one-day old infant presents with papules and pustules on the face, trunk, and proximal
extremities. Palms and soles are spared. The patient is otherwise doing well. A biopsy shows
numerous eosinophils. The diagnosis is:
A. Acropustulosis of infancy
B. Herpes infection
C. Erythema toxicum neanotorum
D. Scabies

330
E. Transient neonatal pustular melanosis
►C

Papule and pustules on an erythematous base appear on a healthy infant within 48 hours, and
typically last 2-3 days in erythema toxicum neanotorum. The lesions may occur anywhere, but
usually spare the palms and soles. Gram stain reveals numerous eosinophils. A 1 day-old infant
would not have had a chance to acquire a scabies infestation. Acropustulosis of infancy typically
presents at 3-4 months and the pathology shows vesicles with many neutrophils. A herpes
infection would not show eosinophils. Transient neonatal pustular melanosis shows sterile sub
corneal pustules.

142
This medicine causes degranulation of mast cells
A. NSAIDs
B. ASA
C. opiates
D. polymixin B
E. All of these answers are correct
►E

Mast cell degranulators include NSAIDs, ASA, opiates, polymixin B, radiocontrast,


tubocurarine, succinylcholine, and others.

143
Herpes gestationis is most commonly associated with:
A. Lymphoma
B. Multiparity
C. Grave's disease
D. Inflammatory bowel disease
E. Lupus
►C

Herpes gestationis (HG) typically occurs in the second or third trimester, and clinically presents
as urticarial papules and plaques around the umbilicus which progress to involve the rest of the
body. HG has been associated with Grave's disease.

144
Which of the following forms of epidermolysis bullosa is autosomal recessive?
A. Weber-Cockayne
B. Dowling-Meara

331
C. Junctional EB with pyloric atresia
D. EB simplex
E. Hyperplastic cockayne-touraine
►C

The forms of junctional epidermolysis bullosa are inherited in an autosomal recessive fashion.
These include Herlitz, non-Herlitz, JEB with pyloric atresia, JEB inversa, and GABEB.
Recessive dystrophic EB (Hallopeau-Siemens) is also a recessive subtype of dystrophic EB. EB
simplex is typically inherited in an autosomal dominant fashion except EBS with muscular
dystrophy which is AR. Hyperplastic cockayne-touraine is a subtype of dominant dystrophic EB.

145
A patient with a history of leukemia that had severe, persistent painful stomatitis extending from
the lips into the pharynx, larynx and esophagus. Conjunctival involvement may lead to blindness.
The cutaneous changes are polymorphic, ranging from erythematous macules to lichenoid
papules to blisters and erosions. This patient most likely has:
A. Paraneoplastic pemphigus
B. Erythema multiforme
C. Cicatricial pemphigoid
D. Chemotherapy induced stomatitis
E. Persistent herpes simplex
►A

This patient has paraneoplastic pemphigus. It is an uncommon blistering disease with a variety of
clinical patterns and target antigens associated with underlying malignancy. It is most likely
associated with lymphoma, leukemia, Castleman tumor, or thymoma. Diagnosis is by indirect
immunofluorescence is positive on rat bladder epithelium.

146
Which type of epidermolysis bullosa has the greatest risk of developing squamous cell
carcinomas?
A. Weber-Cockayne
B. Dowling-Meara
C. Herlitz
D. EB Simplex with muscular dystrophy
E. Hallopeau-Siemens type of recessive dystrophic EB
►E

Dystrophic EB is due to a defectin collagen 7, leading to decreased or absent anchoring fibrils in


the sublamina densa. The three types of dominant dystrophic EB include Hyperplastic cockayne-

332
Touraine, Albopapuloid Pasini, and Bart's. The Recessive form of dystrophic EB is also known
as Hallpeau Siemens and patients present at birth owith generalized bullae which lead to erosions
and atrophic scarring, mitten deformities, flexion contractures, oral and esophageal scarring and
strictures and dysplastic teeth. There is an increased risk of squamous cell carcinomas.

147
Patients with Senear-Usher syndrome are most likely to present with:
A. Erythematous papules and plaques around the umbilicus
B. Transient vesicles on the oral mucosa
C. Severely pruritic grouped vesicles symmetrically distributed primarily on
extensor surfaces
D. Erythema multiforme-like oral ulcerations which are severe
E. Erythematous crusts and hyperkeratotic lesions on the nose, ears, cheeks, scalp,
and chest
►E

Senear-Usher syndrome, also known as pemphigus erythematosus, is thought to be an overlap


between lupus erythematosus and pemphigus foliaceous. The DIF resembles that of lupus and
pemphigus foliaceous with granular IgG and C3 at the dermoepidermal junction and a fishnet
intraepidermal pattern. 80% of patients have a positive lupus band.

148
Mutations in beta-catenin are most commonly associated with:
A. Naxos disease
B. Bullous pemphigoid
C. Pilomatricomas
D. Ectodermal dysplasia with skin fragility
E. Ocular cicatricial pemphigoid
►C

The armadillo family of proteins constitute of group of proteins which function in cellular
adhesion. These include beta-catenin, plakoglobin and plakophilins. Mutations in beta-catenin
have been associated with some pilomatricdomas and colorectal carcinomas. Naxos disease has
been linked to a mutation in plakoglobin, while ectodermal dysplasia with skin fragility has
beenlinked to a mutation in plakophilin 1. Ocular cicatricial pemphigoid is associated with
antibodies to beta4-integrin.

149

333
A woman in her 2nd trimester of pregnancy presents to clinic with urticarial plaques and papules
around her umbilicus, chest and extremities. Tense vesicles are present within a few of the
erythematous plaques. This woman”s condition is most commonly associated with:
A. Lymphoma
B. Multiparity
C. Grave”s Disease
D. Inflammatory Bowel Disease
E. Lupus
►C

Herpes gestationis (HG) typically occurs in the second or third trimester, and clinically presents
as urticarial papules and plaques around the umbilicus which progress to involve the rest of the
body. HG has been associated with Grave's disease. Hormonal factors influence the disease
manifestation. This condition can be seen in pregnant women, menstruating women, and women
taking oral contraceptives.

150
The best substrate for pemphigus vulgaris for indirect immunofluorescent testing is:
A. Monkey esophagus
B. Rat bladder
C. Salt split skin
D. Guinea pig esophagus
E. Rat liver
►A

Pemphigus vulgaris present as flaccid bullous lesions and can affect the mucous membrane. The
best substrate for pemphigus vulgaris is monkey esophagus.

151
The best first line treatment for dermatitis herpetiformis is:
A. Methotrexate
B. Dapsone
C. Colchicine
D. Prednisone
E. IVIG
►B

The most effective drug for dermatitis herpetiformis is dapsone. The dose varies between 50 and
300mg daily. Side effects include hemolytic anemia, leukopenia, methemoglobinemia, and rarely

334
agranulocytosis or peripheral neuropathy. Sulfapyridine is also a very effective treatment for
dermatitis herpetiformis.

152
Patients diagnosed with variegate prophyria have skin fragility and also can have neurological
issues. They have a defect in:
A. Protoporphyrinogen oxidase
B. Ferrochelatase
C. Porphobilinogen deaminase
D. Uroporphyrinogen III synthase
E. Uroporphyrinogen decarboxylase
►A

Patients with variegate porphyria have photosensitivity and neurological issues with also flaccid
skin that heals with crusting. They can also have a higher incidence of skin infection. They have
a defect in the protoporphyrinogen oxidase.

153
Fecal isocoproporphyrin is seen in:
A. Porphyria cutanea tarda
B. Harderoporphyria
C. Variegate porphyria
D. Acute intermittent porphyria
E. Coproporphyria
►A

Fecal isocoproporhyrin is characteristic of porphyria cutanea tarda.

154
A 60 year old gentleman presents with a recurrent pustular eruption. Biopsy demonstrates
subcorneal pustules with abundant neutrophils and a serum protein electrophoresis shows a
monoclonal IgA gammopathy. The diagnosis is?
A. Subcorneal pustular dermatosis (Sneddon Wilkinson disease)
B. Subcorneal pustular dermatosis type of IgA Pemphigus
C. Intraepidermal neutrophilic type of IgA Pemphigus
D. Cicatricial pemphigoid
E. Pemphigus vulgaris
►A

335
Sneddon-Wilkenson (subcorneal pustular dermatosis) is a neutrophilic dermatosis that is often
difficult to distinguish from pustular psoriasis. Clinically, patients have pustules in clusters in the
folds especially the axilla, and groin. There is no identified antigen for sneddon wilkinson. For
IgA pemphigus, the pustules are in a serpiginous or circinate pattern on the abdomen, axilla and
groin. The antigen for IgA pemphigus is desmocollin 1 and 2. Treatment options include acitretin
and dapsone for sneddon wilkinson. Both sneddon wilkinson and IgA pemphigus are associated
with a monoclonal IgA gammopathy.

155
A 46-year-old man presents with desquamation of the bilateral hands. He reports this has been
present since childhood. There are no associated systemic findings. Which enzyme is most likely
implicated?
A. Tissue transglutaminase I
B. Tissue transglutaminase III
C. Tissue transglutaminase V
D. Lysyl oxidase
E. Lysyl hydroxylase
►C

Tissue transglutaminases are calcium dependent enzymes implicated in several dermatologic


diseases. Transglutaminase I is faulty in lamellar icthyosis and non-bullous congenital
icthyosiform erythroderma, while transglutaminase III is implicated in dermatitis herpetiformis.
Transglutaminase V mutations have been associated with keratolysis exfoliativa perstans. Lysyl
oxidase and lysyl hydroxylase are implicated in Ehlers-Danlos Syndrome.

336
337
Chapter -13-
Photobiology and Photosensitivity Disorders
1
Currently the most common cause of photoallergic contact dermatitis is:
A. Halogenated salicylanilides
B. Musk ambrette
C. 6-methylcoumarin
D. Sunscreens
E. Mercaptobenzothiazole
►D

Halogenated salicylanilides, musck ambrette and 6-methylcoumarin used to cause most of the
cases of photoallergic contact dermatitis. They have been essentially eliminated from soaps and
fragrances. Sunscreen ingredients are now the most common cause.

2
Advantages of narrowband UVB over PUVA therapy are the following EXCEPT:
A. Need for protective eyewear
B. No nausea
C. Safe in childhood
D. Safe in pregnancy
E. More effective in treating thick plaques of CTCL
►E

UVB, both narrowband and broadband, is less penetrating into the skin than UVA. Therefore,
PUVA is more effective for thick plaques of CTCL. The UVB does not reach to the bottom of
the plaques.

3
What mutation is responsible for this clinical presentation in a patient with thyroid dysfunction
and chronic candidal infections?
A. AIRE
B. PTEN
C. TRP1
D. fumarate hydratase
E. LYST
►A

338
APECED syndrome named for the clinical features of autoimmune polyendocrinopathy,
candidiasis, ectodermal dystrophy and is caused by a mutation in AIRE (autoimmune regulator).
13% of these patient have vitiligo.

4
This patient presented with hyperpigmented streaks after a vacation in the Caribbean. The most
likely diagnosis is:
A. PMLE
B. CAD
C. Melasma
D. Phytophotodermatitis
E. Actinic prurigo
►D

Phytophotodermatitis requires exposure to certain plants or fruits followed by sunlight. Parsnips,


parsley, figs, limes, celery, bergamot oranges, and others contain psoralens that react with UVA.
Initially there is erythema and blistering followed by streaked hyperpigmentation.

5
A patient demonstrates a positive photopatch test to musk ambrette and a lowered MEDB. The
correct diagnosis is most likely:
A. PMLE
B. Solar urticaria
C. CAD
D. Actinic prurigo
E. Photoallergic contact dermatitis
►C

Only chronic actinic dermatitis (CAD) has a lowered MEDB. That finding helps distinguish
CAD from photoallergic contact dermatitis in which one sees a positive photopatch test also.

6
Which of the following statements regarding phototesting is correct?
A. The UVB MED is performed using narrowband UVB light sources
B. The UVA MED is performed using a narrowband UVA source
C. To test for a visible light reaction, a slide projector is used as a light source
D. Repeated MEDs to UVB or UVA, given to different test sites over several days
can be used to reproduce lesions of polymorphous light eruption
E. A single large dose of visible light is the best way to reproduce lesions of
polymorphous light eruption

339
►C

Phototesting is done prior to initiating phototherapy or during provocative induction. To test for
a visible light reaction, a slide projector is used as a light source. The other choices are incorrect.
The UVB MED and UVA MED is performed using BROADBAND UVB and BROADBAND
UVA light sources respectively. Repeated MED's to UVB or UVA given to the SAME site may
be used to reproduce lesions of PMLE. Alternatively a single large dose of UVA or UVB can be
used to reproduce PMLE.

7
This disease can be brought on by:
A. Visible light
B. UVA II
C. UVA I
D. UVB
E. All of these answers are correct
►E

Solar urticaria can be brought on by UVB or UVA or visible light or combinations of those
wavelengths.

8
The best definition for a MED is:
A. The dose of ultraviolet radiation that produces barely perceptible erythema that
completely fills the test square
B. The dose of ultraviolet radiation that produces a easily visible "sunburn"
C. The dose of ultraviolet radiation that is one step below the first visible erythema
D. The dose of ultraviolet radiation that produces pronounced erythema that
completely fills the test square
E. The dose of ultraviolet radiation that produces bullae that completely fills the
test square
►A

An MED is the dose of ultraviolet radiation that produces barely perceptible erythema that
completely fills the test square. The remaining options are incorrect.

9
Of the following which one is most characteristic of photoxocity rather than photoallergy?
A. Onset in hours to days
B. Eczematous dermatitis

340
C. Cross-reactivity to chemically similar agents
D. Often caused by furocoumarins
E. Relatively low incidence
►D

Furocoumarins may cause a phytophotodermatitis that is a phototoxic reaction. Such reactions


occurs with high frequency in the population, occur within minutes to hours from exposure, and
appear like an exaggerated sunburn. The other answers are common with photoallergy, such as
seen with sunscreens, fragrances, and various systemic medications like chlorpromazine.

10
Lichen planus-like lesions on sun-exposed areas may be seen in patients being treated with:
A. Fenofibrate
B. Ketoprofen
C. Quinolones
D. All of these answers are correct
E. None of these answers are correct
►A

Lichenoid medication photosensitivity, with LP-like lesions on sun-exposed areas has been
associated with treatment with antimalarials, thiazides, demethylchlortetracycline, fenofibrate,
enalapril, quinine, and quinidine.

11
Which of the following is not true about UVB radiation?
A. Responsible for sunburn
B. More intense in the summer than winter months
C. Peaks at noon
D. Decreased with high wind velocity
E. Virtually all blocked by car window glass
►D

The UVB band extends from 290 to 320 nm. The UVB spectrum is recognized as the primary
cause of sunburn, skin cancer, and other harmful effects on human skin. The UVA band extends
from 320 to 400 nm. This spectrum is further subdivided into UVA-2 (320 to 340 nm) and UVA-
1 (340 to 400 nm). The UVA spectrum is recognized as a cause of immediate and delayed
tanning reaction of skin, and several other effects including photoaging, skin photosensitization,
and immunosuppression. UVC radiation comprises wavelengths shorter than 290 nm (from 200
to 290 nm). Notably, window glass filters out ultraviolet wavelengths shorter than 320 nm, so
both UVB and UVC are effectively filtered by car window glass. UVB radiation is more intense

341
during summer months compared to winter months and peaks during midday hours. It has been
postulated that physical factors such as high temperature, high humidity, and wind can all
increase susceptibility to UV-induced carcinogenesis.

12
Hereditary PMLE of Native Americans:
A. Is more similar to PMLE than to actinic prurigo
B. Often presents with chelitis and conjunctivitis
C. Is not treated with thalidomide
D. Rarely persists into adulthood
E. All of these answers are correct
►B

Hereditary PMLE of Native Americans is similar to actinic prurigo but persists much more
frequently into adulthood. 75% of patients have a positive family history. It presents with a
papular, excoriated, eczematous dermatitis that occurs predominantly on the face. Chelitis and
conjunctivitis are common. It may be treated with thalidomide.

13
UVA II encompasses which wavelengths ?
A. 290-320 nm
B. 320-400 nm
C. 400-450 nm
D. 320-340 nm
E. 340-400 nm
►D

UVA can be divided into UVA II (320-340 nm) and UVA I (340-400 nm).

14
Which of the following patients would be the most likely to present with chronic actinic
dermatitis?
A. A 4 year-old Native American boy
B. A teenage girl
C. A 64 year-old farmer
D. A 32 year-old woman
E. A 32 year-old man
►C

342
Patients with chronic actinic dermatitis are usually middle-aged to elderly males who present
with a chronic eczematous dermatitis in a photodistribution without history of current exposure
to a photosensitizer.

15
Ultraviolet radition has been shown to do all of the following in in vitro and in vivo studies
EXCEPT:
A. Alter the ability of antigen-presenting cells to present antigen
B. Suppress the induction of delayed-type hypersensitivity
C. Increase circulating levels of IL-6
D. Decrease circulating levels of IL-1
E. Induce suppressor T-cells
►D

UV-irradiated mice have been shown to have defective antigen presentation and a decreased
number of antigen-presenting cells, which prevents a normal delayed-type hypersensitivity
response. UVR causes the release of immunosuppressive factors, with induction of suppressor T-
cells and increases in circulating levels of cytokines, including IL-1, IL-6, and TNF.

16
Which of the following hypoglycemics is the most common cause of photosensitivity?
A. Insulin
B. Sulfonylureas
C. Metformin
D. Thiazolidinediones (i.e. rosiglitazone)
E. Piroxicam
►B

The sulfonylurea hypoglycemics for diabetes are the most common type of medication of this
class of medications. Piroxicam is not a hypoglycemic agent, but a common NSAID cause of
photoallergy.

17
Which of the following is the most common photodermatosis?
A. Hydroa vacciniforme
B. Chronic actinic dermatitis
C. Actinic prurigo
D. Polymorphous light eruption
E. Solar urticaria
►D

343
Polymorphous light eruption is the most common photodermatosis. It is a idiopathic disease that
usually appears in the first three decades of life and is more common in fair-skinned females.
The pathogenesis is unclear, but is believed to be related to a type IV hypersensitivity reaction.
Most lesions are erythematous pruritic papules, with the plaque form being less common.
Lesions appear symetrically on exposed areas after a delay of several hours to several days.
Patients with mild disease are treated with sun avoidance and a broad spectrum sunscreen. In
more severe cases, hardening and desensitization can be accomplished or antimalarials can be
used for resistant cases. For brief, sunny vacations, a short course of prednisone can be helpful.
The other options are less common forms of idiopathic photosensitivity disorders.

18
Treatment of CAD might include:
A. UV filters for car windows
B. Broad spectrum sunblock to which the patient is not allergic
C. Incandescent bulbs for home and office lighting
D. PUVA
E. All of these answers are correct
►E

CAD patients are exquisitely sensitive to UVB, often sensitive to UVA, and sometimes sensitive
to visible light. Answers a, b, and c are correct because those treatments prevent UV and visible
radiation from reaching the patient. PUVA hardens and desensitizes the skin.

19
The portion of the electromagnetic spectrum that produces a particular biologic effect is called
the:
A. Action Spectrum
B. Absorption Spectrum
C. Cutaneous Effect Spectrum
D. Effective Spectrum
E. Spectral Activity
►A

The action spectrum is the wavelengths that produce a certain biologic effect. The action
spectrum for photosensitivity from exogenous chemicals is usually in the UVA range. The
radiation that is absorbed by those chemicals is called their absorption spectrum.

20
Photoonycholysis has been attributed to:

344
A. Amiodarone
B. Chlorpromazine
C. Quinine
D. Furosemide
E. Tricyclic antidepressants
►C

Photoonycholysis is a manifestation of medication photosensitivity that has been attributed to


quinolones, tetracyclines, psoralens, and quinine.

21
Narrow band UVB consists of what wavelength?
A. 300-306nm
B. 308-310nm
C. 311-313nm
D. 312-320nm
E. 320-330nm
►C

Narrowband UVB is much less erythemogenic with regard to physical units (mJ/cm2) than
broadband UVB. Narrowband UVB is 311-313nm.

22
A normal MEDB on untanned Caucasian skin ranges from approximately:
A. 2-6 mJ/cm2
B. 15-40 mJ/cm2
C. 20-70 mJ/cm2
D. 70-140 mJ/cm2
E. 140-200 mJ/cm2
►C

The MEDB can vary from institution to institution. In one institution, it may range from 20-70
mJ/cm2. In another institution, it may range from 30-90 mJ/cm2.

23
Which of the following is true regarding actinic prurigo?
A. Lesions generally continue through late adulthood
B. Lesions persist for 1-2 days
C. Lesions never occur on non-sun-exposed areas
D. Cheilitis is frequently seen

345
E. Thalidomide has been ineffective for the majority of patients
►D

Actinic prurigo is an idiopathic photosensitivity disorder. Lesions are excoriated papules and
nodules that begin in childhood and remit in puberty. They can last for several months and may
occur on non-sun-exposed areas. Thalidomide has been very effective for treating the majority of
patients with actinic prurigo.

24
Actinic prurigo (AP) differs from PMLE in all of the following except:
A. The lesions of AP begin in childhood
B. The lesions of AP occur on all sun-exposed areas
C. Lesions of AP may occur on non-sun exposed areas
D. Cheilitis is common in AP and common in PMLE
E. Outbreaks of AP are not as clearly related to sun exposure
►D

Cheilitis is common in actinic prurigo and not a usual accompaniment in PMLE.

25
Which of the following statements about UVR and DNA is correct?
A. UVR alters DNA
B. UVA is much more efficient than UVB in inducing DNA damage
C. Cells from patients with actinic keratoses have more DNA repair capacity than
controls
D. UVA is most effective in producing pyrimidine dimer
E. Pyrimidine dimers can not activate oncogenes
►A

UVR alters DNA. UVB is much more efficient than UVA in inducing DNA damage. It can
create pyrimidine dimers that may activate oncogenes and lead to cancer. Cells from patients
with actinic keratoses have less DNA repair capacity than controls.

26
UVB acts on what compound to form pre-Vitamin D3 ?
A. 5-dehydrocholesterol
B. 9-hydrocholesterol
C. 7-deoxycholesterol
D. 7-dehydrocholesterol
E. DNA

346
►D

UVB converts 7-dehydrocholesterol in the skin to pre-Vitamin D3, which then thermally
isomerizes to form Vitamin D3.

27
All of the following are true regarding polymorphous light eruption except:
A. Pruritic
B. Abnormal metabolism of arachidonic acid
C. Hardening occurs with subsequent episodes
D. Lesions heal without scarring
E. Anti-Ro antibody positive
►E

Polymorphous light eruption is the most common photodermatosis that is characterized clinically
by the abnormal occurrence of pruritic, erythematous, edematous papules following exposure to
UV radiation. Lesions heal without scarring. It tends to affect women 2-3x more than men.
Positive Anti-ro antibodies should raise the suspicious for subacute cutaneous lupus
erythematosus (tends also to be less pruritic).

28
Absorption of UV radiation generates singlet oxygen in the skin by which chromophore?
A. Melanin
B. Urocanic acid
C. Keratin
D. Hemoglobin
E. Water
►B

Urocanic acid and DNA are biologically important chromophores. DNA absocrbs uVB directly
inducing changes between adjacent pyrimidine bases on one strand of DNA. Cyclopyrimidine
dimers, particularly thymine dimers or less commonly (6-4) photoproducts may be generated.
Urocanic acid is a second biologically important chromophore in the skin and is a by-product of
filaggrin breakdown. One photon of light contains enough energy to generate singlet oxygen.

29
UVB converts 7-dehydrocholesterol in the skin to pre-vitamin D3. What is the catalyst of the
next step, the conversion to vitamin D3?
A. Thermal isomerization
B. Enzymatic conversion in the skin

347
C. Cell mediated conversion
D. No catalyst is needed
E. Enzymatic conversion in the liver
►A

7-dihydrocholesterol is present in the skin and is converted to previtamin D3 by UV in the


spectrum of 290-315 nm. The Previtamin D3 then thermally isomerizes to form vitamin D3
which enters the circulation. 1-hydroxylation occurs in the liver and 25-hydroxylation in the
kidney resulting in the final form of 1,25-dihydroxyvitamin D3.

30
Which of the following statements about electromagnetic radiation is MOST correct?
A. Electromagnetic radiation can be conceptualized as packets of power called
photons
B. The energy of photons is proportional to the wavelength
C. The energy of photons is inversely proportional to the frequency
D. Longer wavelengths penetrate the skin more deeply
E. Electromagnetic radiation is measured in watts
►D

Longer wavelengths penetrate the skin more deeply. Electromagnetic radiation can be
conceptualized as packets of ENERGY called photons. The energy of photons is proportional to
the FREQUENCY and inversely proportional to WAVELENGTH. Electromagnetic radiation is
measured in wavelength.

31
The immunologic effects of UVR include all of the following except:
A. Alteration of Langerhans cell morphology
B. Decrease in IL-1, IL-6, TNF-alpha
C. Suppression of delayed-type hypersensitivity
D. Alteration of lymphocyte population
E. Alteration of Langerhans cell function
►B

UVR causes an increase of circulating cytokines (IL-1, IL-6, TNF-alpha).

32
The xenon arc solar simulator, whose spectrum is in this slide, is useful for:
A. MED-B testing
B. MED-A testing

348
C. Photopatch testing
D. MED-B testing and MED-A testing
E. MED-B testing, MED-A testing, and Photopatch testing
►D

Solar simulator radiation contains both UVB and UVA in sufficient quantities to be useful for
MED-B and MED-A testing. The field size is too small for photopatch testing.

33
A patient presents with solar urticaria. What tests should be considered initially?
A. ANA
B. ANCA
C. Urinalysis
D. CXR
E. ESR
►A

Solar urticaria is an idiopathic, type I photosensitivity disorder. Rare cases have been associated
with erythropoietic protoporphyria (EPP) and lupus erythematosus (LE). Blood tests for LE and
appropriate screening for EPP should be performed including ANA, Ro/La, Urine & Stool
porphyrins.

34
The portion of the electromagnetic spectrum that produces a particular biologic effect is known
as the:
A. Absorption spectrum
B. Action spectrum
C. Photobiologic spectrum
D. Minimal erythema dose
E. Active spectrum
►B

The action spectrum is the portion of the electromagnetic spectrum that products a particular
biologic effect (e.g. erythema, delayed tanning). The absorption spectrum is the portion of the
electromagnetic spectrum that is absorbed by a particular absorbing molecule, or chromophore.
The minimal erythema dose is the dose of UV radiatino that produces barely perceptible
erythema during phototesting.

35
Which of the following drugs is commonly known to produce photosensitivity?

349
A. Quinidine
B. Sulfonylureas
C. Griseofulvin
D. All of these answers are correct
E. None of these answers are correct
►D

Quinidine, sulfonlyureas, and griseofulvin are all known to cause photosensitivity.

36
The MPD of Oxsoralen plus UVA is:
A. Measured at 24 hours
B. Equal to one-half the patients MEDA
C. Tested on the patient”s calf or abdomen
D. Helpful in starting PUVA therapy
E. None of these answers are correct
►D

MPD stands for the minimal phototoxic dose. For Oxsoralen plus UVA, the MPD is measured at
48-72 hours. Testing is done on the upper buttock or forearm.

37
The main contributor to erythema of the skin with exposure to the sun is:
A. UVC
B. UVB
C. UVA1
D. UVA2
E. Visible light
►B

UVB in natural sunlight is the main contributor to erythema. UVB erythema reaches a maximum
in 6-24 hours. UVA accounts for 15-20% of sunlight erythema despite that there is much more
UVA than UVB in sunlight.

38
Which of the following statements is correct regarding antigen presenting cells after UVR
exposure?
A. Have increased ability to prime UV-irradiated mice to subcutaneously injected
hapten

350
B. UV-irradiated mice have normal antigen presentation, allowing a normal
delayed-type hypersensitivity response
C. There is a decrease in the number of antigen presenting cells
D. APC's have increased ability to prime UV-irradiated mice to subcutaneously
injected protein
E. APC's have increased ability to prime UV-irradiated mice to applied contact-
sensitizing agents
►C

APC's have depressed ability to prime UV-irradiated mice to subcutaneously injected hapten or
protein and to applied contact-sensitizing agents. UV-irradiated mice have defective antigen
presentation, preventing a normal delayed-type hypersensitivity response. There is a
REDUCTION in number of antigen presenting cells.

39
In solar urticaria wheals may be brought on by:
A. UVA
B. UVB
C. UVC
D. Visible light
E. UVA, UVB and Visible light
►E

Some patients react to visible light or UVA or UVB. Others react to combinations of visible
light, UVA, and UVB.

40
The UVAII portion of the electromagnetic spectrum extends from:
A. 200-290 nm
B. 290-320 nm
C. 320-400 nm
D. 320-340 nm
E. None of these answers are correct
►D

UVA extends from 320-400nm. UVA is further subdivided into UVAII (320-340nm) and UVA1
(340-400nm).

41

351
What range of ultraviolet radiation has been shown to be most efficient in inducing neoplasia in
mice?
A. 280-320nm
B. 320-340nm
C. 340-400nm
D. 400-760nm
E. >760nm
►A

Midrange UVR 280-320nm (UVB range) has been shown to be most efficient in inducing
neoplasia in mice. Long-wave UVA, when added to UVB may accelerate carcinogenesis.

42
Immediate pigment darkening:
A. Is associated with an increase in melanocyte number
B. Is predominately brought on by UVB
C. Start 45-60 minutes after exposure
D. Is predominately brought on by UVA and visible light
E. Is caused by an increase in tyrosinase activity
►D

Immediate pigment darkening appears almost as soon as irradiation occurs. It is due to photo-
oxidation of preexisting melanin.

43
The irradiance of a UV source is measured in:
A. Joules
B. Seconds
C. Watts
D. Millijoules
E. Centimeters
►C

The irradiance of a UV source in watts x the time in seconds equals the UV dose in joules.

44
Medication photosensitivity is caused by all except:
A. Thiazides
B. Cephalosporins
C. Phenothiazenes

352
D. Quinolones
E. Doxycycline
►B

Cephalosporins do not cause drug photosensitivity. Neither do the penicillins.

45
Which medication reactivates UVB- and PUVA-induced erythema?
A. Dacarbazine
B. 5-FU
C. Vinblastine
D. Methotrexate
E. None of these answers are correct
►D

Methotrexate reactivates UVB- and PUVA-induced erythema.

46
Most fluorescent UV sources are:
A. High pressure xenon arc lamps
B. Low pressure xenon arc lamps
C. Low pressure argon lamps
D. High pressure tungsten lamps
E. Low pressure mercury vapor lamps
►E

The mercury vapor in the fluorescent bulbs is excited by electric current. Then the mercury emits
radiation at 254 nm. This radiation is absorbed by the phosphor lining the bulb.

47
Which of the following statements is true regarding UV radiation, erythema, and pigmentation?
A. UVB erythema reaches a maximum in 24-36 hours
B. The chromophores involved with UVB erythema are melanosomes
C. Immediate pigment darkening is brought on by UVA and visible light
D. Immediate pigment darkening fades within 12-24 hours after exposure
E. Delayed tanning, which becomes visible about 72 hours after exposure, is
largely brought on by UVA.
►C

353
UVB in natural sunlight is the main contributor to erythema. UVB erythema reaches a maximum
in 6-24 hours. The chromophores involved with UVB erythema are not clear but appear to
involve nucleic acids. Immediate pigment darkening is brought on by UVA and visible light, and
fades within minutes after exposure. Delayed tanning becomes visible about 72 hours after UVB
exposure. UVA contributes to a lesser extent to delayed tanning.

48
Fluorescent UVA bulbs used for phototesting or PUVA therapy have a peak emission at:
A. 254 nm
B. 311 nm
C. 312 nm
D. 352 nm
E. 468 nm
►D

254 nm is the wavelength of the radiation emitted by mercury vapor lamps. Narrowband UVB
emits 311-312 nm. Fluorescent UVA bulbs used for phototesting or PUVA therapy have a peak
emission at 352 nm.

49
All of the following are true regarding actinic reticuloid except:
A. Affects elderly men
B. CD8+ T cells in lesional skin
C. Is a premalignant condition
D. Atypical dermal mononuclear cell infiltrate
E. Generalized lymphadenopathy common
►C

Actinic reticuloid is a type of chronic actinic dermatitis. Ive et al. introduced the disease as a
severe dermatosis with no apparent photoallergen. It generally affects elderly males and is
characterized by infiltrated erythematous plaques on an eczematous background in exposed sites
with lymphadenopathy. Histopathologically, it may resemble cutaneous T cell lymphoma.
However, there is a trend towards a lower CD4+/CD8+ ratio. It is not considered a premalignant
condition.

50
Most bulbs used for PUVA have a peak output predominantly in the following range:
A. 290 nm – 320 nm
B. 320 nm – 340 nm
C. 340 nm – 400 nm

354
D. 350 nm – 360 nm
E. 390 nm – 410 nm
►D

The UVA emitted by these bulbs is absorbed by psoralens, causing covalent bonding of
psoralens to DNA.

51
A patient presents with erythematous pruritic papules on exposed areas that appear in the spring.
They appear between 2 hours and 2 days after exposure. Which of the following statements is
NOT correct?
A. This is an idiopathic disease that appears in the first three decades of life
B. It is more common in fair-skinned females
C. The pathogenesis is unclear, but may be related to type IV hypersensitivity
reactions
D. The diagnosis described above is solar urticaria
E. Vesicles and eczematous dermatitis is uncommon
►D

The diagnosis described above is polymorphous light eruption. The time to development of
lesions is important in distinguishing between PMLE and solar urticaria. Solar urticaria usually
develops 10-30 minutes after UVR exposure.

52
A watt is a measurement of:
A. Power
B. Fluence
C. Energy
D. Heat
E. Distance
►A

A watt is a measurement of power or irradiance of a UV source. Fluence and energy is measured


in joules. These are related by the formula Joules/cm2=Watts/cm2xseconds.

53
The active spectrum for cutaneous vitamin D3 synthesis is:
A. 220-290nm
B. 290-320nm
C. 320-400nm

355
D. 400-410nm
E. Both First and Second Choice
►E

The source states that vitamin D3 synthesis occurs at wavelengths < 320 making choices 220-
290nm and 290-320nm correct.

54
This malnourished individual presented with crusting and hyperpigmentation in a
photodistribution. The best diagnosis is:
A. CAD
B. Photoallergic contact dermatitis
C. Scurvy
D. Pellagra
E. PMLE
►D

Pellagra is characterized by the triad of diarrhea, dermatitis, and dementia. The dermatitis begins
as a burning erythema in sun-exposed areas. There may be bullae and erosions. This is followed
by a dry, brittle, scaling and hyperpigmented phase. Pellagra is due to a deficiency of niacin and
tryptophan.

55
A patient presents with blue-gray pigmentation on sun-exposed areas but does not have
involvement of the sclerae, lunulae or mucous membranes. Which of the following medications
is could be causing this pigmentation?
A. Clindamycin
B. Chlorpromazine
C. Ciprofloxacin
D. Fluoxetine
E. Sertraline
►B

Amiodarone, chlorpromazine and tricyclic antidepressants all are capable of causing blue-gray
pigmentation on sun-exposed areas without involvement of the sclerae, lunulae or mucous
membranes.

56
When solar urticaria is a consideration for phototesting:
A. 7 test squares of increasing UV doses should be exposed

356
B. The lower back should not be used for testing
C. An MED (B) and MED (A) should not be performed
D. An additional reading at 15 minutes after exposure should be performed
E. Visible light will not evoke the lesions
►D

When solar urticaria is a consideration, an additional reading at 15 minutes after exposure is


important, as wheals begin within 10-30 minutes after exposure and last for about one hour. The
face and hands may not show lesions as they are chronically exposed to sun. Some patients react
to either visible light or UVA or UVB. Others react to both UVB and UVA, both UVA and
visible light, or all three.

57
All of the following statements regarding chronic actinic dermatitis are true EXCEPT:
A. The MEDB on phototesting is markedly diminished in patients with this
condition
B. Many cases begin as photoallergic contact dermatitis or drug photosensitivity
C. Fluorescent bulbs are safer for these patients than are incandescent bulbs
D. Azathioprine has been used successfully to treat this disorder
E. Many patients have a lowered threshold to shorter wavelength visible light
►C

Chronic actinic dermatitis (CAD) usually occurs in middle-aged to elderly males who present
with a chronic, eczematous dermatitis in a photodistribution, though there is no history of current
exposure to a photosensitizer. Phototesting is very helpful in diagnosing CAD. The MEDB is
markedly diminished, and the MEDB site may show an eczematous or infiltrated appearance.
Many of the patients have a lowered MEDA as well, and may have a lowered threshold to
shorter wavelength visible light in the blue-violet end of the spectrum. Many cases of this
idiopathic disorder are thought to have begun as photoallergic contact dermatitis or as a drug
photosensitivity with broadening of the photosensitivity to include the UVB range. It is unclear
why photosensitivity persists when the photosensitizer is no longer present. Treatment includes
strict sun avoidance and sun protection. Incandescent bulbs with longer wavelengths, far from
the blue-violet end of the visible spectrum, should be used instead of fluorescent bulbs, which
have significant blue-violet radiation. Topical and oral steroids, oral azathioprine and
cyclosporine, and PUVA have all been used to treat patients with CAD.

58
The differential diagnosis of this patient would include:
A. Photoallergic contact dermatitis
B. Airborne contact dermatitis

357
C. Chronic actinic dermatitis
D. Drug photosensitivity
E. All of these answers are correct
►E

Photoallergic contact, airborne contact, chronic actinic dermatitis, and drug photosensitivity can
all result in erythema or eczematous patches in sun exposed areas. Subtle clues may help
distinguish them such as involvement of the submental area in a airborne contact dermatitis.

59
The most helpful phototest to document this photosensitivity disorder would be:
A. Repeated doses of UVA and UVB
B. MEDBB
C. MEDNB
D. Photopatch tests
E. All of these answers are correct
►A

Repeated doses of ultraviolet radiation can sometimes elicit lesions of PMLE. More patients
react to UVA radiation than to UVB radiation.

60
A patient presents with onycholysis after sun exposure. Which of the following medications
would be least likely as a cause of this presentation?
A. Quinolones
B. Tetracyclines
C. Psoralens
D. Quinine
E. Chlorpromazine
►E

Chlorpromazine is associated with blue-gray pigmentation on sun-exposed areas and is not


associated with photoonycholysis. Quinolones, tetracyclines, psoralens and quinine can cause
photoonycholysis.

61
Phototoxic reactions:
A. Are immunologically mediated
B. Occur only in predisposed individuals
C. Rarely occur on the first exposure to the chemical

358
D. Are called "photoreactive" if they produce damage through reactive oxygen
species
E. Resolve with hyperpigmentation
►E

A phototoxic reaction appears as a exaggerated sunburn with erythema and sometimes blistering,
resolving with hyperpigmentation. It is a nonimmunologic reaction that could occur in all
individuals given enough of the chemical and enough UVR. It can occur on the first exposure to
the chemical and the UVR. Phototoxic reactions that produce damage through reactive oxygen
species are called "photodynamic."

62
Which of the following conditions would be least likely to be photo-exacerbated?
A. Herpes simplex
B. Pellagra
C. Transient acantholytic dermatosis
D. Psoriasis
E. Pyridoxine deficiency
►D

Psoriasis is improved by phototherapy, especially at the wavelengths of 311-312 nm in most


cases. Photosensitive psoriasis can occur, but is the least likely choice to be photo-exacerbated.
The other listed conditions are worsened by UV/sunlight exposure.

63
Treatment of this condition might include:
A. Azathioprine
B. Cytoxan
C. Cyclosporin
D. Antimalarials
E. Acitretin
►D

Sun avoidance, sunblocks, protective clothing, and topical steroids are sufficient for most
patients with PMLE. Other patients may require hardening with UVB or PUVA. Rare patients
require antimalarials.

64
Which of the following statements about the spectrums of UVR that cause of solar urticaria is
MOST correct?

359
A. Visible light causes solar urticaria
B. UVA causes solar urticaria
C. UVB causes solar urticaria
D. Both UVA and UVB cause solar urticaria
E. Patients can react to visible light, UVA and/or UVB
►E

Some patients react with wheals to either visible light or UVA or UVB. Others react to both
UVA and visible radiation. Some react to both UVB and UVA, and some patients react to UVB,
UVA and visible light.

65
Narrowband UVB is effective for psoriasis and can be used in pregnancy and in childhood. It is
also less carcinogenic than PUVA. The wavelength of narrowband UVB is:
A. 311-312nm
B. 315-317nm
C. 300-302nm
D. 318-320nm
E. 317-319nm
►A

Narrowband UVB is 311-312nm. It is as effective as oral PUVA in clearing psoriasis without the
necessity of ingesting a photosensitizer and without the need for eye protection.

66
Which of the following statements about the hypothetical effect of UVR on cancer induction is
correct?
A. UVR induces transformation of keratinocytes with expression of tumor-
associated antigens
B. UVR alters APC function, by increasing the number of antigen-presenting cells
C. UVR inhibits the release of immunosuppressive factors
D. Suppressor T-cells are suppressed
E. UVR increases the ability of Langerhans cells to present antigen
►A

UVR induces transformation of keratinocytes with expression of tumor associated antigens. It


alters APC function by reducing the number of antigen presenting cells. It also promotes the
release of immunosuppressive factors, induce suppressor t-cells, and decreases the ability of
Langerhans cells to present antigen.

360
67
The UVC portion of the electromagnetic spectrum extends from:
A. 10-200 nm
B. 200-290 nm
C. 290-320 nm
D. 400-760 nm
E. None of these answers are correct
►B

Ultraviolet C does not reach the earth's surface. It is filtered out by the ozone layer. It extends
from 200-290 nm.

68
Phototesting of PMLE patients reveals:
A. Normal MEDB, reduced MEDA.
B. Normal MEDB, elevated MEDA.
C. Normal MEDB, normal MEDA.
D. Lowered MEDB, lowered MEDA.
E. Lowered MEDB, normal MEDA.
►C

The MEDB and MEDA are normal in PMLE patients. Only with multiples of the MEDB or
MEDA can one often reproduce the lesions.

69
A MED phototest should be read at:
A. 2 hours
B. 24 hours
C. 48 hours
D. 12 hours
E. 96 hours
►B

MED testing should be read 24 hours after delivery of the doses. An additional reading at 15
minutes is important when solar urticaria is a consideration.

70
Regarding the UVR effects on contact dermatitis and delayed-type hypersensitivity, which of the
following statements is correct?

361
A. Mice exposed to long-term, high-dose UVR demonstrate increased splenic APC
function
B. There are increased delayed-type hypersensitivity responses
C. There are diminished contact hypersensitivity responses
D. Induction of sensitization is increased
E. There are increases in production of Th2 type cytokines
►C

Mice exposed to short-term, high-dose UVR demonstrate decreased splenic APC function. There
are diminished delayed-type hypersensitivity and contact hypersensitivity responses. Induction of
sensitization is decreased. Th2 cytokines are not increased following UVR exposure.

71
UVB converts 7-dehydrocholesterol in the skin to:
A. 25-hydroxyvitamin D
B. 1,25-dihydroxyvitamin D
C. Calcitriol
D. Previtamin D3
E. None of the above
►D

UVB converts 7-dehydrocholesterol in the skin to previtamin D3, which then thermally
isomerizes to form vitamin D3. It is hydroxylated in the liver and then in the kidney to form 25-
hydroxyvitamin D and 1,25-dihydroxyvitamin D, respectively.

72
Which of the following statements about the light sources for phototesting/phototherapy is
correct?
A. The most common light sources are incandescent bulbs
B. Phototherapy bulbs are low-pressure sulfur vapor lamps with the inner surface
coated by a specific phosphor
C. The mercury vapor is excited by electric current and emits a line spectrum of
254 nm
D. The phosphor emits a discoherent spectrum of various wavelengths
E. Broadband UVB bulbs emit throughout the UVB range and also include some
UVC
►C

Fluorescent bulbs are commonly used for phototherapy. These bulbs are low-pressure mercury
vapor lamps with the inner surface coated by a specific phosphor. The phosphor emits a

362
continuous spectrum of various wavelengths. The mercury vapor is excited by electric current
and emits a line spectrum of 254 nm. Broadband UVB bulbs emit throughout the UVB range and
also include some UVA, not UVC.

73
Oxsoralen plus UVA results in the following except:
A. Forms monofunctional adducts
B. Binds to pyrimidine bases
C. Can form DNA crosslinks
D. Suppresses DNA synthesis
E. Has immunomodulating effects
►B

Oxsoralen, in the presence of UVA, forms covalent bonds to pyrimidine bases on DNA.

74
A 56-year-old patient has a chronic, eczematous dermatitis in a photodistribution, though there is
no history of current exposure to a photosensitizer. There is relative sparing of the upper lids,
behind the ears, under the nose and the finger webs. The histology is indistinguishable from:
A. Mycosis fungoides
B. Actinic keratosis
C. Photodermatitis
D. Lichenoid dermatitis
E. Polymorphous light eruption
►A

This patient has chronic actinic dermatitis and some patients can have exacerbations year round.
The histology of this condition is indistinguisable from mycosis fungoides with atypical
mononuclear cells. Circulating Sezary cells have been found in some of those patients.

75
Which patient most likely has chronic actinic dermatitis?
A. A 30-year old female with erythematous pruritic papules on the chest that recur
each summer
B. A girl with excoriated papules and nodules on sun-exposed and non-sun-
exposed areas with cheilitis
C. A Native American adult with papules excoriated dermatitis on the face with
cheilitis
D. A middle-aged male with recurring wheals that begin 20 min after sun exposure
E. An elderly man with persistent eczematous dermatitis in a photodistribution

363
►E

Chronic actinic dermatitis typically affects middle-aged to elderly males and present as a
chronic, eczematous dermatitis in a photodistributed area with relative sparing of the upper lids,
behind the ears, and submental area. Occasionally non-sun-exposed areas are involved. These
patients show histology that resembles mycosis fungoides. Importantly, these patients also have
altered phototesting, usually with a diminished MEDb thought MEDa may also be decreased.
Recurring erythematous pruritic papules and less likely plaques can be seen in polymorphous
light eruption. Children with excoriated papules on sun-exposed and non-sun-exposed areas with
cheilis is typical of actinic prurigo. Hereditary polymorphous light eruption of Native Americans
presents with an excoriated facial dermatitis. Solar urticaria characteristically begin 10-30 min
after exposure and last for about one hour.

76
Actinic prurigo (AP) differs from polymorphous light eruption (PMLE) in that:
A. Chelitis is more frequently seen in PMLE
B. Lesions of AP usually begin after puberty
C. Lesions of PMLE occur on all sun-exposed areas
D. Lesions of AP may persist for months, even into the winter
E. Lesions of PMLE may occur on non-sun-exposed areas
►D

Actinic prurigo (AP) may be a distinct entity, or an HLA-restricted subset of polymorphous light
eruption (PMLE). AP differs from PMLE in that the lesions of AP always begin in childhood
and often remit in puberty, the lesions of AP occur on all sun-exposed areas and may persist for
months, even into the winter, and the lesions of AP may occur on non-sun-exposed areas. In
addition, outbreaks of AP are not as clearly related to sun exposure, and chelitis is frequently
seen in AP, not PMLE.

77
The wavelength range that most effectively induces cutaneous immunosuppression is:
A. 200-290nm
B. 290-320nm
C. 320-340nm
D. 340-400nm
E. 400-410nm
►B

364
290-320nm (UVB) is most effective in suppressing cutaneous immunity; mechanisms include
depletion of Langerhans cells, induction of regulatory T cells, and keratinocyte secretion of such
immunosuppressive cytokines as IL-10 and TNF-alpha.

78
Which of the following statements about ultraviolet C is incorrect?
A. UV-C does not reach the earth's surface
B. UV-C is absorbed by atmospheric ozone
C. UV-C has an electromagnetic spectrum from 200-290 nm
D. UV-C has higher energy than UV-B
E. UV-C has a higher wavelength than UV-B
►E

UV-C has wavelengths of 200 - 290 nm. UV-B has wavelengths of 290 - 320 nm. UV-C has a
lower wavelength, not higher. All of the other listed statements about UV-C are correct.

79
Lichen planus like lesions on sun-exposed areas may be seen in patients receiving which
medication?
A. Fenofibrate
B. Griseofulvin
C. Alprazolam
D. All of these answers are correct
E. None of these answers are correct
►A

LP-like lesions (which may be confluent) on sun-exposed areas have been seen in patients
receiving antimalarials, thiazides, demethylchlortetracycline, fenofibrate, enalapril, quinine, and
quinidine.

80
A patient that rarely burns and usually tans is which of the following skin types?
A. Type I
B. Type II
C. Type III
D. Type IV
E. Type V
►C

365
Skin type I always burns and never tans Skin type II usually burns and rarely tans Skin type III
rarely burns and usually tans Skin type IV never burns and alway tans Skin types V,VI are highly
pigmented individuals

81
A common cause of medication induced photoallergy is:
A. Psoralens
B. Acitretin
C. Ibuprofen
D. Naproxen
E. Piroxicam
►E

All of the listed medications are causes of photosensitivity, but only piroxicam is a cause of
photoallergy.

82
The most common cause(s) of topical phototoxicity today in the United States is(are):
A. Psoralens
B. Halogenated salicylanilides
C. Musk ambrette
D. 6-methyl-coumarin
E. PABA and non-PABA sunscreen ingredients
►A

Topical phototoxicity is most commonly caused by psoralens. Topical 8-methoxypsoralen is


used therapeutically to treated psoriasis, localized vitiligo, and hand/foot eczema. Psoralens in
certain plants, fruits, and vegetables can produce phytophotodermatitis. Topical photoallergy has
in the past been caused by halogenated salicylanilides, as well as musk ambrette and 6-methyl-
coumarin in fragrances. These compounds produced photoallergic contact dermatitis, and have
been removed from marketed products. PABA, its esters, and non-PABA sunscreen ingredients
are the most common causes of topical photoallergy, not topical phototoxicity

83
The most common presentation of a patient with medication photosensitivity is:
A. Photoonycholysis
B. Lichenoid eruptions
C. Diffuse erythema in sun-exposed areas
D. Pseudoporphyria
E. Fixed erythematous patch

366
►C

Most patients with medication photosensitivity present with diffuse erythema in sun-exposed
areas. In some patients, the eruption is eczematous and covered areas are spared.
Photoonycholysis, lichenoid eruptions and pseudoporphyria do occur with mediation
photosensitivity, but are not the most common presentation. Fixed erythematous patch is not
seen with this type of reaction.

84
Narrowband UVB bulbs emit predominantly at:
A. 290-320 nm
B. 311 nm
C. 352 nm
D. 305 nm
E. 360 nm
►B

Narrowband UVB (311-312 nm) is more effective than broadband UVB for psoriasis, vitiligo,
and other skin disorders.

85
Phytophotodermatitis can be seen with the following fruits / vegetables:
A. Potatoes and leeks
B. Tomatoes and bananas
C. Celery and radishes
D. Parsnips and limes
E. Lemons and pears
►D

Psoralens in certain plants, fruits, and vegetables can cause phytophotodermatitis. The most
common ones are limes, figs, parsley, parsnip, bergamot oranges, and celery.

86
Which of the following statements is true regarding pseudoporphyria?
A. Porphyrins may be normal
B. It has been associated with furosemide
C. Clinically it may be indistinguishable from porphyria cutanea tarda
D. All of these answers are correct
E. None of these answers are correct
►D

367
Pseudoporphyria is a phototoxic reaction that clinically and histologically resembles porphyria
cutanea tarda. In pseudoporphyria, however, there is no porphyrin abnormality. It has been
associated with numerous medications, including nalidixic acid, tetracyclines, amiodarone,
furosemide, and ketoprofen.

87
Native american can have hereditary PMLE that is persistent through adulthood. Patients can
have a positive family history in:
A. 75% of cases
B. 60% of cases
C. 50% of cases
D. 35% of cases
E. 25% of cases
►A

Hereditary PMLE of native americans can have a specific HLA type that is predominant with a
75% of patients having a positive family history. Patients have a papular, excoriated dermatitis
that occurs on the face and cheilitis and conjunctivitis are common.

88
Treatment of polymorphous light eruption includes all of the following EXCEPT:
A. Topical steroids
B. Antimalarials
C. PUVA therapy
D. Systemic corticosteroids
E. All of these answers are correct
►E

Most patients with PMLE have mild disease that can be treated by sun avoidance and sun
protection. Topical steroids can be used to treat clinical lesions. For severe cases, hardening and
desensitization can be accomplished with UVB, UVB plus UVA, or PUVA. Antimalarials can be
used for resistant cases. A short course of prednisone (20-40 mg) is effective for brief, sunny
vacations.

89
Solar urticaria:
A. Is an idiopathic, type IV photosensitivity disorder
B. Can present with headache, nausea, and syncope
C. Rarely lasts for more than 6 months to 1 year

368
D. Usually occurs to only UVB radiation
E. Is not benefitted by antihistamines
►B

Solar urticaria is an idiopathic, type I photosensitivity disorder. Mediator release during


widespread whealing may result in headache, nausea, wheezing, faintness, and syncope. It
usually lasts for many years. Some patients react with wheals to either visible light or UVA or
UVB. Others react to both UVA and visible radiation. Some react to both UVB and UVA, and
some patients react to UVB, UVA, and visible radiation. Treatment includes sun avoidance and
protection, and H-1 antihistamines may be of partial benefit.

90
Which of the following is true regarding immediate pigment darkening?
A. Contributes to constitutive skin color
B. Caused by UVA radiation
C. Prominent in lightly pigmented individuals
D. Requires the synthesis of new melanin
E. Becomes prominent 48 hr after exposure
►B

Tanning develops in two phases, early (transitory) and late (stable). The immediate darkening is
in response to UVA and is related to photo-oxidation of pre-existing melanin.

91
Lesions of PMLE typically appear:
A. About one hour after exposure
B. Hours to days after exposure
C. Days to two weeks after exposure
D. 15-30 minutes after exposure
E. Immediately
►B

The history of a delay of several hours to several days after exposure is important to the
diagnosis.

92
Actinic prurigo can occur in areas that are exposed to the sun. Cheilitis is frequently seen and the
treatment that has been effective is:
A. Thalidomide
B. Hydrochloroquine

369
C. Prednisone
D. Tetracycline
E. Cyclosporin
►A

Actinic prurigo can persist for months even in the winter. Lesions can occur on sun and non-sun-
exposed areas. Thalidomide has been very effective for the majority of patients.

93
Which of the following is NOT true regarding polymorphous light eruption?
A. Usually appears in the first three decades
B. May be a manifestation of a type IV hypersensitivity reaction
C. Vesicles and an eczematous dermatitis are a common presentation
D. Not all exposed areas show lesions
E. It may occur through windowglass, which filters out UVB
►C

Polymorphous light eruption is the most common photodermatosis. It is an idiopathic disease


that usually appears in the first three decades. Pathogenesis is unclear but it may be related to a
type IV hypersensitivity reaction. Most lesions are erythematous, pruritic papules. The plaque
form is less common, and vesicles and an eczematous dermatitis are rare. Not all exposed areas
show lesions, but the same areas are affected year after year. It may improve as the summer
progresses. It may occur through windowglass, which filters out UVB.

94
Which of the following is a manifestation of dermatoheliosis?
A. Hydroa vacciniforme
B. Actinic prurigo
C. Chronic actinic dermatitis
D. Brachioradial pruritis
E. Cutis rhomboidalis nuchae
►E

Dermatoheliosis, also known as photoaging, is induced by chronic sun exposure. Clinical


variants include cutis rhomboidalis nuchae, which appears as leathery skin on the posterior neck
with exaggerated skin markings. Other variants include striated beaded lines (small yellowish
papules and plaques along the sides of the neck), Dubreuilh elastoma (translucent papule which
may mimic a basal cell carcinoma), and Favre-Racouchot syndrome (nodular elastoidosis with
cysts and comedones). Hydroa vacciniforme, actinic prurigo, chronic actinic dermatitis, and
brachioradial pruritis are photosensitivity disorders.

370
95
Which spectrum of UV is responsible for the conversion of 7-dehydrocholesterol in the skin to
pre-vitamin D3?
A. 200-290 nm
B. 290-320 nm
C. 320-340 nm
D. 340-400 nm
E. 10-200 nm
►B

Ultraviolet B with the spectrum of 290-320 nm is responsible for the conversion of 7-


dehydrocholesterol in the skin to pre-vitamin D3.

96
The typical patients with polymorphous light eruption are:
A. Fair-skinned females in their 40”s and 50”s.
B. Type IV-skinned males in their 40”s and 50”s.
C. Type V-skinned females in their 20”s and 30”s.
D. Fair-skinned males in their 20”s and 30”s.
E. Fair-skinned females in their teens and 20”s.
►E

PMLE most commonly appears in fair-skinned females during the first three decades of life. It
may be related to type IV hypersensitivity.

97
Oral PUVA has been used to treat which of the following diseases:
A. CTCL
B. Mastocytosis
C. Graft-versus-host disease
D. Generalized granuloma annulare
E. All of these answers are correct
►E

PUVA has been reported to have helped patients with all the diseases listed.

98
Sunscreens can be either chemical absorbers or physical blockers. All of the following protect
against UVA except for:

371
A. PABA and esters
B. Titanium dioxide
C. Zinc oxide
D. Avobenzone
E. Ecamsule
►A

PABA and esters are ingredients in chemical absorbers in sunscreens and do not protect against
UVA. All the other ingredients protect against UVA. Titanium dioxide and zinc oxide are
physical blockers.

99
A 8-year-old child develops umbilicated vesicles on sun-exposed areas that resolve with crusting
followed by pock like scars. This patient most likely has:
A. Hydroa vacciniforme
B. Hereditary PMLE of native americans
C. Solar urticaria
D. Actinic Prurigo
E. Polymorphous light eruption
►A

Hydroa vacciniforme is a rare photosensitivity disorder occurring in childhood. They are


characterized by umblicated vesicles on sun-exposed areas. Resolution occurs with crusting
followed by pock-like scar.

100
UVB is about how many times more erythermogenic than UVA ?
A. 10
B. 100
C. 1000
D. 10000
E. 5
►C

Though UVB is 1000 times more erythermogenic than UVA, UVA is much more plentiful in
sunlight. Therefore, UVA does contribute to sunlight erythema.

101
Which of the following statements regarding hereditary PMLE of Native Americans is true?
A. It will remit in puberty

372
B. Specific HLA types predominate in Native Americans
C. 5-10% of patients have a family history of this eruption
D. Cheilits and conjunctivitis are uncommon
E. Patients have an urticarial erupion.
►B

Native Americans have a high rate of PMLE and there is some evidence of a genetic
predisposition. It tends to have a specific HLA predominance, continues through adulthood, and
commonly presents as a papular, excoriated, eczematous dermatitis predominantly on the face.
Cheilitis and conjunctivitis are common. Up to 75% of patients have a positive family history.

102
The solar simulator is not useful for phototherapy because of its:
A. High output of ultraviolet radiation
B. High output of infrared radiation
C. Low output of UVB
D. Low level of ionizing radiation
E. Small field size
►E

The coin-sized field of the solar simulator prevents this source from being useful for
phototherapy. It is useful in MED testing.

103
A joule is a measurement of:
A. Energy
B. Power
C. Irradiance
D. Wavelength
E. Distance
►A

A joule is a measurement of energy dose or fluence. Power or irradiance is measured in watts.


These are related by the formula Joules/cm2=Watts/cm2xseconds.

104
Patients with chronic actinic dermatitis (CAD) typically show on phototesting:
A. Lowered MEDB, usually lowered MEDA, sometimes positive photopatch tests
B. Lowered MEDB, usually normal MEDA, sometimes positive photopatch tests
C. Normal MEDB, lowered MEDA, sometimes positive photopatch tests

373
D. Normal MEDB, normal MEDA, sometimes positive photopatch tests
E. None of the above
►A

The hallmark of CAD is a lowered MEDB. Often the patients have a lowered MEDA. It is felt
that many of CAD patients began with photoallergic contact dermatitis, so some have positive
photopatch tests.

105
Blue-gray pigmentation of the lunulae:
A. Can be seen in patients on amiodarone
B. Occurs only after many years of treatment with chlorpromazine
C. Is seen in argyria and not in most medication photosensitivities
D. Is a rare side effect of treatment with tricyclic antidepressants
E. None of these answers are correct
►C

Blue-gray pigmentation on sun-exposed areas can be seen during treatment with amiodarone,
chlorpromazine, and tricyclic antidepressants. In these medication photosensitivities, there is no
involvement of the sclera, lunulae, or mucous membranes, as in argyria (prolonged contact with
or ingestion of silver salts).

106
Which of the following is true about UV light:
A. UVA radiation is 1000 times greater than UVB during midday hours
B. UVB radiation is 1000 times more erythrogenic than UVA
C. Sunlight early in the morning and late in the day contains relatively more UVB
D. UVAII light is 340-400nm
E. Clouds absorb most UVA light
►B

UVA light is found b/w 320 and 400nm and is broken up into UVAI(340-400nm) and
UVAII((320-340nm). UVB light is found between 290 and 320nm. UVA radiation is 100 times
greater than UVB during midday hours and sunlight early in the morning and late in the day
contains relatively more UVA. UVB radiation is 1000 times more erythrogenic than UVA.
Cloud cover is a poor UV absorber.

107

374
A patient presents with signs of porphyria cutanea tarda. Porphyrin screens are negative. Which
of the following medications on the patient's medication list is your top choice for
discontinuation?
A. Naproxen
B. Fenofibrate
C. Enalapril
D. Chloroquine
E. Multivitamin
►A

NSAIDs are a frequent offender in causing pseudoporphyria. The other listed medications are not
frequent causes of this type of skin reaction.

108
Regarding renal transplant recipients, which of the following statements is correct:
A. Have rates of SCC similar to the general population
B. Have a 36-fold increased risk of BCC
C. Have a 36-fold increased risk of SCC
D. Should have skin cancer screening at the same rate as the general population
E. Have an decreased risk for melanoma
►C

The rates of SCC in renal cell transplant recipients is increased at a rate of 36x. The rates of skin
cancer are higher than in the general population.

109
Ultraviolet radiation from the sun causes all of the following acute effects in the skin EXCEPT:
A. Redistribution of melanosomes from a perinuclear position into dendrites
B. Epidermal thickening
C. Mast cell degranulation
D. Photooxidation of preexisting melanin
E. All of these answers are correct
►E

All of these statements are acute effects of UV raditation on the skin. Immediate pigment
darkening, which fades within minutes after exposure, is brought on by UVA and visible light. It
is caused by photooxidation of preexisting melanin and a redistribution of melanosomes from a
perinuclear position into dendrites. Epidermal thickening is mainly a UVB-induced phenomenon.
Mast cell degranulation, with release of histamine and other mast cell products, also occurs as a
result of UV radiation.

375
110
A patient presents with purple polygonal pruritic papules on sun exposed areas. Which of the
following of his medication would not be suspect for causing this eruption?
A. Enalapril
B. Quinidine
C. Fenofibrate
D. Furosemide
E. Hydrochlorothiazide
►D

Furosemide (Lasix) is not a cause of lichenoid drug reactions, but can cause pseudoporphyria.
The others listed are causes of lichenoid drug reactions. Others are antimalarials,
demethylchlortetracycline and quinine.

111
Common side effects of PUVA include all of the following except:
A. Nausea
B. Hair loss
C. Painful erythema
D. Prolonged pruritus
E. Squamous cell carcinoma
►B

Alopecia is not a usual side effect of PUVA treatment.

112
The action spectrum for photoallergy is mostly in which spectrum?
A. 320-425nm
B. 290-320nm
C. 311-312nm
D. 200-290nm
E. 400-760nm
►A

The action spectrum for photoallergic dermatoses refers to the specific wavelengths of light that
evoke the photosensitive reaction. This falls mostly within the UVA region and may spill into the
visible light region for photoallergy (320-425nm). 200-290nm refers to the UVC region; 290-
320nm refers to the UVB region; 311-312nm refers to narrowband UVB region; and 400-769nm
refers to the visible light region.

376
113
As a result of ultraviolet radiation:
A. There is mast cell degranulation and release of histamine
B. Prostaglandins are increased
C. Epidermal thickening occurs
D. All of these answers are correct
E. None of these answers are correct
►D

Effects of ultraviolet radiation include mast cell degranulation with release of histamine and
other mast cell products, increases in certain prostaglandins and interleukins, and epidermal
thickening, which is mainly a UVB-induced phenomenon.

114
The following compound exerts immunosuppressive effects in the skin following exposure to
UV-radiation:
A. Cis-urocanic acid
B. 7-dehydrocholesterol
C. IL-12
D. Delta aminolevulinic acid
E. Amino-levulinic acid
►A

Trans-urocanic acid is an epidermal chromophore that isomerizes to cis-urocanic acid following


exposure to UV radiation. Cis-urocanic acid has been shown to be immunosuppressive, for
example, by impairing the induction of contact allergy in mouse models. The mechanism of this
immunosuppressive effect is unclear.

115
Ultraviolet radiation from the sun cause ALL of the following acute effects EXCEPT?
A. Epidermal thickening
B. Photosynthesis of vitamin D
C. Sunburning
D. Immediate pigment darkening
E. Increased immune surveillance
►E

377
UV radiation causes acute effects including: photosynthesis of vitamin D, sunburning, immediate
pigment darkening, delayed tanning, epidermal thickening and immunologic effects such as
DECREASED immune surveillance.

116
Which of the following is the most likely cause of photosensitivity?
A. Quinolones
B. Sulfonamides
C. Doxycycline
D. Minocycline
E. Penicillin V
►C

Doxycycline is the tetracycline derivative most likely to cause photosensitivity. Quinolones and
sulfonamides will also cause this with ingestion. Minocycline is the least photosensitizing of the
tetracycline derivatives. Penicillin is not a common cause of photosensitivity.

117
Rare cases of which photosensitivity disorder have been associated with erythropoietic
protoporphyria?
A. Hydroa vacciniforme
B. Solar urticaria
C. Actinic prurigo
D. Polymorphous light eruption
E. Chronic actinic dermatitis
►B

Rare cases of solar urticaria have been associated with erythropoietic protoporphyria (EPP),
lupus erythematosus (LE), and with certain drugs. Blood tests for LE and EPP should be
performed in patients with this diagnosis.

118
Which of the following statements about UVR in vivo is correct?
A. induces skin cancers
B. normalizes Langerhans cell morphology and function
C. decreases circulating levels of cytokins (IL-1, IL-6 and TNF)
D. normalizes cell trafficking
E. normalizes proportions of lymphocyte subtypes in peripheral blood
►A

378
UVR can induce skin cancer. UVR in vivo INCREASES circulating levels of cytokins (IL-1, IL-
6 and TNF) and alters Langerhans cell morphology, cell trafficking, and the proportion of
lymphocyte subtypes in peripheral blood.

119
Which of the following genodermatoses is NOT worsened by sunlight?
A. Darier's disease
B. Kindler syndrome
C. Hartnup disease
D. Rothmund-Thompson syndrome
E. Job syndrome
►E

Many genodermatoses can be exacerbated by sunlight including Darier's, Kindler, Rothmund


Thompson, Hartnup (pellagra changes).

120
Initial treatment of this disease should include:
A. Sun avoidance, sunblocks, beta carotene
B. Sun avoidance, sunblocks, desensitization
C. Hydroxychloroquine
D. Car UV filters, PUVA, sunblocks
E. Sun avoidance, sunblock, antihistamines
►E

Treatment of solar urticaria is difficult. Sun avoidance is the most important aspect of treatment.
A broad-spectrum sunblock and antihistamines.

121
This middle-aged man demonstrates infiltrated, dusky plaques on all sun-exposed areas. The
most likely diagnosis is:
A. PMLE
B. CAD (Chronic actinic dermatitis)
C. Actinic prurigo
D. Drug photosensitivity
E. Photoallergic contact dermatitis
►B

The thick, infiltrated plaques on sun-exposed areas are typical of the actinic reticuloid variety of
CAD.

379
122
The highest energy visible photons are in which portion of the visible spectrum?
A. Green
B. Blue-violet
C. Red-Orange
D. Yellow
E. All of these answers are correct
►B

In the visible spectrum, the blue-violet portion has the shortest wavelength and the highest
energy. The red-orange portion has the longest wavelength and the lowest energy.

123
Which of the following statements is true regarding ultraviolet carcinogenesis?
A. Mid-range ultraviolet radiation is less efficient in inducing neoplasia in mice that
is long wave UVR
B. Suppressor T-cells arise in UV-irradiated hosts only after tumors have
developed
C. Cells from patients with actinic keratoses have normal DNA repair capacity
D. UVA, when added to UVB, may accelerate carcinogenesis
E. UVA is most effective in producing pyrimidine dimers, which may activate
oncogenes
►D

Mid-range UVR (280-320 nm) is more efficient in inducing neoplasia in mice, but long wave
UVA, when added to UVB, may accelerate carcinogenesis. Suppressor T-cells induce
susceptibility to tumors, and appear to arise in UV-irradiated hosts prior to tumors developing,
thus playing a role in carcinogenesis. Cells from patients with AKs have less DNA repair
capacity than controls. UVB is most effective in producing pyrimidine dimers, which may
activate oncogenes, particularly in the formation of BCCs and SCCs.

124
All of the following are true about UVA radiation except:
A. 10 times more abundant than UVB
B. penetrates to a greater depth in the dermis than UVB
C. responsible for phototoxic drug reactions
D. approximately 50% of exposure occurs in the shade
E. virtually all blocked by car window glass
►E

380
The UVA band extends from 320 to 400 nm. This spectrum is further subdivided into UVA-2
(320 to 340 nm) and UVA-1 (340 to 400 nm). The UVA spectrum is recognized as a cause of
immediate and delayed tanning reaction of skin, and several other effects including photoaging,
skin photosensitization, and immunosuppression. The majority of the ultraviolet radiation at the
earth's surface is UVA (95 to 98%) with only 2 to 5% comprised of UVB. As UVC is completely
absorbed by the stratospheric ozone layer, it does not comprise ultraviolet radiation hitting the
earth's surface. Much of the UV radiation after reaching the atmosphere becomes scattered by the
time it hits the earth's surface. Due to this "sky radiation", it is possible to sunburn even if one is
exposed only to the shade. Notably, window glass filters out ultraviolet wavelengths shorter than
320 nm, so only UVB (290 to 320 nm) and UVC (200 to 290 nm) are effectively filtered by car
window glass. Although UVA penetrates deeper into the dermis than UVB, UVB radiation is
much more erythmogenic. Finally, most common photosensitizers have action spectrums in the
UVA range, and, as a result, UVA radiation is responsible for most phototoxic drug reactions.

125
Photoexacerbated genodermatoses include all except:
A. Tuberous sclerosis
B. Cockayne's Syndrome
C. Hailey-Hailey Disease
D. Hartnup Disease
E. Darier's Disease
►A

Tuberous sclerosis is not associated with photosensitivity.

126
The potent photosensitizer, 5-methoxypsoralen, is contained in which of the following contact
allergens?
A. Oil of Bergamot
B. Balsam of Peru
C. Tuliposide A
D. Usnic acid
E. Eugenol
►A

Oil of Bergamot contains 5-methoxypsoralen and may cause of 'Berloque dermatitis,' an intense
post-inflammatory hyperpigmentation due to an antecedent phytophotodermatitis that may be
subclinical.

381
127
Lumisterol is an inactive epidermal reservoir of which vitamin?
A. A
B. C
C. D
D. E
E. K
►C

Lumisterol and tachysterol are inert byproducts created during the biosynthesis of vitamin D.
When Previtamin D3 is exposed to light, it can result in photoisomerization of previtamin D3 to
lumisterol and tachysterol. If previatmin D3 is depleted, lumisterol and tachysterol can become
converted back to previtamin D3.

128
In solar urticaria wheals typically:
A. Begin at 45 minutes after exposure and last 2 hours (h.)
B. Begin at 1-2 h. and last 2-4 h
C. Begin at 15-30 minutes and last 1 h
D. Begin at 30-60 minutes and last 1-4 h
E. Begin at 15-30 seconds and last about 15 minutes
►C

The wheals of solar urticaria begin 10-15 minutes after exposure and last for about an hour. Solar
urticaria is usually idiopathic. Rare cases are associated with EPP, SLE, and certain medications.

129
Which of the following cell types induce susceptibility to tumor growth?
A. Suppressor T-cells
B. NK cells
C. Helper T-cells
D. Mast cells
E. Langerhans cells
►A

The suppressor T-cells induce susceptibility to tumor. These cells appear to arise in UV-
irradiated hosts prior to tumor developing, and play a role in carcinogenesis.

130

382
A 40-year-old patient presents with widespread plaque-type psoriasis without arthritis. She had
already applied potent topical steroids, calcipotriene, and tazarotene. The treatment of choice
would be:
A. PUVA
B. Cyclosporine
C. Narrowband UVB
D. Etretinate
E. Methotrexate
►C

The other choices involve internal medications and therefore have the potential for more side
effects than with narrowband UVB. Phototherapy may be impractical for some patients, and
therefore systemic therapies should be offered.

131
Phototoxicity and photoallergy from agents typically involve absorption of:
A. UVA
B. UVB
C. UVB and UVA
D. UVA and visible light
E. UVB, UVA and visible light
►A

Topical and systemic agents that produce phototoxicity and/or photoallergy usually have action
spectra in the UVA range.

132
The best location for phototesting patients suspected of photosensitivity is:
A. Affected skin of the buttock
B. Unaffected skin of the lower back
C. Affected skin of the ventral forearm
D. Unaffected skin of the upper back
E. Unaffected skin of the outer thighs
►B

Patients with suspected photosensitivity can be tested on unaffected skin of the buttocks, lower
back or ventral forearm. Effected skin should not be used for testing.

133
What is the wavelength of a Wood's light?

383
A. 290nm
B. 311nm
C. 330nm
D. 365nm
E. 410nm
►D

A Wood's light emits ultaviolet light at a wavelenth of 365nm and is produce by bassing light
through a Wood's filter which is composed of nickel oxide containing glass.

134
Most patients with PMLE require treatment with:
A. Sunscreen and sun avoidance between 11 am and 3pm
B. UVB hardening/desensitization
C. Chloroquine
D. Prednisone
E. Cyclophosphamide
►A

Most patients have mild disease that can be treated by sun avoidance, especially between 11 am
and 3pm; a broad spectum sunscreen and clothing with a tight weave. In more severe cases, UV
hardening, antimalarials or prednisone can be used. Cyclophosphamide is not used in PMLE.

135
What is the most likely cause of these lesions on the cheek of this 8 year-old boy?
A. Acne
B. Herpes simplex
C. Varicella
D. Hydroa vacciniforme
E. Polymorphous light eruption
►D

Hydroa vacciniforme is a rare photodermatosis of childhood which occurs on areas of sun-


exposed skin. The lesions leave depressed and atrophic scars. The condition tends to
spontaneously resolve in adulthood.

136
The main condition on the differential for polymorphous light eruption is lupus erythematosus.
Which of the following tests should NOT be performed to help make this distinction?
A. Antinuclear antibody

384
B. Anti-SSA
C. Anti-SSB
D. Skin biopsy for routine staining and direct immunofluorescence
E. SED rate
►E

All of the listed tests are helpful in distinguishing between PMLE and lupus except a SED rate,
which is a non-specific marker of systemic inflammation.

137
Ultraviolet light spectrum most completely encompasses which of the following spectrums?
A. 200 - 400 nm
B. 290 - 400 nm
C. 10 - 400 nm
D. 400 - 700 nm
E. 760 - 1200 nm
►C

The ultraviolet spectrum encompasses Vacuum UV from 10-200nm, UV-C from 200-290nm,
UV-B from 290-320nm and UV-A from 320-400nm. The visible light spectrum is from 400-
700nm. 700-1200 nm is part of the infrared spectrum.

385
Chapter -14-
Plants and Creatures of Dermatologic Significance
1
Which of the following plant families commonly causes phytophotodermatitis?
A. Rubiaceae
B. Compositae
C. Urticaceae
D. Ranunculaceae
E. Apiaceae
►E

The Apiaceae family (formerly Umbelliferae) includes parsley, celery, parsnip, hogweed, and
fennel, all potential causes of phytophotodermatitis.

2
Crusted scabies is a risk factor for all the follow patients except:
A. HIV
B. Elderly
C. Transplant patients
D. Children
E. Farmers
►E

All the following are risk for developing crusted scabies or Norwegian scabies, which are seen in
immuno-compromised hosts except for farmers. Crusted scabies are infested with innumerable
mites. The mites can survive for up to a week.

3
Pineapples contain which of the following proteolytic enzymes?
A. None of these answers are correct
B. All of these answers are correct
C. Bromelin
D. Phorbol esters
E. Ranunculin
►C

386
Bromelin is a proteolytic enzyme found in pineapples. It is granted access to dermal vessels by
calcium oxalate-induced fissures. Phorbol esters are found in plants of the Euphorbiaceae family.
Ranunculin is an irritant found in buttercups (Ranunculaceae family).

4
Which disease is transmitted by this insect?
A. Visceral leishmaniasis
B. African sleeping sickness
C. Chagas disease
D. Dracunculiasis
E. Onchocerciasis
►E

The insect depicted is the black fly, Simulium species. The black fly is the vector for
onchocerciasis, which is caused by onchocerca volvulus. Clinical manifestation of this disease
include atrophy, depigmentation and thickening of the skin. In addition, the infection may cause
"river blindess" and onchocercomas, which are nodules containing microfilariae.

5
Lipase is a notable active component of the venom of which of the following spiders?
A. Lycosidae
B. Lactrodectus
C. Hidippus
D. Chiracanthium
E. Loxosceles
►D

The venom of Chiracanthium (sac spiders) contains lipase. The venom of Lycosidae (wolf
spiders) is notable for histamine; Lactrodectus (black widow spiders) for alpha lactrotoxin;
Hidippus (jumping spiders) for hyaluronidase; and Loxosceles (brown recluse spiders) for
sphingomyelinase-D.

6
Identify this caterpillar:
A. Hag moth caterpillar
B. Puss caterpillar
C. Greenback caterpillar
D. Saddleback caterpillar
E. Io caterpillar
►D

387
This is the saddleback caterpillar, as it appears that the caterpillar is wearing a saddle.

7
Necrosis may be seen in which of the following:
A. Allergic contact dermatitis
B. Contact urticaria
C. Dermatographism
D. Irritant contact dermatitis
E. Acute urticaria
►D

Necrosis may be seen in severe irritant reactions. The other reactions do not involve necrosis.

8
Mosquitoes cause yellow fever and dengue. The cutaneous reaction is due to the female inserting
her blood tube, injecting irritating salivary secretions that anticoagulate blood. Another name for
mosquitoes are:
A. Anopheles
B. Dipetalonema perstans
C. Pulex irritans
D. Ctenocephalides felis
E. Trypanosoma cruzi
►A

The other word for mosquitoes is anopheles that causes malaria. It causes yellow fever and
dengue and most mosquitoes prefer black skin, young skin, warm to cool skin and scented skin.
They also refer bright colors also.

9
Which of the following statements regarding Theraphosidae is correct?
A. Bites do not generally produce any systemic toxicity
B. These creatures possess urticating hairs that are of dermatologic significance
C. These creatures can cause visual loss by causing a chronic granulomatous
reaction (ophthalmia nodosa)
D. All of these statements are correct
E. None of these statements are correct
►D

388
Theraphosidae (tarantulas) are large, brown to black, hairy spiders frequently found in the
southwest. They are of dermatologic importance due to urticating hairs. Tarantula bites generally
do not produce any systemic toxicity. The spider releases hairs in the direction of the perceived
attacker; hairs can penetrate the skin as deeply as the reticular dermis. If they injure the cornea,
they can cause a chronic granulomatous reaction (ophthalmia nodosa) and loss of vision.

10
Identify this insect:
A. Tabanid fly
B. Water flea
C. Tsetse fly
D. Reduviid bug
E. Black fly
►A

The insect indentified is a fly of the Tabanid family, which encompasses horseflies and deerflies.
The genus Chrysops of this family is the vector for loiasis. Clinical manifestations of this
infection include painful, localized subcutaneous nonpitting edema, calabar swellings and
eosinophila. Conjunctival migrations are common.

11
Ananas comosus causes an irritant dermatitis attributed to which irritant?
A. Phorbol esters
B. Capsaicin
C. Thiocyanates
D. Bromelin
E. Ranunculin
►D

Ananas comosus is the pineapple. Pineapples contain the irritants bromelin (a proteolytic
enzyme) and calcium oxalate (which causes fissures that grant bromelin access to dermal
vessels). Capsaicin is found in chili peppers. Phorbol esters are found in spurges, crotons,
poinsettas, and machineel trees. Thiocyanates are found in garlic, mustard, and radishes.
Ranunculin is found in buttercups.

12
Which of the following infections is commonly known to occur from contact with infected
sheep?
A. Infectious pustular dermatitis
B. Cutaneous anthrax

389
C. Oculoglandular syndrome of Parinaud
D. Erysipeloid
E. Infectious pustular dermatitis and cutaneous anthrax
►E

Infectious pustular dermatitis is another name for orf, or ecthyma contagiosum. It is transmitted
to humans by a parapoxvirus in sheep, goats, and reindeer. Cutaneous anthrax is also called
"woolsorter's disease" and is caused by Bacillus anthracis, a gram-positive rod transmitted from
contact with carcasses of infected sheep, cows, goats, and horses. The oculoglandular syndrome
of Parinaud is the combination of granulomatous conjunctivitis and preauricular
lymphadenopathy seen in cat-scratch disease.

13
Identify this tick:
A. Ixodes dammini
B. Dermacentor andersoni
C. Dermacentor variabilis
D. Amblyomma americanum
E. Rhipicephalus sanguineus
►D

This is Amblyomma americanum which transmits Lyme as well as RMSF.

14
The majority of naturally-occurring cases of anthrax:
A. Are extra-cutaneous
B. Are acquired through ingestion of spores
C. Are oropharyngeal
D. Are pulmonary
E. Are cutaneous
►E

The majority (95%) of naturally-occurring cases of anthrax are of the cutaneous form, acquired
from direct contact with the carcasses of dead sheep, cows, goats, and horses. Naturally-
occurring pulmonary, gastrointestinal (acquired by ingestion of spores) and oropharyngeal
infection with anthrax is less common.

15
Contact urticaria is a:
A. Type I hypersensitivity

390
B. Type II hypersensitivity
C. Type III hypersensitivity
D. Type IV hypersensitivity
E. Undetermined hypersensitivity
►A

Contact urticaria is an example of a type I hypersensitivity reaction. The classic example is a


latex induced contact urticaria. Type I reactions (immediate hypersensitivity reactions) involve
immunoglobulin E (IgE)–mediated release of histamine and other mediators from mast cells and
basophils.

16
Ehrlichiosis is classified in two primary forms. These are:
A. Monocytic and granulocytic
B. Endemic and epidemic
C. Intracellular and extracellular
D. Primary and secondary
E. None of these answers are correct
►A

Ehrlichiosis comes in two forms: monocytic, which invades mononuclear WBCs, and
granulocytic, in which peripheral PMNs show intracytoplasmic inclusions. Patients present with
fever, chills, headache, and myalgias, along with leukopenia, anemia, and thrombocytopenia.

17
A 54 year-old man with psoriatic arthritis on methotrexate and infliximab calls you after he is
bitten deeply on the ankle by his daughter's 2 year-old Chihuahua. He is concerned that his
immunocompromised status puts him at risk for infection and potential complications from this
bite. You correctly tell him:
A. As long as the dog has been vaccinated, he need not worry about serious
infections
B. Capnocytophaga from dog bites can cause sepsis in immunocompromised
victims
C. Eikenella corrodens is the most common cause of infection following dog bites
D. Pasteurella multocida infection is frequent after dog bites but not cat bites
E. Rickettsial pox can be seen after dog bites
►B

Local infection and cellulitis is a concern following domesticated animal bites; vaccination may
prevent against rabies, but not other infections. Cat bites most frequently result in Pasteurella

391
multocida infections, while dog bites result in Pasteurella canis. However, most bites are
polymicrobial with mixed aerobes and anaerobes. Infrequently systemic infections may result in
brain or lung abscess, endocarditis, or sepsis. Sepsis is more frequent in immunocompromised
victims and may be due to Capnocytophaga or Pasteurella.

18
A patient who experiences an allergic contact dermatitis in reaction to an extract from the Pinus
palustris tree may also be sensitized to:
A. Rosin
B. Mascara
C. Varnish
D. None of these answers are correct
E. All of these answers are correct
►E

Colophony, derived from the Pinus palustris tree, may cross react with rosin, medications,
mascara, adhesives, bandages, varnish, wax, paper products, and dental cement.

19
The insect pictured is:
A. A vector of relapsing fever
B. A vector of Lyme borreliosis
C. A vector of babesiosis
D. The cause of maculae cerulea
E. Ornithonyssus
►B

The insect pictured is a male Amblyomma americanum tick. This tick can serve as a vector for
Lyme borreliosis and Rocky Mountain Spotted Fever. Relapsing fever is transmitted by
Ornithodoros moubata. Babesiosis is trasnmitted by Ixodes dammini. Maculae cerulea are seen
in severe cases of crab lice. Ornithonyssus is the fowl mite.

20
Allergic contact dermatitis is a:
A. Type I hypersensitivity
B. Type II hypersensitivity
C. Type III hypersensitivity
D. Type IV hypersensitivity
E. Undetermined hypersensitivity
►D

392
Allergic contact dermatitis is an example of a Type 4 hypersensitivity reaction (delayed type
hypersensitivity reaction). Type 4 reactions are mediated by T-cells. Other types of
hypersensitivity reactions are described below. Type I reactions (immediate hypersensitivity
reactions) involve IgE–mediated release of histamine and other mediators from mast cells and
basophils. Type II reactions (cytotoxic hypersensitivity reactions) involve immunoglobulin G or
immunoglobulin M antibodies bound to cell surface antigens, with subsequent complement
fixation. Type III reactions (immune-complex reactions) involve circulating antigen-antibody
immune complexes that deposit in postcapillary venules, with subsequent complement fixation.

21
A patient comes to you with complaints of numerous erythematous pruritic papules under her
bathing suit after swimming in the ocean. What is the most likely diagnosis?
A. Swimmer”s itch
B. Hot tub folliculitis
C. Seabather”s eruption
D. Hydrozoa contact
E. Nematocyst stings
►C

Sea bather”s eruption is seen after ocean exposure and can be intensely pruritic. It is due to
jellyfish larvae (Linuche unguiculata) and can be prevented by rinsing off soon after coming out
of the water. Swimmer”s itch is a cercarial dermatitis caused by avian schistosomes which
occurs after freshwater exposure. It is usually seen on exposed areas. Portuguese man of war
(hydrozoa) stings can lead to hemorrhagic lesions with vesiculations.

22
Which of the following statements regarding Megalopyge opercularis is correct?
A. Are also called "bed bugs" by lay people
B. Contact with this creature often causes immediate pain, urticaria, and
erythematous papules or purpura in a tram-track pattern
C. Bites are usually asymptomatic, initially, but then form linear purpuric macules
D. These creatures can be vectors of many diseases from typhus to plague
E. These creatures, found in Hawaii, are nocturnal carnivores that produce painful
wounds by discharging venom
►B

Megalopyge opercularis (puss caterpillar) is one of the most widespread causes of caterpillar
dermatitis. Contact with this creature often causes immediate pain, urticaria, and erythematous
papules or purpura in a tram-track pattern. Bites of Cimicidae (bedbugs) are usually

393
asymptomatic, initially, but then form linear purpuric macules. Fleas can be vectors of many
diseases from typhus to plague. Chilopoda (centipedes) are nocturnal carnivores that produce
painful wounds by discharging venom. Scolopendra is a species of centipede found in Hawaii.

23
Which of the following vectors transmits Borrelia duttonii?
A. Ixodes pacificus
B. Rhipicephalus sanguineus
C. Ornithodoros moubata
D. Amblyomma americanum
E. none of the above
►C

Borrelia duttonii is a pathogenic organism of relapsing fever. It is transmitted by Ornithodoros


moubata. Ixodes pacificus is one of the vectors of Lyme Borreliosis. Rhipicephalus sanguineus is
a vector of Ehrlichiosis. Amblyomma americanum is a vector of Lyme Borreliosis and Rocky
Mountain Spotted Fever.

24
Which of the following is not a hallmark of allergic contact dermatitis?
A. Eosinophils
B. CD4+ Lymphocytes
C. Langerhans cells
D. Neutrophils
E. Vesicles
►D

Langerhans cells present the antigens to CD4+ lymphocytes in the dermis and after traveling
through lymphatics to regional lymph nodes. Eosinophils may also be present. Neutrophils are
not characteristic of this reaction.

25
A butcher presents with numerous verrucous papules of the hands and fingers. Based on her
occupation, you consider a diagnosis of infection with which HPV types?
A. HPV 16 and 18
B. HPV 1 and 2
C. HPV 2 and 7
D. HPV 6 and 8
E. HPV is not associated with this condition
►C

394
Meat, fish, and poultry handlers often suffer from hand and finger verrucae, usually caused by
human papilloma virus types 2 and 7.

26
Solenopsin D is a:
A. Hemolytic factor
B. Phospholipase
C. Piperdine derivative
D. Histamine-releaser
E. Hemolytic factor, piperdine derivative, histamine-releaser
►E

Solenopsin D is a venom which is a hemolytic factor, a piperidine derative and leads to


histamine release. The venom from honeybeeds contains phospholipase A .

27
An iguana is a pet that can harbor this organism that has been known to cause cellulitis with
severe systemic manifestations. The organism is most likely:
A. Serratia marcescens
B. Salmonella
C. Cheyletiella
D. Dermayssus
E. Glyciphagus
►A

An iguana is an inexpensive pet that can carry herpes, Serratia marcescens and salmonella.
Serratia marcescens causes severe cellulitis in humans.

28
Which of the following mites is also known as the chigger and often causes intense pruritus on
the ankles, legs, or belt line?
A. Ornithonyssus
B. Glyciphagus
C. Demodex
D. Allodermanyssus sanguineus
E. Trombicula alfreddugesi
►E

395
Trombicula alfreddugesi (chigger mite, harvest mites) frequently causes intense pruritus on the
ankles, legs, or belt line. Contact occurs during summer and fall. In sensitized individuals,
papular urticaria, vesiculation, or a granulomatous reaction with fever and lymphadenopathy
may occur. Allodermanyssus sanguineus (house mouse mite) is the vector of rickettsial pox.
Demodex folliculorum is a mite that can live within the sebaceous glands of hair follicles and
may be important in some subsets of patients with acne rosacea, as well as patients with HIV
infection and folliculitis. Glyciphagus (cheese mite) is the cause of grocer's itch. Ornithonyssus
(fowl mite) can carry Western equine encephalitis.

29
Flying squirrels can carry many infections, including which of the following?
A. Epidemic typhus
B. Staphlyococcus sp
C. Toxoplasma gondii
D. none of these infections are correct
E. all of these infections are correct
►E

Flying squirrels can carry many infections, including Toxoplasma gondii, Staphylococcus sp.,
and Rickettsia prowazekii (via the body lous, causing epidemic typhus).

30
A patient has an necrotic lesions with extensive gangrene. In children, it can cause fever, chills,
vomiting, joint pain and hematuria with shock and death. The spider has a violin-shaped marking
on the abdomen and is therefore the:
A. Loxosceles
B. Lactrodectus
C. Lycosidae
D. Hidippus
E. Chiracanthium
►A

This patient has a bit from the brown recluse spider also known as the Loxosceles. The venom
content is the sphingomyelinase-D. Treatments have been disappointing such as dapsone,
steroids, aspirin, and anti-venom.

31
Plants from the family Moraceae include which of the following?
A. Ficus carica
B. Fennel

396
C. Burning bush
D. All of these answers are correct
E. None of these answers are correct
►A

Ficus carica (fig tree) is a member of the family Moraceae and a cause of phytophotodermatitis.
Burning bush is a member of the family Rutaceae. Fennel is a member of the family Apiaceae.

32
Snake bite antivenin is derived from:
A. Human antibody
B. Mouse antibody
C. Antigen extracted directly from the attacking snake
D. Horse antibody
E. Pooled IVIg
►D

Following envenomation, antivenins are derived from horses.

33
Onions and chives:
A. Contain diallyl disulfide
B. Are members of the family Anacardiacea
C. Contain pentadecylcatechol
D. Are members of the same family as Gingko
E. Cross-react with artichoke in sensitized patients
►A

Onions, garlic, and chive are members of the family Alliaceae and contain diallyl disulfide (also
allylpropyl disulfide, allicin).

34
Erysipeloid is an infection that causes a sharply marginated and polygonal patches of bluish
erythema. Patients usually develop a solitary violaceous and tender nodule that vesiculates. The
most likely source of this infection is:
A. Pigs
B. Deer
C. Plants
D. Tress
E. Tics

397
►A

Erysipeloid is caused by a gram positive rod called Erysipelothrix insidiosa or rhusiopathiae.


This type of bacteria is found in fish, birds, mammals, and shellfish. It usually affects people
who work with these animals (farmers, butchers, fish handlers, and veterinarians can be
affected). Treatment is with erythromycin or penicillin.

35
The vector for Babesia microti is also a vector for:
A. None of these answers are correct
B. All of these answers are correct
C. Francisella tularensis
D. Rickettsia rickettsii
E. Borrelia burgdorferi
►E

The vector for Babesia microti (pathogenic organism of babesiosis), is Ixodes dammini, which is
also a vector for Borrelia burgdorferi, the pathogenic organism of Lyme borreliosis. Francisella
tularensis (the pathogenic organism of tularemia) is transmitted by Dermacentor andersoni and
Dermacentor variabilis. Rickettsia rickettsii (the pathogenic organism of Rocky Mountain
Spotted Fever) is transmitted by Dermacentor andersoni, Dermacentor variabilis, and
Amblyomma americanum.

36
Which of the following infections is caused by a gram-positive rod?
A. Erysipeloid of Rosenbach
B. Brucellosis
C. Cat-scratch disease
D. Glanders
E. Milker's nodule
►A

Erysipeloid of Rosenbach is an infection with Erysipelothrix insidiosa or Erysipelothrix


rhusiopathiae, gram-positive rods. Brucellosis is caused by infection with Brucellae (gram-
negative rod). Cat-scratch disease is caused by infection with Bartonella henselae, a gram-
negative rod. Glanders is caused by infection with Burkholderia Mallei, a gram-negative rod.
Milker's nodule is caused by the paravaccinia viurs, a parapoxvirus related to the orf virus.

37
Identify this insect:

398
A. Phlebotomous sandfly
B. Tsetse fly
C. Reduviid bug
D. Water flea
E. Mango fly
►C

This a picture of the reduviid bug, also called the kissing bug or assassin bug. It is the vector for
American Trypanosomiasis (Chagas disease), which is caused by Trypanosoma cruzii. The
reduviid bug prefers to bite at mucocutaneous junctions.

38
A 21 year old college student returning back to school from a spring break trip presents with 12
days of headache, fever, and a skin lesion with annular erythema on her upper back measuring 15
cm in diameter. This multi-system disease is most likely caused by?
A. Treponema pallidum
B. Borrelia burgdorferi
C. Staphylococcus aureus
D. Francisella tularensis
E. Pseudomonas aeruginosa
►B

Lyme disease is a multisystem disease caused by Borrelia burgdorferi that is transmitted to


humans by ticks of the genus Ixodes. Patients present with variable constitutional symptoms
including HA, fever, malaise, and arthralgias. 75% of patients develop erythema chronicum
migrans that begin as an erythematous papule that can progress to an annular erythema reaching
20 cm or more in size.

39
A 47 year old park ranger developed a diffuse, erythematous, eczematous eruption after fighting
a forest fire. What is the most likely allergen causing this airborn contact dermatitis?
A. Diallyl disulfide
B. Pyrethrins
C. Alphamethylene
D. Psoralen
E. D-usnic acid
►E

399
Lichen causes airborne allergic contact dermatitis in lumberjacks, forest workers, and people
exposed to firewood, funeral wreaths, and masculine fragrances. The primary allergen is d-usnic
acid.

40
Insects from the species pictured:
A. Transmit malaria
B. Trasmit yellow fever
C. Transmit dengue fever
D. Transmit malaria and yellow fever
E. Trasmit yellow fever and dengue fever
►E

The insect pictured is an Aedes mosquito. Aedes transmits yellow fever and dengue fever.
Anopheles mosquitoes transmit malaria.

41
When doing patch testing for compositae sensitivity, which of the following substances is used?
A. Colophony
B. Pentadecacatechols
C. Rhus
D. Sesquiterpene lactone mix
E. Tuliposide A
►D

Colophony cross reacts with turpentine resin, hairpiece adhesive, mascara, dental cement and
others. Pentadecacatechols are in the rhus family and are found in poision ivy, oak and sumac in
addition to cross reacting with mango and cashew. Tuliposide A is found in Peruvian lily
(alstomeria), and tulip.

42
Poison sumac is:
A. A member of the Toxicodendron genus
B. A source of tuliposide A
C. The common name for Myroxylon balsamum
D. The common name for Pinus palustris
E. A source of colophony
►A

400
Poison ivy, oak, and sumac are members of the Anacardiacea family, Toxicodendron genus.
Alstromeria (Peruvian lily) is the source of tuliposide A. Myroxylon balsamum is the source of
Balsam of Peru. The Pinus palustris tree is the source of colophony.

43
Cantharadin is derived from:
A. Ranunculaceae
B. Pentadecacatechol
C. Solanaceae
D. Compositae
E. Lytta vesicatoria
►E

Ranunculaceae (buttercup) causes irritant dermatitis. Pentadecacatechol is the rhus antigen found
in poison ivy, oak and sumac. Solanaceae (chili pepper) contains capsaicin. Pyrethrin is derived
from compositae (chrysanthemum flower). Lytta vesicatoria (spanish fly) is a blister beetle from
which cantharadin is made.

44
Immunologic contact urticaria is frequently caused by:
A. Hevea brasiliensis
B. Plants of the Urticaceae family
C. Plants of the Euphorbiacea family
D. Answers A and C
E. None of these answers are correct
►A

Contact urticaria occurs after direct contact, and can be immunologic (IgE) or non-immunologic
(toxin-mediated). Type I hypersensitivity, or IgE mediated reactions, require previous
sensitization. Frequent causes of immunologic eruptions include latex proteins (Hevea
brasiliensis). Plants such as stinging nettles (Urticaceae family) and spurge nettle
(Euphorbiaceas) cause nonimmunologic urticaria, which occurs without previous sensitization in
any host.

45
Head lice is found in children with six legs and a long narrow body. The louse lives for one
month. It is seen with the naked eye and the nits are seen in the scalp. The most likely organism
is:
A. Pediculus capitis
B. Pthirus pubis

401
C. Chilopoda
D. Callitroga americana
E. Pediculus humanus var. corporis
►A

The organism that causes head lice is called Pediculus capitus. The treatment recommendations
include two applications of a pediculicide on week apart. There is some resistance to permethrin
5% cream.

46
Match the photo to the plant identified:
A. Alliaceae
B. Primulaceae
C. Anacardiacea
D. Compositae
E. Alstromeria
►E

Alstromeria is pictured.

47
The toxin holothurin, released by which of the following marine creatures, can cause
conjunctivitis in exposed swimmers?
A. None of these answers are correct
B. Sea cucumber
C. Linuche unguiculata
D. Edwardsiella lineate
E. Chironex fleckeri
►B

Sea cucumbers are cucumber-shaped bottom-dwellers that can cause conjunctivitis due to release
of a toxin called holothurin to which nearby swimmers can be exposed. The larva of Linuche
unguiculata (thimble jelly fish) and Edwardsiella lineate (sea anemone) are causes of seabather's
eruption (or sea lice). Chironex fleckeri is the box jellyfish; its stings are extremely toxic and
may lead to shock and death, especially in small adults and children.

48
This caterpillar has spines, which can cause hemorrhagic papules in a grid like pattern:
A. Megalopye opercularis
B. Lonomia caterpillars

402
C. Black swallow caterpillars
D. Saddleback caterpillars
E. Costa rican caterpillars
►A

Megalopye opercularis also known as the puss caterpillar has spines, which can cause
hemorrhagic papules in a grip like patter. The saddleback caterpillar can cause swelling of the
hands, nausea and a bad rash that takes time to clear.

49
This spider can cause acutely painful and may swell but do not cause necrosis. The venom
contains neurotoxins and can cause vomiting, violent cramps, paralysis, and spasms. The most
likely spider is:
A. Lactrodectus
B. Loxosceles
C. Lycosidae
D. Phidippus
E. Cheiracanthium
►A

Lactrodectus is the black widow spider. The patient can within hours develop chills, vomiting,
violent cramps, paralysis, spas,s and symptoms mimicking an acute abdomen. An antivenin may
be helpful up to 90 hours after the bite.

50
Which of these spiders is bright green with red spots, with black spines on its legs?
A. Phidippus formosus
B. Loxosceles reclusa
C. Latrodectus mactans
D. Peucetia viridans
E. Centruroides sculpturatus
►D

Peucetia viridans (green lynx spider) is common in the southern United States. It is bright green
with red spots; the legs have black spines. The bite is painful, with tenderness and pruritus.
Phidippus formosus (jumping spider) is an aggressive spider with dark body hairs and various
white patterns. They are the most common biting spiders in the United States. Loxosceles reclusa
(brown recluse) is a tan spider with a violin-shaped marking on the abdomen. The bite may be
painless, however the patient may develop erythema, a vesicle, and eventual necrosis.
Latrodectus mactans (black widow) is a large, black, shiny spider with an hourglass shaped white

403
or red marking on the abdomen. Bites are acutely painful, and may swell but do not cause
necrosis; the venom contains neurotoxins. Centruroides sculpturatus is a primary scorpion
species in the U.S.

51
Which of the following is the most common cause of dermatitis in florists?
A. Calcium oxalate
B. Mechanical dermatitis
C. Tuliposide A
D. Primin
E. Sesquiterpene lactones
►A

Calcium oxalate is present in the bulbs and stems of Narcissus (daffodil).

52
Which of the following contains active furocoumarins?
A. Lichens
B. Chrysanthemum
C. Fig
D. Mango
E. Primrose
►C

The fig tree may cause a phytophotodermatitis.

53
A patient comes into the office and states that they have been bitten by a cat. The most common
organism to cause infection in the skin is:
A. Pasturella multocida
B. Pasturella canis
C. Eikenella corrodens
D. Streptococcus pyogenes
E. Staphylococcus aureus
►A

Cat bits are caused by pasturella multocida. All bites should be treated with augmentin.
Pasturella canis is the most common organism in dogs. Eikenell corrodens is the most common
organism for human bits.

404
54
What is the initial treatment of choice for this dermatitis?
A. Observation
B. 5% acetic acid
C. Topical petrolatum jelly
D. Topical antibiotics
E. Warm compresses
►B

The linear appearance of the dermatitis is typical of jellyfish stings. To prevent further
nematocyte activation, 5% acetic acids should be applied for 15-30 minutes. Papain found in
meat tenderizers or 70% isopropyl alcohol may also be applied. Warm compresses are
contraindicated as they may increase systemic absorption of the venom.

55
Which organism causes peliosis?
A. Brucellae
B. Pseudomonas mallei
C. Bartonella hensalae
D. Burkholderia pseudomallei
E. Bacillus anthracis
►C

Peliosis hepatis are seen in bacillary angiomatosis caused by Bartonella henselae which usually
occurs in patients with AIDS. Brucelliosis is also known as Malta fever and is caused by
ingesting raw goat mil and unpasteurized goat cheese; skin lesions occur in 20% of patients as
violaceous papulonodules on the trunk and legs. Burkholderia pseudomallei causes Whitmore
disease or meliodosis characterized by pulmonary disease and septicemia. Anthrax is caused by
Bacillus anthracis, a gram-positive rod, and can cause pulmonary, gastrointestinal, or cutaneous
disease.

56
Which of the following contains significant levels of naturally occurring psoralens?
A. Potatos
B. Parsnips
C. Carrots
D. Tomatoes
E. Peas
►B

405
Significant quantities of psoralens are present in fruits and vegetables such as limes, lemons,
figs, and parsnips. Other furocoumarine(psoralen)-containing plants that can produce
phytophotodermatitis are celery, dill, lime bergamot,parsley, meadow grass, mokihana (Pelea
anisata) berries, mustard, and St. John's wort

57
Match the photo to the plant identified:
A. Poison ivy
B. Gingko
C. Poison oak
D. Mango leaves
E. Poison sumac
►C

Poison oak is identified by leaf shape - irregular lobing and varied leaf size.

58
What is cutaneous larva migrans caused by?
A. Ancylostoma caninum and A. Brazilense
B. Toxoplasma gondii
C. Onchocerca volvulus
D. Shistosoma hematobium, S. japonicum & S. mansoni
E. Strongyloides stercoralis
►A

Cutaneous larva migrans is caused by the hookworms, Ancylostoma caninum and A. Brazilense.
Toxoplasma gondii causes toxoplasmosis, which is usually minor and self-limiting but can have
serious or even fatal effects on a fetus whose mother first contracts the disease during pregnancy
or on an immunocompromised human or cat. Onchocerca volvulus is a nematode which is spread
to humans by the bite of the blackfly (genus: Simulium). This causes onchocerciasis, also known
as reiver blindness, which is the world”s second leading cause of blindness. Shistosoma
hematobium, S. japonicum & S. mansoni are flatworms that causes schistosomiasis, which is
acquired by swimming or playing in infected water. Strongyloides stercoralis is a nematode
which causes strongyloidiasis, which mainly affects the GI tract as well as red hive-like areas
near the anus

59
Identify this plant:
A. Alstromeria
B. Toxicodentron

406
C. Myroxylon balsamum
D. Ragweed
E. Primrose
►E

This is primrose (primulaceae) a cause of allergic contact dermatitis.

60
Phytophotodermatitis is caused by which of the following wavelengths of light?
A. 100-200 nm
B. 200-290 nm
C. 290-320 nm
D. 311 nm
E. 320-400 nm
►E

UVA rays are responsible for the phototoxic reaction which occurs with phytophotodermatitis.

61
This patient acquired an infection with a microorganism that may have been inoculated into the
skin from contact with:
A. All of these answers are correct
B. None of these answers are correct
C. Grasses
D. Sphagnum moss
E. Rose thorns
►A

This patient has sporotrichosis, which is caused by Sporothrix schenckii. Sporothrix schenckii
can be inoculated into the skin by grasses, sphagnum moss, and rose thorns.

62
A 22 year-old man presents with an irregular laceration on the dorsal metacarpals following a fist
fight in which he struck someone in the mouth. The tissue surrounding the wound is
erythematous, edematous, indurated, warm, and tender. There is some purulence of the wound.
You consider infection with:
A. Pasteurella multocida
B. Eikenella corrodens
C. Capnocytophaga
D. Erysipelothrix rhusiopathiae

407
E. Pasteurella canis
►B

Human bites may result in infection with Eikenella corrodens, a gram-negative bacillus that is
part of the normal flora of the human mouth. This infection can follow fist fights with blows to
the mouth.

63
Ophthalmia nodosa may result following injury by which of the following creatures?
A. Theraphosidae
B. Chironex
C. Lactrodectus
D. Cheyletiella
E. Loxosceles
►A

Tarantulas are relatively large, hair spiders belonging to the Theraphosidae family. Many are
kept as household pets given their longevity (they may live to 20 years or more) and slow-
moving, relatively sedentary lifestyle. However, when agitated, tarantulas may engage in rubbing
their hind legs against their abdomen, resulting in release of numerous urticating, barbed hairs.
These projectile hairs may then lodge in the conjunctiva, cornea, iris, or even retina, resulting in
multifocal and chronic granulomatous inflammation known as ophthalmia nodosa. Similar
reactions may be seen secondary to ocular penetration of the sensory setae of caterpillars.

64
The hobo spider is the leading cause to necrotic arachnidism in the Pacific Northwest. The spider
is larger, with a herringbone-striped pattern on the abdomen and the bite can be painless but
develop erythema and numbness. This spider is also known as:
A. Tegenaria agrestis
B. Loxosceles reclusa
C. Lactrodectus mactans
D. Peucetia viridans
E. Phidippus formosus
►A

The other name is Tegenaria agrestis. The necrotic eschar will develop with slow healing.
Systemic reactions can consist of headache, visual disturbances, and hemodynamic changes.

65
Sesquiterpene lactones are found in which of the following plants?

408
A. Alstromeria
B. Pinus palustris
C. Artichoke
D. Chive
E. Gingko
►C

Sesquiterpene lactones are sensitizers found in plants of the Asteraceae (Compositae) family.
Members include ragweed, pyrethrum, chrysanthemum, weeds, feverfew, and artichoke. The
sensitzer found in alstromeria (Peruvian lily) is tuliposide A (a glycoside). The Pinus palustris
tree is the source of colophony. Chives contain diallyl disulfide. Gingko fruit pulp is a sensitizer.

66
Match the photo to the plant identified:
A. Poison ivy
B. Gingko
C. Poison oak
D. Mango leaves
E. Poison sumac
►E

Poison sumac leaves grow in groupings of 7-13 with a single leaf at the tip.

67
Patients with an allergy to the compositae family may have an allergic reaction to which of the
following drugs?
A. Pyruvic acid
B. Pyrethrins
C. Psoralens
D. Propylene glycol
E. Pseudoephedrine
►B

Pyrethrins are derived from chrysanthemum flowers. Patients with allergy to these flowers or
ragweed may experience allergic symptoms after using pyrethrins.

68
When bitten with the spider in the photograph, the active agent in the venom is:
A. Alpha lactrotoxin
B. Histamine

409
C. Hyaluronidase
D. Lipase
E. Sphingomyelinase-D
►A

This is a black widow spider which can be identified by its shiny black body with characteristic
red hourglass marking on the abdomen.

69
Unilateral swelling of the eyelid at the site of the bite of the assassin bug is called what?
A. Unilateral Chagas' disease
B. Romana's sign
C. Rothschild's sign
D. Russell's sign
E. Raynaud's sign
►B

Rothschild's sign is alopecia of the lateral 1/3 of the eyebrows secondary to hypothyroidism.
Russell's sign is the development of callouses on the dorsal dominant hand. Raynaud's sign is the
development of acrocyanosis.

70
A 45-year old man had exposure to an exotic pet and afterwards develops fevers and a rash. He
is diagnosed with epidemic typhus. Which of the following animals did he most likely come in
contact with?
A. Hedgehog
B. Iguana
C. Chinchilla
D. Cockatoo
E. Flying squirrel
►E

Flying squirrels can be carriers of many infections including Toxoplasma gondii,


Staphylococcus, and Rickettsia prowazekki, the latter of which causes epidemic typhys. African
and European hedgehogs have been known to transmit salmonella and Mycobacterium marinum,
respectively. Iguanas have been demonstrated to be carriers of Serratia marcescens, a herpes-type
virus, and salmonella. Chinchillas are frequently colonized with Trichophyton mentagrophytes
and gram-negative bacteria like Klebsiella pneumoniae which can be dangerous especially in
immunocompromised patients. Cockatoos and pigeons can transmit Cryptococcus neoformans.

410
71
The Phidippus fornosus is an aggressive spider with dark body hairs and is the most common
biting spider in the U.S. and is found in the south. The other name for it is:
A. Jumping spider
B. Lynx spider
C. Wolf spider
D. Brown recluse spider
E. Necrotic spider
►A

The jumping spider is the most common cause of biting spiders There are no systemic symptoms
known but the bit is painful with toxic venom.

72
Capsaicin causes which of the following?
A. Decreased firing of nerve fibers transmitting pain
B. immediate increased substance P release
C. immediate decreased substance P release
D. Blocking the influx of sodium ions into the cell
E. Creating a barrier between nerves and their stimuli
►B

Repeated substance P release leads to eventual depletion of this agent which causes pain
transmission.

73
Which of the following may NOT occur after contact with the organism in the photo?
A. Purpura in a tram-track pattern
B. Urticaria
C. Tram-track calcification on skull x-ray
D. Loss of vision
E. Pruritus
►C

The most common response is erythema, pruritus and urticaria. Purpura and hemorrhage in a
tram-track or ladder pattern may be seen as well. Ophthalmia nodosa may occur if hairs injure
the cornea. Tram-track calcification on skull films is seen in Sturge-Weber syndrome.

74

411
Sharp hairs on plants of the Urticaceae family contain which of the following toxins that are
released into the skin causing rapid edema, pruritus, and burning?
A. Histamine
B. Serotonin
C. Acetylcholine
D. All of these answers are correct
E. Histamine and acetylcholine
►D

Sharp hairs on plants such as stinging nettles (Urticaceae family) contain toxins (histamine,
serotonin, and acetylcholine) which are released into the skin causing rapid edema, pruritus, and
burning.

75
Which of these mites is the most likely cause of a pruritic papulovesicular eruption in a baker?
A. Dermatophagoides
B. Dermanyssus
C. Acarus
D. Cheyletiella
E. none of the above
►C

Acarus (grain mite) causes so-called "baker's itch." Dermatophagoides (dust mite) causes allergic
reactions, Dermanyssus (fowl mite) causes equine encephalitis, and Cheyletiella causes walking
dandruff in dogs and cats.

76
Which of the following plants are members of the Solanaceae family?
A. Prickly pear
B. Chili pepper
C. Garlic
D. Radish
E. Crotons
►B

Chili peppers belong to the Solanaceae family. Prickly pears (Opuntia vulgaris), crotons
(Euphorbiaceae family), garlic (Alliaceae family), and radishes (Brassicaceae family) are not
members of this family.

77

412
Furocoumarins, such as 5-methoxypsoralen or 8-methoxypsoralen:
A. Cause a photoallergic reaction after contact with the skin, followed by exposure
to ultraviolet light in the 320-400nm range
B. Cause a type IV delayed hypersensitivity reaction after exposure to all
wavelengths of UV light
C. Cause a phototoxic reaction after contact with the skin, followed by exposure to
ultraviolet light in the 280-320nm range
D. Cause a phototoxic reaction after contact with the skin, followed by exposure to
ultraviolet light in the 320-400nm range
E. Cause a photoallergic reaction after contact with the skin, followed by exposure
to ultraviolet light in the 280-320nm range
►D

Unlike photoallergic reactions, which involve the immune system (type IV or delayed
hypersensitivity), phytophotodermatitis is non-immunologic. Photoallergic reactions are more
frequently caused by other, non-plant sources. Furocoumarins, such as 5-methoxypsoralen or 8-
methoxypsoralen, cause a phototoxic reaction after contact with the skin, followed by exposure
to ultraviolet light in the UVA range (320 to 400nm).

78
Which of the following is false regarding this image?
A. This is the tsetse fly
B. This is a glossina fly
C. A large abscess may occur following a bite
D. This transmits sleeping sickness
E. This transmits trypanosomes
►C

Cutaneous reactions tend to be minimal from the bite of the tsetse, or glossina, fly which
transmits trypanosomes and sleeping sickness.

79
The necrosis and hemolysis that can result from a brown recluse spider bite may be attributed to
which of the following toxins?
A. Alpha lactrotoxin
B. Sphingomyelinase-D
C. Histamine
D. Hyaluronidase
E. Lipase
►B

413
The brown recluse spider (Loxosceles reclusa) is a tan spider with a violin-shaped marking on
the thorax. Its bite may be painless, however the patient may develop erythema, a vesicle, and
eventual necrosis. This may result in the "red, white, and blue sign." Systemic reactions
(viscerocutaneous loxoscelism) may occur. The venom contains several toxins, however
sphingomyelinase D seems to be causative of the necrosis and hemolysis.

80
The bug that is responsible for Romana's sign in a person at the site of the bit of the bug is
known as the :
A. Reduviid bug
B. Pulex irritans
C. Cimicidae
D. Anopheles
E. Solenopsis
►A

The reduviid bugs are members of the Triatoma species. They are bloodsuckers and transmit
Chaga's disease by carring trypanosoma cruzi. The Romana's sign is unilateral eyelid swelling at
the site of the bite of the assassin bug.

81
Which of the following is a common cause of seabather's eruption?
A. Edwardsiella lineate
B. Linuche unguiculata
C. Thimble jelly fish
D. Sea anemone
E. All of these answers are correct
►E

Seabather's eruption (or sea lice) can be sporadic or part of an outbreak. Clinically, it appears as
dermatitis beneath areas covered by swim suits. The geographic distribution is most frequent
from the Caribbean to Bermuda, however there have been three outbreaks at Long Island, NY
beaches. A variety of cnidarian larvae can cause this eruption, including thimble jelly fish
(Linuche unguiculata) and sea anemone (Edwardsiella lineate). Quick removal of bathing suits
and rinsing can help with prevention. Symptom relief is the mainstay of therapy.

82
Mechanical irritant dermatitis can be caused by:
A. Glochids

414
B. latex
C. Bromelin
D. Capsacin
E. All of these answers are correct
►A

Mechanical irritant dermatitis can be caused by the direct effects of thorns and barbs. Small
glochids (barbed hairs or brisles) or large thorns (on cacti for example) can become imbedded
into the skin causing injury and resultant dermatitis. Secondary infection can be caused by
microorganism inoculation.

83
The majority of phytophotodermatitis is caused by plants from which family?
A. None of these answers are correct
B. Apiaceae
C. Liliaceae
D. Alliaceae
E. Compositae
►B

The Apiaceae family (formerly Umbelliferae) includes parsley, celery, parsnip, hogweed, and
fennel, and causes the majority of phytophotodermatitis.

84
Which creature is a human flea that can also be seen on dogs?
A. Pulex irritans
B. Ctenocephalides canis
C. Lytta vesicatoria
D. Liponyssoides sanuineus
E. Pediculus humanus
►A

Pulex irritans is the human flea; it can also be seen on dogs. More frequent infestations on
domestic animals are by Ctenocephalides felis and canis (on cats and dogs respectively). Lytta
vesicatoria is a blister beetle from which cantharadin is derived. Pediculus humanus var. corporis
is the human body louse.

85
Ecthyma contagiosum or infectious pustular dermatitis is transmitted from animal to humans by
fomites. The host is:

415
A. Sheep
B. Cow
C. Grass
D. Ticks
E. Pigs
►A

Ecthyma contagiosum also known as orf is caused by parapoxvirus and the host is sheep, goat, or
reindeer. It is transmitted to humans by direct contact or fomites. Animal and meat handlers are
the most susceptible to getting orf.

86
Which snake is part of the Crotalidae family?
A. Rattlesnake
B. Garden snake
C. Water snake
D. Coral snake
E. Cobra
►A

The Crotalidae family includes rattlesnakes and copperheads. The coral snake is part of the
elapidae family.

87
A previously healthy 25-year-old man presents to the ER 8 hours after experiencing an extremely
painful spider bite. The bite site is erythematous and indurated, without necrosis or eschar. The
patient is vomitting and experiencing chills, violent cramps, and abdominal pain and rigidity.
The surgical consultant suspects an acute abdomen. You correctly propose that:
A. The patient needs anti-venom after a likely Lactrodectus bite
B. The patient has likely experienced a hobo spider bite
C. The patient needs anti-venom after a likely Phidippus formosus bite
D. The patient has a likely Lycosidae bite and needs supportive care
E. The patient has a likely Loxosceles bite and should go to the OR for excision
and debridement
►A

The patient has signs and symptoms a black widow spider bite, caused by Lactrodectus. Anti-
venom may be helpful up to 90 hours after the bite.

88

416
Which of the statements regarding Tegenaria agrestis is correct?
A. Stings from this scorpion can produce cardiovascular complications
B. Contact with this creature results in erythematous papules, hemorrhage, or
purpura in a classic "tram-track" pattern
C. This creature is the leading cause of necrotic arachnidism in several states of the
Pacific Northwest
D. This ant injects a venom that contains a hemolytic factor, solenopsin D
E. None of these answers are correct
►C

Tegenaria agrestis (the Hobo spider) is the leading cause of necrotic arachnidism in several states
of the Pacific Northwest. These spiders are large, with a herringbone-striped pattern on the
abdomen. Contact with caterpillars (Lepidoptera) may produce lesions in a classic "tram-track"
pattern. Solenopsis (fire ants) ants inject a venom that contains a hemolytic factor, solenopsin D,
that causes a release of histamine and other vasoactive amines from mast cells.

89
This organism is:
A. Pediculus humanus
B. Pulex irritans
C. Chigoe flea
D. Pediculus capitis
E. Pthirus pubis
►E

This is a crab louse. It is shorter and wider than the head and body louse. Note the crab-like
claws.

90
A tarantulas is known for its urticating hair. The spider releases hairs in the direction of the
attacker and can injure the eye. The chronic eye condition that can occur is:
A. Ophthalmia nodosa
B. Ophthalmia erythema
C. Ophthalmia streaks
D. Ophthalmia papilledema
E. Ophthalmia angioid
►A

417
Tarantulas release hair and can cause chronic granulomatous reaction called ophthalmia nodosa
and loss of vision. The spider releases hair and can penetrate the skin as deeply as the reticular
dermis.

91
This snake, the coral snake, is a member of what family?
A. Crotalidae
B. Lepidoptera
C. Elapidae
D. Theraphosidae
E. Apiaceae
►C

The Crotalidae family includes rattlesnakes and copperheads. Lepidoptera are caterpillars.
Theraphosidae is the scientific name for tarantulas. Apiaceae (Umbelliferae) is a plant family
which includes parsley, celery and fennel. Elapidae is the correct family of snakes, including
coral snakes.

92
“Strimmer” dermatitis is caused by which of the following?
A. M. marinum
B. Pseudomonas
C. Cryptococcus
D. Chervil
E. Lime
►D

Strimmer dermatitis is a phytophotodermatitis caused by spray of plant matter when using a


weed-whacker or similar trimming tool.

93
Which species is a human flea that can also be seen on dogs?
A. Pulex irritans
B. Pediculus humanus
C. Liponyssoides sanuineus
D. Lytta vesicatoria
E. Ctenocephalides canis
►A

418
Pulex irritans is the human flea; it can also be seen on dogs. More frequent infestations on
domestic animals are by Ctenocephalides felis and canis (on cats and dogs respectively). Lytta
vesicatoria is a blister beetle from which cantharadin is derived. Pediculus humanus var. corporis
is the human body louse.

94
Which disease is transmitted by this insect?
A. African sleeping sickness
B. Chagas disease
C. Dracunculiasis
D. Visceral leishmaniasis
E. Onchocerciasis
►B

This a picture of the reduviid bug, also called the kissing bug or assassin bug. It is the vector for
American Trypanosomiasis (Chagas disease), which is caused by Trypanosoma cruzii. The
reduviid bug prefers to bite at mucocutaneous junctions.

95
On slide preparation, many organisms were noted on this patient, as well as occasional blue
macules on the buttocks. He most likely has:
A. Sarcoptes scabiei infestation
B. Pediculus humanus infestation
C. Chigoe fleas
D. Pediculus capitis infestation
E. Pthirus pubis infestation
►E

It is often very easy to find crab lice on physical exam in contrast with scabies. Head lice
infestations are usually limited to the scalp. Body lice are not often found on the patient, more
commonly in clothing seams. Maculae caerulea can be seen in widespread involvement as in this
case.

96
Match the photo to the plant identified:
A. Alliaceae
B. Primulaceae
C. Toxicodendron
D. Compositae
E. Alstromeria

419
►D

This is ragweed, a member of the compositae (or asteraceae) family.

97
Name this creature:
A. Demodex folliculorum
B. Ornithonyssus sylviarum
C. Pthirus pubis
D. Pediculus capitis
E. Pediculus humans corporis
►D

This is the head louse (Pediculus Humans Capitis) has a narrow body. The body louse or
Pediculus humanus var. corporis has an elongated abdominal region . The pubic louse (pthirus
pubis) is shorter, and squatter.

98
Which chemical is present in large quantities of this arachnid's venom?
A. Sphingomyelinase D
B. Alpha lactrotoxin
C. Histamine
D. Hyaluronidase
E. Lipase
►C

The spider shown is the Wolf spider (Lycosidae). This Australian spider's bites contains large
amounts of histamine. Bites are painful with edema, erythema and subsequent lymphangitis.

99
Which one of the following diseases is not transmitted by the Ixodes tick?
A. Babesiosis
B. Ehrilichiosis
C. Lyme
D. Tick paralysis
E. Rocky Mountain Spotted Fever
►E

Babesiosis, ehrlichiosis, Lyme disease, and tick paralysis can all be transmitted by the Ixodes
tick. This tick has three main types: I. pacificus in California, I. ricinus in Europe, and I.

420
scapularis in the eastern US. Rocky Mountain Spotted Fever is typically transmitted by
Amblyoma americanum or Dermacentor ticks.

100
The female Sarcoptes scabiei var. hominis mite can lay up to 100 eggs. It has an ovoid, flat body
and how many short legs?
A. None of these answers are correct
B. 12
C. 6
D. 8
E. 10
►D

The female Sarcoptes scabiei var. hominis mite has an ovoid, flat body and 8 short legs.

101
A butcher‟s nodule is caused by which of the following?
A. HPV 24
B. HPV 16, 18
C. HPV 1
D. HPV 2, 7
E. HPV 13, 32
►D

HPV 24 causes common warts. HPV 16 and 18 are found in genital warts; 1 is for plantar warts
and types 13 and 32 are seen in Heck‟s Disease (focal epithelial hyperplasia).

102
All of the following are food mites except:
A. Acarus
B. Tyrophagus
C. Glyciphagus
D. Dermatophagoides
E. All of the listed answers are food mites
►D

Acarus (grain mite), Tyrophagus (grocery mite), and Glyciphagus (cheese mite) are all food
mites that ingest foodstuffs. They can produce papular urticaria or vesicopapular eruptions.
Dermatophagoides is the dust mite, and causes environmental allergic reactions.

421
103
A chef presents with bilateral hand cellulitis after several days working with a new raw fish dish.
Cultures grow Streptococcus iniae. What type of fish was he likely working with?
A. Cod
B. Mackerel
C. Tilapia
D. Freshwater salmon
E. Farm salmon
►C

Streptococcus iniae is an important cause of cellulitis, endocarditis, mengingoencephalitis, and


septicemia in fish handlers and cooks. It is found in tilapia, red drum, hybrid striped bass, and
rainbow trout.

104
Phytophotodermatitis caused by skin exposure to limes is caused by:
A. Ascorbic acid
B. Tuliposide A
C. Furocoumarin
D. D-limonene
E. Sesquiterpene lactone
►C

Phytophotodermatatitis (phototoxic eruptions from plants) are relatively commonly seen in


florists, nursery personnel, gardeners, and farm workers. Plants that cause this eruption may
produce photoxins such as furocoumarins Fig, fennel, lime, lemon, parsley, celery, dill, and
hogweed are all causes of phytophotodermatitis.

105
Ticks of the species shown:
A. Can transmit Lyme borreliosis and tularemia
B. Can transmit Lyme borreliosis but not Rocky Mountain Spotted Fever or
tularemia
C. Can transmit Rocky Mountain Spotted Fever and Lyme borreliosis
D. Can transmit Rocky Mountain Spotted Fever and tularemia
E. Can transit tularemia but not Rocky Mountain Spotted Fever
►D

422
The tick pictured is a male Dermacentor variabilis tick. Dermacentor variabilis can transmit
Rocky Mountain Spotted Fever and tularemia. Lyme borreliosis is transmitted by Ixodes
dammini, Ixodes pacificus, and Amblyomma americanum.

106
Which of the following statements regarding orf is correct?
A. The papillomatous stage of lesion progression preceeds the regenerative stage
B. Lesions last indefinitely without treatment
C. The papular stage of lesion progression follows the target stage
D. Orf is also known as "pseudocowpox."
E. The virus is very sturdy and survives many months in cold weather
►E

Orf, also known as ecthyma contagiosum or infectious pustular dermatitis, is transmitted to


humans by a parapoxvirus in sheep, goats, and reindeer. This is a self-limited condition with an
excellent prognosis lasting approximately 6 weeks. Lesion morphology changes through a series
of stages:
papular (red, elevated) →► target (nodule with red center, white middle ring, red periphery)
→► acute (red, weeping) →► regenerative (thin, dry crust overlying lesion) →►
papillomatous →► regressive. The virus is very sturdy and survives many months in cold
weather.
Milker's nodule, caused by the paravaccinia virus (a parapoxvirus, related to the orf virus),
infects cows and can be transmitted to dairy workers, and is also known as pseudocowpox.

107
Leishmania donovani is transmitted by:
A. Callitroga americana
B. Phlebotomus
C. Glossina
D. Attagenus megatoma
E. Anopheles
►B

Leishmania donovani is transmitted by Phlebotomid sandflies. Callitroga americana is the most


important cause of cutaneous myiasis in the United States. Glossina, the tsetse fly, transmits
sleeping sickness. Attagenus megatoma is a carpet beetle, the larvae of which can cause an
allergic contact dermatitis. Anopheles mosquitoes transmit malaria.

108

423
A infectious pustular dermatitis develops in a farmer. The virus is known to be very sturdy and
survives many months in the cold winter. There is localized lymphadenopathy and crusting. The
most likely source that it came from is:
A. Animals
B. Insects
C. Plants
D. Trees
E. Ticks
►A

This describes Orf. It is also known as ecthyma contagiosum. It is transmitted to humans by


parapoxvirus in sheep, goats, and reindeer. It is self limited with an excellent prognosis and last
for 6 weeks.

109
Babesia microti:
A. Is pathogenic in an infection that is endemic on Martha's Vineyard
B. Is an intracellular parasite
C. Is transmitted by Ixodes dammini
D. All of these answers are correct
E. None of these answers are correct
►D

Babesiosis is caused by an intracellular RBC parasite, Babesia microti. It is transmitted by the


larvae of Ixodes dammini. This disease is endemic in eastern Long Island, Martha's Vineyard,
and Nantucket, and carries an increased risk in those with T-cell depression or after splenectomy.
The condition is associated with fever, drenching sweats, myalgias, and hemolytic anemia.

110
The venom from which of the following creatures contains phospholipase A?
A. Theraphosidae
B. Honeybee
C. Cimicidae
D. Ctenocephalides canis
E. Solenopsis invicta
►B

Honeybees (Hymenoptera) leave a barbed ovipositor and paired venom sacs impaled into their
victim. The honeybee dies after stinging, but other hymenoptera do not. The venom from the
honeybee contains phospholipase A. Theraphosidae (tarantulas) are of dermatologic importance

424
due to the presence of urticating hairs. Cimicidae (bedbugs) cause initially asymptomatic bites
that then form linear, purpuric, pruritic macules and erythematous papules. Ctenocephalides
canis is a flea that causes frequent infestations on dogs. Solenopsis invicta is the imported fire
ant, and attacks in groups. Solenopsis inject a venom that contains a hemolytic factor, solenopsin
D.

111
This patient complains of intense itching at this site. KOH examination is negative. She denies
any recent travel, however does take her 2 year old to the local playground where there is a
sandbox. Which of the following is appropriate?
A. Obviously this is dermatophyte despite a negative KOH. Treat with topical
antifungals
B. Inform the patient she is contagious and should isolate herself from her children
C. Slowly extract the organism
D. Give her clobetasol ointment
E. Treat with po ivermectin
►E

This is hookworm, or cutaneous larva migrans; systemic ivermectin or topical thiobendazole


would be appropriate treatments. The causative organism is the larvae of Ancylostoma
braziliensis or A. canium.

112
The fire ants inject a venom that contains a hemolytic factor. It can cause a release of histamine
and other vasoactive amines from mast cells. The hemolytic factor is:
A. solenopsin D
B. phospholipase A
C. Cimicidae
D. Endotoxin A
E. Exotoxin B
►A

The fire ants inject a venom that contains a hemolytic factor, solenopsin D a piperidine
derivative that causes a release of histamine and other vasoactive amines from mast cells. The bit
becomes a sterile pustule.

113
This is a:
A. Moth caterpillar
B. Puss caterpillar

425
C. Woolybear caterpillar
D. Brownbear caterpillar
E. Furrybear caterpillar
►C

This is a woolybear caterpillar.

114
Which of the following plants are members of the family Rutaceae?
A. Burning bush
B. Bergamot orange
C. Rue
D. All of these answers are correct
E. None of these answers are correct
►D

The family Rutaceae includes lime, rue, burning bush, bergamot orange, and Hawaiian lei
flowers. Plants from this family commonly cause phytophotodermatitis.

115
A 52 year-old gardener presents with innumerable linear hyperpigmented streaks across the chest
and arms. He states that several weeks previously he had a painful, erythematous, blistering
eruption at the same site. The most likely botanical cause for this eruption is:
A. Hogweed
B. Peruvian lily
C. Plants of the Alliaceae family
D. Ragweed
E. Lime
►A

The gardener has so-called "strimmer dermatitis." The use of a weed-whacker may result in a
spray of weeds (such as cow parsley, wild chervil, and hogweed) leading to this
phytophotodermatitis on exposed areas such as the chest and arms.

116
A meat handler has extensive verrucous papules that are cauliflower like lesions on the dorsal,
palmar and periungual aspect of the hand. The HPV subtype that causes butcher's wart is:
A. HPV 2, 7
B. HPV 3, 5
C. HPV 7, 9

426
D. HPV 1, 4
E. HPV 11, 13
►A

Butcher's wart is caused by HPV 2, 7. It is from meat and poultry handlers. It is not caused by
the animal papillomavirus type.

117
If suspicious for scabies infestation, which of the following areas is least likely to be involved?
A. Fingerwebs
B. Nipples
C. External genitalia
D. Occipital scalp
E. Umbilicus
►D

Erythematous papules and burrows are commonly seen in webspaces, nipples, external genitalia,
umbilicus and axilla in sarcoptes scabiei infestation.

118
A goat farmer presents with fever and flu-like symptoms, as well as violaceous papulonodules on
the trunk and lower extremities. You consider Malta fever in your differential diagnosis. Which
of the following statements regarding this diagnosis is correct?
A. Treatment is with doxycycline
B. Treatment is with rifampin
C. Skin lesions are infrequently seen
D. All of these answers are correct
E. None of these answers are correct
►D

Malta fever is another name for Brucellosis. This infection is caused by ingesting raw goat milk
and unpasteurized goat cheese infected with Brucellae (gram-negative rods). It is infrequent in
the US, occurring mainly in veterinarians and farmers. Acute brucellosis has very non-specific
features and presents as a flu-like illness. Skin lesions are infrequent (20% of cases) but are
usually violaceous papulonodules on the trunk and lower extremities. Treatment is with
doxycycline or rifampin.

119
This tick transmits:
A. Lyme

427
B. Relapsing fever
C. Rocky mountain spotted fever
D. Erlichiosis
E. Babesiosis
►C

This is Dermacentor variabilis which transmits Rocky Mountain Spotted Fever as well as
Tularemia.

120
Caterpillars can cause a variety of reactions with human contact. The eye reaction is a foreign
body granulomatous reaction to caterpillar septae or tarantula hairs:
A. Opthalmia nodosa
B. Red scotomas
C. Granulomatous nodosa
D. Koplicks spots
E. Angiod streaks
►A

Opthalmia nodosa is a foreign body granulomatous reaction to caterpillar septae or tarantula


hairs. It can cause an inflammatory response in the eye.

121
Which of the following plants contains thiocyanates that can cause an irritant dermatitis?
A. Garlic
B. Mustard
C. Radish
D. Plants of the Brassicaceae family
E. All of these answers are correct
►E

Thiocyanates are found in garlic (Alliaceae family), mustard, and radish (Brassicaceae family).

122
Which family of plants is most commonly implicated in phytophotodermatitis caused by
furocoumarins?
A. Apiaceae
B. Rutaceae
C. Moraceae
D. Papilonaceae

428
E. Compositae
►A

The Apiaceae (formerly umbelliferae) family is the most common cause of


phytophotodermatitis. The members of this family include cow parsley, celery, wild parsnip,
false bishop's weed, giant hogweed, angelica, meadow grass, fennel, wild carrot, caraway, and
coriander.

123
All of the following plants commonly cause a phytophotodermatitis except:
A. Hawaiian lei flowers
B. Ficus carica
C. Celery
D. Myroxylon balsamum
E. All of these plants commonly cause a phytophotodermatitis
►D

Myroxylon balsamum is the source of Balsam of Peru, a common sensitizer of allergic contact
dermatitis. Hawaiian lei flowers, Ficus carica (fig tree), and celery all are known to commonly
cause phytophotodermatitis.

124
Which of the following infections is caused by a gram-negative rod?
A. Malta fever
B. Farcy
C. Bacillary angiomatosis
D. All of these answers are correct
E. None of these answers are correct
►D

Malta fever is another name for Brucellosis, an infection caused by Brucellae (gram-negative
rod). Farcy is another name for Glanders, an infection with the gram-negative rod Pseudomonas
mallei. Bacillary angiomatosis is caused by infection with Bartonella henselae, a gram-negative
rod.

125
A patient develops a chronic granulomatous reaction of the eye after a spider encounter near the
cornea. The patient was most likely injured by which spider?
A. Theraphosidae
B. Lactrodectus mactans

429
C. Loxosceles reclusa
D. Tegenaria agrestis
E. Lycosidae
►A

Theaphosidae, or the tarantulas, is a hairy spider that has urticating hairs and can cause
ophthalmia nodosa and blindness if it injures the cornea. Lactrodectus mactans, or the black
widow spider, possesses alpha lactrotoxin that leads to symptoms of systemic toxicity,
abdominal pain, paralysis, vomiting, etc. Loxosceles reclusa, also known as the brown recluse
spider, contains sphingomelinase which leads to extensive gangrene and necrosis. Tegenaria
agrestis is the hobo spider which is the leading cause of necrotic arachnidism in the Pacific
Northwest. The Lycosidea spiders, or wolf spiders, cause painful bites that may lead to
lymphangitis or an eschar.

126
This caterpillar can produce an anticoagulant venom that can cause internal hemorrhaging, renal
failure and hemolysis:
A. Lonomia caterpillars
B. Puss caterpillar
C. Megalopye opercularis
D. Geometrid caterpillar
E. Black swallowtails
►A

The Lonomia caterpillars produce an anticoagulant venom, which can cause internal
hemorrhaging, renal failure and hemolysis. The puss caterpillar has spines that can cause
hemorrhagic papules in a grid like pattern.

127
Cantharidin creates a vesicle by which of the following mechanisms?
A. Disruption of keratinocyte-keratinocyte adhesion
B. Instability of the hemidesmosome
C. Subcorneal layer separation
D. Antibodies to desmosomal proteins
E. Local recruitment of neutrophils
►A

Desmosomal adhesions are interrupted by proteases with eventual tonofilament detachment.

128

430
The reaction caused by this plant is mediated by:
A. Langerhans cells
B. Eosinophils
C. IgE
D. Mast cells
E. Histamine
►A

This is ragweed, which causes allergic contact dermatitis. Mast cells, IgE and histamine are
characteristic of contact urticaria. Allergic contact dermatitis is caused by langerhans cells
presenting antigen to CD4+ lymphocytes leading to a cell-mediated delayed hypersentitivity
reaction.

129
The skin lesions with cutaneous larva migrans are typically seen on which body location?
A. Face
B. Scalp
C. Distal lower extremity
D. Perianal area
E. Trunk
►C

Cutaneous larva migrans is usually acquired on sandy beaches. The hookworm can penetrate the
human skin, usually through the foot. A serpiginous inflammatory reaction can be seen
highlighting the burrow of the larvae in the skin. This is a self-limited reaction, however pruritus
can be significant, therefore oral treatment with thiabendazole or ivermectin is often given.
Topical 10% thiabendazole also is effective.

130
Thorns and barbs on blackberry bushes have been known to cause secondary infection through
inoculation of:
A. Mycobacterium kansaii
B. Sporothrix schenckii
C. Mycobacterium ulcerans
D. All of these answers are correct
E. None of these answers are correct
►A

Thorns and barbs on blackberry bushes have been known to cause secondary infection through
inoculation of Mycobacterium kansaii. Sporothrix schenckii can be innoculated by grasses,

431
sphagnum moss, and rose thorns. Mycobacterium ulcerans has been innoculated by spiky
tropical vegetation.

131
A patient works selling gourmet cheeses and complains of an urticarial and vesicopapular
eruption. Which of the following organisms is most implicated in his condition?
A. Acarus
B. Glyciphagus
C. Pyemotes
D. Dermanyssus
E. Ornithonyssus
►B

Food and grain mites can cause papular urticaria or vesicopapular eruptions. Glyciphagus,
Acarus, and Pyemotes are the cheese, grain, and straw itch mites respectively. Dermanyssus and
Ornithonyssus are fowl mites which most often affect office workers, homemakers, and bird
fanciers.

132
Which of the following mites is a cause of "walking dandruff" in dogs and cats?
A. All of these answers are correct
B. None of these answers are correct
C. Cheyletiella
D. Acarus
E. Allodermanyssus sanguineus
►C

Cheyletiella mites are harbored by dogs and cats and cause walking dandruff. The pet is
asymptomatic, but people handling the pet may experience pruritus when mites feed on skin.
Diagnosis is by microscopic examination of cellophane tape applied to the pet's skin. Acarus is
the grain mite, and causes baker's itch. Allodermanyssus sanguineus is the house mouse mite,
and the vector of rickettsial pox.

133
This creature can do the following EXCEPT:
A. Be used as a natural control for the tick population
B. Cause anaphylaxis
C. Transmit Yersinia pestis
D. Cause a painful sting
E. Build large characteristic mounds

432
►C

This is a fire ant. In some parts of the world, they are used as a natural tick control as they are
scavengers and consumers of tick eggs. They can cause anaphylaxis in an allergic person. They
do not transmit disease. The mounds are characteristic in appearance.

134
A veterinarian presents with an edematous pustule on the right arm, with nodules along the
lymphatic drainage route. You consider a diagnosis of glanders. What treatment do you
recommend?
A. Reassurance. No treatment is necessary, as infection is self-limited and should
resolve within 6 weeks
B. Antibiotic treatment with streptomycin combined with tetracycline
C. Surgical excision
D. Surgical excision followed by antibiotic treatment with streptomycin combined
with tetracycline
E. Surgical excision followed by antibiotic treatment with rifampin and ethambutol
►D

Farcy, or glanders, is caused by infection with the gram-negative rod Burkholderia mallei. It
most frequently infects horses, donkeys, and mules; humans can occasionally also be infected.
An edematous nodule, pustule, or vesicle develops at the inoculation site. Nodules are often
present along the lymphatic drainage route and are called "farcy buds." Importantly, nasal
ulceration and septum perforation can be caused by glanders. A chronic form leads to deep,
painful abscesses. Treatment is with surgical excision of the lesion followed by streptomycin
combined with tetracycline. Additional antibiotics that can be used include: Imipenem,
ciprofloxacin, novobiocin, gentamicin, ceftrazidime, sulfonamides.

433
Chapter -15-
Dermatologic and Cosmetic Surgery
1
Which of the following is a potential side effect of ambulatory tumescent liposuction?
A. Muscle atrophy
B. Breast enlargement
C. Decreased appetite
D. Abdominal perforation
E. Pulmonary embolus
►B

Breast enlargement is a relatively common and unexpected side effect of tumescent liposuction.
Since the majority of these patients report increased breast size in the absence of weight gain,
some authors postulate shifting hormone ratios as the etiology of this paradoxical breast
augmentation. Abdominal perforation, respiratory failure and pulmonary embolus are
complications that are seen almost exclusively in liposuction patients that receive general
anesthesia or intravenous sedation.

2
Which is an advantage of a split-thickness skin graft over a full-thickness skin graft?
A. Better color match
B. Better texture match
C. Less contraction
D. Decreased infection risk
E. Improved survival
►E

The major advantage of a split-thickness skin graft over a full-thickness graft is the increased
survivability. Full-thickness skin grafts have better color and texture match and less contraction.
There is no significant difference in infection risk.

3
Which of the following sutures is the first to be absorbed?
A. Catgut
B. Polyglycolic acid
C. Polyglactin 910
D. Polydioxanone
E. Polypropylene
►A

434
Catgut has a variable rate of absorption but typically lasts about 7-14 days. Thus, it would be the
first suture to be absorbed. Polyglactin 910 (Vicryl) and polyglycolic acid (Dexon) are both
absorbed in about 90 days. Polydioxanone (PDS) is an absorbable monofilament which lasts
approximately 180 days. Polypropylene (Prolene) is not an absorbable suture.

4
Which of the following injectable local anesthetics is the longest lasting?
A. Lidocaine
B. Bupivacaine
C. Meppivacaine
D. Procaine
E. Diphenhydramine
►B

Bupivacaine is the longest lasting injectable local anesthetic-120-240 minutes without


epinephrine, 180-420 minutes with epinephrine. Lidocaine lasts 30-60 minutes without
epinephrine, 120-360 with epinephrine. Mepivacaine last 45-90 minutes without epinephrine,
120-360 with epinephrine. Procaine lasts 15-60 minutes. Diphenhydramine lasts 30 minutes.

5
Which of the following demonstrates the highest risk of metastasis?
A. SCC on the ear
B. SCC within a scar
C. SCC on the lip
D. SCC on the scalp
E. SCC on the nose
►B

Location of tumor is an important risk factor for metastasis of squamous cell carcinoma.
Compared with a 10% likelihood of metastasis for tumors located on the ear or the lip, an SCC
developing in the scar, however, has been estimated to metastasize at a rate as high as 30-40%. .

6
The post-operative complication shown in the photograph is most commonly seen with which
type of reconstruction?
A. Rhombic flap
B. Nasolabial flap
C. Rotation flap
D. Full-thickness skin graft

435
E. Secondary intention
►B

Trap door deformity is believed to result from insufficient undermining. This surgical
complication is most often associated with the nasolabial transposition flap. Intralesional
corticosteroids may be beneficial in improving the cosmetic outcome.

7
Which is the following is true regarding cellulite?
A. It is caused by lipoatrophy of fat.
B. Adipocytes in the gluteofemoral region are more responsive to lipolysis.
C. Norepinephrine is the only hormone that affects lipolysis.
D. It occurs in up to 50% of postpubertal females.
E. Caucasian women tend to get cellulite more than Asian women.
►E

According to the JAAD CME article in March 2010, cellulite is caused by the herniation of
subcutaneous fat, not lipoatrophy, within fibrous connective tissue. It is prevalent in almost all
postpubertal females. It is most notable in the pelvic region, lower limbs, and abdomen. The
adipocytes in the gluteofemoral region are larger and are influenced by female sex hormones.
They are metabolically more stable and resistant to lipolysis. The hormones that acutely affect
lipolysis in adipocytes are catecholamines (epinephrine and norepinephrine) and insulin.

8
Which of the following lasers has the greatest depth of penetration in the skin?
A. Pulsed dye laser (585 nm)
B. Diode (800 nm)
C. Nd: YAG (1064 nm)
D. Erbium: YAG (2940 nm)
E. CO2 (10,600 nm)
►C

The Nd: YAG laser emits energy at 1064nm near the infrared range and penetrates the skin to the
level of the deep dermal blood vessels. The target chromophore for this laser is melanin, thus its
primary use is in treating lesions such as nevus of Ota and removing black tattoo pigment.
Although both the erbium:YAG and CO2 lasers have longer wavelengths than the Nd: YAG,
they are ablative lasers which only penetrate to the level of the stratum corneum and superficial
epidermis, respectively.

436
The maximum dose of lidocaine with epinephrine when used in tumescent anesthesia is:
A. 3 mg/kg
B. 4.5 mg/kg
C. 7 mg/kg
D. 20 mg/kg
E. 50 mg/kg
►E

The maximum recommended dosage of lidocaine in adults is 4.5 mg/kg without epinephrine, 7.0
mg/kg with epinephrine, and 55 mg/kg in tumescent anesthesia for liposuction.

10
Which of the following chemical peels does not need to be neutralized?
A. Salicylic acid
B. TCA 40%
C. Glycolic acid
D. Lactic acid
E. Phenol
►A

Salicylic acid is a very superficial chemical peel that is used for acne, milia, keratolysis. This
peel localizes to the pores given its lipophilic nature. It is a self-neutralizing peel.

11
The use of imiquimod (Aldara) for the treatment of superficial basal cell carcinoma is advocated
at which treatment regimen?
A. Five times a week for 6 weeks
B. Five times per week for 4 weeks
C. Three times a week for 6 weeks
D. Three times per week for 4 weeks
E. Three times a week for 10 weeks
►A

Imiquimod (Aldara) was FDA-approved in 2004 for the treatment of superficial basal cell
carcinoma. The recommended treatment schedule is once daily, five times per week for a total of
six weeks. Geisse J et al. Imiquimod 5% cream for the treatment of superficial basal cell
carcinoma: results from two phase III, randomized, vehicle-controlled studies.

12
Which of the following topical antibacterial agents may cause neutropenia?

437
A. Mupirocin
B. Silver sulfadiazine
C. Polymyxin
D. Bacitracin
E. Povodine-iodine
►B

Silver sulfadiazine is most commonly used to prevent infection in second and third-degree burn
patients. Rare cases of leukopenia, neutropenia and kernicterus have been reported in patients
using this topical antibacterial agent. Consequently, silver sulfadiazine should be used cautiously
and avoided in infants, nursing mothers and pregnant women. Silver sulfadiazine also has the
potential to cause a hypersensitivity reaction in patients with sulfa allergy.

13
The surgical instrument shown in the photograph is a:
A. Needle holder
B. Hemostat
C. Gradle scissor
D. Towel clamp
E. Iris scissor
►B

A hemostat is shown in the photograph. This instrument is commonly used in cutaneous surgery
for clamping off blood vessels.

14
Which one of the following best estimates the percent strength of a wound one month after
surgery with a primary closure?
A. 30%
B. 50%
C. 70%
D. 80%
E. 90%
►B

One month after surgery with a primary closure a wound is approximately at 50% of its ultimate
strength (this is 35% of the original strength of the skin). Two weeks after surgery it is at 5%
strength. Three weeks after surgery it is at 20%. It never reaches 100% of its original strength, its
maximum strength is 90%.

438
15
The most appropriate laser to treat a port wine stain on an infant‟s cheek would have which of
the following wavelengths?
A. 488 nm
B. 532 nm
C. 585 nm
D. 694 nm
E. 810 nm
►C

The 585nm pulsed dye laser (PDL) targets intravascular oxyhemoglobin and is considered the
treatment of choice for most benign vascular lesions. The original PDL had a wavelength of 577
nm which was later modified to 585 nm to achieve deeper penetration yet still maintainvascular
specificity.

16
A patient with a basal cell carcinoma on the lower eyelid has Mohs surgery. Once tumor-free
margins were obtained, the surgeon repairs the wound with a full thickness skin graft. Which of
the following statements is true about this repair method?
A. Graft should be at least 25% larger than the defect
B. Graft rarely survives in this location
C. Graft should be the same size as the defect
D. Healing by secondary intention would minimize the risk of ectropion
E. Graft should be at least 25% smaller than the defect
►A

To account for wound contraction and minimize the risk of ectropion, full thickness skin grafts
on the lower eyelid should be sized such that the graft is at least 25% larger than the actual size
of the wound. Jewett BS and Shockley WW. Reconstructive options for periocular defects.
Otolaryngol Clin North Am 2001; 34(3):601-625.

17
The anatomic structure identified in the photograph is called the:
A. Triangular fossa
B. Concha
C. Antihelix
D. Scaphoid fossa
E. Tragus
►A

439
The triangular fossa of the ear is shown in the photograph. This anatomic structure is bordered
by the crura of the antihelix.

18
Jessner‟s solution contains all of the following except:
A. Salicylic acid
B. Resorcinol
C. Glycolic acid
D. Ethanol
E. Lactic acid
►C

Jessner‟s solution is a combination of resorcinol, salicylic acid, and lactic acid in ethanol that is
used as a superficial peeling agent. The advantage of Jessner‟s solution is that timing is
unnecessary and neutralization is not performed. The lactic acid is an alpha hydroxy acid.

19
All of the following statements are true regarding imiquimod (Aldara) except:
A. It is FDA-approved for the treatment of actinic keratoses
B. It is FDA-approved for the treatment of a 1.2 cm superficial basal cell on the
scalp
C. It is FDA-approved for the treatment of a 1.9 cm superficial basal cell on the
chest
D. It is FDA-approved for the treatment of a 1.5 cm superficial basal cell on the leg
E. It is FDA-approved for the treatment of a 2.0 cm superficial basal cell on the
back
►B

Imiquimod (Aldara) is FDA-approved for the treatment of actinic keratoses and the treatment of
superficial basal carcinoma. With respect to basal cell carcinoma, treatment is indicated for
primary tumors that are 2.0 cm or less, and that are located on areas of the body excluding the
face, scalp and anogenital region.

20
A physician using a carbon dioxide laser to treat verruca should be aware that all of the
following complications can occur except:
A. Ocular damage
B. Transmission of viral disease
C. Purpura
D. Recurrence of lesion

440
E. Scarring
►C

The carbon dioxide laser uses a 10,600 nm wavelength to target water as a chromophore. Lasers
that target water may damage the cornea. The carbon dioxide laser can be used to destroy
epidermal and dermal lesions such as warts. HPV virus has been recovered in the laser plume
after treatment with carbon dioxide laser. Bovine papilloma virus recovered in the plume was
even found to transmit disease to calf skin. Scarring and recurrence have also been found to be
potential complications.

21
Which anatomic structure is likely to be severed during this repair?
A. Mental nerve
B. Angular artery
C. Buccal nerve
D. Labial artery
E. Marginal mandibular nerve
►D

The photograph demonstrates a wedge resection which is used to repair defects of the lower lip.
During this repair, the labial artery will be severed since it courses superficially between the
mucosa and the underlying muscle.

22
The anatomic structure identified in the photograph is called the:
A. Soft triangle
B. Columella
C. Cupid‟s bow
D. Philtral ridge
E. Vermillion border
►D

The philtral ridges are located at the upper middle lip and form part of the aesthetically important
cupid‟s bow.

23
Which topical antiseptic works via denaturation of proteins?
A. Alcohols
B. Chlorhexidine
C. Iodine

441
D. Hexachlorophene
E. Triclosan
►A

Alcohols exert their antimicrobials effect via denaturation of proteins ie DNA, RNA, lipids, etc.
Chlorhexidine, hexachlorophene, and triclosan disrupt the microbial cell membrane. Iodine and
iodophores work via oxidation/substitution for elemental form of iodine, thereby precipitating
proteins.

24
While contemplating the repair of a large cheek defect after Mohs surgery, you begin to
anesthetize an elderly lady who weighs 110 lbs. What is the maximum amount of 1% lidocaine
with 2.5% epinephrine that this patient can receive?
A. 10 ml
B. 35 ml
C. 50 ml
D. 350 ml
E. 500 ml
►B

The maximum recommended dosage of 1% lidocaine with 2.5% epinephrine in adults is 7mg/kg.
The patient weighs 110 lbs (or 50 kg) which allows her 350 mg or 35 ml (1% lidocaine has 10mg
of lidocaine perml) of the anesthetic.

25
Reticulate eythema is a side effect seen with which treatment?
A. Mesotherapy
B. Sclerotherapy
C. Diode laser
D. Cryotherapy
E. Pulsed dye laser
►C

Reticulate erythema is a recently-reported side effect of diode laser treatment. High energy
fluences and a history of chilblains are believed to be predisposing risk factors. Laser treatment
should be discontinued at the first sign of this complication.

26
The nasolabial two stage flap uses depends on which artery for flap survival:
A. Supratrochlear artery

442
B. Superior labial artery
C. Angular artery
D. Lateral Nasal artery
E. Inferior labial artery
►C

Angular artery provides blood supply for the nasolabial transposition flap.

27
The photograph demonstrates which of the following?
A. Split-thickness skin graft
B. Mohs layer
C. Full-thickness skin graft
D. Dog ear
E. Composite graft
►C

A full-thickness skin graft (FTSG) is composed of the entire epidermis and dermis.
Subcutaneous tissue must be removed since the fat may compromise the viability of the graft.

28
In a patient who is allergic to paraphenylinediamine, which of the following anesthetics should
be avoided?
A. Bupivicaine
B. Etidocaine
C. Mepivicaine
D. Prilocaine
E. Benzocaine
►E

Paraphenyelenediamine is a common allergen found in permanent hair dyes. In individuals who


are allergic to paraphenylendiamine, may also have allergic reactions to ester anesthetics, like
benzocaine. The other answer choices are amide anesthetics. Other crossreactants to
paraphenyelediamine include the preservative parabens, sulfonamides, and thiazide diuretics.

29
Aging skin demonstrates all of the following characteristic changes except:
A. Loss of elasticity
B. Hypertrophy of sebaceous glands
C. Decreased number of Langerhans cells

443
D. Increase in subcutaneous fat
E. Decreased dermal collagen
►D

The loss and redistribution of subcutaneous fat is a characteristic finding of the aging face. The
forehead, temporal fossae, malar cheeks and perioral region are the most commonly affected
areas. Knowledge of senescent changes in fat distribution has altered the cosmetic surgeon‟s
approach of the aging face to one which includes augmentation rather than one of simply lifting
and tightening the skin.

30
Which cosmetic injectable provides dermal augmentation through the harvesting of a patient”s
fibroblasts?
A. Restylane
B. Isolagen
C. Hylaform
D. Juvaderm
E. Dermalogen
►B

Isolagen is an emerging technology whereby a patient”s own fibroblasts are isolated from a skin
biopsy, reproduced and then re-introduced into the patient”s treatment site.

31
Which of the following sterilization methods has the negative side effect of dulling instruments?
A. chemical autoclave
B. steam autoclave
C. dry heat
D. cold sterilization
E. gas sterilization
►B

Steam autoclave is the most common type of sterilization used in the office setting. It is not to be
used with heat-sensitive plastics. A negative side effect of using steam autoclave is that it can
dull sharp instruments.

32
The classic purpose for serrated scissors is:
A. For work on thicker tissues (back)
B. For work on delicate areas around the eyes and the ears

444
C. For gripping tissue while cutting
D. For tissue undermining
E. For sharp dissections of tissue
►C

Serrated scissors are used primarily for gripping tissue to prevent sliding while cutting.

33
Which of the following is NOT a form of electrosurgery?
A. electrocautery
B. electrodesiccation
C. electrofulguration
D. electrocoagulation
E. electrosection
►A

Electrosurgery is a broad term to describe the use of electricity to produce thermal tissue
damage. Electrocautery is a method of direct heat transfer to tissue by a heated element. No
electric current is transferred.

34
The most important function of a postoperative wound dressing is:
A. Provide a barrier to infection
B. Decrease tension on the wound
C. Create a moist environment
D. Prevent suture dehiscence
E. Immobilize the wound
►E

Providing hemostasis to a wound is the most important function of a wound dressing. This is
particularly critical in the first few hours postoperatively when the vasoconstrictive effects of
epinephrine are wearing off and the wound is at risk of bleeding or hematoma formation. The
provision of a barrier to infection and creation of a moist, occlusive environment are also
necessary functions of a dressing but are not as critical as immobilizing the wound and ensuring
hemostasis.

35
What surgical dressing would you use if you wished to debride a wound, reduce pain, and
provide a cooling effect?
A. Alginates

445
B. Foams
C. Hydrocolloid
D. Hydrogel
E. Films
►D

Hydrogels are 70-90% water, decrease pain, provide a cooling effect, and are useful for dry and
necrotic wounds. Foams are suitable for use on light-to-medium exuding wounds. Hydrocolloids
ensure the moist wound environment, promote the formation of granulation tissue, and provide
pain relief by covering nerve endings with both gel and exudate. Large quantities of alginates are
used each year to treat exudating wounds, such as leg ulcers, pressure sores, and infected surgical
wounds.

36
Which laser would be the best choice for the treatment of a red cosmetic tattoo around the lips?
A. Carbon dioxide
B. Pulsed dye
C. Q-switched ruby
D. Alexandrite
E. Nd:YAG
►C

Q-switched lasers can be effectively used to remove tattoos. However, immediate darkening of
white, flesh colored, and pink/red tattoos has been reported after treatment with Q-switched
lasers. Both ferric oxide and titanium dioxide (found in tattoo pigments) may be both be reduced
after heating by the laser and produce dramatic darkening.

37
Blue nodules in the skin are a potential complication of which cosmetic injectable?
A. Radiesse (hydroxyapatite)
B. Silicone
C. Restylane (hyaluronic acid)
D. Cosmoderm (collagen)
E. Sculptra (poly L-lactic acid)
►C

Although hyaluronic acid is believed to be inert in humans, rare reports exist of granulomatous
foreign body reactions to this material. Skin biopsy of a Restylane nodule demonstrates
multinucleated giant cells surrounding a blue amorphous material.

446
38
A patient has a 2.0 cm surgical defect on the left lower eyelid after having Mohs surgery. Which
management option would be the least appropriate?
A. Primary closure
B. Secondary intention
C. Full-thickness skin graft
D. Tripier flap
E. Rhombic flap
►B

In lower eyelid reconstruction, tension should be oriented parallel to the lower eyelid to prevent
the development of ectropion. Secondary intention would be the least appropriate management
option in this location because normal wound contraction would increase the risk of ectropion.

39
The use of dermabrasion to improve the cosmesis of a scar is best performed how long after the
initial surgery?
A. 1 week
B. 3 weeks
C. 6 weeks
D. 6 months
E. 1 year
►C

Dermabrasion is the process of surgically planing or abrading the epidermis and dermis and is
usually carried out with a rapidly rotating wire brush or diamond fraise. Following dermabrasion,
reepithelialization from adnexal structures occurs with remodeling and replacement of collagen
bundles in the papillary and reticular dermis. This can result in improvement of surgical or acne
scars. When used in the post-operative period for surgical scars, dermabrasion is usually carried
out 6 to 8 weeks following the procedure. Regional dermabrasions are routinely carried out
under local anesthesia. Post-procedure complications include scarring, pigmentary alterations,
persistent erythema, and infection. Contraindications for dermabrasion include recent use of
isotretinoin and recent facelift, browlift or other procedure involving extensive undermining.
Reactivation of herpes labialis can be prevented in the lip area by administering antiviral
prophylaxis.

40
A patient with a deep vertical frown line in the glabellar region requests Botox. Treatment will
mostly target which one of the following muscles?
A. Oculi

447
B. Orbicularis oculi
C. Depressor superciliaris
D. Levator palpebrae superioris
E. Corrugator superciliaris
►E

Although the procerus and orbicularis oculi muscles are contributory, the development of
vertical glabellar frown lines is mostly due to the overactivity of the corrugator supercilaris.
These muscles function to pull the eyebrow inferomedially.

41
All of the following are alpha-hydroxy acids except:
A. Lactic acid
B. Citric acid
C. Glycolic acid
D. Tartaric acid
E. Salicylic acid
►E

Alpha-hydroxy acids are naturally occurring carboxylic acids found in many foods. The alpha-
hydroxy acids include glycolic, lactic, malic, citric, and tartaric acids. Factors that determine the
intensity of the peel include the concentration of the acid, pH, degree of buffering, vehicle
formulation, frequency of application, conditions of delivery, and the length of time the acid is
placed on the skin. Salicylic acid is a type of beta-hydroxy acid.

42
Which post-operative complication is shown in the photograph?
A. Trap door deformity
B. Hypertrophic scar
C. Ectropion
D. Temporal nerve paralysis
E. Eclabium
►C

Ectropion occurs when the eyelid margin turns outward away from the eye. In lower eyelid
reconstruction, tension should be oriented parallel to the lower eyelid to prevent the development
of ectropion.

43
Treatment of acne scarring would be most effective with which modality?

448
A. Pulsed dye laser
B. Intense pulsed light
C. Photodynamic therapy
D. Radiofrequency
E. Nd:YAG laser
►E

Treatment with the nonablative 1064-nm Q-switched Nd:YAG provides significant improvement
in skin topography in patients with atrophic acne scars. The further improvements that are seen at
6-month follow-up suggest that dermal remodeling is a process that continues long after
treatment.

44
The anatomic structure identified in the photograph is called the:
A. Nasal ala
B. Soft triangle
C. Columella
D. Philtral ridge
E. Nasal groove
►C

The columella is one of the cosmetic subunits of the nose. It is formed by the medial crura of the
alar cartilage.

45
A surgeon plans to repair a defect on the lower lip with a wedge resection. All of the following
statements are true regarding wedge resection of the lower lip except?
A. a V-shaped excision is often used
B. the vermillion border is marked prior to excision
C. the repair involves a two layered closure of skin and mucosa
D. up to 1/3 of the lip can be excised and closed primarily
E. the mental crease should not be crossed
►C

The lower lip has such great elasticity that full-thickness lesions that involve up to 1/3 of the lip
can be excised and repaired primarily with excellent cosmetic and functional results. This type of
repair requires a three layered closure of mucosa, skin and muscle. Larrabee W. In: Principles of
Facial Reconstruction. Lippincott-Raven: Philadelphia, 1995; 182-183.

46

449
This is used to describe a the setting for electrosurgery that is AC, bipolar, low voltage, and high
amp:
A. Electrocaogulation
B. Hyfrecation
C. Electrosection
D. Electrolysis
E. Electrocautery
►A

Electrocaogulation is set on AC, bipolar, low voltage, and high amp. All the other ones do not fit
the description.

47
When can a patient who undergoes follicular-unit hair transplantation expect hair growth of the
grafted follicles to begin?
A. 7 to 10 days
B. 1 month
C. 2 to 3 months
D. 6 months
E. 9 months
►C

The use of follicular-unit grafts, which contain one to four hair follicles, represents the
advancement in both surgical technique and aesthetic outcome in the field of hair transplantation.
The grafted hair follicles typically begin to grow within 8 to 10 weeks of implantation and are
expected to survive for the individual”s lifetime.

48
Signs and symptoms of lidocaine toxicity include all of the following except:
A. Circumoral numbness
B. Ototoxicity
C. Slurred speech
D. Nystagmus
E. Seizure
►B

The first signs of lidocaine toxicity are CNS symtpoms that resemble inebriation with alcohol.
These symptmoms include stupor, dysarthria, circumoral numbenss and dizziness. Further
increases in toxicity leads to nausea, metallic taste, twitching, and seizures. Ototoxicity is not one
of the toxic manifestations seen with lidocaine toxicity. Without epinephrine the maximum dose

450
of lidocaine 4mg/kg. For a 70 kg individual, this is 300 mg or 30 ml of a 1% lidocaine solution.
For a preparation of lidocaine with epinephrine the maximum dose is 7mg/kg. For a 70 kg
individual, this is 500 mg or 50 ml of a 1% solution.

49
To avoid injury to the facial nerve, undermining in the region of the temple should be performed
at which anatomic level?
A. Dermal-epidermal junction
B. Reticular dermis
C. Subcutaneous fat
D. Above superficial fascia (SMAS)
E. Above the frontalis muscle
►C

In the region of the temple, lateral to the eyebrow, the temporal branch of the facial nerve lies in
the superficial fascia. Once above the eyebrow, the nerve is located deep to the frontalis muscle.
In the region of the temple, it is therefore prudent to undermine in the superficial subcutaneous
fat.

50
Paradoxical darkening has been associated with Q-switched ruby, Q-switched alexandrite, Q-
switched Nd:Yag treatment of which tattoo color?
A. Green
B. Red
C. Purple
D. Black
E. Blue
►B

Paradoxic darkening of flesh-tone, red, and white tattoo inks with QSRL, Q-switched Nd:YAG,
and Q-switched alexandrite lasers has been reported. The tattoo pigments most associated with
this phenomenon are iron oxide and titanium dioxide. Ferric sulfide is converted to ferrous
sulfate by the laser.

51
Which type of flap moves almost entirely in one direction in a linear fashion
A. Rhombic flap
B. Advancement flap
C. Rotation flap
D. Transposition flap

451
E. Bilobed flap
►B

An advancement flap is one that moves almost entirely in one direction or in a linear fashion.
Rotation flap alters the direction of major closure tension forces. Transposition flap redirects and
redistributes tension. Rhombic and bilobed flaps are both examples of transposition flaps.

52
Jessner's solution includes:
A. Lactic acid, alcohol, glycolic acid, resorcinol
B. Lactic acid, alcohol, glycolic acid, TCA
C. Lactic acid, alcohol, salicylic acid, glycolic acid
D. Lactic acid, resorcinol, salicylic acid, glycolic acid
E. Lactic acid, alcohol, salicylic acid, resorcinol
►E

Jessner's is a superficial peeling agent, which may be used to treat dyschromia and texture
abnormalities. TCA and glycolic acid are not ingredients in this formulation.

53
Which potential complications is seen exclusively with phenol peels?
A. Milia
B. Hypopigmentation
C. Contact dermatitis
D. Prolonged erythema
E. Laryngeal edema
►E

The principle concept of chemical peeling is to utilize a chemical agent to produce a controlled
wound in the skin. Many different types of side effects can occur with chemical resurfacing and
are typically related to the depth of the wound. Complications which are common to all peeling
agents include: milia; acne; pigmentary changes; contact dermatitis; scarring; infection;
prolonged erythema; textural changes and cold sensitivity. Unlike other peeling agents, phenol
has the potential to cause severe adverse reactions which are exclusive to its use only. They
include atrophy, cardiac arrhythmias and laryngeal edema.

54
The antiptosis subdermal suspension threads used in facial rejuvenation are composed of:
A. Poly-L-lactic acid
B. Polyglactin

452
C. Polypropylene
D. Polydioxanone
E. Polyglycolic acid
►C

The Aptos subdermal suspension thread technique is a minimally invasive procedure which
targets the ptotic changes seen with facial aging. 2-0 and 3-0 polypropylene threads are tunneled
in the dermis to lift and suspend the skin and subcutaneous tissue.

55
Alopecia neoplastica is most commonly associated with which malignancy?
A. Lymphoma
B. Squamous cell carcinoma
C. Breast carcinoma
D. Colon carcinoma
E. Melanoma
►C

While various malignancies have a predilection for metastasis to the scalp where they can cause
hair loss, alopecia neoplastica is most commonly associated with metastatic breast carcinoma.

56
In organ transplant recipients, all of the following factors increase susceptibility to the
development of skin cancer except:
A. Fair skin (Fitzpatrick types I-III)
B. History of chronic sun exposure
C. Duration of immunosuppression
D. History of HPV infection
E. CD8 lymphocytopenia
►E

All organ transplant recipients are at increased risk for the development of cutaneous
malignancies. However, the above-mentioned factors, with the exception of choice E, place these
individuals at further risk. CD4 lymphocytopenia, rather than CD8, is another identified risk
factor as is older age, history of actinic keratoses and history of skin cancer.

57
A Z-plasty is performed to:
A. Decrease tension on a wound
B. Obtain wound eversion

453
C. Shorten the length of an excision
D. Correct dog-ears
E. Make use of excess tissue
►A

A Z-plasty is a transposition flap used most commonly in the treatment of contractures and scars.
The main advantages to performing a Z-plasty are to decrease the tension on a wound, change
the orientation of a scar, and camouflage a scar by breaking it up into smaller components.

58
Which of the following sutures is best used for mucous membranes?
A. Catgut
B. Silk
C. Polyglactin 9, 10
D. Nylon
E. Polydioxanone
►B

Silk is a natural nonabsorbable suture used mainly for eyelid, lip, or intraoral surgeries as they
are more comfortable to the patient on these surfaces. This suture has excellent handling as it is
less stiff and flows easily through soft tissue. However, it has high tissue reactivity, low tensile
strength, and is associated with a higher risk of infection.

59
On average, how much does a full-thickness graft contract when removed from a donor site?
A. 15%
B. 30%
C. 50%
D. 60%
E. 80%
►A

On the average, a full-thickness graft may contract by about 15% when removed from a donor
site.

60
Pre-testing for allergy to collagen is required for which cosmetic filler?
A. Radiance
B. Artecoll
C. Perlane

454
D. Poly L lactic acid
E. Silicone
►B

Artecoll is a nonbiodegradable injectable filler composed of microspheres of


polymethylmethacrylate which are suspended in bovine collagen. As with other types of
injectable bovine collagen, patients must be tested for allergy to bovine collagen prior to
treatment.

61
Which tattoo pigment has most commonly been associated with phototoxic reactions?
A. Titanium dioxide
B. Mercuric sulfide
C. Carbon
D. Iron oxide
E. Cadmium sulfide
►E

Cadmium sulfide produces a yellow tattoo. Phototoxic reactions are most commonly caused by
cadmium sulfide tattoo pigment. Red tattoos have also been associated with phototoxic tattoo
reactions. However, this is thought thought to be due to the addition of cadmium sulfide which
enhances and brightens the red color.

62
Which of the following is true regarding a Baker-Gordon peel?
A. 88% phenol is the most important component for efficacy
B. It contains salicytic acid
C. It contains resorcinol
D. It is considered a medium depth peel
E. Croton oil is considered the most important component for efficacy
►E

Croton oil is considered the most important component for efficacy. It is a epidermolytic that
enhances penetration. A Baker Gordon peel is a deep peel that contains 88% phenol, tap water,
croton oil, and septisol.

63
All of the following statements are true regarding nerves blocks on the face except:
A. Anesthesia is placed around a nerve trunk
B. Lidocaine with epinephrine may be used

455
C. Numbness occurs in areas other than the operative site
D. Tissue distortion is minimized
E. Hemostasis is achieved using epinephrine
►E

While regional nerve blocks offer many advantages in cutaneous surgery, hemostasis is not one
of them. Since the anesthetic is injected distant to the operative site, the vasoconstrictive effects
of the epinephrine, and hence hemostasis, are not provided at the surgical site.

64
Which statement is true regarding relaxed skin tension lines (RSTLs)?
A. they occur as a result of increased elastic tone
B. they lie parallel to the underlying muscles
C. the long axis of a wound often lies in the direction of the RSTL
D. incisional scars should be placed perpendicular to RSTLs
E. smiling minimizes the appearance of RSTLs
►C

Relaxed skin tension lines are creases in the skin that are present at rest and develop as a
consequence of the decreased elasticity that occurs with aging and solar damage. These lines lie
perpendicular to the underlying musculature and can be accentuated by smiling or frowning. An
undermined circular wound will often form an oval shape due to muscular tension, and will have
its long axis oriented within these relaxed tension lines. Knowledge of RSTLs is important in
cutaneous surgery because placing incisions within these lines will ensure the most favorable
cosmetic result.

65
Which needle shape is most commonly used in cutaneous surgery?
A. 1/4 circle
B. 3/8 circle
C. 1/2 circle
D. 5/8 circle
E. compound curve
►B

Needles are either straight or curved. Curved needles have their curvature described either as a
fraction of a circle or a compound curve. The greater the fraction of a circle, the more pronation
and supination of the wrist required by the surgeon to place the needle. The 3/8 circle needle is
easy to use in large, superficial areas and is the most commonly used needle for cutaneous
surgery.

456
66
Sensory innervation of the glabella and mid-forehead is provided by which nerve?
A. infratrochlear nerve
B. supraorbital nerve
C. supratrochlear nerve
D. temporal branch of the facial nerve
E. zygomatic branch of the facial nerve
►C

The supratrochlear, supraorbital and infratrochlear nerves are branches of the ophthalmic
division of the trigeminal nerve, supplying sensory innervation to the face. The supratrochlear
nerve exits the orbit approximately one centimete lateral to the midline and traverses upward
along the medial supraorbital rim along the supratrochlear notch. It provides sensory innervation
to the medial upper eyelid and glabella, the medial anterior forehead, and the scalp. The
supraorbital nerve supplies the lateral forehead and scalp as well as the upper eyelid. The
infratrochlear nerve provides sensory innervation to the root of the nose and to portions of the
medial canthus. The facial nerve provides motor innervation to muscles of the face. The temporal
branch of the facial nerve supplies motor innervation to the upper orbicularis oculi and the
frontalis muscle, as well as the anterior and superior auricular muscles. The zygomatic branch of
the facial nerve provides innervation to the inferior portion of the orbicularis oculi, zygomaticus
minor and zygomaticus major muscles, as well as levator labii superioris. (Leffell's Manual of
Skin Surgery. 1997 p44.) (Surgical Anatomy of the Skin, Salasche et al, p 113-115.)

67
True statements regarding skin cancer in organ transplant recipients include all of the following
except:
A. 65 fold increase in development of SCC compared with general population
B. Mohs micrographic surgery indicated for in-transit metastases
C. Cutaneous malignancies develop 3-5 years after organ transplantation
D. Extent of tumor development related to degree of immunosuppression
E. Skin cancer is the most common cancer in transplant patients
►B

In-transit metastases is a common manifestation of metastatic disease.

68
Which type of collagen is the first to be deposited in a healing wound?
A. Type I collagen
B. Type II collagen

457
C. Type III collagen
D. Type IV collagen
E. Type VII collagen
►A

Coagulation/Inflammation, tissue formation, and scar/remodeling are the three phases important
in wound healing. Collagen deposition occurs during the second phase. Fibroblasts migrate into
the wound along the fibrin-fibronectin matrix deposited in the initial clot. The fibroblasts
subsequently produce type I and III collagen, elastin, and proteoglycans. Collagen type III is the
predominant collagen in early wound healing.

69
The cosmetic subunits are designated based upon their similarity in all of the following except:
A. Texture
B. Color
C. Sebaceous features
D. Hair density
E. Eccrine features
►E

The cosmetic unit are based on their similarity in texture, color, solar exposure, hair density, and
sebaceous features. They are not from eccrine features.

70
Which of the following cosmetic injectables does not illicit an inflammatory response?
A. Radiesse (hydroxyapatite)
B. Artecoll (polymethylmethacrylate)
C. Silicone
D. Zyplast (collagen)
E. Sculptra (poly L-lactic acid)
►A

Radiesse is an injectable, biodegradable filler composed of calcium hydroxyapatite


microspheres. It is a normal constituent of bone and is therefore biocompatible. Current studies
are examining the role of hydroxyapatite in augmenting the craniofacial skeleton.

71
The „snap test‟ is performed prior to surgery at which anatomic location?
A. Hand
B. Ear

458
C. Mouth
D. Eyelid
E. Neck
►D

The snap test determines the laxity of the lower eyelid. It should be performed prior to surgery,
laser resurfacing or Botox injections of the lower eyelid to assess the potential for ectropion
formation. Ratner D et al. Cutaneous laser resurfacing. J Am Acad Dermatol 1999; 41(3):365-
389.

72
Which of the following local anesthetics has the longest duration of action?
A. Lidocaine
B. Bupivacaine
C. Procaine
D. Benzocaine
E. Mepivacaine
►B

Bupivacaine is an amide type of local anesthetic. Its duration of action is approximately 3-5
hours and is the longest one listed. Following bupivacaine is mepivacaine, lidocaine, and
procaine. The duration of action is based on the amine portion of the molecule.

73
Chlorohexidine should not be used around the following area:
A. Eyes
B. Nose
C. Forehead
D. Arms
E. Feet
►A

Chlorhexidine or hibiclens can cause keratitis and allergic reaction if used directly in contact
with the eye. It can cause ocular keratitis. It can also cause tympanic membrane damage.

74
Healing by secondary intention will offer the most favorable cosmetic result at which location?
A. Nasal tip
B. Malar cheek
C. Chin

459
D. Forehead
E. Medial canthus
►E

Indications for healing of full-thickness skin wounds by secondary intention include infected
wounds, patients who refuse reconstructive surgery or who are poor surgical risks for
reconstructive surgery, and resection of tumors with high rates of recurrence. In addition, second
intention, or granulation, can be utilized when the cosmetic result is anticipated to be superior or
equal to reconstruction. Disadvantages of healing by granulation include prolonged wound
healing time and somewhat unpredictable cosmetic results. Anatomic sites lending themselves
best to second intention healing, with excellent results, are the concave areas on the face. These
include the medial canthus, the nose-cheek junction, the auricular concha, the preauricular cheek,
and the retroauricular scalp. The nasal tip, malar cheek, chin and forehead are all convex surfaces
on the face.

75
Which porphyrin is targeted in aminolevulinic acid-photodynamic therapy?
A. Protoporphyrin IX
B. Uroporphyrinogen
C. Coproporphyrin III
D. Coproporphyrinogen
E. Porphobilinogen
►A

Protoporphyrin IX is the substrate for the final rate-limiting step of heme synthesis. This
enzymatic reaction is catalyzed by ferrochelatase. The preferential uptake of aminolevulinic acid
by cancerous and pre-cancerous cells results in higher concentrations of protoporphyrin IX. This
can then be selectively targeted by photodynamic therapy.

76
Which tattoo pigment has been most commonly associated with pseudolymphomatous
reactions?
A. Mercuric sulfide
B. Cadmium sulfide
C. Phthalocyanine dyes
D. Malachite
E. Cobalt
►A

460
Some delayed type hypersensitivity reactions may lead to pseudolymphoma, most commonly
caused by red tattoo pigment (mercuric sulfide).

77
A patient undergoing Mohs surgery undergoes a rotation flap from the lateral canthus/cheek to
close an infraorbital defect. What is the name of this flap?
A. Rieger flap
B. Mustarde flap
C. O to Z
D. Bilobed
E. Abbe flap
►B

This patient had a Mustarde flap, a rotation flap from the lateral canthus or cheek that close the
lower eyelid or infraorbital defect. A Rieger flap is a dorsal nasal rotation flap with backcut at
the glabella; it is also knows as a hatchet flap or glabellar turn-down flap. O to Z is a double
rotation flap. A bilobed is a transposition flap most often used on the nose. Finally, and Abbe
flap, or a lip-switch flap, is a lower lip flat of mucosa, muscle, skin, and subucutis that is
transposed superiorly to the repair an upper lip defect based on inferior labial artery blood
supply.

78
Which of the following immunosuppressive agents may exert a protective effect against the
development of skin cancers in organ transplant patients?
A. Cyclosporine
B. Rapamycin
C. Tacrolimus
D. Corticosteroids
E. Azathioprine
►B

Rapamycin (also called sirolimus) is a macrolide antibiotic and a structural analog of FK 506. It
is a potent immunosuppressive agent which inhibits mTOR (a member of P13K family kinases).
Despite its immunosuppressive effects, preliminary data show a decreased incidence of skin
cancer in organ transplant patients treated with rapamycin and postulate that it may exert a
protective effect against cutaneous malignancies.

79
The relaxed skin tension lines overlying the scapula are:
A. Do not exist over the scapula

461
B. Oriented vertically
C. Oriented circumferentially
D. Oriented horizontally
E. Perpendicular to the underlying muscle groups
►E

Relaxed skin tension lines (RSTLs) are creases in the skin that are present at rest and develop as
a consequence of the decreased elasticity that occurs with aging and solar damage. These lines
lie perpendicular to the underlying musculature. They tend to run diagonally on the upper back
whereas on the central trunk, they are typically circumferential. Knowledge of RSTLs is
important in cutaneous surgery because placing incisions within these lines will ensure the most
favorable cosmetic result.

80
While removing sutures from an excision performed on the lateral neck, your patient reports
decreased sensation of the helix and lobule of the ipsilateral ear. Which of the following nerves
was likely injured during surgery?
A. Auriculotemporal nerve
B. Lesser occipital nerve
C. Auricular branch of vagus nerve
D. Greater auricular nerve
E. Spinal accessory nerve
►D

Sensory innervation of the ear is provided by the greater auricular nerve, the auriculotemopral
nerve, the lesser occipital nerve and the auricular branch of the vagus nerve. The greater
auricular nerve is composed of the C2 and C3 branches of the cervical plexus and lies in the
posterior triangle of the neck. It provides sensation to the helix, antihelix, antitragus, posterior
auricle and the lobule. The spinal accessory nerve is a motor nerve which innervates the
trapezius muscle. This nerve is also subject to injury in the lateral neck but would cause motor
deficits rather than sensory deficits.

81
The type of repair depicted in the photograph is a:
A. Rhombic flap
B. Nasolabial flap
C. Rotation flap
D. Full-thickness skin graft
E. Island pedicle flap
►E

462
The island pedicle flap is a type of advancement flap commonly used for medial cheek defects.
The closure can often be camouflaged in the melolabial fold.

82
Full thickness skin grafts are more likely than split thickness skin grafts to demonstrate:
A. Alopecia
B. Contraction
C. Necrosis
D. Hypohidrosis
E. Hypopigmentation
►C

Full thickness skin grafts have more adnexal structures, vessels, and structures to supply.
Therefore, they are more prone to necrosis. However, they are less prone to contraction,
alopecia, hypohidrosis, and hypopigmentation for similar structural reasons.

83
The use of topical vitamin K has been shown to:
A. Decrease the appearance of infraorbital pigmentation
B. Minimize the appearance of telangiectasia
C. Reduce the severity of laser-induced purpura
D. Increase collagen production
E. Induce keratinocyte differentiation
►C

Treatment of benign vascular lesions with the pulsed dye laser often produces significant
postoperative purpura. Topical vitamin K has been shown to decrease the severity of laser-
induced purpura although its mechanism of action has yet to be determined. No other cosmetic
effects of topical vitamin K have been proven to be statistically significant.

84
What part of the eye may be damaged by exposure to irradiation from the carbon dioxide laser?
A. Retina
B. Lens
C. Sclera
D. Cornea
E. Iris
►D

463
The carbon dioxide laser operates at a wavelength of 10,600 nm and targets water as a
chromophore. Because of the high water content of the cornea, it may be damaged by exposure
to irradiation from the carbon dioxide laser. Exposure to the erbium may cause corneal damage
as well.

85
The rhytids identified in the photograph are referred to as:
A. Marionette lines
B. Relaxed skin tension lines
C. Crow‟s feet
D. Nasolabial folds
E. Bunny lines
►E

Bunny lines are located on the dorsum of the nose and are formed by the contraction of the
nasalis muscle.

86
The Shaw scalpel is used:
A. As a form of electrocautery
B. To enhance cyst wall removal
C. For incision and drainage
D. For thicker tissue (back)
E. For delicate work around the eyes and ears
►A

The Shaw scalpel provides the ability to cut through tissue with minimal blood loss. This
instrument employs an elctronically heated metal cutting balde that heats up while it cuts through
tissue, and coagulates fine vessels when they are divided. This instrument has been used
frequently for treatment of rhinophyma and in parotid gland surgeries.

87
Cosmetic units are defined by each of the following anatomic boundaries except:
A. Cheek
B. Glabellar region
C. Nasofacial sulcus
D. Philtral ridge
E. Nasolabial folds
►A

464
The term cosmetic unit is used to define regions of the face by specific characteristics such as
color, texture, presence of hair, and prominence of sebaceous glands. In dermatologic surgery, it
is preferable to conceal incisions within the boundary lines of cosmetic units. In addition,
moving skin from one cosmetic unit to another should be avoided when possible to minimize the
appearance of apposed skin of dissimilar quality.

88
The “trapdoor effect” that can occur with a flap can be avoided by which of the following
techniques:
A. Wide undermining
B. Using additional sutures
C. Performing the flap in a two staged procedure
D. Using a flap with length to width ratio of less than 3:1
E. Avoid thinning the flap
►A

Wide undermining of the primary defect can release tension and prevent tenting or the trap door
deformity of the flap.

89
A patient has a 1.5cm surgical defect on the right nasal sidewall that reveals the lateral cartilage.
Which management option would be the least appropriate?
A. Primary closure
B. Secondary intention
C. Full-thickness skin graft
D. Cheek transposition flap
E. Forehead flap
►C

Full-thickness skin grafts (FTSGs) depend upon a viable, vascular bed at the recipient site in
order to survive. Exposed cartilage is an avascular tissue and therefore would not be able to
support the metabolic requirements of a FTSG.

90
Which of the following treatment modalities for basal cell carcinoma has the highest long term
cure rate?
A. MOHS surgery
B. Radiation
C. Cryotherapy
D. Electrodessication and curettage

465
E. Surgical excision
►A

Five year cure rates of basal cell carcinomas (BCC) treated with MOHS is about 99%. The other
options listed range from 90-92% five year cure rates.

91
Each of the following are branches of the internal carotid artery except:
A. Ophthalmic artery
B. Supratrochlear artery
C. Dorsal nasal artery
D. Angular artery
E. Supraorbital artery
►D

The angular artery is a branch of the external carotid artery. It arises from the facial artery and
courses superiorly along the nasofacial angle until it reaches the area of the medial canthal
tendon. At this location, the angular artery anastamoses with the dorsal nasal branch of the
ophthalmic artery establishing a communication between the internal and external carotid arterial
systems.

92
The following flaps are types of transposition flaps except:
A. Rhombic flap
B. Nasolabial
C. Bilobed
D. A to T
E. Z plasty
►D

The Z plasty, Bilobed, Rhombic, nasolabial flaps are all examples of transposition flaps. In each
of these, the flap is transposed over a normal piece of skin to be placed in the recipient site.
Whereas, the A to T flap, is an advancement flap, in which tissue is moved in a linear direction
to cover the defect.

93
Compared to intact skin, what is the tensile strength of a wound 6 months after surgery?
A. 30%
B. 50%
C. 70%

466
D. 90%
E. 100%
►C

Tissue remodeling allows the host to develop a scar that is approximately 70% of the original
strength of the skin.

94
A patient with end stage liver disease comes to your surgical practice. Which of the following
anesthetics should be used with this patient?
A. Benzocaine
B. Etidocaine
C. Lidocaine
D. Mepivicaine
E. Prilocaine
►A

Benzocaine, an ester, should be used as it is the only anesthetic mentioned above that is
metabolized by plasma cholineseterase and then is excreted by the kidney and not the liver.

95
Preliminary studies with deoxycholic acid demonstrate efficacy in the treatment of:
A. Superficial rhytids
B. Submental fat
C. Soft tissue augmentation
D. Postinflammatory hyperpigmentation
E. Acne scarring
►B

Deoxycholic acid is a detergent that dissolves fat. It is ATX-101.

96
Which of the following suture materials is most likely to cause infection?
A. Silk
B. Vicryl (polyglactin)
C. Prolene (polypropylene)
D. Nylon
E. PDS (polydioxanone)
►A

467
Silk is a nonabsorbable braided suture which has been shown to aid in the production of
infection. It is believed that the interstices of braided or twisted suture materials enhance the
potential for developing infection by providing privileged sites which harbor bacteria.

97
Formation of granulomas is a potential complication of treatment with:
A. Isotretinoin
B. Autologous fat
C. CO2 resurfacing
D. Silicone
E. Hydroxychloroquine
►D

Silicone is a synthetic, viscous compound which is composed of long polymers of


dimethylsiloxanes. Silikon-1000 is one silicone product which is available in the United States
and is FDA-approved for the ophthalmic treatment of complicated retinal detachment. The
number 1000 refers to the product”s viscosity which is measured in centistokes. As a reference,
water has a viscosity of 100 centistokes and mineral oil has a viscosity of about 350 centistokes.
With respect to cosmetic applications, silicone is not FDA-approved for soft tissue augmentation.
In addition to the occurrence of hypersensitivity reactions and product migration, there have been
many reports of granuloma formation after silicone injection, even many years post-treatment.

98
What nerve innervates the skin between the 1st and 2nd toes?
A. saphenous nerve
B. posterior tibial nerve
C. sural nerve
D. superficial peroneal nerve
E. deep peroneal nerve
►E

deep peroneal nerve innervates the skin between the 1st and 2nd toes. The saphenous innervates
the instep and medial ankle. The posterior tibial innervates the heel and middle sole of the foot.
The sural nerve innervates the 5th toe and lateral side of the sole. The superficial peroneal nerve
innervates the skin of the toes other than the outside of the 5th toe and in between the 1st and 2nd
toes.

99
Injection of Botox at the location identified in the photograph would denervate which muscle?
A. Orbicularis oculi

468
B. Frontalis
C. Corrugator superciliaris
D. Procerus
E. Levator palpebrae superioris
►D

The development of vertical glabellar frown lines is due to the overactivity of the orbicularis
oculi, procerus and corrugator supercilaris muscles. These muscles function to pull the eyebrow
inferomedially.

100
Which of the following complications has been reported with infraorbital injections of Botox?
A. Nystagmus
B. Blindness
C. Festooning
D. Astigmatism
E. Photophobia
►C

Festooning of the lower eyelid is a recently reported complication of infraorbital Botox


injections. The affected patient had a prior lower lid blepharoplasty which likely weakened the
orbicularis oculi muscle. The author postulates that festooning resulted from further muscle
weakness due to Botox treatment.

101
Which structual component of local anesthetics (e.g. lidocaine) is responsible for the onset of
activity?
A. Aromatic ring
B. Intermediate chain
C. Amine end
D. Length of the carbon chain
E. None of the above
►A

Local anesthetics like lidocaine contain 3 principle structural elements. The aromatic ring
determines the onset of activity, the intermediate chain defines the class (amide vs. ester), and
the amine end is responsible for the duration of action.

102
Botulinum toxin type A cleaves which protein in the presynaptic neuron?

469
A. Acetylcholine
B. Snap-25
C. Serotonin
D. Synaptobrevin (VAMP)
E. Syntaxin
►B

Botulinum toxin type A is FDA approved for the treatment of glabellar wrinkles. Botulinum
toxins act by a three step process of binding, internalization by receptor medicated endocytosis
and enzymatic activation. It has specific light chain intracellular binding sites and different sites
of action on different SNARE (synaptosomal associated protein receptor [SNAP]) proteins. The
SNARE proteins are intimately involved in releasing acetylcholine at presynaptic terminals.
Botulinum toxin A cleaves SNAP-25 whereas B cleaves synaptobrevin.

103
Which of the following criteria carries the worst prognosis for a patient with a squamous cell
carcinoma?
A. Size of tumor > 1 cm
B. Depth of invasion > 4 mm
C. Perineural invasion
D. Anatomic location
E. Immunosuppression
►B

Many factors contribute to the aggressive behavior of cutaneous squamous cell carcinoma. For
example, tumors in immunosuppressed patients and tumors with the histologic subtypes
desmoplastic and acantholytic are often biologically aggressive squamous cell carcinomas.
Compared with other risk factors, depth of invasion >4 mm and size > 2cm demonstrate the
greatest risk for metastasis.

104
The cutaneous lip and chin are divided into cosmetic units by which anatomic boundary?
A. Vermillion border
B. Nasolabial fold
C. Marionette lines
D. Mental crease
E. Philtral ridge
►D

470
The term cosmetic unit is used to define regions of the face by specific characteristics such as
color, texture, presence of hair, and prominence of sebaceous glands. In dermatologic surgery, it
is preferable to conceal incisions within the boundary lines of cosmetic units. The mental crease,
as demonstrated in the photograph, divides the cutaneous lip and chin.

105
A 78 year old man with an infiltrative basal cell carcinoma of the left nasal ala. what is the best
treatment option?
A. MOHS surgery
B. 5-fluorouracil cream
C. Electrodessication and curettage
D. Standard excision
E. Imiquimod
►A

Based on the histopathologic classification and location, this infiltrative basal cell carcinoma
(BCC) is best treated with MOHS surgery to preserve normal tissue and structure. 5-fluorouracil
cream is used to treat superficial BCC, squamous cell carcinoma in situ (SCCIS), and actinic
keratoses. Electrodessication and curettage is used to treat SCCIS and superficial BCC. Standard
excision would be possible if preservation of normal tissue and structure were not as crucial as in
this area, the nasal ala. Imiquimod is used to treat actinic keratoses and superficial basal cell
carcinoma.

106
Which of the following facial rejuvenation techniques creates microthermal zones?
A. Radiofrequency
B. Ablative CO2 laser
C. Intense pulsed light
D. Dermabrasion
E. Fractional photothermolysis
►E

Fractional photothermolysis utilizes an infrared laser to create thousands of microthermal zones


with normal skin remaining between these zones. The zones of normal skin support rapid re-
epithelialization and result in much faster healing times than with traditional resurfacing
procedures.

107
All of the following are considered the danger zones of the face except:
A. Frontal branch of the temporal artery at the temple

471
B. Facial artery as it crosses the mandibular rim
C. Angular artery as it courses near the nose
D. Posterior auricular artery runs near sulcus dividing mastoid from the posterior
ear
E. Anterior aspect of the jaw where the angular artery crosses the jaw
►E

All of the following are considered danger zones of the face except for the anterior aspect of the
jaw where the angular artery crosses the jaw. The angular artery is most dangerous when it
courses near the nose.

108
The most appropriate suture for closing a wound on the buccal mucosa is:
A. Prolene
B. Silk
C. Monocryl
D. Braided nylon
E. PDS
►B

The most appropriate suture for closing a wound on the buccal mucosa is silk. Other sutures may
be used as well, but classically, silk is used because of its soft nature.

109
Which ocular structure is at most risk of injury from an erbium: YAG laser?
A. Lens
B. Retina
C. Cornea
D. Vitreous humor
E. Iris
►C

Injury from lasers may occur via direct or indirect ocular exposure. Damage is generally
wavelength specific. Laser that target hemoglobin or pigment may cause damage to the retinal
pigment or vasculature while lasers that target water as a chromophore (carbon dioxide and
erbium) can damage the cornea.

110
A patient who had liposuction 5 weeks ago presents with multiple firm nodules at the cannula
insertion sites. You suspect the diagnosis is:

472
A. Foreign body granuloma
B. Herpes simplex infection
C. Organized hematoma
D. Mycobacterial infection
E. Cold panniculitis
►D

Atypical mycobacterial infections are occurring with increasing frequency after cosmetic surgery
procedures. These infections typically occur 4-14 weeks after a procedure as a late-occurring
complication. Firm nodules at the treatment site or dehiscence of a previously healed wound may
be presenting signs of atypical mycobacterial infection.

111
The tissue temperature required for destruction of basal cell or squamous cell skin cancers with
cryotherapy:
A. -40 to 50” C
B. 0 to -10 C
C. 10 to 20 C
D. -60 to- 70 C
E. -10 to -20 C
►A

-40 to -50 “C is the temperature required in the treated skin to effectively destroy basal cell and
squamous cell skin cancers. To treat malignant skin lesions a thermocoupling device can be used
to measure the exact temperature of the skin while freezing. After anesthetizing the skin, this
device is inserted into the estimated depth of the skin lesion. Cryotherapy is then performed until
the thermometer reaches a temperature of approximately -50 degrees Celsius. Keratinocytes need
to be frozen to -50 degrees Celsius for effective destruction, while melanocytes are destroyed at -
5 degrees Celsius. To treat cutaneous malignancies,-50 Degress Celsius is the effective
temperature. For benign skin lesions such as verrucae, -20 degrees Celsius is effective.

112
All of the following are branches of the internal carotid artery except:
A. Ophthalmic artery
B. Angular artery
C. Supraorbital artery
D. Dorsal nasal artery
E. Supratrochlear artery
►B

473
Branches of the internal carotid artery include the ophthalmic artery, supraorbital artery, dorsal
nasal artery, supratrochlear artery, palpebral artery, and lacrimal artery. Branches of the external
carotid artery include the superficial temporal artery, facial artery (inferior labial, superior labial,
angular branches), maxillary artery, posterior auricular, and occipital artery. The external and
internal carotid arteries anastamose in the periorbital region.

113
Which of the following is the major antioxidant in the human epidermis?
A. Superoxide dismutase
B. Coenzyme Q10
C. Ascorbic acid
D. Glutathione
E. Alpha-tocopherol
►E

Research into the preventive role of topical antioxidants in photoaging is based on the free
radical theory of aging. All of the choices above are antioxidants which are naturally found in the
skin, however, alpha-tocopherol is the major antioxidant in human epidermis. Epidermal
depletion of this vitamin has been shown to be an early and sensitive marker of environmental
oxidative damage.

114
Which of the following histochemical stains can be used intraoperatively during Mohs
micrographic surgery to confirm the diagnosis of extramammary Paget's disease?
A. Neuron-specific enolase
B. Toluidine blue
C. S-100
D. Cytokeratin 7
E. Myeloperoxidase
►D

Cytokeratin 7 is a structural component of the cytoskeleton that stains poorly differentiated


tumors of the epithelium. This low molecular weight cytokeratin positively stains Paget cells.
Neuron-specific enolase stains merkel cell carcinoma, toluidine blue stains mast cells and
myeloperoxidase stains cells with myeloid differentiation such as leukemia cells. S-100 protein
is a non-specific stain that is commonly used as an adjunctive marker in the diagnosis of
melanoma.

115

474
A 40 year-old woman from Southern California has wrinkles at rest on her forehead, scattered
telangiectasia on her nose and a few seborrheic keratoses on her chest. What category in
Glogau's photoaging classification scale does this patient represent?
A. Type 1
B. Type 2
C. Type 3
D. Type 4
E. Type 5
►C

Glogau‟s photoaging scale is a classification system that employs clinical markers of


photodamage to determine an individual‟s level of photoaging. The scale ranges from Type 1
through Type 4, with Type 4 demonstrating the most extensive photodamage. Type 3 on the
Glogau scale is classified as advanced photoaging and describes a patient who is typically 50
years of age or older, has wrinkles at rest, telangiectasias, obvious dyschromias and visible
keratoses.

116
What is the term used to describe the parallel nature of coherent light waves is?
A. monochromicity
B. coherence
C. collimation
D. energy
E. power
►C

Collimation is the parallel nature of coherent light waves. Monochromaticity is the emission of a
single wavelength. Coherence is the term for light waves traveling in phase. Energy is the
fundamental unit of work. Power is the rate of energy delivery measured in watts.

117
Which of the following cosmetic injectables is FDA-approved for the treatment of lipoatrophy?
A. Radiance
B. Silicone
C. Botox
D. Sculptra
E. Isolagen
►D

475
Sculptra (called New-Fill outside of the US) is a biodegradable filler composed of poly-L-lactic
acid, the same material used in absorbable sutures. It is biocompatible and nonallergenic and was
approved by the FDA in August 2004 for the treatment of HIV-associated lipoatrophy.

118
Which of the following is true regarding lidocaine:
A. 1% lidocaine is equal to 1g/10ml
B. Duration with no epinephrine is 4-6 hours
C. Maximum dose with no epinephrine is 7mg/kg
D. Beta blockers increase lidocaine levels
E. Allergy most commonly occurs to propylene glycol preservatives
►D

Beta blockers increase lidocaine levels. The rest of the answers are false. 1% lidocaine is equal to
1g/100ml or 10mg/ml. Duruation of lidocaine with no epi is 30-60 minutes. Maximum dose of
lidocaine with no epi is 4.5 mg/kg, with epi it is 7 mg/kg. Allergy to lidocaine is most commonly
due to paraben preservatives.

119
Which can work in a wet field?
A. Electrocautery
B. Electrodessication
C. Electrofulguration
D. Electrosection
E. Electrocoagulation
►A

Electrocautery is the only one to work in a wet field as it has no current and works via heat.

120
Which of the following is the earliest symptom of lidocaine toxicity:
A. Tachycardia
B. Perioral tingling
C. Nystagmus
D. Cyanosis
E. Seizure
►B

The maximum dosage of lidocaine is 4.5 mg/kg without epinephrine and 7.0 mg/kg with
epinephrine. Signs of lidocaine toxicity start at 1-5 microgm/ml with increased anxiety,

476
talkativeness, tinnitus, numbess/tingling around lips, metallic taste, double vision. Higher levels
of toxicity may cause nystagmus, muscle twitching, tremor and finally seizures and respiratory
arrest.

121
Activation of the procerus muscle causes
A. Periocular wrinkles
B. Wrinkles at the nasal root
C. Wrinkles on the forehead
D. Perioral wrinkles
E. Accentuation of the melolabial folds
►B

The procerus muscle causes wrinkling at the nasal root and is often targeted with Botox therapy
for improved cosmesis.

122
Which nerve provides the sensory innervation to the tragus?
A. Auricular branch of vagus
B. Facial
C. Glossopharyngeal
D. Auriculotemporal
E. Greater auricular
►D

The auriculotemporal nerve which is a branch of the mandibular branch of the trigeminal nerve
innervates the tragus. The sensory innervation to the auricle is provided by the greater auricular
nerve, and to a lesser extent the lesser occipital nerve. The external auditory canal, concha, and
posauricular sulcus supplied variably by 3 nerves - the auricular branch of the vagus (CNX), the
facial nerve (CN7), and the glossopharyngeal (CNIX).

123
Which of the following topical antibacterial agents demonstrates activity against Pseudomonas
species?
A. Mupirocin
B. Erythromycin
C. Polymyxin
D. Bacitracin
E. Clindamycin
►C

477
Polymyxins are peptides produced by the organism Bacillus polymyxa. They are bactericidal
against Pseudomonas aeruginosa, Proteus mirabilis, Enterobacter, Klebsiella and Escherichia
coli. Since they provide little coverage against gram-positive organisms, polymyxins are often
combined with other antibacterial agents to increase their spectrum of activity.

124
Which of the following lasers uses a solid as its medium?
A. Argon laser
B. pulsed dye laser
C. alexandrite
D. krypton
E. carbon dioxide
►C

Solid media lasers include alexandrite, KTP, ruby, Nd Yag, diode. Liquid media is found in the
pulsed dye laser. Gas media is used in argon, carbon dioxide, krypton, and cooper vapor laser.

125
The use of EMLA cream is contraindicated in patients with which of the following?
A. Atopic dermatitis
B. Deomycin allergy
C. Sickle cell anemia
D. Methemoglobinemia
E. Peripheral neuropathy
►D

The most serious adverse effect of eutectic lidocaine and prilocaine (EMLA) is
methemaglobinemia. A metabolite of prilocaine oxidizes hemoglobin to methemoglobin, which
is less efficient in release of oxygen leading to tissue hypoxia. Patients with congenital or
idiopathic methemoglobinemia or infants under 1 year of age are at higher risk for the
development of this side effect.

126
Which of the following has been implicated in the promotion of skin aging?
A. alpha-tocopherol
B. homocysteine
C. proanthocyanidin
D. beta carotene
E. alpha-lipoic acid

478
►B

Homocysteine is a sulfhydryl-containing amino acid produced during the conversion of


methionine to cysteine. Homocysteine can promote defects in fibrillin-1 and stimulate matrix
metalloproteinase production, leading to collagen and elastic fiber degradation. Homocystinuria,
caused by a deficiency in cystathionine beta-synthase, presents with osteoporosis, thin skin, and
striae.

127
Which of the following parameters determines the wavelength of a laser?
A. Medium
B. Fluence
C. Spot size
D. Pulse duration
E. Lens length
►A

Laser light is monochromatic light that is emitted at a single wavelength. The wavelength of the
laser is determined by the medium in the optical cavity of the laser through which the light
passes. The medium may be solid, liquid or gas.

128
Which of the following lasers may cause milia formation as a post-procedure complication?
A. Excimer
B. KTP
C. Pulsed Dye
D. ND-Yag
E. Erbium
►E

The erbium (2940 nm) and carbon dioxide (10,600 nm) lasers may both cause milia formation
after laser skin resurfacing.

129
Which of the following cosmetic injectables can be seen on routine dental x-rays?
A. Botox
B. Zyplast
C. Radiesse
D. Cosmoderm
E. Restylane

479
►C

Radiesse is an injectable, biodegradable filler which is composed of calcium hydroxylapatite


microspheres. Calcium hydroxylapatite is a normal constituent of bone and thus can be seen on
radiographic imaging. Botox is botulinum toxin; Zyplast is bovine collagen; Cosmoderm is a
non-animal form of collagen; and Restylane is a non-animal form of hyaluronic acid.

130
A Z- plasty is performed to:
A. Reorient a scar
B. Shorten the length of an excision
C. Close a wound on a convex surface
D. Obtain wound eversion
E. Make use of skin from an area of laxity
►A

A Z-plasty is a type of transposition flap that may be used to reorient a scar, especially when the
scar crosses skin tension lines. It is also used to change the scar length or vector and ease scar
contractures.

131
A history of anaphylaxis is a contraindication to treatment with which cosmetic injectable?
A. Sculptra
B. Myobloc
C. Radiance
D. Cosmoderm
E. Restylane
►E

Restylane is a hyaluronic acid gel produced by the Streptococcus species of bacteria. The
package insert for Restylane warns that its use is contraindicated in patients with “severe
allergies manifested by a history of anaphylaxis or the presence of multiple severe allergies”.

132
For high risk surgical patients, when is prophylaxis for infective endocarditis ideally first given?
A. 2 days prior to the procedure
B. 1 day prior to the procedure
C. 1 week prior to the procedure
D. 60 minutes prior to the procedure
E. after the procedure

480
►D

Antibiotics are usually given 60 minutes prior to the procedure. The American Heart Association
advises it can be given up to 2 hours after the procedure if not given before. Infections are
usually polymicrobial, but staph and Strep are of prime concern.

133
All of the following are true statements regarding the immunohistochemical stain Melan-A
except:
A. Antigen present in 80-100% of melanomas
B. Recognized by CD8+ T cells
C. Sensitive marker for melanocytic nevi
D. Effective in frozen and paraffin-embedded tissue
E. Reliably stains desmoplastic melanoma
►E

E Melan-A is a 22-kDa cytoplasmic melanosome-associated glycoprotein also known as MART-


1 (melanoma antigen recognized by T cells). It is a sensitive marker for both melanocytic nevi
and melanoma and may be used on frozen tissue sections as well as paraffin-embedded tissue.
Although the antigen is present in > 80% of melanomas, it does not reliably stain desmoplastic or
spindle cell melanomas.

134
Endocarditis prophylaxis is recommended for which
A. Rheumatic heart disease
B. Atrial septal defects
C. Hypertrophic cardiomyopathy
D. Mitral valve prolapse with regurgitation
E. Previous bacterial endocarditis
►E

Preoperative antibiotics are recommended for endocarditits prophylaxis in select high risk
patients that undergo high risk procedures. High risk procedures are procedures that involve
manipulation of gingival tissue or the periapical region of teeth or perforation of the oral mucosa.
High risk patients are, according to the guidelines, those with the following: Prior infective
endocarditis, Prosthetic cardiac valves, Unrepaired cyanotic congenital heart defects, including
palliative shunts and conduits Congenital heart defects completely repaired with prosthetic
material or a device, whether placed by surgery or by catheter intervention, during the first 6
months after the procedure, Repaired congenital defects with residual defects at the site or
adjacent to the site of a prosthetic patch or prosthetic device, Cardiac transplants and

481
development of cardiac valvulopathy. Patient groups that may have received routine antibiotic
prophylaxis in the past but are no longer candidates for it include those with mitral and aortic
valve disease, rheumatic heart disease, or structural disorders like ventricular or atrial septal
defects or hypertrophic cardiomyopathy, according to the AHA statement. The revised
guidelines were developed with the participation of and have been endorsed by the American
Dental Association, the Infectious Diseases Society of America, and the American Academy of
Pediatrics. Classically 1 gram of Dicloxacillin or cephalexin is given 1 hour preoperatively and
an additional 500 mg is given 6 hours post op. Clindamycin can be givenin those patients who
are penicillin allergic.

135
Which of the following medicines has a newly reported side effect of eruptive epidermoid cysts?
A. Minocycline
B. Retinoids
C. Imiquimod
D. Denileukin difitox
E. Potassium iodide
►C

Imiquimod is a topical immunomodulator which induces the toll like receptor 7 and is used in the
treatment of genital warts, actinic keratoses, and superficial basal cell carcinomas. It has a newly
reported side effect of eruptive epidermoid cysts. Minocycline has side effects including a lupus-
like syndrome and hepatitis. Retinoids has numerous side effects including hypertriglyceridemia,
elevated liver enzymes, dryness of skin, teratogenicity, and most recently inflammatory bowel
disease. Denileukin difitox, an engineered protein combining interleukin-2 and Diphtheria toxin,
used in mycosis fungoides may cause vascular leak syndrome characterized by hypotension,
edema, and pleural effusion. Potassium iodide may lead to the Wolff-Chaikoff effect in which
thyroid hormone synthesis is inhibited from excess iodides that block organic iodides from
binding in the thyroid.

136
The role of imiquimod in the treatment of superficial basal cell carcinoma is based on the
induction of which cytokine?
A. Interleukin-2
B. Tumor growth factor-alpha
C. Interleukin-4
D. Interferon-gamma
E. Tumor necrosis factor-beta
►D

482
Imiquimod is an immunomodulator which induces the production of cytokines that are involved
in cell-mediated immunity. These cytokines include interferon-alpha, interferon-gamma,
interleukin-1, interleukin-10 and tumor necrosis factor-alpha. Induction of these cytokines results
in antiviral and antitumor activity in vivo.

137
The O to Z flap is what type of flap?
A. Rotation
B. Advancement
C. Transposition
D. Pedicle
E. Bilobed
►A

The O to Z flap is a bilateral rotation flap in which tissue is moved from two ends to cover a
central defect. It is often used on the scalp or lower extremities in locations where tissue laxity is
not present.

138
The pain associated with Botulinum A Exotoxin injection is attributed to the:
A. Needle gauge
B. PH
C. Preservative-free saline
D. Exotoxin
E. Temperature
►C

Botulinum A exotoxin is used for multiple reasons, most often for the treatment of dynamic
facial lines. In a double-blind, randomized controlled study, investigators found that botulinum A
exotoxin reconstituted with preservative-containing saline less painful than with preservative-
free saline.

139
This muscle overlies the nasal bone and attaches to the nasal root and is also responsible for the
"bunny lines" of the nose:
A. Procerus
B. Nasalis
C. Obicularis oculi
D. Zygomatic major
E. Zygomatic minor

483
►B

The nasalis muscle is responsible for the foreshortening of the nose. It is responsible for the
'bunny lines."

140
What risk does an organ transplant recipient have of developing melanoma?
A. Same risk as general population
B. Less risk than general population
C. 3-fold increased risk
D. 10- fold increased risk
E. 50-fold increased risk
►C

Organ transplant recipients are at risk for developing many cutaneous tumors. The risk of
melanoma is 3-fold that of the general population whereas these patients have a 65-fold
increased risk of developing squamous cell carcinoma.

141
The main advantage of selecting 4-0 Vicryl rather than 5-0 Vicryl to suture a wound is:
A. Smaller suture diameter
B. Increased tensile strength
C. Increased knot security
D. Smaller needle
E. Increased suture memory
►B

Sutures are classified according to the United States Pharmacopeia (USP) criteria. This
classification system specifies the diameter of a given suture material that is required to produce
a certain tensile strength. The smaller the cross-sectional diameter of a suture material, the higher
the USP number that is assigned. Thus, 4-0 Vicryl will have greater tensile strength and a larger
cross-sectional diameter than 5-0 Vicryl.

142
An M-plasty is performed to:
A. Decrease tension on a wound
B. Obtain wound eversion
C. Shorten the length of an excision
D. Reorient a scar
E. Make use of excess tissue

484
►C

An M-plasty is a variation of the fusiform excision where either one or both ends of the ellipse
are modified. The main advantages to performing an M-plasty are to shorten the length of an
excision and to correct dog-ears.

143
Which anesthetic has the shortest duration of action?
A. Mepivacaine
B. Procaine
C. Bupivacaine
D. Tetracaine
E. Prilocaine
►B

Procaine has the shortest duration of action, only lasting about 30-60 minutes. It is an ester
anesthetic and is commonly used in dentistry.

144
This agent should not be used in pregnant patients and in children because it can be teratogenic:
A. Hexachlorophene
B. Chlorhexidine
C. Providone iodine
D. Bentadine
E. Hibiclens
►A

Hexachlorophene should not be used in children or in pregnant women. It has a potential for
neurotoxicity and can be teratogenic.

145
Which one of the following lasers would be effective in the treatment of rhinophyma?
A. Pulsed dye laser
B. Ruby laser
C. Alexandrite laser
D. KTP laser
E. Carbon dioxide laser
►E

485
The carbon dioxide laser is a 10,600 nm laser that can be used to treat rhinophyma. The
advantage of the carbon dioxide laser over steel or dermabrasion is that the laser is relatively
bloodless. The erbium YAG can also be used to treat rhinophyma.

146
Which tattoo pigment is most commonly associated with allergic reactions, eczematous and
granulomatous?
A. Mercuric sulfide
B. Titanium dioxide
C. Cadmium sulfide
D. Cobalt
E. Carbon
►A

Allergic reactions have been reported with several different types of tattoo pigment. The most
commonly associated tattoo pigment however is mercuric sulfide. Tattoo with mercuric sulfide
produces a red color.

147
When would use of this technique be most beneficial?
A. To prevent post-operative bleeding
B. To prevent post-operative infection
C. To shorten the length of the wound
D. To increase the width of the wound
E. To prevent pincushioning
►C

M-plasty is used to shorten wound length and is often when it is not desirable to extend the
ellipse into a nearby structure.

148
Damage to the zygomatic branch of the facial nerve leads to which of the following:
A. Corner of the mouth droop
B. Inability to pucker lips
C. Eyelid ectropion and inability to close eyelid
D. Unilateral eyelid ptosis
E. Winged scapula
►C

Damage to the zygomatic nerve causes eyelid ectropion and inability to close eyelid.

486
149
A Mohs surgery fellow experiences burning and tingling of her fingertips 6 months after starting
her fellowship. She most likely has been exposed to which one of the following chemicals?
A. Chlorhexidine gluconate
B. Hematoxylin
C. Povodine-iodine
D. Hexachlorophene
E. Toluidine blue
►D

Hexachlorophene is an antibacterial agent that was first introduced in 1944. Its use was
discontinued in the 1970s when it was discovered to have neurotoxic side effects.

150
Which of the following is a complication of Thermage treatment?
A. Keloid formation
B. Lipoatrophy
C. Granuloma formation
D. Hyperpigmentation
E. Herpes simplex infection
►B

Thermage (ThermaCool) is a nonablative radiofrequency-based system which uses volumetric


heating to induce tightening of the skin and dermal remodeling. Lipoatrophy is a late-occurring
potential complication of this technology.

151
Which of the following surgical prepatory solutions is teratogenic?
A. Povidine-iodine
B. Chlorhexidine
C. Ethyl and Isopropyl alcohol
D. Hexachlorophene
E. Benzalkonium chloride
►D

Hexachlorophene or pHisoHex has shown to be toxic to developing embryos in animal studies


and thus is prohibited for use in pregnant females. Chlorhexidine is a common ingredient in oral
rinses(Peridex) and surgical prep solutions(Hibiclens) can cause ototoxicity if in contact with the
middle ear and ocular toxicity if in contact with the eyes. Ethyl alcohol is effective against both

487
gram positive and gram negative organisms. Both Povidine-Iodine (Betadine)and Benzalkonium
chloride (Zephiran) can cause an allergic contact dermatitis.

152
The cutaneous lip and chin are divided into cosmetic units by which anatomic boundary?
A. Vermillion border
B. Nasolabial fold
C. Marionette lines
D. Mental crease
E. Philtral ridge
►D

The term cosmetic unit is used to define regions of the face by specific characteristics such as
color, texture, presence of hair, and prominence of sebaceous glands. In dermatologic surgery, it
is preferable to conceal incisions within the boundary lines of cosmetic units. The mental crease
divides the cutaneous lip and chin.

153
The tensile strength of a wound 6 months after surgery compared to intact skin is:
A. 15%
B. 30%
C. 50%
D. 70%
E. 100%
►D

The tensile strength of a wound approaches 70% of normal skin strength at 6 months
postoperatively. Wound healing is a process which takes many months, yet the tensile strength of
a wound never exceeds 80% of the tensile strength of intact skin. It is therefore critical to use
long-lasting subcuticular sutures to minimize the tension on a healing wound and prevent scar
widening.

154
How long after cutaneous infiltration of lidocaine with epinephrine is maximal vasoconstriction
achieved?
A. 1 minute
B. 7 minutes
C. 15 minutes
D. 30 minutes
E. 1 hour

488
►C

The use of epinephrine with local anesthesia has two main advantages. Firstly, the epinephrine is
a vasoconstrictor and assists in controlling bleeding during surgery. Secondly, and also a direct
result of its vasoconstrictive effects, epinephrine helps prolong the duration of the anesthetic
agent 100% to 200% by delaying its absorption from the surgical site. Although the anesthetic
properties of lidocaine take effect within the first few minutes of infiltration, the vasoconstrictive
effects of epinephrine require approximately 15 minutes to fully develop.

155
All of the following statements are true regarding the „MCW Melanoma Cocktail‟ except:
A. it is a mixture of monoclonal antibodies to MART-1
B. it is performed intraoperatively
C. it is a mixture of monoclonal antibodies to Melan-A
D. it demonstrates micrometastases in sentinel lymph nodes
E. it is a mixture of polyclonal antibodies to tyrosinase
►B

The „MCW Melanoma Cocktail‟ is an immunostain made up of monoclonal antibodies to


MART-1, Melan-A and tryrosine. It is used intraoperatively during sentinel lymph node biopsy
and allows for rapid and accurate determination of micrometastases. Shidham VB et al.
Optimization of an immunostaining protocol for the rapid intraoperative evaluation of melanoma
sentinel lymph node imprint smears with the „MCW Melanoma Cocktail‟. Cytojournal
2004;1(1):2.

156
Which cosmetic filler substance is contraindicated in the glabella?
A. Bovine collagen
B. Human collagen
C. Hyaluronic acid
D. Calcium hydroxyapatite
E. Poly-L-lactic acid
►A

As placement of bovine collagen is typically deeper in the tissues compared to other filler, its use
has resulted in vascular occlusion and necrosis in the glabellar area, and is therefore
contraindicated.

157
Which of the following cosmetic injectables has the longest duration of action?

489
A. Botox
B. Sculptra (poly L-lactic acid)
C. Restylane (hyaluronic acid)
D. Cosmoderm (collagen)
E. Myobloc
►B

Sculptra (called New-Fill outside of the US) is a biodegradable filler composed of poly-L-lactic
acid, the same material used in absorbable sutures. Preliminary studies have demonstrated
longevity of the filler at two years post-treatment. Valantin MA et al. Polylactic acid implants
(New-Fill) to correct facial lipoatrophy in HIV-infected patients: results of the open-label study
VEGA. AIDS. 2003;17:2471-2477.

158
What is the ratio of sodium bicarbonate to 1% lidocaine in a buffered lidocaine solution?
A. 1:1
B. 1:2
C. 1:5
D. 1:10
E. 1:25
►D

The use of sodium bicarbonate in 1% lidocaine reduces the acidity of the local anesthetic.
Unbuffered lidocaine has pH of approximately 5.5 -7.0 and lidocaine with epinephrine has a pH
of approximately 3.5 - 5.0. The addition of sodium bicarbonate at a ratio 1:10 raises the pH to a
more physiologic level, thereby reducing the stinging sensation associated with injection.
However, the addition of bicarbonate also reduces the shelf-life of the lidocaine.

159
Which of the following local anesthetic agents should not be used in children?
A. Bupivicaine
B. Etidocaine
C. Mepivicaine
D. Prilocaine
E. Benzocaine
►D

Prilocaine should bot be used in children given the risk of


methemoglobinemia.Methemoglobulinemia in children can occur from exposure to oxidizing
substances such as aniline dyes, prilocaine or pyrimidine. In methemoglobinemia, the iron in

490
hemoglobin is oxidized from the ferrous state (Fe 2+) to ferric (Fe3+) resulting in the inability to
transport oxygen and carbon dioxide. Clinically, this condition results in cyanosis.

160
Treatment with which of the following lasers has been effective in psoriasis?
A. Carbon dioxide laser
B. Excimer laser
C. Ruby Laser
D. Pulsed dye laser
E. Alexandrite laser
►B

The excimer laser is a 308 nm wavelength laser that has been used to treat psoriasis. The excimer
laser has some advantages over ultraviolet therapy. By treating only involved skin, higher doses
can be used and clearance may occur with fewer treatments.

161
Which of the following is a vasoconstrictor in the absence of epinephrine?
A. lidocaine
B. procaine
C. cocaine
D. bupivacaine
E. prilocaine
►C

Cocaine is the only anesthetic that vasoconstricts in the absence of epinephrine. Cocaine is an
ester anesthetic. Full vasoconstriction effects of epinephrine requires 7-15 minutes. Epinephrine
is pregnancy category C.

162
The most important function of a postsurgical wound dressing is to
A. Immobilize the wound
B. Prevent wound infection
C. Prevent scar formation
D. Reduce pain
E. Evert wound edges
►A

Postsurgical wound dressings are intended to aid in hemostasis, prevent infection and provide a
semi-occlusive environment. The most important postoperative function is to immobilize the

491
wound and surrounding tissue to allow the newly formed platelet plugs to remain intact. Winton
GB, Salasche SJ. Wound dressings for dermatologic surgery. J Am Acad Dermatol 1985; 13:
1026-44

163
Topical vitamin K has been shown to:
A. Increases epidermal differentiation
B. Reduce severity of postoperative purpura
C. Decrease epidermal pigmentation
D. Improve fine winkling
E. Improve skin hydration
►B

Topical vitamin K reduced post later treatment purpura when applied after (not before) treatment
of telangectasia with the 585 nm pulsed dye laser.

164
Which material is used to coat the jaws of the needle holder in the photograph?
A. Gold
B. Tungsten carbide
C. Copper
D. Stainless steel
E. Nickel
►B

Tungsten carbide is believed to be a harder more durable material that allows for better grasping
of the needle. Needle holders with tungsten carbide jaws usually have gold-plated handles.

165
What is the tattoo pigment responsible for most lichenoid reactions?
A. Titanium dioxide
B. Carbon
C. Mercuric sulfide
D. Chromates
E. Iron oxide
►C

Lichenoid tattoo reactions are not as common as eczematous reactions, both of which are most
commonly caused by mercuric sulfide. Lichenoid reaction are likely to be mediated by delayed
hypersensitivity to a lymphocytic T-cell infiltrate.

492
166
Which one of the substances is not a component of a Jessner's peel?
A. Ethanol
B. Salicylic acid
C. Lactic acid
D. Phenol
E. Resorcinol
►D

Jessner's is a light-medium chemical peel that contains salicylic acid, lactic acid, ethanol, and
resorcinol. Phenol is used in deep peels, ie the Baker-Gordon peel which also contains croton oil,
water, and septisol. Phenol may induce cardiac arrythmias and liver or renal failure; hydration is
important in all phase of treatment.

167
Which of the following cosmetic injectables binds water to create volume, has a low allergic
reaction profile, and lasts 6 to 12 months?
A. Botulinum toxin
B. Calcium hydroxylapatite
C. Human-derived collagen
D. Hyaluronic acid
E. Silicone
►D

Hyaluronic acid is a natural component of human connective tissue. As a cosmetic filler,


hyaluronic acid is an attractive alternative to currently available filler substances: its ability to
bind water creates volume and plumps the skin; its duration of action of approximately 6-12
months is appreciably longer than conventional collagen; and since hyaluronic acid is chemically
identical across all species, hypersensitivity reactions are rare.

168
The use of EMLA cream is contraindicated in patients with which of the following?
A. Atopic dermatitis
B. Neomycin allergy
C. Sickle cell anemia
D. Methemoglobinemia
E. Peripheral neuropathy
►D

493
EMLA is a topical anesthetic composed of a eutectic mixture of 2.5% lidocaine and 2.5%
prilocaine. The major concern when using EMLA is the potential risk of methemoglobinemia.
Patients with glucose-6-phosphate deficiency and patients taking methemoglobin-inducing
agents (dapsone, phenytoin, sulfonamides) are more susceptible to developing
methemoglobinemia. In addition, care should be taken when using EMLA in infants less than 3
months of age because of the incomplete maturation of their NADH-methemoglobinemia
reductase system.

169
Which part of the eye is most likely to be damaged to exposure to a pulsed dye laser?
A. Retina
B. Cornea
C. Lens
D. Conjuctiva
E. Sclera
►A

The pulsed dye laser operates at 585-nm and targets hemoglobin as a chromophore. It can pass
through the cornea and damage choroidal and retinal vasculature. Several pigment-specific lasers
(eg, ruby, alexandrite, Nd:YAG) may also damage the retina by targeting the retinal pigment.
The eye damage is related to the chromophore of the laser. The lasers that target water damage
the cornea.

170
The repair demonstrated in the photograph is a:
A. Split-thickness skin graft
B. Interpolation flap
C. Full-thickness skin graft
D. Rotation flap
E. Transposition flap
►B

The postauricular flap is useful for repairing extensive defects of the helical rim, particularly
when cartilage is involved. It is an example of an interpolation flap.

171
Epinephrine should be avoided with anesthetics in which of the following conditions?
A. Hypothyroidism
B. Diabetes Mellitus
C. Renal failure

494
D. Liver failure
E. Pheochromocytoma
►E

Epinephrine should be avoided in patients with a pheochromocytoma.

172
The type of repair depicted in the photograph is a:
A. Rhombic flap
B. Nasolabial flap
C. Rotation flap
D. Full-thickness skin graft
E. Island pedicle flap
►A

The rhombic flap is a type of transposition flap. The classic rhombic flap is designed with two 60
degree angles and two 120 degree angles. The point of maximum tension is at the closure of the
donor site.

173
The following are true statements regarding the design of an excisional ellipse except:
A. Ellipse includes pathologic and normal tissue
B. Length of ellipse is at least 3 times its total width
C. The angle at each of the apices is 15º
D. Incision lines are placed in relaxed skin tension lines
E. Long axis of ellipse is oriented perpendicular to free margins
►C

Thoughtful design of an elliptical excision is important for obtaining optimum tissue diagnosis,
tumor clearance, wound closure and cosmetic result. The above statements are all true except
that the angles at the apices of the ellipse should be 30º.

174
All of the following thrombotic complications have been reported after discontinuing aspirin
therapy prior to surgery except:
A. Stroke
B. Myocardial infarction
C. Pulmonary embolism
D. Transient ischemic attack
E. Cerebral embolism

495
►C

Kovich et al. JAAD 2003 evaluated thrombotic complications in patients who had stopped either
coumadin or aspirin peri-operatively. Thrombotic complications in patients who stopped
warfarin included stroke, TIA, myocardial infarction, cerebral embolism, death, DVT,
pulmonary embolus, and blindness. Thrombotic complications in patients who stopped aspirin
included stroke, TIA, myocardial infarction, cerebral embolism and death. No DVT or
pulmonary embolus was reported.

175
A 60-year-old man receives cephalexin prophylaxis 60 minutes prior to his Mohs surgery for a
BCC on the nose. Which condition of his most likely necessitated prophylaxis?
A. History of CABG
B. Secundum atrial septal defect
C. Mitral valve prolapse with regurgitation
D. History of rheumatic fever without valve dysfunction
E. Pacemaker
►C

Prior to Mohs patients who are at high risk should receive prophylaxis int he form of cephalexin
(or clindamycin or azithromycin if penicillin-allergic) 30-60 minutes prior to the procedures for
non-oral skin. High risk patients include those with prosthetic valves, a history of bacterial
endocarditis, mitral valve prolapse with regurgitation, mitral valve prolapse without regurgitation
in MEN > 45 years, any valve dysfuntion, cardiac malformation, hypertrophic cardiomyoopathy,
CNS shunts, and more. Some low-risk examples (no prophylaxis) include s/p CABG, pacemaker,
murmur, or mitral valve prolapse without regurgitation.

176
The ultraviolet radiation that is responsible for photoallergic reactions is:
A. UVA
B. UVB
C. UVC
D. UVD
E. Board UV
►A

UVA is known to cause photoallergic reactions. It penetrates glass and goes deep into the dermis
and can cause immediate erythema.

177

496
The most common adverse reaction seen with betadine is:
A. Ototoxity
B. Teratogenicity
C. Allergic contact dermatitis
D. Seizures
E. Corneal damage
►C

Most common side effect seen with betadine is allergic contact dermatitis, secondary to the
iodine component.

178
A patient with a large nasal tip defect had reconstruction of his wound with a forehead flap. After
what postoperative interval should the patient schedule the inset of this flap?
A. 1 week
B. 3 weeks
C. 2 months
D. 3 months
E. 6 months
►B

The forehead flap is often the reconstructive option of choice for large nasal defects. The flap has
a vertically-oriented blood supply which is supplied primarily from the supratrochlear arteries.
Pedicle division and inset of this flap are typically performed 3 weeks after the initial surgery.

179
A patient with a squamous cell carcinoma of the helical rim had excisional surgery followed by
closure of the wound with an advancement flap. Which would be the most appropriate antibiotic
for this patient to receive postoperatively?
A. Dicloxacillin
B. Cephalexin
C. Erythromycin
D. Ciprofloxacin
E. Trimethoprim-sulfamethoxazole
►D

Infection after cutaneous surgery is typically due to Staphylococcal and Streptococcal organisms.
However, Pseudomonas aeruginosa may be normal flora of the ear and thus can potentially
complicate ear surgery by causing severe infection of the external ear canal (malignant otitis

497
externa). Ciprofloxacin would be the antibiotic of choice for this patient because it is effective
against Pseudomonas species.

180
Which of the following determines the wavelength of a laser?
A. Fluence
B. Q switch
C. Medium
D. Spot size
E. Pulse duration
►C

The wavelength of a laser is determined by the medium.

181
Treatment with which modality would be most effective for the patient shown?
A. Pulsed dye laser
B. Intense pulsed light
C. Photodynamic therapy
D. Radiofrequency
E. Erbium:YAG laser
►B

The patient shown has multiple solar lentigines. Targeting the melanosomes in these benign,
superficial pigmented lesions would be most successfully accomplished with the intense pulsed
light source.

182
Which of the following absorbable sutures lasts the longest?
A. Catgut
B. Polyglycolic acid
C. Polyglactin 910
D. Polydioxanone
E. Polypropylene
►D

Polydioxanone (PDS) is an absorbable monofilament which lasts approximately 180 days.


Catgut has a variable rate of absorption but typically lasts about 7-14 days. Polyglactin 910
(Vicryl) and polyglycolic acid (Dexon) are both absorbed in about 90 days. Polypropylene
(Prolene) is not an absorbable suture.

498
499
Chapter -16-
Dermatopharmacology
1
Concomitant administration of retinoid or ketoconazole with doxorubicin may lead to which side
effect?
A. Dermatomyositis-like reaction
B. Papulopustular eruption
C. Capillary leak syndrome
D. Sticky skin
E. Leg ulcers
►D

Doxorubicin when given with retinoids or ketoconazole may lead to sticky skin. A
dermatomyositis-like reaction and leg ulcers may be seen with hydroxyurea. Epidermal growth
factor receptors such as erlotinib and cetuximab can produce papulopustular eruptions. Capillary
leak syndrome is seen with interleukins.

2
Which of the following vehicles is correctly defined
A. Ointment-water in oil emulsion
B. Gel-oil in water emulsion
C. Cream-Semisolid emulsion in alcohol base
D. Lotion/Solution-Cream in water
E. Foam - powder in cream
►A

Various vehicles are used for different clinical situations. Ointments are water in oil emulsions.
Creams are oil in water emulsion. Gels are semisolid emulsion in alcohol base .
Lotions/Solutions are powder in water (some oil). Foams are liquid and/or solid materials in a
gaseous medium.

3
Which of the following supplements is most likely to decrease hemolysis associated in patients
taking dapsone?
A. Vitamin A
B. Vitamin B6
C. Folic acid
D. Vitamin D

500
E. Vitamin E
►E

Adverse effects from dapsone are both pharmacologic and idiosyncratic and include hemolytic
anemia, methemoglobinemia, agranulocytosis, hypersensitivity syndrome and neuropathy. Of
these, the first two are pharmacologic and anticipated, to some degree, in most patients treated
with dapsone. However, the magnitude of toxicity varies greatly among individuals on the drug.
Methemoglobinemia is the formation of methemoglobin in the blood, which has a decreased
oxygen-carrying capacity compared with hemoglobin and can result in cyanosis. The reaction is
related to the N-hydroxy metabolites of dapsone, which are potent oxidants. G6PD-deficient
individuals are more susceptible to oxidative stresses, including those from dapsone metabolites,
and a baseline G6PD level is recommended prior to initiation of dapsone therapy. Vitamin E
(800 IU/day) has been suggested to provide a small amount of protection against
methemoglobinemia and hemolysis, however, the clinical benefit of this strategy is unclear.

4
Potentially fatal ventricular arrhythmias can occur with concomitant use of cisapride and:
A. Astemizole
B. Digoxin
C. Terbinafine
D. Erythromycin
E. Atorvostatin
►D

Co-administration of erythromycin with the antihistamines terfenedine and astemizole or the


gastrointestinal promobility agent cisapride increases the risk of torsade de pointes and is
contraindicated. These drugs are no longer available in the US.

5
Fomivirsen is a single-stranded antisense oligonucleotide FDA-approved for the treatment of:
A. HSV infection
B. HIV infection
C. EBV infection
D. CMV infection
E. HPV infection
►D

Fomiversen (Vitraene) represents a new class of therapeutic agents known as antisense drugs. It
is indicated for the treatment of CMV retinitis in patients with AIDS. The most common adverse
effects are ocular inflammation and increases in intraocular pressure.

501
6
All of the following agents exert their function in a cell-cycle specific manner except:
A. Cyclophosphamide
B. Methotrexate
C. Azathioprine
D. Hydroxyurea
E. 5-fluorouracil
►A

Cyclophosphamide is a cell-cycle nonspecific agent, which produces DNA cross-linkages at any


point in the cell cycle. Methotrexate, azaithioprine, and hydroxyurea are S-phase specific
cytotoxic agents. 5-fluorouracil is a cell-cycle specific pyrimidine antagonist.

7
All of the following drugs are known to cause pseudoporphyria except for:
A. Dapsone
B. Tetraycycline
C. Amiodarone
D. NSAIDs
E. Ampicillin
►E

All of the following drugs cause pseudoporphyria except ampicillin. Dapsone, tetracycline,
amiodarone, NSAIDs, and nalidixic acid.

8
Thryoid function tests should be checked before and during therapy with which of the following
medications?
A. Colchicine
B. Gold
C. Potassium iodide
D. Thalidomide
E. Azathioprine
►C

The Wolff-Chaifkoff effect, which is the inhibition of thyroid hormone synthesis from excess
iodides which block organic iodides from binding in the thyroid, can be observed in patients on
potassium iodide therapy. In patients with normal thyroid function, autoregulatory mechanisms

502
allow for appropriate escape from this effect. In patients with impaired autoregulatory
mechanisms, the Wolff-Chaikoff effect can lead to hypothyroidism.

9
The antihistamine with strong H1 and H2 blockade is:
A. Chlorpheniramine
B. Cetirizine
C. Cimetidine
D. Cyproheptadine
E. Doxepin
►E

Doxepin, a tricyclic antidepressant, has H1 and H2 antihistamine activity.

10
Which of the following is an adverse effect of oral retinoid therapy?
A. Mania
B. Fibroydysplasia ossicficans progressiva
C. Vascular calcification
D. Hypertension
E. Hypertrichosis
►B

Adverse effects of oral retinoid therapy: Cutaneous: Xerosis, palmoplantar digital desquamation,
retinoid dermatitis, photosensitivity, pyogenic granulomas, stickiness sensation, Staph aureus
infxns Hair: Teolgen effluvian, abnormal hair texture, dryness Nails: Fragility with nail
softening, paronychia, onycholysis Occular: Dry eyes w/visual blurriness,
Blepharoconjunctivitis, Photophobia Oral: Cheilitis, dry mouth, sore mouth and tongue Nasal:
Nasal mucosal dryness, decreased mucous secretion, epistaxis Musculoskeletal: Arthralgias,
myalgias, fatigue and muscle weakness, tendinitis Neurological: Headache, mild depression
Gatrointestinal: N/V/D, adb pain

11
In regards to liposuction, what is the maximum amount of tumescent anesthesia (lidocaine)
recommended for an adult?
A. 15 mg/kg
B. 25 mg/kg
C. 35 mg/kg
D. 45 mg/kg
E. 55 mg/kg

503
►E

Technically, 35 mg/kg is considered the safe and average amount recommended. However,
maximum amount of lidocaine has been reported to be 55 mg/kg. Tumescent anesthesia is a
method where dilute lidocaine is injected into subcutaneous fat for the liposuction procedure.

12
Finasteride is a specific inhibitor of:
A. Dihydrotesterone reductase
B. Type I 5 alpha reductase
C. Type II 5 alpha reductase
D. Aromatase
E. Testosterone synthetase
►C

Finasteride, a type II 5 alpha reductase inhibitor, given as a 1mg tablet daily, is effective in
preventing further hair loss and in increasing the hair counts to the point of cosmetically
appreciatable results in men ages 18 to 41 with mild to moderate hair loss at the vertex, in the
anterior midscalp, and the frontal region.

13
This medication is used for the treatment of scabies in pregnant women:
A. Precipitated sulfur 6%
B. Thiabendazole
C. Malathion
D. Ivermectin
E. Permethrin
►A

Precipitated sulfur 6% can be used for the treatment of scabies in pregnant women. Scabies can
be in bedding and in clothes and should be washed in hot water after the patient is treated.

14
Women with unwanted hair have the option of using a cream to help in the reduction of facial
hair. This cream is:
A. Eflornithin
B. Spironolactone
C. Finasteride
D. Hydroxy-progesterone
E. Androstenedione

504
►A

Women that have unwanted facial hair can use eflornithine and is also known as Vaniqa. It is
applied to the skin for the reduction of unwanted facial hair. It inhibits ornithine decarboxylase
located in the root of the hair follicle.

15
This antifungal medication has a side effect of causing rare fulminant hepatitis risk,
gynecomastia and impotence:
A. Terbinafine
B. Itraconazole
C. Fluconazole
D. Griseofulvin
E. Ketoconazole
►E

Ketoconazole causes rare fulminant hepatitis risk, gynecomastia, and impotence. It inhibits CYP
450 and is a triazole and a keratinophilic.

16
All of the following cause a SLE like eruption except:
A. Anticonvulsants
B. Isoniazid
C. Hydralazine
D. Minocycline
E. Phenophalein
►E

Phenophalein is an ingredient in laxatives and causes fixed drug eruption. Anticonvulsants,


Isoniazid, hydralazine, minocycline, procainamide, penicilline, and D-penicillamine all cause
SLE-like eruption.

17
This drug chelates polyvalent cations resulting in the inhibition of the metal dependent enzymes
that degrade peroxides in the fungal cell wall:
A. Ciclopirox
B. Griseofulvin
C. Nystatin
D. Terbinafine
E. Itraconazole

505
►A

Ciclopirox chelates polyvalent cations resulting in the inhibition of metal dependent enzymes
that degrade peroxides in the fungal cell wall.

18
Which of the following antifungal agents is contraindicated in patients with a history of
porphyria?
A. Griseofulvin
B. Ketoconazole
C. Fluconazole
D. Itraconazole
E. Terfinafine
►A

Griseofulvin has been reported as a potential exacerbator of acute intermittent porphyria, and
thus is contraindicated in patients with a history of porphyria.

19
All the following have been known to cause acute generalized exanthematous pustulosis except:
A. Beta-lactam antibiotics
B. Macrolide antibiotics
C. Mercury
D. Cephalosporins
E. Hydrochlorothiazides
►E

Hydrochlorothiazide causes a lichenoid eruption. Beta-lactam antibiotics, macrolide antibiotics,


Mercury, and cephalosporins all have been known to cause AGEP.

20
Which of the following has been associated with a lichenoid drug eruption?
A. Dabigatran
B. Hydrochlorothiazide
C. Acetaminophen
D. Erythromycin
E. Nicotinamide
►B

506
Lichen-planus-like (lichenoid) drug eruptions have been reported with: antimalarials, B-blockers,
captopril, gold, penicillamine, HCTZ, NSAIDs. Lichenoid drug reactions are often
photodistributed.

21
Which of the following is true of the live attenuated varicella vaccine when given to healthy
adults?
A. It was marginally effective
B. It provides only short term protection (6 months)
C. It protects even individuals who never had serconversion or whose antibody
levels were undetectable from severe varicella zoster viral disease
D. The disease process will be accelerated
E. Disseminated zoster is frequently seen
►C

When live attenuated varicella vaccine is given to healthy adults, it protects even individuals
who never had serconversion or whose antibody levels were undetectable from severe varicella
zoster viral disease.

22
The steroid with the least minerocorticoid activity is:
A. Hydrocortisone
B. Cortisone
C. Prednisone
D. Methylprednisolone
E. Prednisolone
►D

Of the corticosteroids listed, the steroid with the lowest mineralcorticoid activity is
methylprednisolone. Minerocorticoids act on the kidney to decrease the rate of sodium excretion
(with accompanying retention of water). Triamcinolone, dexamethasone, and betamethasone also
have low mineralcorticoid activity.

23
Which of the following is a low sedation metabolite of hydroxyzine?
A. Fexofenadine
B. Cetirizine
C. Loratadine
D. Cyproheptadine
E. Ranitidine

507
►B

Cetirizine is a second-generation H1 antihistamine that is a low sedation metabolite of


hydroxyzine.

24
Gold is used for its anti-inflammatory effects and inhibits macrophage and neutrophil
phagocytosis. It is effective in inhibiting degradative epidermal lysosomal enzymes which
contribute to blister formation. This reaction that can occur with gold causes acute flushing,
dizziness, hypotension and fainting:
A. Nitritoid reaction
B. Leucovorin reaction
C. Juxheimer reaction
D. Golden reaction
E. Lupus like reaction
►A

Nitritoid reaction is after a gold injection that causes acute flushing, dizziness, hypotension, and
fainting. Gold is used in treat severe lupus erythematosus and psoriatic arthritis. The most
common reaction is mucocutaneous side effects such as stomatitis, cheilitis and lichen planus
like eruptions and pityriasis rosea like eruptions.

25
What family of medications is associated with xerosis?
A. Sulfonylureas
B. Beta blockers
C. Calcium channel blockers
D. Loop diuretics
E. Cholesterol lowering agents
►E

Medications that alter the lipid composition of the epidermis and stratum corneum may impair
the normal barrier function of the skin. Cholesterol lowering medications like HMG-CoA
reductase inhibitors and niacin may cause xerosis through this mechanism.

26
Which of the following is the highest potency topical corticosteroid?
A. Halobetasol propionate cream 0.05%
B. Triamcinolone acetonide ointment 0.1%
C. Fluocinolone acetonide cream 0.025%

508
D. Hydrocortisone valerate ointment 0.2%
E. Mometasone furoate ointment 0.1%
►A

Halobetasol propionate cream 0.05% is a superpotent Class 1 topical steroid. Triamcinolone


acetonide ointment 0.1% and hydrocortisone valerate ointment 0.2% are medium potency class 4
topical steroids. Fluocinolone acetonide cream 0.025% is a medium potency class 4 topical
steroid. Mometasone furoate ointment 0.1% is a high potency class 2 topical steroid.

27
Which of the following substances is not found in Castellani's paint?
A. Resorcinol
B. Acetone
C. Magenta
D. Phenol
E. Lactic acid
►E

Castellani's paint is a fungicidal and bactericidal agent. It contains boric acid, resorcinol, acetone,
water, industrial methylated spirit, magenta, and phenol. Magenta may stain clothing and skin
while phenol may be toxic in children.

28
Which of the following locations has the highest percutaneous absorption of topical
corticosteroids?
A. Scrotum
B. Axilla
C. Back
D. Arm
E. Palm
►A

Absorption through the scrotum is about 42 times greater than the ventral forearm. Although
less, other occluded sites such as the axilla have increased percutaneous absorption. The palms
and soles have the least due to the thick stratum corneum.

29
A patient is using imiqimod for her warts. This medicine uses which toll like receptor to act on:
A. 2
B. 3

509
C. 9
D. 5
E. 7
►E

Imiquimod acts on toll like receptor 7. It can be used in the treatment of warts, actinic keratosis,
superficial basal cell carcinomas and squamous cell carcinomas in-situ. It also up-regulates TNF
alpha, IFN gamma and alpha and IL-12.

30
Approximately what percentage of patients with drug hypersensitivity syndrome will have liver
function test abnormalities?
A. Less than 10%
B. 25%
C. 50%
D. 75%
E. Close to 100%
►C

Drug hypersensitivity syndrome is characterized by fever, skin eruption and internal organ
involvement. Drugs associated with drug hypersensitivity syndrome include sulfonamindes,
dapsone, anticonvulsants (carbamezapine, phenobarbitol, lamotrigine), anti-retrovirals (ritonovir,
nevirapine) and minocycline. Approximately 50% of patients will have abnormal liver enzymes.

31
Which of the following biologic therapies is pregnancy category C?
A. Etanercept
B. Alefacept
C. Infliximab
D. Efalizumab
E. Adalimumab
►D

All of the above drugs are pregnancy category B except efalizumab which is category C.

32
Patients that are on methotrexate with renal disease and also using NSAIDs or TMP/SMX with
no folate supplementation are at a greater risk for:
A. Pancytopenia
B. Liver fibrosis

510
C. Liver cirrhosis
D. Renal failure
E. Renal insufficiency
►A

Patients on methotrexate with renal disease and on NSAIDs and TMP/SMX with no folate
supplementation are at a great risk for pancytopenia. Leukovorin is given under conditions of
methotrexate induced myelosuppression.

33
Which of the following side effects has not been reported in association with intravenous
immune globulin?
A. Headache
B. Flushing
C. Hypotension
D. Stevens-Johnson syndrome
E. Anaphylaxis
►D

IVIG is used to treat several diseases including graft versus host disease, connective tissue
disease, and autoimmune bullous dermatoses. Adverse effects include infusion reactions
(headache, flushing, chills, myalgia, wheezing, Tachycardia, lower back pain, nausea, or
hypotension). Anaphylaxis occurs rarely. Disseminated intravascular coagulation, transient
neutropenia, and aseptic meningitis syndrome has been reported. Cutaneous adverse effects
include eczematous eruptions and alopecia.

34
The Wolff-Chaikoff effect is associated with what medication?
A. Bexarotene
B. Zidovudine
C. Potassium iodide
D. Hydroxychloroquine
E. Thalidomide
►C

The Wolff-Chaikoff effect is the inhibition of thyroid hormone synthesis from excess iodides
which block organic iodides from binding in the thyroid. In patients with normal thyroid
function, autoregulatory mechanisms allow for escape from this effect. In patients with impaired
autoregulatory mechanisms, the Wolff-Chaikoff effect can lead to hypothyroidism. Thyroid
function should be evaluated and monitored with patients started on potassium iodide.

511
35
This patient also has a diagnosis of acne vulgaris (diagnosed 10 years ago). What is the most
likely explanation for these cutaneous changes?
A. Peripheral vascular disease
B. Diabetes mellitus
C. Drugs hypersensitivity reaction to penicillin
D. Minocycline-induced hyperpigmentation
E. Leukocytoclastic vasculitis
►D

This patient has minocyclin-induced hyperpigmentation. Hyperpigmentation is one of the most


commonly observed side effects of minocycline and occurs regardless of dosage or treatment
duration (although most often seen in patients after long-term treatment with the drug). Three
distributions can be seen: Type I (blue-black pigmentation in sites of inflammation/scars), Type
II (blue-black pigmentation on the anterior lower legs, and Type III (muddy brown diffuse
pigmentation on normal, sun-exposed skin). This patient has Type II pigmentation.

36
A female patient with adult acne is placed on spironolactone for her acne vulgaris. As a
physician, you warn her that there is a black box warning against patients with a :
A. Personal history of breast cancer
B. Personal history of thyroid cancer
C. Personal history of bladder cancer
D. Personal history of colon cancer
E. Personal history of rectal cancer
►A

Spironolactone is an antiandrogenic potassium sparing diuretic. It can be used for women with
adult acne that is recalcitrant to treatment. Although it is not first line, it helps to block androgens
that cause acne in the jawline. Spironolactone has a black box warning against patient with a
personal history of breast cancer or family history of breast cancer.

37
Clinical evidence of hypothyroidism can be induced by which drug?
A. Bexarotene
B. Gold
C. Griseofulvin
D. Acitretin
E. Isotretinoin

512
►A

Bexarotene can cause a central hypothyroidism with low TSH and T4. You must titrate the
levothyroxine dose to the free T4 as it is a central hypothyroidism which is defined as
insufficient TSH to stimulate an otherwise normal thyroid gland.

38
All of the following are true regarding water-soluble retinoids EXCEPT:
A. They are undetectable in the serum after 1 month of stopping therapy
B. They include isotretinoin
C. They include etretinate
D. They include bexarotene
E. They have very little lipid deposition
►C

Isotretinoin, acitretin, and bexarotene are water-soluble, with very little lipid deposition.
Etretinate is 50 times more lipophilic than acitretin, with increased storage in adipose tissue.

39
Which of the following may cause an acneiform eruption?
A. Fluoxetine
B. Valproic acid
C. ACTH
D. Finasteride
E. Methotrexate
►C

Many medications are associated with acneiform eruptions, including halogens (bromide and
iodide), androgenic hormones such as testosterone, ACTH, corticosteroids, isoniazid (INH),
lithium, phenytoin, and vitamins B2, B6 and B12.

40
Peak vulnerability to thalidomide occurs between which days of gestation?
A. Days 1-14
B. Days 15-20
C. Days 21-36
D. Days 37-56
E. Days 57-70
►C

513
Peak vulnerability to thalidomide occurs between days 21 to 36 of gestation, during which only a
single dose will cause birth defects to occur. Birth defects associated with thalidomide include
phocomelia (underdevelopment of arms and legs, the most common birth defect), ear
malformation, and gastrointestinal and urogenital defects.

41
Which of the following drugs is correctly matched to its target enzymes?
A. Tacrolimus-Thymidine kinase
B. Methotrexate-Inosine monophosphate dehydrogenase
C. Mycophenolate Mofetil-Phospholipase A2
D. Acyclovir-DNA polymerase
E. Terbinafine-Cytochrome p450
►D

Drug/Specific enzyme inhibited Tacrolimus - Calcineurin Methotrexate - Dihydrofolate


reductase Mycophenolate mofetil - Inosine monophosphate dyhydogenase Acyclovir - DNA
Polymerase Terbinafine - Squalene epoxidase

42
Which of the following drugs has been known to cause pyogenic granuloma?
A. Daunorubicin
B. Mithramycin
C. Isosfamide
D. Capecitabine
E. Paclitaxel
►D

Systemic retinoids, indinavir and capecitabine have all been describe to cause pyogenic
granulomas.

43
Which biologic agent is administered intramuscularly?
A. Alefacept
B. Efalizumab
C. Etanercept
D. Infliximab
E. None of the above
►A

Alefacept is given intramuscularly.

514
44
Which chemotherapeutic agent has been reported to cause acral sclerosis with Raynaud's
phenomenon?
A. Bleomycin
B. 5-Fluorouracil
C. Methotrexate
D. Actinomycin
E. Interferon
►A

Bleomycin is an antibiotic that induces single strand breaks in the DNA. Reactions to bleomycin
include flagellae hyperpigmentation, acral sclerosis with Reynaud's, penile calcification, and a
morbilliform eruption. Etreinate is stored in the lipid for years and can be re-esterified to acitretin
after ETOH. It should not be used for women that are wanting to get pregnant because it can be
stored in the fat for years.

45
The retinoid that is stored in lipid for years and can be re-esterified as acitretin with ETOH is:
A. Etretinate
B. Acitretin
C. Tretinoin
D. Isotretinoin
E. Bexarotene
►A

Etreinate is stored in the lipid for years and can be re-esterified to acitretin after ETOH. It should
not be used for women that are wanting to get pregnant because it can be stored in the fat for
years.

46
Bone marrow suppression can occur more often in individuals taking azathioprine with
genetically low allele activity of what enzyme?
A. Inosine monophosphate dehydrogenase
B. Thiopurine methyltransferase
C. Glucose-6-phosphatase
D. Dihydrofolate reductase
E. Epoxide hydroxylase
►B

515
Thiopurine methyltransferase is one of three enzymes that metabolize azathioprine along with
xanthine oxidase and HGPRT. Decreased activity of TPMT leads to heightened metabolism by
HGPRT into the active 6-thioguanine. This may lead to increased toxicity including bone
marrow suppression.

47
Cefaclor has been associated with increased incidence of what in children?
A. Mononucleosis-like syndrome
B. Anaphylaxis
C. Transaminitis
D. Serum sickness reaction
E. Generalized tonic-clonic seizures
►D

The use of cefaclor has been associated with an increased incidence of serum sickness in
children.

48
Methotrexate can cause hepatotoxicity and uncommonly cause acute pneumonitis. The
cumulative doses at or above this amount can have an increase risk for inducing liver fibrosis and
cirrhosis:
A. 2.0 grams
B. 3.0 grams
C. 4.0 grams
D. 5.0 grams
E. 6.0 grams
►C

Methotrexate is an antimetabolite that can be used for psoriasis and immunobullous diseases. At
a cumulative dose at or above 4.0 grams can risk inducing liver fibrosis and cirrhosis. The liver
biopsy is the gold standard diagnostic test.

49
Which of the following chemotherapeutic agents has been linked to acneiform eruptions?
A. Bleomycin
B. Cytarabine
C. Cisplatin
D. Doxorubicin
E. Cetuximab
►E

516
Cetuximab is a chimeric anti-epidermal growth factor receptor antibody that is FDA approved to
treat advanced colorectal cancer. Acneiform eruptions have been reported to occur in up to 1/3 of
patients.

50
Neutrophilic eccrine hidradenitis is a side effect of which therapeutic agent?
A. Interferon-alpha
B. Cytarabine
C. Interferon type I
D. Intravenous immune globulin
E. Granulocyte colony stimulating factor
►B

Neutrophilic eccrine hidradenitis most commonly occurs in the setting of a patient with acute
myelogenous leukemia being treated with cytarabine. Clinical manifestations include tender,
erythematous macules, papuls and plaques on the trunk, neck and extremities which resolve
within a few days. Histologically, this drug eruption is defined by the presence of dense
neutrophilic infiltrate within and around eccrine glands, with necrosis of eccrine epithelial cells.

51
Cyclosporin A should not be consumed with grapefruit juice due to:
A. Induction of CYP2D6 by grapefruit juice
B. Inhibition of CYP2D6 by grapefruit juice
C. Induction of CYP3A4 by grapefruit juice
D. Inhibition of CYP3A4 by grapefruit juice
E. Grapefruit juice binds cyclosporine, inactivating it in the GI tract.
►D

Grapefruit juice is an inhibitor of CYP3A4 at the intestinal mucosal membrane. The intestinal
CYP3A4 is involved in "first pass" metabolism - thus with inhibition of this enzyme in the gut,
less cyclosporine is metabolized, allowing for greater absorption of cyclosporine. Saquinivir also
can be affected by this "first pass" inhibition by grapefruit juice.

52
Which of the following statements regarding dapsone and sulfapyridine is true?
A. They exert their anti-inflammatory actions by stimulating the myeloperoxidase
activity of polymorphonuclear leukocytes
B. Dapsone hypersenstivity syndrome is characterized by lymphocytosis

517
C. Sulfapyridine has a similar but often more severe side effect profile than
dapsone
D. Concomittant administration of cimetidine has been shown to increase the risk
of methemoglobinemia
E. None of these answers are correct (all are false)
►E

None of the above statements are true. Dapsone and sulfapyridine exert their anti-inflammatory
actions by inhibiting the myeloperoxidase activity and chemotactic abilities of
polymorphonuclear leukocytes. Dapsone hypersenstivity syndrome is characterized by
eosinophilia, as well as a severe mononucleosis-like reaction, including fever, erythroderma,
hepatitis, and even death. Sulfapyridine has a similar but often less severe side effect profile.
Cimetidine has been shown to provide some protection against methemoglobin formation.

53
All of the following retinoids are excreted in the urine EXCEPT:
A. Bexarotene
B. Etretinate
C. Isotretinoin
D. Tretinoin
E. Acitretin
►A

Bexarotene is excreted via hepatobiliary excretion. The others are excreted in bile and urine.

54
Which drug has been associated with cholestatic hepatitis?
A. Clindamycin
B. Erythromycin
C. Rifampin
D. Doxycycline
E. Ampicillin
►B

The estolate form of erythromycin has been associated with cholestatic hepatitis.

55
A patient taking daily prednisone is advised to switch to alternate day dosing to decrease the risk
of:
A. Glaucoma

518
B. Aseptic bone necrosis
C. Cataracts
D. Adrenal crisis
E. Osteoporosis
►D

Long-term therapy with oral corticosteroids can result in numerous adverse effects, including
elevated risks of glaucoma, cataracts, hypertension, diabetes, osteoporosis, adrenal axis
suppression, and aseptic bone necrosis. Alternate-day dosing or oral corticosteroids lowers the
rate of adrenal axis suppression. It is hypothesized that during the off day, cell mediated
immunity, white blood cells subset levels, and potassium excretion are normalized while the
anti-inflammatory benefits of the drug persist. Alternate-day corticosteroid therapy should be
employed once adequate disease control has been attained with daily dosing. Of note, the risk of
cataracts, osteoporosis, and other adverse effects of long-term corticosteroid use are not
minimized with alternate-day dosing.

56
The mechanism of action of mycophenolate mofetil most closely resembles that of what other
drug?
A. Cyclophosphamide
B. Doxorubicin
C. Azathioprine
D. Hydroxyurea
E. Chlorambucil
►C

Both azaithioprine and mycophenolate mofetil directly interfere with purine synthesis.

57
The form of erythromycin most likely to cause jaundice is:
A. Ethylsuccinate
B. Estolate
C. Stearate
D. Gluceptate
E. Lactobionate
►B

The form of erythromycin that most likely causes jaundice is estolate.

58

519
Which cell type is increased by glucocorticoids?
A. Neutrophils
B. Monocytes
C. T-cells
D. B-cells
E. Eosinophils
►A

Glucocorticoids alter the balance of circulating leukocytes, causing an increase in the number of
polymorphonuclear leukocytes and diminishing the numbers of lymphocytes, eosinophils, and
monocytes.

59
This antiviral medicine doesn't require phosphorylation and may be used for acyclovir
recalcitrant disease:
A. Foscarnet
B. Gancyclovir
C. Famciclovir
D. Valacyclovir
E. Penciclovir
►A

Foscarnet is a non-competitive inhibitor of viral DNA polymerase. It does not require


phosphorylation and can be used for acyclovir recalcitrant disease.

60
A patient taking azathioprine for bullous pemphigoid develops a hypersensitivity syndrome
characterized by fever and shock. Approximately how long ago did the patient start this
medication?
A. 14 days
B. 24 hours
C. 3-4 days
D. 6 weeks
E. 1 week
►A

Azathioprine is a purine analog which is used to treat multiple inflammatory disorders, including
autoimmune bullous disease, vasculitis, and severe dermatitis. A hypersensitivity syndrome with
fever and shock can occur at around 14 days.

520
61
Which drug has recently been shown to be of use in chemoprevention of squamous cell
carcinoma in recessive dystrophic epidermolysis bullosa:
A. Isotretinoin
B. Cyclosporine
C. Mycophenolate mofetil
D. Methotrexate
E. Topical tar
►A

Despite low therapeutic responses of advanced cancers to retinoids, these drugs appear to have a
promising role in chemoprevention. Patients with oral leukoplakia, actinic keratoses, arsenic
keratoses, and Bowen's disease can benefit from retinoid therapy.

62
Which of the following statements regarding sunscreens is true?
A. Physical blockers absorb ultraviolet light and convert it to lower energy
wavelengths
B. Menthyl anthranilate is a UVB absorber
C. Padimate O is a UVB absorber
D. Photoallergy has not been reported to benzophenones
E. PABA and its derivates do not cross react with sulfonamides
►C

Physical blockers reflect and scatter UV rays, whereas chemical sunscreens absorb UV light and
convert the absorbed energy into longer lower energy wavelengths. Menthyl anthranilate is a
UVA blocker. Padimate O, a PABA derivative, is a UVB blocker. Photoallergy has been
reported with increasing frequency to benzophenones. Allergic contact allergy can occur with
PABA and its derivatives, which can cross react with azodyes, aniline, procaine, benzocaine,
paraphenylenediamine, and sulfonamides.

63
Which of the following best describes the contents of a cream?
A. A semisolid emulsion of oil in water
B. A semisolid transparent emulsion
C. An aqueous or alcohol-based substance that may contain a salt in solution
D. A semisolid grease/oil, sometimes also containing powder, but little or now
water
E. An ointment with a high proportion of powder
►A

521
A cream is a semisolid emulsion of oil in water that contains a preservative to prevent
overgrowth of microorganisms. A gel is a semisolid transparent, nongreasy emulsion. A lotion is
a liquid vehicle that is aqueous or alcohol-based that may contain a salt in solution. An ointment
is a semisolid grease/oil, sometimes also containing powder, but little or now water; usually there
is no preservative needed. A paste is an ointment with a high proportion of powder that gives it a
stiff consistency.

64
Which of the following bones will develop the most dramatic changes in density following oral
glucocorticoids?
A. Femur of 72 year old female
B. Femur of 72 year old male
C. Vertebrae of 22 year old male
D. Vertebrae of 22 year old female
E. Vertebrae of 72 year old female
►C

Although post menopausal women are at greatest risk for fractures from steroid therapy (they
have the weakest bones to begin with), it is young men who have the most dramatic decrease in
bone density. Young men have the highest bone density on average and therefore will show the
greatest change with steroid therapy. Trabecular bone (ribs, vertebrae) are more affected than
cortical bones (long bones) due to higher metabolic rate of the former. Also, most
demineralization occurs during the first 6-12 months of therapy. This highlights the fact that
even young men who are not traditionaly at risk for fractures, need prophylaxis for osteoporosis
while on systemic steroids.

65
Which of the following statements is not true regarding the tetracycline antiobiotics?
A. Ingestion of zinc salts may impair absorption of tetracycline
B. Tetracycline is not the most common cause of fixed drug eruption
C. Tetracycline is more phototoxic than demeclocycline
D. These antibiotics are effective against Mycoplasma infections
E. Tetracyclines are contraindicated in children less than 9 years of age
►C

Demeclocycline and doxycycline are the most phototoxic of all the tetracyclines.

66
Which of the following medicines reduces the formation of methemoglobin?

522
A. Dapsone
B. Cimetidine
C. Azathioprine
D. Cyclosporine
E. Rifampin
►B

Cimetidine and vitamin E can reduces the formation of methemoglobin, thereby increasing
tolerance to dapsone which can lead to methemoglobinemia. Azathioprine can cause hepatitis,
lymphoproliferative malignancy, infections, and hypersensitivity syndrome. Cyclosporine can
lead to renal dysfunction, hypertension, and ginvigal hyperplasia. Rifampin can cause decreased
effectiveness of oral contraceptives.

67
Which of the following medications is most likely to have caused this reaction?
A. Pseudoephedrine hydrochloride
B. Vancomycin
C. Lithium
D. ACE inhibitor
E. Diclofenac
►A

Fixed drug eruptions occur 30 mintues to 8 hours after ingestion of offending agent. After
rechallenge with the same agent, the fixed drug eruption will recur. Potential causes of a fixed
drug eruption include analgesics, sulfonamides, barbituates, pseudoephedrine and
anticonvulsants. Pseudoephedrine causes non-pigmented fixed drug as shown in the photo.

68
The combination of ethanol and acitretin is potentially problematic because:
A. Ethanol exacerbates the cheilitis caused by acitretin
B. Acitretin increases the toxicity of ethanol
C. Ethanol inhibits the cytochrome p450 system
D. Ethanol promotes the conversion of acitretin to etretinate
E. Ethanol promotes the metabolism of acitretin
►D

Acitretin is an synthetic retinoid with affinity to the retinoic acid receptor (RAR). It can
modulate the proliferation and differentiation of epidermal keratinocytes. The concurrent
injestion of ethanol and alcohol increases the formation of etretinate. Unlike acitretin, etretinate

523
is more lipophilic and accumulates in the fat. It therefore has a much longer elimination half-life,
estiimated at 120 days.

69
This antiparasitic medication disables sodium transport channels in the nerve cell membrane of
the parasite leading to paralysis:
A. Permethrin
B. Lindane
C. Ivermectin
D. Malathion
E. Precipitated Sulfur (6%)
►A

Permethrin is a pyrethroid compound that disables sodium transport channels in the nerve cell
membrane of the parasite, leading to paralysis. It is used in scabies infections.

70
Clofazamine hyperpigmentation has been described as:
A. Dark hyperpigmented streaks in the nails
B. Red-brown hyperpigmentation within skin lesions
C. Blue-gray hyperpigmentation over the anterior shins, palate, ears
D. Slate gray-purple hyperpigmentation in a photo-exposed distribution
E. Yellow discoloration of the skin, sclera
►B

Clofazamine can induce red-brown hyperpigmentation within skin lesions of patients with
Hansen's disease.

71
Which of the following is not an ingredient of Castellani's paint?
A. Resorcinol
B. Industrial methylated spirit
C. Phenol
D. Boric Acid
E. Ethyl acetate
►E

Castellani's paint was named after Sir Aldo Castellani and contains resorcinol, acetone, magenta,
phenol, boric acid, industrial methylated spirit, and water. It is fungicidal and bactericidal with

524
local anesthetic effects. It has been used to treat inflammatory tinea cruris, leg ulcers, and acute
paronychia.

72
The antibiotics that inhibit DNA gyrase is:
A. Fluoroquinolones
B. Rifampin
C. Clindamycin
D. Acyclovir
E. Cephalosporins
►A

Fluoroquinolones inhibit DNA gyrase and is effective against Mycobacterium species, gram-
negative infections and is contraindicated during pregnancy and in children because of evidence
of its deposition in cartilage leading to impaired cartilage formation

73
Itraconazole is used for the treatment of blastomycosis, histoplasmosis, aspergillosis, candidiasis,
and cryptococcosis. It also works to inhibit:
A. 14 alpha demethylase
B. 14 alpha lanosterol
C. 14 alpha ergosterol
D. 14 alpha esterase
E. 14 beta ergosterol
►A

Itracaonazole acts by inhibiting 14 alpha demethylase that blocks the conversion of lanosterol to
ergosterol. It is highly lipophilic and has a affinity for cytochrome p450.

74
Tetracyclines have a risk of photosensitivity. The tetracycline that has the highest risk for
photosensitivity is:
A. Demeclocycline
B. Doxycycline
C. Tetracycline
D. Minocycline
E. Oxycycline
►A

525
Tetracyclines bind the bacterial at the 30S ribosomal subunit. There is a risk of photosensitivity
with this class of drug. Demeclocycline has the highest risk of photosensitivity.

75
This medication is used to treat creeping eruption or cutaneous larva migrans and larva currens:
A. Thiabendazole
B. Precipitated sulfur 6%
C. Malathion
D. Ivermectin
E. Permethrin
►A

Thiabendazole inhibits fumarate reductase, a helminth-specific enzyme. It is used to treat


creeping eruption or cutaneous larva migrans and larva currens.

76
This drug is the longest acting systemic corticosteroid in this group:
A. Betamethasone
B. Methylprednisolone
C. Prednisone
D. Predisolone
E. Hydrocortisone
►A

The answer is betamethasone is the longest acting systemic corticosteroid. The half life is 36-54
hours. Methylprednisolone, prednisone, prednisolone, triamcinolone are intermediate acting and
hydrocortisone is short acting with a half life of 8-12 hours.

77
What drug can potentiate bone marrow suppression when used concomitantly with azathioprine?
A. NSAIDS
B. Salicylates
C. Sulfonamides
D. Phenytoin
E. Allopurinol
►E

Concomitant allopurinol use, which inhibits xanthine oxidase, can lead to excess toxic purine
analogs via increased metabolism of azaithioprine via the HGPRT pathway, causing bone
marrow suppression.

526
78
What is used to reduce bladder toxicity from cyclophosphamide?
A. Mesna
B. Leukovorin
C. Cimetidine
D. Vitamin E
E. Folic acid
►A

Mesna or sodium 2-mercptoethanesulfonate, has been used to reduce bladder toxicity from
cyclophosphamide.

79
Methotrexate toxicity with myelosuppression is treated with what agent?
A. Folinic acid
B. Folic acid
C. Oral methylene blue
D. Cimetidine
E. Vitamin E
►A

Leukovorin, or folinic acid, is able to bypass dihydrofolate reductase in the cell division
pathway. It is administered under conditions of methotrexate-induced myelosuppression.
Leukovorin is also available as an oral form, and may be administered as a continual part of
methotrexate therapy, instead of folic acid. It is now believed that neither folic acid nor folinic
acid inhibit the efficacy of methotrexate�s antipsoriatic actions.

80
This antihistamine is used for diarrhea in mastocytosis:
A. Cromolyn
B. Doxepin
C. Cimetidine
D. Ranitidine
E. Hydroxyzine
►A

Cromolyn sodium blocks mast cell degranulation and is used for controlling diarrhea in
mastocytosis.

527
81
In addition to minocycline, which of the following drugs has been associated with drug-induced
lupus erythematosus-like syndrome?
A. Amiodarone
B. Itraconazole
C. Rifampin
D. Doxycycline
E. Hydralazine
►E

Drugs associated with drug-induced SLE include minocycline, hydralazine, procainamide,


isonaizid, penicillamine, and anti-convulsants.

82
The most specific marker of drug-induced lupus is:
A. Anti-histone Ab
B. ANA
C. Anti-Ro Ab
D. Anti-La Ab
E. Anti-ds DNA Ab
►A

Anti-histone Ab is most specific for drug-induced lupus. In addition to minocycline, hydralazine,


procainamide, isonaizid (INH), penicillamine and anti-convulsants have been associated with
drug-induced lupus-like syndrome.

83
The combination of doxorubicin and which medications has been reported to cause sticky skin?
A. Vancomycin
B. Amphotercin
C. Ketoconazole
D. G-CSF
E. Cisplatin
►C

Polsen et. al. reported a 29% incidence of sticky skin in patients treated with high dose
ketoconazole and doxorubicin for prostate cancer. Other medications reported to cause this
include etretinate and tretinoin. Polsen JA, Cohen PR, Sella: Acquired cutaneous adherence in
patients with androgen-independent prostate cancer receiving ketoconazole and doxorubicin:
medication-induced sticky skin. J Am Acad Dermatol: 32 (4):571-5 1995

528
84
A patient is diagnosed with pseudo-PCT with a normal porphyrin profile. All the medications are
involved in the cause of pseudo-PCT except:
A. Penicilin
B. Furosemide
C. Naproxen
D. Tetracycline
E. Nalidixic acid
►A

All of the above medications are involved in the pathophysiology of pseudo-PCT except for
penicillin. Patients with pseudo-PCT have normal porphyrin profile.

85
Which of the following statements regarding drug interactions is true?
A. Drugs that induce CYP3A enzymes may decrease levels of drugs which act as
substrates for CYP3A
B. CYP3A inhibitors may increase levels and cause toxicity of drugs metabolized
by cytochrome P-450
C. Terbinafine is not metabolized by cytochrome P-4503A4
D. Tobacco induces P-450 enzymes
E. All of the above are true
►E

The most relevant drug interactions in dermatology involve the hepatic biotransformation
pathways catalyzed by the cytochrome P-450 isoenzymes from the subfamilies CYP3A3/4.
Drugs that induce CYP3A enzymes may decrease levels of drugs which act as substrates for
CYP3A. CYP3A inhibitors may increase levels and cause toxicity of drugs metabolized by
cytochrome P-450. Terbinafine is not metabolized by cytochrome P-450, but inhibits CYP2D6-
mediated metabolism. Tobacco induces P-450 enzymes.

86
Side effects of PUVA include all of the following except:
A. Headache
B. Pruritus
C. Neutropenia
D. Insomnia
E. Nausea
►C

529
Side effects of psoralen with ultraviolet A light (PUVA) include side effects which are due to
phototoxic effects including pruritus, photoonycholysis, friction blisters, ankle edema and
hypertrichosis. In addition, there are adverse effects from methoxypsoralen including
gastrointestinal and neurologic effects, hepatotoxicity and exanthems. Neutropenia is not a
described side effect of PUVA.

87
The agent of choice used to acutely lower methemoglobin levels in patients taking dapsone is:
A. Cimetidine
B. Oral methylene blue
C. Homocysteine
D. Vitamin E
E. Glucose-6-phosphatase
►B

Cimetidine and vitamin E have both been known to provide prophylaxis against methemoglobin
formation. G6PD-deficient individuals are at greater risk of hematologic toxicity from dapsone.

88
The class of antibiotics that inhibit protein synthesis by binding to the 30S ribosomal subunit is:
A. Tetracyclines
B. Cephalosporins
C. Penicillins
D. Macrolides
E. Rifampin
►A

Tetracyclines inhibit protein synthesis by binding to the 30S ribosomal subunit. It is effective
agains both gram positive and gram negative organisms and mycoplasma infections

89
What antifungal is the best choice for a patient with mucocutaneous candidiasis who is currently
taking antacids?
A. Itraconazole
B. Fluconazole
C. Ketoconazole
D. Terbinafine
E. Griseofulvin
►B

530
Ketoconazole and itraconazole require an acidic environment. Fluconazole, however, does not
require an acidic environment and can work safely and effectively in patients taking antacids,
which can raise gastric pH levels.

90
Which of the following agents has UVA activity?
A. Sulisobenzone
B. PABA
C. Cinnamates
D. Salicylates
E. Padimate A
►A

Sulisobenzone is a benzophenone, a UVA blocker. The others are UVB blockers. Padimate A is
a PABA derivative.

91
Treatment with isotretinoin has been shown to cause increased colonization of the skin with
which of the following organisms?
A. Streptococcus pyogenes
B. Pseudomonas aeruginosa
C. Staphylococcus aureus
D. Pityrosporum orbiculare
E. Demodex folliculorum
►C

Staphylococcus aureus colonization tends to correlate with isotretinoin-induced reduction in


sebum production and may lead to infections. This complication may possibly be prevented with
pulsed intranasal mupirocen therapy. There has been a report of staphylococcus endocarditis in a
patient with underlying aortic insufficiency.

92
The laboratory abnormality most associated with cyclosporine is:
A. Hypermagnesemia
B. Hyponatremia
C. Hyperkalemia
D. Hypouricemia
E. Increased LDH
►C

531
The laboratory abnormalities associated with cyclosporine are decreased magnesium, increased
potassium, and increased uric acid. Renal function and blood pressure must also be monitored
closely in patients using cyclosporine.

93
An 87 year old female with chronic lymphocytic leukemia develops disseminated varicella-
zoster infection. She is hospitalized for treatment. Rapid intravenous infusion of acyclovir has
been associated with what complication?
A. Reversible obstructive nephropathy
B. Disseminated intravascular coagulation
C. Thrombocytopenia
D. Pulmonary fibrosis
E. Serum sickness
►A

Acyclovir is a guanosine analog. It is preferentially phosphorylated by viral thymidine kinase. It


inhibits viral DNA polymerase, halting viral DNA synthesis by chain termination. Rapid
intravenous infusion of acyclovir has been associated with a reverisble obstructive nephropathy.

94
The anti-HIV medication best known for causing a severe reaction which can result in fatality
upon rechallenge is:
A. Indinavir
B. Didanosine
C. Zidovudine
D. Abacavir
E. Nevirapine
►D

The hypersensivity reaction associated with abacavir usually resolves with cessation of the drug,
however upon rechallenge the reaction can be life-threatening.

95
All of the following are reported cutaneous side effects of zidovudine EXCEPT:
A. Trichomegaly
B. Diffuse and oral hyperpigmented macules
C. Hyperpigmented streaks in nails
D. Periungual pyogenic granulomas
E. None of the above (all are reported side effects)

532
►E

All of the following are cutaneous side effects reported with zidovudine, a nucleoside HIV
reverse transcriptase inhibitor. Periungal/paronychial eruptions resulting in pyogenic granuloma-
like lesions have also been reported with other HIV medications, including indinavir and
lamivudine.

96
Imiquimod has proven beneficial in the treatment of all of the following lesions except:
A. Erythroplasia of Queyrat
B. Superficial basal cell carcinoma
C. Actinic keratoses
D. Squamous cell carcinoma
E. Extramammary Paget‟s disease
►D

Imiquimod is an immune response modifier that acts via the toll like receptor 7. It induces
cytokines including interferon alpha, interleukins 1, 5, 6, 8, 10, and tumor necrosis factor. It has
been used to treat Erythroplasia of Queyrat, superficial basal cell carcinomas, actinic keratoses,
extramammary Paget‟s disease, and warts.

97
A patient takes oral clindamycin for a deep soft tissue infection due to a gram-positive organism.
The mechanism of action of clindamycin is most similar to which of the following antibiotics?
A. Erythromycin
B. Levofloxacin
C. Doxycycline
D. Penicillin
E. Rifampin
►A

Clindamycin is particularly effective against anaerobic and gram-positive organisms, particularly


those causing deep tissue infections. It works by binding to the 50S ribosomal subunit thereby
inhibiting protein synthesis. Macrolide antibiotics such as erythromycin also work by this
mechanism. Fluoroquinolones block DNA gyrase. Tetracyclines inhibit the 30S subunit of
ribosomes. Penicillin blocks transpeptidation of the bacterial cell wall.

98
Which antifungal is fungicidal?
A. Itraconazole

533
B. Terbinafine
C. Fluconazole
D. Clotrimazole
E. Griseofulvin
►B

Terbinafine is an allylamine antifungal that is fungicidal. It works via noncompetitive inhibition


of squalene epoxydation. Amphotericin is also fungicidal. The azoles are fungistatic and inhibit
14-alpha-demthylase to prevent ergosterol synthesis. Griseofulvin disrupts microtubule
formation.

99
Ketoconazole and itraconazole are potent inhibitors of which one of the following cytochrome
p450 isozymes?
A. 2C9
B. 2D6
C. 3A4
D. 1A2
E. 2C18
►C

Ketoconazole has been shown to be the strongest inhibitor of cytochrome p450 (CYP) 3A4.
Itraconazole is an inhibitor of CYP 3A4, whereas fluconazole inhibites CYP 2C9 significantly
more than minimal inhibitory role of CYP 3A4. Terbinafine inhibits CYP 2D6.

100
What is the most common cause of nonpigmented fixed drug eruption?
A. Naproxen
B. Tetracycline
C. Sulfonamides
D. Phenolphthalein
E. Pseudoephedrine
►E

Pigmented incontinence is usually prominent in a fixed drug eruption; yet occasionally, fixed
drug reactions do not result in long-lasting hyperpigmentation. The so-called nonpigmented fixed
drug eruption is distictive. Pseudoephedrine hydrochloride is by far the most common cause.

101

534
Of the new biologic therapies for psoriasis, which agent requires weekly CD4 T-cell count
monitoring?
A. Efalizumab
B. Alefacept
C. Infliximab
D. Etanercept
E. Adalimumab
►B

Alefacept is a human LFA-3/IgG fusion protein, which blocks LFA-3 on antigen presenting cells
from interacting with CD-2 on T-cells, preventing T-cell stimulation. Alefacept also eliminates
activated memory-effector T-cells, so weekly CD4 T-cell counts are required.

102
Which antihistamine has suppressor T-cell inhibitory activity?
A. Doxepin
B. Promethazine
C. Fexofenadine
D. Cimetidine
E. Cromolyn sodium
►D

Cimetidine is an H2 antihistamine that has suppressor T-cell inhibitory activity, by competitively


blocking their H2 receptors. Immunomodulatory effects are useful for treating mucocutaneous
candidiasis, verruca vulgaris, and condyloma acuminata.

103
Vascular leak syndrome has been associated with which chemotherapeutic agent?
A. Denileukin diftitox
B. Cytarabine
C. Methotrexate
D. Bleomycin
E. Interferon
►A

Denileukin difitox (brand name Ontak) is approved for the treatment of cutaneous T-cell
lymphoma. Denileukin difitox is a fusion protein composed from a portion of IL-2 with
diphtheria toxin. The chemotherapy is taken up by cells expressing high-affinity IL-2 receptors.
Capillary leak syndrome presenting with hypotension, edema, pleural effusions, and weight gain
caused by fluid retention has been reported in patients receiving denileukin difitox.

535
104
Which class of medication works by inhibiting DNA gyrase?
A. Quinolones
B. Tetracyclines
C. Aminoglycosides
D. Cephalosporins
E. Carbapenems
►A

Penicillins, cephalosporins, carbapenems, and vancomycin work on the cell wall. Quinolones
inhibit DNA gyrase. Tetracyclines and aminoglycosides work at the 30 S ribosomal subunit.

105
Methemoglobinemia is an adverse reaction to treatment with which agent?
A. Dapsone
B. Methotrexate
C. Plaquenil
D. Azathioprine
E. Cyclophosphamide
►A

Dapsone is a lipid-soluble sulfone drug that is used widely in dermatology in a variety of


conditions including dermatitis herpetiformis, leprosy, and neutrophilic dermatoses. Adverse
effects from dapsone are both pharmacologic and idiosyncratic and include hemolytic anemia,
methemoglobinemia, agranulocytosis, hypersensitivity syndrome and neuropathy. Of these, the
first two are pharmacologic and anticipated, to some degree, in most patients treated with
dapsone. However, the magnitude of toxicity varies greatly among individuals on the drug.
Methemoglobinemia is the formation of methemoglobin in the blood, which has a decreased
oxygen-carrying capacity compared with hemoglobin and can result in cyanosis. The reaction is
related to the N-hydroxy metabolites of dapsone, which are potent oxidants. G6PD-deficient
individuals are more susceptible to oxidative stresses, including those from dapsone metabolites,
and a baseline G6PD level is recommended prior to initiation of dapsone therapy. In the event of
emergent methemoglobinemia, oral methylene blue (100-300 mg/day) can be used to decrease
methemoglobin levels. However, if the patient is G6PD deficient, this strategy is ineffective.

106
Neutrophilic eccrine hidradenitis is associated most often with which chemotherapeutic agent?
A. 5-fluorouracil
B. Cytarabine

536
C. Dactinomycin
D. Doxorubicin
E. Vinca alkaloids
►B

Neutrophilic eccrine hidradenitis occurs in neutropenic patients with malignancies, usually acute
myelogenous leukemia. Cytarabine is the most commonly associated medication.

107
Etretinate is this times much more lipophilic than acitretin with increased storage in adipose
tissue:
A. 10x
B. 20x
C. 30x
D. 40x
E. 50x
►E

Etretinate is 50x more lipophilic than acitretin with increased storage in adipose tissue. The
highly lipid soluble etretinate lasts several years in the fatty tissues, in the presence of ethanol
and acitretin is re-esterified to etretinate.

108
This drug is associated with hemorrhagic cystitis which can lead to bladder carcinoma:
A. Cyclophosphamide
B. Methotrexate
C. Cyclosporin
D. Azathioprine
E. Mycophenolate mofetil
►A

Cyclophosphamide disrupts DNA cross linkage. It is the treatment of choice for Wegener's and
there is a risk for hemorrhagic cystitis. In up to 40% of patients there is a risk for transitional cell
bladder cancer.

109
Which of the following statements regarding podophyllin is NOT correct?
A. It is derived from the May apple plant
B. It binds tubulin
C. It arrests cells in telophase

537
D. It is contraindicated in pregnancy
E. None (all of these statements are true)
►C

Podophyllin is a crude cytotoxic extract from the May apple plant. It is antimitotic, arresting
cells in metaphase (not telophase) by binding to the protein tubulin. It may be teratogenic and
should not be used in pregnancy.

110
Tazarotene is what category for safety in pregnancy?
A. Category A
B. Category B
C. Category C
D. Category D
E. Category X
►E

Category X drugs include: acitretin, etretinate, estrogens, finasteride, 5-fluorouracil, flutamide,


isotretinoin, methotrexate, stanozolol, thalidomide, and tazarotene.

111
A 59 year-old woman is diagnosed with acute myelogenous leukemia. Induction chemotherapy
is initiated. A few days later the patient develops tender erythematous plaques on her face. What
is the most likely culprit drug?
A. Methotrexate
B. Cytarabine
C. 5-fluorouracil
D. Cyclophosphamide
E. Hydroxyurea
►B

The most likely diagnosis is neutrophilic eccrine hidradenititis. Cytarabine is the usual offending
agent.

112
Hemorrhagic cystitis is a risk of what chemotherapy?
A. Chlorambucil
B. Cyclophosphamide
C. Vinca alkaloids
D. Hydroxyurea

538
E. Azaithioprine
►B

Bladder toxicity is due to the acrolein metabolite of cyclophosphamide. Hemorrhagic cystitis is


associated with the increased risk of transitional cell carcinoma of the bladder. Mesna has been
used to reduce the toxic effect. The risk of cystitis is avoided by adequate fluid intake, frequent
voiding, and careful screening for hematuria.

113
The most common cause of fixed drug eruption is:
A. Tetracyclines
B. Macrolides
C. Naproxin
D. Pencillins
E. Cephalosporins
►A

The most common cause of fixed drug eruption is tetracyclines. There is also a risk of brown
discoloration of the teeth and delayed bone growth it should not be used in children in less than 9
years old.

114
A poikiloderma of the dorsal hands with a band-like distribution over the fingers and toes,
diffuse hyperpigmentation and leg ulcers upon withdrawal of the medication has been described
with this medication:
A. Hydroxyurea
B. Fluorouracil
C. Mycophenolate mofetil
D. Cyclophosphamide
E. Chlorambucil
►A

Hydroxyurea is a cytotoxic agent that is used to treat psoriasis, scleromyxedema, and Sweet's
syndrome. Poikiloderma of the dorsal hands with a band-like distribution over the fingers and
toes, diffuse hyperpigmentation, and leg ulcers upon withdrawal of the medication have been
described. Radiation recall, acral erythema and dermatomyositis like reactions are other rare side
effects.

115
Which of the following is not associated with calcipotriene?

539
A. Inactivation by basic pH
B. Increase in interleukin-10
C. Decrease in interleukin-2
D. Potential risk for photosensitivity
E. Inhibition of NF-kB
►A

Calcipotriene is relatively unstable and inactivated by acidic pH, ie with ammonium lactate and
salicylic acid. The other statements are true. Psoriasis is classically a TH1 disease and this
medicine upregulates TH2 cytokines it is commonly used to treat this condition. It has mild anti-
inflammatory properties via its inhibition of NF-kB. It also inhibits cell proliferation and induces
cell differentiation.

116
A patient is diagnosed with infectious mononucleosis and experiencing fever, pharyngitis, and
lymphadenopathy. The patient develops a morbilliform eruption after treatment with this
antibiotic:
A. Ampicillin
B. Penicillin
C. Erythromycin
D. Doxycycline
E. Minocycline
►A

The patient has infectious mononucleosis and is developing a morbilliform eruption from
treatment with ampicillin. It affects teenagers and young adults. They also develop malaise,
headache, pharyngitis, hepatosplenomegaly and fevers.

117
Which of the following statements regarding retinoids is true?
A. Tretinoin and isotretinoin are second-generation synthetic retinoids
B. Etretinate is a first-generation retinoid
C. Second-generation retinoids are polyaromatic retinoids
D. Tazarotene is a fourth-generation selective retinoid
E. Bexarotene is a third-generation retinoid
►E

First-generation retinoids are tretinoin (all-trans RA) and isotretinoin (13-cis RA). Second-
generation synthetic retinoids are etretinate, which was replaced by its metabolite acitretin.

540
Third-generation (polyaromatic retinoids) include the arotinoids, tazarotene, adapalene, and
bexarotene.

118
A patient has a single oval erythematous patch on his abdomen. Biopsy reveals pigment
incontinence and basal vacuolization. A diagnosis of fixed drug eruption is made. What is the
most common cause of fixed drug eruption?
A. Tetracycline
B. NSAIDs
C. Phenylephrine
D. Amoxicillin
E. Simvastatin
►A

A fixed drug eruption is a single or multiple oval or annular erythematous patches that arise as a
result of exposure to a systemic medication. Repeat exposure to the offending drug can cause
reappearance of the lesions in the same location. Tetracycline is the most common cause of fixed
drug eruption.

119
Which of the following statements regarding antifungal medications is TRUE?
A. Griseofulvin is safe for patients with variegate porphyria
B. Ketoconazole has been associated with gynecomastia
C. Griseofulvin is a cytochrome P450 3A4 inhibitor
D. Terbinafine is fungistatic
E. Ketoconazole is fungicidal
►B

Medications that can precipitate acute attacks in variegate porphyria include barbiturates,
estrogen, griseofulvin, sulfonamides and ethanol. Ketoconazole can produce impotence and
gynecomastia by interfering with androgen synthesis. Ketoconazole inhibits cytochrome (CYP)
P450 and most concerning, can rarely cause fulminant hepatitis. Griseofulvin induces CYP P450,
not inhibits it. Terbinafine is fungicidal along with amphotericin B. Ketoconazole is fungistatic.

120
Keratinocyte differentiation is enhanced by retinoids with all of the following EXCEPT:
A. Increased filaggrin production
B. Increased keratohyalin granules
C. Stimulation of ornithine decarboxylase
D. Odland body secretion of lipids

541
E. Increased keratin filaments
►C

Keratinocyte differentiation is enhanced by retinoids with increased filaggrin production,


increased keratohyalin granules, keratin filaments, and Odland body secretion of lipids.
Retinoids directly inhibit ornithine decarboxylase and therefore lessen inflammatory hyperplasia.

121
A patient is taking 15 mg of methotrexate each week for psoriasis. Which of the following
medications would be safe for use in this patient?
A. Tetracyclines
B. Phenytoin
C. Ibuprofen
D. Aspirin
E. Acetaminophen
►E

Methotrexate is a S-phase specific antimetabolite which competitively and irreversibly blocks


dihydrofolate reductase from catalyzing the formation of tetrahydrofolate, an important cofactor
in thymidylate and purine synthesis. Tetracyclines, phenytoin, phenothiazines, chloramphenicol,
NSAIDs, salicylates, and sulfonamides can all increase methotrexate levels by displacement of
plasma proteins. Acetaminophen is safe for concurrent use with methotrexate.

122
Over use of of which medication may lead to this clinical image?
A. Topical steroid
B. Topical antibiotic
C. Calcipotriene
D. Imiquimod
E. Hydroquinone
►E

Exogenous ochronosis has been reported with prolonged use of high concentration
hydroquinone. On pathology, a characteristic ochre colored deposit is noted between the collagen
bundles.

123
Which of the following sclerotherapy agents can have a disulfiram like reaction?
A. polidocanol
B. sodium morrhuate

542
C. glycerin
D. hypertonic saline
E. sodium tetradecyl sulfate
►A

Polidocanol is a detergent type of sclera agent and can have a disulfiram like reaction. It also has
a very rare risk of anaphylaxis. Sodium morrhuate is also a detergent sclera agent and has the
highest risk of anaphylaxis.

124
Which of the following is the most common adverse effect of Thalidomide therapy?
A. Diarrhea
B. Sedation
C. Skin discoloration
D. Photosensitivity
E. Hypertension
►B

Thalidomide was introduced in the late 1950's as a "safe" sleeping aide. It readily penetrates the
CNS, where it exerts a hyposedative effect comparable with barbiturates. By far, the most
common adverse effect from thalidomide is sedation, which in many patients may require that
primarily night-time doses be utilized.

125
The t 1/2 of isotretinoin is:
A. 1 hour
B. 7 hours
C. 20 hours
D. 2 days
E. 120 days
►C

The t 1/2 of isotretinoin is 20 hours. The other answers list the t 1/2 times of various retinoids.

126
A 56 year-old man presents with blue-gray discoloration on his face, ears, and dorsal hands.
What is the most likely offending agent?
A. Minocycline
B. Amiodarone
C. Chloroquine

543
D. Quinacrine
E. Clofazimine
►B

The patient presents with blue-gray discoloration in sun-exposed areas. The most likely
offending agent is amiodarone. Blue-gray discoloration from minocycline usually occurs on legs.
Chloroquine usually causes blue-gray discoloration in the sclerae, teeth, buccal mucosa, nail
beds, and pretibial areas. Quinacrine causes yellow discoloration of skin and conjunctiva.
CLofazimine usualy causes a red-brown discoloration.

127
The treatment of choice for erythema nodosum leprosum (ENL) is:
A. Thalidomide
B. Clofazamine
C. Rifampin
D. Rifampin and clofazamine
E. Isoniazid, rifampin and clofazamine
►A

Erythema Nodosum Leprosum is a type 2 reaction of leprosy. It most commmonly ocurrs in


lepromatous leprosy. Clinically, it presents as painful nodules and is associated with fever,
malaise, anorexia, and arthralgias. The treatment of choice is thalidomide.

128
Which of the following statements regarding the ocular toxicities of the antimalarial drugs is
NOT true?
A. Premaculopathy associated with changes in visual fields without visual loss is
reversible if the antimalarial is discontinued
B. True retinopathy is associated with "bull's eye" pigment deposition, central
scotoma, and diminished visual acuity
C. Risk of retinopathy is greatest for quinacrine, followed by chloroquine
D. Chloroquine and hydroxychloroquine should not be given together because of an
additive effect on retinotoxicity
E. The 4-aminoquinolones may have significant associated ocular toxicity
►C

Risk of retinopathy is greatest with chloroquine and does not exist for quinacrine.

129

544
What is the recommended period for contraception after cessation of acitretin therapy in the
United States?
A. 1 month
B. 3 months
C. 1 year
D. 2 years
E. 3 years
►E

Etretinate has a prolonged half-life of 80-160 days. The levels may persist up to 3 years in the
body. Acitretin can be converted to etretinate in the presence of ethanol.

130
Raynaud's phenomenon is a potential adverse effect of what chemotherapeutic agent?
A. 5-fluorouracil
B. Doxorubicin
C. Bleomycin
D. Vinca alkaloids
E. Hydroxyurea
►C

Bleomycin use has been associated with Raynaud's phenomenon occurring in digits treated with
intralesional therapy for periungual and plantar warts.

131
Which antibiotic is the treatment of choice for pregnant patients with Rocky Mountain Spotted
Fever?
A. Amoxicillin
B. Doxycycline
C. Erythromycin
D. Chloramphenicol
E. Trimethoprim
►D

Chloramphenicol is the treatment of choice for pregnant patients with Rocky Mountain Spotted
Fever. In non-pregnant patients, the treatment of choice is Doxycycline.

132
One of the main concerns for prescribing azathioprine to a gout patient with a normal level of
thiopurine methyltransferase on chronic allopurinol is:

545
A. Azathioprine does not work
B. Allopurinol does not work
C. Patient may develp photosensitivity
D. Patient may become pancytopenic
E. No concerns
►D

Allopurinol inhibits xanthine oxidase, an enzyme that catabolizes azathioprine. In patients


concurrently taking bothe medications, the allopurinol shunts more 6-MP from the xanthine
oxidase catabolic pathway to the hypoxanthine-guanine
phosphoribosyltransferase(HGPRT)anabolic pathway, creating an excess of purine analogs. This
in turn may lead to to excessive immunosuppression and risk of pancytopenia.

133
Which of the following oral agents has been effective in the treatment of Norwegian scabies?
A. Thiabendazole
B. Ivermectin
C. Mebendazole
D. Griseofulvin
E. Metroniddazole
►B

Ivermectin (Stromectol) is an anti-helminthic agent currently FDA-approved for the treatment of


strongyloides and onchocerciasis. Several publications have reported efficacy of this agent in the
treatment of scabies and head lice. Due to its low rate of adverse effects, its high rate of
effectiveness, and its ease of administration, some authors consider this agent to be the treatment
of choice for scabies and head lice. The mechanism of action of ivermectin is blockade of
glutamate-gated, chloride ion channels, with adverse effects on nerve and muscle resulting in
paralysis and death of the helminth or mite. The drug has a very low affinity for mammalian
chloride channels resulting in its relatively low toxicity. Adverse effects are rare and have been
associated with accidental intoxication. It should be avoided when there is compromise of the
blood-brain-barrier. Ivermectin is pregnancy category C. There are virtually no associated drug
interactions with oral ivermectin therapy.

134
This retinoid targets RXR receptors:
A. Tretinoin
B. Isotretinoin
C. Acitretin
D. Bexarotene

546
E. Etretinate
►D

Bexarotene targets RXR receptors. It is used in the treatment of mycosis fungoides refractory to
conventional therapy. Side effects include central hypothyroidism and hyperlipidemia. These
should be treated with synthroid, lipitor, and fenofibrate.

135
Which of the following medications mechanism of action is through inhibition of DNA-
dependent RNA polymerase?
A. Penicillin G
B. Cephalexin
C. Tetracycline
D. Ciprofloxacin
E. Rifampin
►E

Rifampin is an antibiotic used frequently for M. leprae and M. tuberculosis. It is the only drug
bactericidal to M. leprae. Its mechanism of action is by inhibiting RNA synthesis by inhibiting
DNA-dependent RNA polymerase. Penicillin G inhibits bacterial cell wall synthesis by blocking
the transpeptidation step. Cephalosporins have a similar mechanism of action, possessing a beta-
lactam ring. Tetracycline inhibits protein synthesis by binding and inhibiting the 30S ribosome
while the fluoroquinolone family inhibits DNA gyrase.

136
The treatment of choice for Wegner's granulomatosis is:
A. Chlorambucil
B. Systemic glucocorticosteroids
C. Cyclophosphamide
D. Methotrexate
E. Azaithioprine
►C

Wegener granulomatosis is an idiopathic disorder characterized by necrotizing granulomatous


vasculitis of the upper and lower respiratory tract, glomerulonephritis, and a variable degree of
small-vessel vasculitis. Testing for c-ANCA is 90% sensitive for the diagnosis when the
presentation is classic. Cyclophosphamide has been used very effectively and now is the usual
drug of choice for induction of remission.

137

547
Which one of the following is not true about the mechanism of action or effects of
glucocorticoids?
A. Decrease fibroblast production of collagen
B. Increase blood glucose
C. Decrease transcription of AP-1
D. Increase transcription of NF-kB
E. Form complexes with intracellular receptors
►D

Glucocorticoids play a diverse role in the human body. They modulate transcription of specific
genes that lead to an increase or decrease in the levels of specific proteins, ie they decrease
transcription of AP-1 and NF-kB. All the other statements are true. Side effects include
osteoporosis, hyperglycemia, hypertension, poor wound healing, peptic ulcers, Cushingoid
features, and muscle weakness.

138
The most common side effect of azathioprine is:
A. Bone marrow suppression
B. Neuropathy
C. Hepatotoxicity
D. Nephorotoxicity
E. Myopathy
►A

The most common side effect of azathioprine (Imuran) is bone marrow suppresion
(Pancytopenia). It is a rare adverse event resulting from excessive immunosuppression by
azathioprine. To prevent catastrophic bone marrow failure, regular complete blood counts should
be checked.

139
Which of the following systemic corticosteroids has the highest glucocorticoid activity?
A. Betamethasone
B. Methylprednisolone
C. Triamcinolone
D. Cortisone
E. Hydrocortisone
►A

Systemic corticosteroids are commonly used in dermatology. Short acting steroids, cortisone and
hydrocortisone, have the greatest mineralocorticoid activity, while cortisone has the lowest

548
glucocorticoid activity. Intermediate and long-acting steroids, methylprednisolone,
triamcinolone, dexamethasone, and betamethasone, have virtually no mineralocorticoid activity.
Dexamethasone and betamethasone have the highest glucocorticoid activity.

140
Concomitant use of methotrexate and what other drug is contraindicated because of the potential
increased risk of pancytopenia?
A. Folic acid
B. NSAIDs
C. Fish Oil
D. Acetaminophen
E. Systemic retinoids
►B

Drugs that simultaneously inhibit the folate metabolic pathway, such as NSAIDS, dapsone, or
trimethoprim-sulfamethoxazole, can increase hematologic toxicity when combined with
methotrexate.

141
This drug is a synthetic derivative of testosterone with anabolic properties. It is also used to
prevent angioedema that attaches in hereditary angioedema:
A. Stanozolol
B. Finasteride
C. Spironolactone
D. Flutamide
E. Doxepin
►A

Stanozolol and Danazol are synthetic derivatives of testosterone with anabolic properties. It is a
potent fibrinolytic activity used in the treatment of cryofibrinogenemia, lipodermatosclerosis,
and livedoid vasculitis.

142
Which antiparasitic agent is highly flammable?
A. Lindane
B. Permethrin
C. Precipitated sulfur
D. Thiabendazole
E. Malathion
►E

549
Malathion, an organophosphate cholinesterase inhibitor used to treat scabies and head lice, is
flammable.

143
The mechanism action of this cytotoxic agent is via inhibition of IMP dehydrogenase.
A. Azathioprine
B. Methotrexate
C. Hydroxyurea
D. 5-fluorouracil
E. Mycophenolate mofetil
►E

Mycophenolate mofetil (cellcept), a purine analog, blocks de novo purine synthesis by inhibiting
the enzyme inosine monophosphate dehydrogenase.

144
A patient has recalcitrant Sweet\'s syndrome. Hydroxyurea is initiated. Hydroxyurea works by
inhibiting which enzyme?
A. Ribonucleotide reductase
B. DNA gyrase
C. Dihydrofolate reductase
D. Inosine monophosphate dehydrogenase
E. Thymidine kinase
►A

Hydroxyurea is a S-phase specific cytotoxic agent which inhibits ribonucleotide reductase, an


enzyme responsible for converting ribonucleotides to deoxyribonucleotides in DNA synthesis.
Anemia, hepatitis, and renal toxicity are associated adverse effects. Fluoroquinolones inhibit
DNA gyrase. Methotrexate inhibits dihydrofolate reductase. Mycophenolate mofetil inhibits
inosine monophosphate dehydrogenase. Acyclovir utitilizes thymidine kinase to interrupt viral
replication.

145
Which antiparasitic agent acts by inhibiting fumarate reductase?
A. Ivermectin
B. Lindane
C. Permethrin
D. Thiabendazole
E. Cidofovir

550
►D

Thiabendazole inhibits fumarate reductase, a helminth-specific enzyme. It is used to treat


creeping eruption or cutaneous larva migrans and larva currens. Ivermectin blocks glutamate-
gated chloride ion channels, and is used to treat strongyloidiasis, onchocerciasis, and Norwegian
scabies. Lindane is an organochloride which blocks neural transmission, and is effective against
scabies, pubic lice, head lice, and body lice. Permethrin disables sodium transport channels in the
nerve cell membrane of the parasite. Cidofovir is an antiviral nucleotide analogue.

146
A child with systemic medium-vessel vasculitis is treated with a cytotoxic agent. While in the
hospital, he develops new-onset generalized tonic-clonic seizures. Which cytotoxic agent is
associated with tonic-clonic seizures?
A. Chlorambucil
B. Cyclophosphamide
C. Methotrexate
D. Doxorubicin
E. Hydroxyurea
►A

Chlorambucil is a nitrogen mustard derivative that is used as a steroid sparing agent in vasculitis,
Behcet\'s disease, dermatomyositis, and sarcoidosis. In children with nephritic syndrome or
adults with a seizure history, it can cause generalized tonic-clonic seizures.

147
Combination oral contraceptives decrease free testosterone levels by:
A. Directly binding free testosterone
B. Increasing SHBG (sex hormone binding globulin) production
C. Acting as competitive inhibitors of the androgen receptor
D. Acting as a GnRH agonist
E. Acting as a GnRH antagonist
►B

Oral contraceptives decrease free testosterone levels by increasing the production of sex
hormone binding globulin (SHBG).

148
Anemia, leg ulcers, poikilodermatous skin changes, hepatitis, renal toxicity, and acral erythema
are most commonly associated with what medication?
A. 5-fluorouracil

551
B. Hydroxyurea
C. Cyclosporine
D. Methotrexate
E. Doxorubicin
►B

The constellation of adverse effects is most closely associated with hydroxyurea.

149
The medication most acceptable for usage in patients with renal failure is:
A. Tetracycline
B. Minocycline
C. Demeclocycline
D. Oxytetracycline
E. Doxycycline
►E

Renal failure may prolong the half-life of most tetracyclines except doxycycline. Doxycycline is
excreted via the GI tract, unlike the other tetracyclines.

150
A non-pigmenting fixed drug eruption is known to be caused by what agent?
A. Tetracycline
B. Barbiturates
C. Phenopthalein
D. Naproxen
E. Pseudoephedrine
►E

All agents listed are associated with fixed drug eruptions, however, pseudoephedrine
hydrochloride is the one most commonly associated with non-pigmenting fixed drug eruptions.

151
Which cutaneous side effect is a common complication of nitrogen mustard therapy?
A. Bullous drug eruption
B. Telangiectasia
C. Hyperpigmentation
D. Allergic contact dermatitis
E. Fixed drug eruption
►D

552
Topical nitrogen mustard, or mechlorethamine, is an antineoplastic agent which works via
alkylation thereby inhibiting DNA synthesis. Allergic contact dermatitis occurs in two-thirds of
patients who are treated with topical nitrogen mustard in aqueous solution, but occurs in less than
5% of patients treated with the ointment based preparation.

152
Acitretin is a water soluble with very little lipid deposition. The recommended period for
contraception after acitretin therapy is:
A. 1 year
B. 2 years
C. 3years
D. 4 years
E. 5 years
►C

The recommended period for contraception after acitretin therapy is three years. Isotretinoin,
acitretin and bexarotene are water soluble with very little lipid deposition. It is un-detecable in
the serum after one month of stopping therapy.

153
Which one of the following agents when used prophylactically has demonstrated efficacy in
reducing skin cancer in organ transplant patients?
A. Tacrolimus
B. Acitretin
C. Imiquimod
D. Interleukin-2
E. Cyclosporine
►B

Systemic retinoids like acitretin have been used in the chemoprophylaxis of skin cancers. Several
studies have suggested a beneficial effect of retinoids with lowered incidences of skin cancer.
However, following cessation of the retinoids, the incidence normalizes.

154
A 25 year old female patient has a recurrent pustule that occurs in the same place on the lower
lip. The drug that she can use is the drug that is the prodrug of acyclovir. It is used for the
infection of HSV-1 and HSV-2 and can be used for recurrent infections:
A. Valacyclovir
B. Brivudin

553
C. Famciclovir
D. Griseofulvin
E. Dapsone
►A

Valacyclovir is a prodrug of acyclovir and is used in acute HSV 1 and 2 infections. It is


converted in first pass through gastrointestinal tract and liver to acyclovir. It is probably better
for zoster than acyclovir. It is not approved in immunosuppressed patients.

155
Which of the following medications can lead to hematologic toxicity when combined with
methotrexate?
A. Trimethoprim
B. Sulfonamides
C. Dapsone
D. All of these answers are correct
E. None of these answers are correct
►D

All of the above inhibit the folic acid metabolic pathway, and can lead to hematologic toxicity
when combined with methotrexate.

156
What is the difference between podophyllin and podophyllotoxin?
A. Essentially the same; they are interchangable in terms of treatment and side
effects
B. Podophyllotoxin contains quercetin which is a potent mutagens
C. Podophyllin contains kaempherol which is a potent mutagens
D. Podophyllotoxin is a phosphodiesterase inhibitor
E. Podophyllin reversibly binds tubulin inhibiting cells in metaphase
►C

Podophyllotoxin, also known as podofilox or Condolox, is a anti-mitotic agent that reversibly


binds tubulin, arresting cells in metaphase. It is used topically to treat genital warts. Podophyllin,
which has the same mechanism of action, contains kaempero and quercetin which are potent
mutagens. Both are derived from the May Apple plant. Cantharin, an antiviral agent derived
from the Blister beetle, is a phosphodiesterase inhibitior.

157
Which of the following is a potentially irreversible ocular side effects of antimalarial agents?

554
A. Corneal deposition - causing halos, blurred vision, photophobia
B. Neuromuscular eye toxicity
C. Retinopathy
D. Cataracts
E. Pterygium
►C

Three types of ocular adverse effects may develop from antimalarials: corneal deposits,
neuromuscular eye toxicity and retinopathy. Only retinopathy is potentially irreversible. It is
recommended that a patient be evaluated for retinopathy at baseline, then every 6 months by an
ophthamologist. Testing visual acuity, visual fields and performing a funduscopic examination
are considered acceptable for screening purposes.

158
What drug is known to cause scotomas?
A. Dapsone
B. Isotretinoin
C. Quinacrine
D. Chloroquine
E. Gold
►D

True retinopathy is associated with "bull's eye" pigment deposition, central scotoma, and
diminished visual acuity. Chloroquine is at greatest risk of causing retinopathy. Quinacrine is not
associated with the risk of retinopathy.

159
Thalidomide is most associated with what adverse effect?
A. Sensory neuropathy
B. Distal motor neuropathy
C. Oral ulceration
D. Photosensitivity
E. Hypothyroidism
►A

The most common presentation of the neuropathy from thalidomide is a mild proximal muscle
weakness with symmetric painful paresthesias of the distal extremities with accompanying lower
limb sensory loss. Hypothyroidism is a rarely reported adverse effect.

160

555
What medication's mechanism of action is via suppression of the halide-myeloperoxidase
system?
A. Cyclophosphamide
B. Quinicrine
C. Dapsone
D. Melphalan
E. Hydroxyurea
►C

Dapsone inhibits the neutrophil halide-myeloperoxidase system which results in an impaired


respiratory burst and subsequent tissue damage.

161
This retinoid has been shown to cause reversible hypothyroidism and is also a third generation
retinoid:
A. Bexarotene
B. Etretinate
C. Arotinoids
D. Tazarotene
E. Adapalene
►A

Bexarotene is a third generation retinoid. Other include arotinoid, tazarotene, adapalene, and
bexarotene that selectively act on specific receptors. Bexarotene has been shown to cause
reversible hypothyroidism.

162
Which chemical sunscreen has UVB and UVA II absorption capability?
A. Methyl anthranilate
B. Oxybenzone
C. Octyl salicylate
D. Titanium dioxide
E. Padimate O
►B

The benzophenones, oxybenzone and dioxybenzone, have the broadest absorption spectrum of
the chemical sunscreens, with UVB and UVA II range. Methyl anthranilate absorbs UVA II, and
octyl salicylate, and padimate O are UVB-absorbing chemicals. Titanium dioxide is not a
chemical absorber, it is a physical blocker.

556
163
Which site of the body has the highest penetration ability when topical medications applied
A. Face
B. Eyelids
C. Scrotum
D. Mucous membrane
E. Upper arms and legs
►D

Drug penetration varies depending on body site and thickness of stratum corneum. An important
consideration in topical therapy is that abnormal skin may have an altered (increased, decreased,
or absent) stratum corneum, thus changing the body site's barrier function. Regional penetration
of topical therapy is ranked from highest penetration to less penetration as following: Mucous
membrane > scrotum > eyelids > face > chest and back > upper arms and legs > dorsal hands and
feet > palms and soles > nails

164
Penile erosions are a reported side effect associated with which medication?
A. Trimethoprim-sulfamethoxazole
B. Acyclovir
C. Foscarnet
D. Azaithioprine
E. Bleomycin
►C

Genital ulcers have been reported as a complication of foscarnet therapy.

165
The risk of pseudotumor cerebri in patients taking isotretinoin is increased by:
A. Dehydration
B. Concomitant use of tetracycline
C. Concomitant use of TMP-SMX
D. Doses higher than 1.0 mg/kg/day
E. Comorbid affective disorder
►B

The risk of pseudotumor cerebri is increased in patients on isotretinoin and a tetracycline.

166
Which of the following medications is most likely to result in increased carbamazepine levels?

557
A. Minocycline
B. Rifampin
C. Erythromycin
D. Azithromycin
E. TMP-SMX
►C

Eythromycin inhibits the hepatic cytochrome P450 system and can increase serum levels and
potential toxicities of carbamazapene, theophylline, warfarin, digoxin, methylprednisolone.

167
At standard dosages, which of the following is fungicidal?
A. Terbinafine
B. Fluconazole
C. Ketoconazole
D. Itraconazole
E. Griseofulvin
►A

Terbinafine block ergosterol synthesis early in the synthetic pathway by inhibitins squalene
epoxidase. Squalene then accumulates within fungal cells and discupts cell membranes. At
standard dosaging, it is believed to be fungicidal. The other choices are fungistatic.

168
Which of the following statements is true regarding the absorption of antibiotics?
A. Fluoroquinolone absorption is not altered by antacids
B. Tetracycline absorption is impaired by the ingestion of calcium products but not
iron
C. Minocycline absorption is not significantly impaired by the ingestion of calcium
products but it should be taken on an empty stomach
D. Doxycycline absorption is not impaired by the ingestion of dairy products and
calcium
E. None of the above (all of the above statements are false)
►E

Antacids decrease the absorption of fluoroquinolones and should be taken at least 2 hours after
the drug. Tetracycline absorption is impaired by the ingestion of dairy products, calcium, and
iron or zinc salts. Minocycline and doxycycline absorption may be impaired by the ingestion of
those products; they may be taken on an empty stomach or with food. Thus, all of the statements
are false.

558
169
Antibiotics that inhibit the 50S ribosomal subunit during protein synthesis and is effective in
gram negative soft-tissue infections is:
A. Macrolides
B. Tetracyclines
C. Cephalosporins
D. Penicillin
E. Rifampin
►A

Macrolides inhibit the 50S ribosomal subunit during protein synthesis. Arthyromycin and
clarithromycin are effective in the treatment of gram-negative soft-tissue infections.
Erythromycin is effective against acne, pyodermas, erythrasma, and pitted keratolysis.

170
What testing can be performed to assess for methotrexate induced liver fibrosis, in lieu of liver
biopsy?
A. AST/ALT ratio
B. Alkaline phosphatase level
C. procollagen 3 testing
D. procollagen 1 testing
E. procollagen 4 testing
►C

procollagen 3 testing. Procollagen 3 has been shown to be a sensitive marker for methotrexate
induced liver fibrosis. Procollagen 1 is a marker for AFX.

171
This medication can cause a slate gray hyperpigmentation in photo-exposed areas:
A. Amiodarone
B. Clofazamine
C. Lithium
D. NSAIDS
E. Antimalarials
►A

Amiodarone can cause slate gray hyperpigmentation in photo-exposed areas. Histologically,


there are yellow-brown granules are seen in macrophages in the dermis.

559
172
The anti-viral agent used most often for acyclovir-resistent HSV and VZV infections is:
A. Valacyclovir
B. Gancyclovir
C. Foscarnet
D. Famcyclovir
E. Pencyclovir
►C

Foscarnet does not require phosphorylaton for antiviral activity. Therefore, it is achieved against
viruses resistant to acyclovir, famcyclovir, or gancyclovir on the basis of altered-kinase
activities.

173
Which antiviral agent has been associated with fatal thrombotic thrombocytopenic purpura in
AIDS and transplant patients taking high doses?
A. Acyclovir
B. Valacyclovir
C. Famciclovir
D. Penciclovir
E. Cidofovir
►B

Valacyclovir has been associated with severe and even fatal cases of thrombotic
thrombocytopenic purpura / HUS syndrome in AIDS and transplant patients taking high doses.

174
Which of the following medications used in dermatology is known to increase the risk for bone
fractures?
A. Dapsone
B. Isotretinoin
C. Acitretin
D. Prednisone
E. Doxycycline
►D

Long term use of oral corticosteroids has been shown to decrease bone mineral density (BMD).
This decrease in BMD leads to an increased risk for fractures. It was originally thought that
Vitamin A analogues such as isotretinoin and acitretin may also increase the risk of fracture but
in a large case-controlled study published in the May 2010 Archives illustrates that risk of

560
fracture is not associated with Vitamin A analogues. Vitamin A analogues have been associated
with hyperostosis.

175
The Mazzotti reaction is most closely associated with administration of which of the following
medications?
A. capecitabine
B. diethylcarbamazine
C. penicillamine
D. cisplatin
E. ipilimumab
►B

The Mazzotti reaction is characterized by urticaria, fever, chills, sweats, abdominal pain,
arthralgias, and lymphadenopathy. Hypotension and tachycardia may also be present, and the
reaction can be life threatening. Mazzotti reactions are most often associated with the
administration of diethylcarbamazine for the treatment of onchocerciasis, but may be seen during
treatment with other anti-helminthics, such as suramin. The pathogenesis of Mazzotti reactions is
thought to be consequent to abrupt release of parasitic antigens during cell death, prompting an
intense inflammatory cascade. Currently, ivermectin is the treatment of choice for
onchocerciasis, which has a lower incidence of associated Mazzotti reactions (approximately
10%).

176
Which of the following medications is most likely to interefere with the efficacy of oral
contraceptives?
A. Rifampin
B. Tetracycline
C. Doxycycline
D. Trimethoprim-sulfamethaxasole
E. Amoxicillin
►A

Rifampin has been shown to decrease the efficacy of oral contraceptives. It is an inducer of
cytochrome p450 which increases the metabolism of hormones thereby decreasing the efficacy
of oral contraceptives. There is no clear decrease in oral contraceptive efficacy with concomitant
use of ampicillin, ciprofloxacin, clarithromycin, doxycyline, metronidzole, ofloxacin, or
tetracycline.

177

561
Of the medications listed below, the safest to use during pregnancy is:
A. Penicillin
B. Erythromycin estolate
C. Doxycycline
D. Estrogens
E. NSAIDS
►A

Erythromycin estolate is contraindicated in pregnancy because of the risk of cholestatic hepatitis.


Other forms of erythromycin are safer for use in pregnancy. Tetracyclines are category D and
estrogens are category X. NSAIDS may promote persistent fetal circulation or oligohydramnios.

178
Which drug has mucocutaneous side effects which can include stomatitis, chelitis, lichen planus-
like eruptions, and pityriasis rosea-like eruptions?
A. Colchicine
B. Gold
C. Potassium iodide
D. Thalidomide
E. None of the above
►B

Mucocutaneous side effects, which are more common with injectable gold, include stomatitis,
cheilitis, lichen planus-like eruptions, and pityriasis rosea-like eruptions.

179
What is the target antigen for rituximab?
A. CD4
B. CD7
C. CD8
D. CD20
E. CD22
►D

Rituximab is an anti-CD20 monoclonal antibody. CD20 is a B-cell marker and is used to treat
Non-Hodgkins B-cell lymphoma. It is also approved for the treatment of rheumatoid arthritis.

180
Which of the following drugs has been known to cause penile erosions?
A. Gancyclovir

562
B. Cidofovir
C. Foscarnet
D. Penciclovir
E. Abacavir
►C

Foscarnet has been reported to cause penile erosions.

181
For which of the following medications is abdominal cramping and watery diarrhea a very
common side effect that may limit treatment?
A. Colchicine
B. Gold
C. Potassium iodide
D. Thalidomide
E. Chlorambucil
►A

Colchicine is an alkaloid with antimitotic activity that is used in dermatology for its effects on
neutrophils. The most common side effect from colchicine use is gastrointestinal distress with
abdominal cramping and watery diarrhea.

182
This medication is the drug of choice for erythema nodosum leprosum. It inhibits tumor necrosis
factor-alpha and suppresses monocyte and neutrophil phagocytosis. This medication is:
A. Thalidomide
B. Potassium iodide
C. Colchicine
D. Gold
E. Stanazolol
►A

Thalidomide is the drug of choice for erythema nodosum leprosum. It is known for its uses in
HIV associated mucosal ulceration and apthous stomatitis, chronic cutaneous lupus
erythematosus and chronic GVHD.

183
Which of the following is true regarding the use of cyclosporine?
A. Guidelines limit the continuous use of cyclosporine in the US to 5 years
B. Cyclosporine should not be used together with methotrexate

563
C. Cyclosporine is not an effective treatment for psoriatic arthritis
D. Cyclosporine inhibits the activation of antigen presenting cells
E. Cyclosporine has not shown efficacy in the treatment of chronic urticaria
►D

Cyclosporine binds to cyclophilin, an intracellular immunophilin, and inhibits the activity of


calcineurin phosphatase, which is then unable to dephosphorylate NFAT. Guidelines limit the
use of cyclosporine in the US to one year. Cyclosporine is an effective treatment for psoriatic
arthritis, alone or in combination with methotrexate. Cyclosporine inhibits the activation of T-
cells, NK cells, and antigen presenting cells. And cyclosporine has shown some efficacy in
histamine resistant chronic urticaria.

184
You see a patient with dark streaking in the nails, diffuse and oral hyperpigmented macules and
trichomegaly. Which of the following medication are they taking?
A. Imatinib
B. Zidovudine
C. Hydroxyurea
D. Cyclophosphamide
E. Bleomycin
►B

All of the listed medications have been reported to cause nail hyperpigmentation, but the
addition of oral hyperpigmented macules and trichomegaly (lengthened anagen phase of the
eyelashes). AZT can also cause bone marrow suppression with subsequent anemia and
granulocytopenia.

185
Ganciclovir and valganciclovir would be most appropriate to treat which infection?
A. HSV-1
B. HSV-2
C. VZV
D. CMV
E. EBV
►D

CMV is most susceptible to cidofovir, ganciclovir, and valganciclovir. HSV-1 and HSV-2 are
more susceptible to acyclovir and famciclovir. Foscarnet can be used to treat resistant strains of
CMV, HSV-1 and HSV-2.

564
186
All of the following statements are true regarding cyclosporin A EXCEPT:
A. Adverse effects include hypertrichosis and gingival hyperplasia
B. The most common eletrolyte abnormalities are hypokalemia and
hypermagnesemia
C. Forms a complex with cyclophilin, blocking its ability to activate calcineurin,
thus preventing calcineurin from de-phosphorylation NFAT-1
D. Metabolized by the hepatic cytochrome P-450 3A4 enzyme system
E. NSAIDs can potentiate renal toxicity when combined with cyclosporine
►B

The most common electrolyte abnormalities associated with cyclosporin A are hyperkalemia,
hyperuricemia, and hypomagnesemia.

187
Which of the following cytotoxic agents has been associated with poikiloderma of the dorsal
hands with a band-like distribution of the fingers and toes?
A. Intralesional bleomycin
B. Doxorubicin
C. Flurouracil
D. Azathioprine
E. Hydroxyurea
►E

Hydroxyurea has been associated with poikiloderma of the dorsal hands with a band-like
distribution of the fingers and toes. It has also been associated with diffuse hyperpigmentation,
and with leg ulcers upon withdrawal.

188
A 40 year old man with acute promyelocytic leukemia develops scrotal ulcers 1 month after
starting therapy. All tests for STD causes of ulcers as well as CMV and EBV are negative. What
is the most likely cause?
A. All-trans retinoic acid
B. Daunorubicin
C. Methotrexate
D. 6-mercaptopurine
E. Cytarabine
►A

565
All-trans-retinoic acid (ATRA) has been shown to improve the outcome of patients with acute
promyelocytic leukemia (APL). However, various adverse effects of ATRA treatment have been
noted, such as scrotal and genital ulcers.

189
A 15 year old boy presents with a 4 month history of pigmented bands on several fingernails and
toenails. The most like etiology is:
A. Peutz-Jeghers syndrome
B. Chloroquine therapy
C. Minocycline therapy
D. Nevomelanocytic nevi
E. Acral lentiginous melanoma
►C

Melanonychia occurring simultaneously on several nails is most likely to be due to minocycline


therapy. Blue-black pigmentation may be present in nails, skin, scars and sclerae.

190
Which of the following is not a side effect of bleomycin?
A. Erythromelalgia
B. Pulmonary fibrosis
C. Flagellate hyperpigmentation
D. Serpentine supravenous hyperpigmentation
E. Acrosclerosis
►D

All of the above are side effects of bleomycin except serpentine supravenous hyperpigmentation.
This is a side-effect of 5-fluorouracil.

191
Which of the following is considered most safe in pregnancy?
A. Benzoyl peroxide
B. Epinephrine
C. Methotrexate
D. Azelaic Acid
E. Isotretinoin
►D

566
Azelaic Acid is pregnancy category B. The other choices are in less safe pregnancy categories.
Epinephrine is category C, Benzoyl peroxide is category C, and Methotrexate and Isotretinoin
are category X.

192
Which of the following medications would you not want a woman using an IUD as her only
source of contraception to take?
A. Ibuprofen
B. Cefalexin
C. Etanercept
D. Infliximab
E. Lidocaine
►A

NSAIDS (including ibuprofen) and Azathiprine have been associated with IUD failure. The
other listed medications are all pregnancy category B and should be safe for use in this patient.

193
Which of the following statements is NOT true regarding the categories for safety of drug use in
pregnancy?
A. Drugs are category A if controlled studies in humans show no risk to the fetus
B. Drugs are category B if controlled human studies show no risk to the fetus but
may show risk to animals, or if no risk has been shown in animal studies but no
human studies have been conducted
C. Drugs are category C if risk to the human fetus has been demonstrated, but
animal studies are equivocal
D. Drugs are category D if controlled studies show risk to human fetus, but in some
instances benefits may outweigh risks
E. Category X drugs are contraindicated in pregnancy
►C

Drugs are category C if risk to the human fetus cannot be ruled out, studies are lacking, or animal
studies are equivocal. Drugs for which risk to the human fetus has been demonstrated are
pregnancy category D.

194
Which of the following drugs may cause acute generalized exanthematous pustulosis?
A. Ampicillin
B. Azithromycin
C. Cefazolin

567
D. Mercury
E. All of these answers are correct
►E

Acute generalized exanthematous pustulosis (AGEP) has been associated with beta-lactam
antibiotics, macrolide antibiotics, cephalosporins, and mercury.

195
What is the half-life of isotretinoin?
A. 7 hours
B. 20 hours
C. 50 hours
D. 30 days
E. 120 days
►B

The half-life of isotretinoin is 20 hours. The half lives of bexarotene, acitretin, etretinate are 7
hours, 50 hours, 120 days respectively.

196
This drug is a second generation cephalosporin and has activity against gram positive bacilli:
A. Cefazolin
B. Cephradrine
C. Cephalexin
D. Cefaclor
E. Cefotaxime
►D

Cefazolin, cephradrine, cephalexin are all first generation cephalosporin. The correct answer is
cefaclor, which is a second generation cephalosporin. Cefotaxime is a 3rd generation
cephalosporin.

197
A patient is being treated by a dermatologist. Yellowish changes of his sclera and yellowish
discoloration of his dorsal hands and feet is noted. Which medication is most likely responsible?
A. Quinacrine
B. Minocycline
C. Hydroxychloroquine
D. Terbinafine
E. Prednisone

568
►A

Quinacrine frequently produces a yellow discoloration of the sclera and skin, especially over the
dorsal hands and feet. Minocycline and hydroxychloroquine can cause bluish-gray
hyperpigmentation. Terbinafine and prednisone are not associated with alteration of cutaneous
pigmentation.

198
A 10 year old child with a seizure disorder develops a morbiliform eruption and elevated LFT's
two weeks after starting Dilantin therapy. As his physician you:
A. Continue Dilantin and treat rash with topical corticosteroids
B. Discontinue Dilantin and begin Phenobarbital
C. Discontinue Dilantin and begin carbamazepine
D. Restart Dilantin once the rash resolves
E. Discontinue Dilantin and begin valproic acid
►E

Anticonvulsant hypersensitivity syndrome (also drug rash with eosinophilia and systemic
symptoms (DRESS0 and dilantin hypersensitivity syndrome) presents with cutaneous eruption
accompanied by fever, facial edema, lymphadenopathy, leukocytosis and hepatitis. Cross
reactivity is present in all aromatic anticonvulsants including phenytoin, carbamazepine and
phenobarbitol. There is no cross reaction with valproic acid.

199
Which of the following retinoids is most lipophilic?
A. Etretinate
B. Acitretin
C. Tretinoin
D. Bexarotene
E. Isotretinoin
►A

Isotretinoin, acitretin, and bexarotene are water-soluble, with very little lipid deposition. Water-
soluble retinoids are undetectable in the serum 1 month after stopping therapy. Etretinate is 50
times more lipophilic than acitretin. It can last several years in fatty tissues.

200
A patient with acute diarrhea is prescribed antibiotic treatment for his symptoms. He
subsequently suffers from nausea and vomiting after ingesting alcohol. What is the most likely
medication he is taking?

569
A. Ciprofloxacin
B. Azithromycin
C. Penicillin
D. Clindamycin
E. Metronidazole
►E

This patient is most likely taking metronidazole for acute diarrhea secondary to giardella.
Metronidazole causes antabuse-like reactions with ingestion of alcohol.

201
This antihistamine is used for cold urticaria:
A. Cyproheptadine
B. Diphenhydramine
C. Promethazine
D. Chlorpheniramine
E. Hydroxyzine
►A

For cold urticaria, the antihistamine that is first generation is cyproheptadine. First generation
antihistamine have the side effect of sedation, increased appetite, dry mouth, and constipation.

202
All of the following drugs can cause linear IgA dermatosis except:
A. Vancomycin
B. Lithium
C. Amiodarone
D. Captopril
E. Actinomycin D
►E

Linear IgA dermatosis can be due to vancomycin, lithium, amiodarone, captopril, and penicillin.
Actinomycin D can cause folliculitis.

203
Which drug is known to cause an SCLE-like eruption?
A. Tylenol
B. Minocycline
C. Barbituates
D. Furosemide

570
E. Terbinafine
►E

SCLE-like reaction are known to occur most likely from glyburide, griseofulvin, captopril,
hydrochlorothiazide, penicillamine, piroxicam, and terbinafine.

204
Which drug may increase levels of digoxin?
A. Amoxicillin
B. Warfarin
C. Erythromycin
D. Minocycline
E. Cephalexin
►C

Erythromycin inhibits the cytochrome P-450 system, which may result in increased levels of
digoxin, among many other drugs.

205
Which of the following are retinoid side effects?
A. Reversible hypothyroidism
B. Diffuse interstitial skeletal hyperostosis
C. Premature epiphyseal closure
D. Pseudotumor cerebri
E. All of these answers are correct
►E

Bexarotene has been shown to cause reversible hypothyroidism, not hyperthyroidism. Systemic
retinoids have been shown to cause diffuse interstitial skeletal hyperostosis, premature
epiphyseal closure, and pseudotumor cerebri (risk increased with concommitant use of
tetracyclines).

206
Which of the following antiviral agents is NOT phosphorylated by viral thymidine kinase?
A. Acyclovir
B. Famciclovir
C. Gancyclovir
D. Cidofovir
E. Valacyclovir
►D

571
Cidofovir is a nucleotide analogue antiviral agent. It does not require phosphorylation by virus,
but is converted by host cell kinases to a diphosphate.

207
A patient presents with tingling and burning of their lateral upper lip. They report that blisters
will form which then crust and heal. She gets these episodes once or twice each year. You
prescribe acyclovir. What is the mechanism of action of acyclovir when treating this type of
infection?
A. Acts on viral thymidine kinase
B. Inhibits viral DNA polymerase
C. Inhibits viral RNA polymerase
D. Enhances CD8+ T-cell function leading to immune destruction of the virally
infected cells
E. Is a non-competitive inhibitor of viral DNA polymerase at the pyrophosphate
binding site
►B

Acyclovir relies upon the fact that thymidine kinase is produced at a higher rate in herpes
infected cells than in noninfected cells. It is a guanosine analog that is preferentially
phosphorylated by viral thymidine kinase which then inhibits viral DNA polymerase, thus
halting viral DNA synthesis by chain termination. Acyclovir does not inhibit viral RNA
polymerase or boost immune destruction of infected cells. It is not a non-competitive inhibitor of
viral DNA polymerase at the pyrophosphate binding site – this mechanism of action is that of
Foscarnet, also an antiviral active against HSV.

208
A 58-year old female receives thio-TEPA (trietheylenethiophosphoramide) for adenocarcinoma
of the breast. Which of the following cutaneous side effects might she expect?
A. Generalized hyperpigmentation sparing the palmar creases and mucous
membranes
B. Hyperpigmentation of the teeth with permanent pigmentation of the gingival
margin
C. Intense flushing of the skin
D. Hyperpigmentation of the axillae
E. Alternating colors of hair
►D

Thio-TEPA is an alkylating agent used in a variety of cancers. It can cause pruritus, urticaria,
angioedema, and hyperpigmentation localized to occluded areas. Generalized Addisonian-

572
hyperpigmentation may be caused by busulfan. Hyperpigmentation of the teeth with
discoloration of the gingival margins is associated with cyclophosphamide. Intense flushing of
the skin is usually seen with dacarbazine and carmustine. Alternating colors of hair, representing
the "flag sign," can be seen with methotrexate.

209
Which of the following events is most important in the pathogenesis of this painful eruption?
A. Increased expression of FasL
B. Reduction in circulating tumor necrosis factor
C. Reduction in circulating IL-6
D. Overexpression of keratins 6 and 16
E. Cleavage of desmoglein 1
►A

Toxic epidermal necrolysis is a life threatening drug eruption characterized by widespread


epidermal necrosis. The exact etiology of the keratinocyte necrosis has not been fully elucidated.
However, FasL (FasL and Fas are able to trigger apoptosis) has been shown to be upregulated in
TEN. There is also drug induced increased secretion of granulysin from cytotoxic T cells, and
NK cells.

210
For which of the following medications is sedation a very common side effect that may limit
treatment?
A. Colchicine
B. Gold
C. Potassium iodide
D. Thalidomide
E. Chlorambucil
►D

Sedation is a very common side effect of treatment with thalidomide. It is additive with other
sedatives, such as alcohol and barbiturates.

211
Which drug has been associated with an increased incidence of serum sickness in children?
A. Rifampin
B. Clarithromycin
C. Ciprofloxacin
D. Clindamycin
E. Cefaclor

573
►E

Cefaclor has been associated with an increased incidence of serum sickness in children; the other
drugs have not.

212
A patient with a long standing history of nodulocystic acne vulgaris on isotretinoin should avoid
tetracyclines with the drug to avoid complications such as:
A. Vitamine A exaggerated effect
B. Pseudotumor cerebri
C. Hepatotoxicity
D. Elevated triglycerides
E. Arthralgias
►B

Isotretinoin is a oral retinoid that helps with nodulocystic acne. When isotretinoin is combined
with tetracycline there is an increased risk for pseudotumor cerebri. It should not be used
together or in patients with a history of pseudotumor cerebri that has been diagnosed by a
neurologist. Pseudotumor cerebri can be diagnosed by measuring the intraocular pressure, in
which it will be elevated.

213
Alternate-day administration of oral steroids can reduce all of the following side effects except?
A. Growth impairment
B. HPA axis suppression
C. Cataracts
D. Peptic ulcer disease
E. Opportunisitic infection
►C

Alternate-day corticosteroid dosing regimens does not decrease the risks of posterior subcapsular
cataracts, osteoporosis, and possibly osteonecrosis.

214
A 44 year old female with HIV/AIDS develops grouped painful vesicles in a T4 dermatomal
distribution. Valacyclovir is prescribed. What severe reaction has been associated with the use of
valacyclovir in AIDS patients?
A. Thrombotic thrombocytopenia purpura/hemolytic uremic syndrome
B. Stevens-Johnson syndrome
C. Acute renal failure

574
D. Jarisch-Herxheimer reaction
E. Disseminated intravascular coagulation
►A

Valacyclovir is a prodrug of acyclovir, and has enhanced bioavailability and converts rapidly and
completely to acyclovir. Severe and even fatal cases of thrombotic thrombocytopenia
purpura/hemolytic uremic syndrome have been reported in AIDS and transplant patients taking
high doses of valacyclovir.

215
What is the half-life of isotretinoin?
A. 100 days
B. 100 hours
C. 50 hours
D. 20 hours
E. 10 hours
►D

The half-life of isotretinoin is 20 hours.

216
This antibiotic can reduce the efficacy of oral contraceptives and should not be used together:
A. Rifampin
B. Minocycline
C. Doxycycline
D. Clarithromycin
E. Ceftriaxone
►A

Rifampin has been shown to reduce the efficacy of oral contraceptives and should not be used in
conjunction with them. All other antibiotics such as ampicillin, ciprofloxacin, clarithromycin,
tetracyclines, metronidazole, doxycyline and minocycline all do not change the efficacy of oral
contraceptives.

217
The following drugs have been implicated in drug-induced subacute cutaneous lupus
erythematosus:
A. Terbinafine
B. Verapamil
C. Pravastatin

575
D. All the above are correct
E. None of the above are correct
►D

All of the above choices have been implicated in drug-induced subacute cutaneous lupus
erythematosus.

218
Which of the following class of medications has been associated with acquired brachial
dyschromatosis?
A. Non-Steroidal Anti-inflammatory medications
B. ACE-inhibitors
C. Anti-virals
D. HMG-CoA Reductase inhibitors
E. Protease inhibitors
►B

Acquired brachial dyschromatosis is a condition described as asymptomatic, gray-brown patches


with geographic borders, occasionally interspersed with hypopigmented macules, on the dorsum
of the forearms, mostly bilaterally and seen in middle aged women. Epidermal atrophy, basal
layer hyperpigmentation, elastosis and angiectases were histopathologic features. An association
with Civatte's poikiloderma as well as hypertension and/or antihypertensive drugs, especially
ACE-inhibitors, is suggested.

219
Which of the following pairings of antifungal agents and their mechanisms of action is NOT
correct?
A. Terbinafine: Inhibits squalene epoxidase
B. Itraconazole: Inhibits 14-alpha-demethylase
C. Griseofulvin: Disrupts microtubule mitotic spindle formation
D. Ketoconazole: Blocks conversion of lanosterol to ergosterol
E. Fluconazole: Inhibits squalene epoxidase
►E

Terbinafine inhibits squalene epoxidase and blocks the biosynthesis of ergosterol, a sterol
essential to the integrity of fungal cell membranes. Itraconazole inhibits 14-alpha-demethylase,
blocking lanosterol conversion to ergosterol. Griseofulvin disrupts microtubule mitotic spindle
formation causing metaphase arrest. Ketoconazole has a mechanism of action similar to
itraconazole. Fluconazole also inhibits 14-alpha-demethylase, not squalene epoxidase.

576
220
A patient who has been on dapsone for dermatitis herpetiformis develops cyanosis and appears
symptomatic. This patient is diagnosed with methemoglobinemia by his primary care doctor
after doing a complete blood count and hemoglobin. The deficiency that this patient most likely
has is:
A. Glucose-6-phosphate dehydrogenase (G6PD)
B. Agranulocytosis
C. Aplastic Anemia
D. Pancytopenia
E. Xanthine Oxidase Deficiency
►A

Dapsone is used in the treatment of dermatitis herpetiformis. Contraindications include severe


cardiovascular disease, marked renal insufficiency, and sulfonamide allergy. Patients need to be
screened for glucose-6-phosphate dehydrogenase (G6PD) deficiency. This is common among
blacks and Asians.

221
This antiviral drug is known to cause penile erosions that does not require phosphorylation for
antiviral activity and is a non-competitve inhibitor of viral DNA polymerase is:
A. Foscarnet
B. Acyclovir
C. Gancyclovir
D. Valacyclovir
E. Cidofovir
►A

Foscarnet is a noncompetitive inhibitor of viral DNA polymerase at the pyrophosphate binding


site. It does not require phosphorylaton for antiviral activity and therefore active against viruses
resistant to acyclovir, famciclovir or ganciclovir.

222
A patient with AIDS is given combination anti-retroviral therapy. Which of the following
antiviral medications used for HIV is associated with lipodystrophy with abnormal fatty
deposits?
A. Indinavir
B. Abacavir
C. Didanosine
D. Zidovudine
E. Foscarnet

577
►A

Indinavir is a protease inhibitor used for the treatment of HIV. The protease enzyme is responible
for the final assembly of new viral proteins. The protease inhibitors, particularly indinavir, have
been associated with lipodystrophy, which manifests as abnormal fatty deposits known as the
\"buffalo hump\" and \"protease pouch.\"

223
Which of the following medications is associated with anti-myeloperoxidase antibodies?
A. Isoniazid
B. Minocycline
C. Penicillamine
D. TNF-alpha inhibitors
E. Griseofulvin
►B

Minocycline, which is widely used in the treatment of acne, often without adequate supervision,
may induce arthritis and livedo vasculitis associated with anti-MPO (pANCA). (O Elkayam. Ann
Rheum Dis. 1996) Isoniazid most often causes drug-induced lupus associated with anti-histone
antibodies. Penicillamine and TNF-inhibitors are associated with anti-dsDNA antibodies.
Griseofulvin can exacerbate lupus but is most often associated with drug-induced SCLE and
anti-Ro antibodies.

224
Which of the following is NOT known to increase methotrexate levels?
A. Salicylates
B. Tetracyclines
C. Phenothiazines
D. NSAIDs
E. None of these answers are correct(all are known to increase methotrexate levels)
►E

Tetracyclines, phenytoin, phenothiazines, chloramphenicol, NSAIDs, salicylates, and


sulfonamides, among other drugs, can all increase methotrexate levels by displacement of plasma
proteins.

225
Onycholysis is a potential side effect of treatment with which therapy?
A. Dapsone
B. Ketaconazole

578
C. Chloroquine
D. Tetracycline
E. Retinoids
►D

Tetracycline has been associated with onycholysis and photo-onycholysis. Chloroquine may
cause a blue-brown discoloration of the nail bed and retinoids are associated with paronychia.

226
Which of the following medications is associated with painful symmetrical erythema and
paresthesias of the palms and soles?
A. sorafenib
B. cetuximab
C. bortezomib
D. erlotinib
E. gefitinib
►A

Sorafenib is a small-molecule multi-kinase inhibitor used in the treatment of renal cell cancer
and hepatocellular cancer. Notably it inhibits Raf kinase, which functions in the Ras signaling
pathway, in addition to blocking VEGFR-2, VEGFR-3, FLT3, and PDGFRb signaling. Reports
of hand and foot syndrome have been noted, characterized by acral erythema and paresthesias.
This side effect seems to resolve rapidly after discontinuation of treatment.

227
Bleomycin can be used for intralesional therapy for HPV infection, It damages the DNA directly
during the:
A. M phase
B. S phase
C. G1 phase
D. T phase
E. A phase
►A

Bleomycin damages DNA by binding to the M and the G2 phase It is effective in treatment of
HPV and it associated with Raynaud's phenomenon occurring in digits treated with intralesional
therapy for periungual and plantar warts.

228
Which of the following is correct regarding mycophenolate mofetil?

579
A. It is pregnancy category C drug
B. Metronidazole has been shown to increase the bioavailability of mycophenolate
mofetil
C. It depletes the de novo production of guanosine nucleotides
D. It can be toxic in individuals with TPMT deficiency
E. The most common side effect is anemia.
►C

Mycophenolate mofetil is a lymphocyte selective immunosuppressive agent that inhibits de novo


purine synthesis. Specifically, it depletes guanosine nucleotides by inhibiting inosine
monophosphate dehydrogenase. The most commonly reported side effects are GI and are dose-
dependent. Fluoroquinolones, rifampin, and metronidazole have been shown to decrease the
bioavailability of mycophenolate mofetil, which may result in lower circulating levels.
Medications that result in elevated concentrations of mycophenolate mofetil include salicylates
and probenecid. There is also a possibility of reduced concentration of nevirapine when
coadministered with mycophenolate mofetil. It is currently classified as FDA pregnancy category
D. Azathioprine can be toxic in individuals with TPMT deficiency, not mycophenolate mofetil.

229
If a patient develops a phenytoin hypersensitivity reaction, which anticonvulsant is the better
alternative therapy?
A. Carbamezapine
B. Phenytoin
C. Phenobarbital
D. Valproic acid
E. None of these answers are correct
►D

Carbamezapine, phenytoin, and phenobarbital are known to cross-react with one another.

230
Gray-green discoloration of the mid-portion of permanent teeth is a side effect of?
A. Doxycycline
B. Tetracycline
C. Minocycline
D. Fluoroquinolones
E. clindamycin
►C

580
In contrast to tetracycline staining of the teeth, which occurs in childhood and produces a brown
discoloration along the gingival third, minocycline stains the permanent teeth in adults, with a
gray-green discoloration of the mid-portion of the tooth.

231
Dapsone is used for a patient with dermatitis herpetiformis. Dapsone induces anti-inflammatory
effects by primarily inhibiting what type of cell?
A. Polymorphonuclear leukocyte
B. T cell lymphocyte
C. Macrophage
D. Mast cell
E. Langerhans cell
►A

Dapsone is used to treat a wide variety of dermatoses, including dermatitis herpetiformis, bullous
SLE, erythema elevatum diunitum, Sweet\'s syndrome, and others. The anti-inflammtory effect
of this medication is most effective against neutrophils because of inhibition of myeloperoxidase
activity and chemotactic abilities.

232
What antihistamine can cause gynecomastia, impotence, and loss of libido?
A. Cyproheptadine
B. Chlorpheniramine
C. Cimetidine
D. Doxepin
E. Fexofendine
►C

Cimetidine competitively inhibits dihydrotestosterone at the androgen receptor site and can
exhibit anti-androgen effects.

233
Patients that are using dapsone and sulfapyridine may experience hemolytic anemia and
methemoglobinemia. The treatment for methemoglobin is:
A. Methylene blue
B. Cloroquine
C. G6PD
D. Leucovorin
E. Dapsonase
►A

581
Patients on dapsone and sulfapyridine can experience hemolytic anemia and
methemoglobinemia. It is dose related and occur with varying degrees within all individuals.
Oral methylene blue is used in emergency situations to lower methemoglobin levels.

234
What feature best distinguishes lichen planus from lichenoid drug eruption?
A. segmental hypergranulosis
B. parakeratosis
C. eosinophils
D. spongiosis
E. saw tooth pattern
►B

Lichenoid drug reaction and lichen planus are best distinguished clinically. However, there are
clues to lichenoid drug, including parakeratosis and eosinophils. The presence of parakeratosis
has been shown to be more sensitive than eosinophils in the diagnosis of lichenoid drug reaction.

235
What tetracycline is least phototoxic?
A. Minocycline
B. Doxycycline
C. Oxytetracycline
D. Tetracycline
E. Demeclocycline
►A

Minocycline is least phototoxic. Demeclocycline and doxycycline are the most phototoxic of all
the tetracyclines. Onycholysis can accompany tetracycline-induced phototoxicity.

236
Which of the following antifungal agent works by way of inhibiting ergosterol synthesis by
blocking squalene epoxidation:
A. Itraconazole
B. Terbinifine
C. Naftifine
D. Itraconazole and Terbinifine
E. Terbinifine and Naftifine
►E

582
Terbinafine and Naftifine work by way of inhibiting ergosterol synthesis by blocking squalene
epoxidation (B&C). Itraconazole works by inhibiting ergosterol synthesis by blocking Lanosterol
14-alpha demthylase.

237
Which of the following retinoid medications is a second-generation synthetic retinoid?
A. Acitretin
B. Isotretinoin
C. Tretinoin
D. Adapalene
E. Bexarotene
►A

First generation synthetic retinoids include tretinoin and isotretinoin. Second generation
synthetic retinoids are etretinate and acitretin. Tazarotene, adapalene, and bexarotene are third
generation.

238
What is the treatment of choice for methemoglobinemia?
A. Methylene blue
B. Aspirin
C. Iron
D. Hydration
E. Observation
►A

Methylene blue is reduced in the presence of NADPH and diaphorase II to leukomethylene blue,
which then reduces methemoglobin (Fe3+) to hemoglobin (Fe2+)

239
Painful periungual pyogenic granulomas have been associated with what medication?
A. Ketoconazole
B. Indinavir
C. Doxycycline
D. Tazarotene
E. Valacyclovir
►B

Periungual pyogenic granulomas and painful paronychial eruptions have been reported in
association with various anti-HIV medications including, indinavir, zidovudine, and lamivudine.

583
240
A 21-year old male presents with a sudden onset of fever, rash, facial edema, leukocytosis and
hepatitis four weeks after starting phenytoin for seizures. This patient most likely had which risk
factor for developing this skin condition:
A. Slow acetylator
B. Deficiency of epoxide hydroxylase
C. Renal failure
D. Recent vaccination
E. Thyroid disease
►B

This patient has anticonvulsant hypersensitivity syndrome most likely resulting from a
deficiency of epoxide hydroxylase. Slow acetylators are also predisposed to this condition after
being given sulfonamides. Renal failure is a risk factor with allopurinol administration. Recent
vaccination and thyroid disease are not known risk factors.

241
A 52 year old male presents from home 5 days after excision of a basal cell carcinoma on his left
shin with pain, erythema, and a purulent discharge from the surgical site. He is placed on
Cephalexin for a total of 10 days. Two days later, he complains of worsening pain, redness, and
discharge. Which of the following is the best antibiotic choice?
A. Doxycycline
B. Amoxicillin
C. Azithromycin
D. Polymixin B
E. Ciprofloxacin
►A

This patient presents with a surgical wound infection unresponsive to cephalexin. This is likely
secondary to community acquired MRSA infection. Doxycycline is the best choice for
community acquired MRSA infection. Bactrim, not listed, is another option. Amoxicillin,
Azithromycin, and Ciprofloxacin are not typically used to treat MRSA infections as resistance is
high with these medications. Polymixin B is a topical antibiotic with anti-pseudomonal
properties.

242
Dapsone inhibits the myeloperoxidase activity of PMNs. One of the motor side effects of
dapsone are:
A. Motor peripheral neuropathy

584
B. Motor central neuropathy
C. Sensory peripheral neuropathy
D. Sensory central neuropathy
E. Dermatonal sensory changes
►A

Patients that are on dapsone can experience motor peripheral neuropathy. They can also
experience hemolytic anemia and methemoglobinemia.

243
Which one of the following sites has the highest percutaneous absorption of topical drugs?
A. Scrotum
B. Eyelid
C. Chest
D. Acral area
E. Mucous membrane
►E

Mucous membranes have the highest absorption of topical medicines. Percutaneous absorption,
from highest to lowest, is as follows: mucous membrane > scrotum > eyelids > face > chest and
back > upper extremity > lower extremity > acral.

244
Terbinafine exerts its antifungal activity by what manner?
A. Inhibition of 14-a demethylase
B. Inhibition of squalene epoxidase
C. Inhibition of epoxide hydroxylase
D. Interference with cell respiratory processes
E. Direct binding to membrane sterols, increasing permeability
►B

Terbinafine, an allylamine, interferes with ergosterol synthesis by inhibiting squalene epoxidase.


The azoles inhibit 14-a demethylase. Nystatin is a polyene which binds irreversibly to membrane
sterols, resulting in a permeability shift. Ciclopirox does not appear to affect sterol biosynthesis
but instead interferes with cell respiratory processes.

245
This antibiotic inhibits RNA synthesis by inhibiting DNA-dependent RNA polymerase. It also
causes orange-red discoloration of urine and tears. This antibiotic is:
A. Rifampin

585
B. Clindamycin
C. Fluoroquinolones
D. Acyclovir
E. Tetracycline
►A

Rifampin inhibits RNA synthesis by inhibiting DNA dependent RNA polymerase. It is effective
in tuberculosis and atypical mycobacterial infections. It is also effective in cutaneous
leishmaniasis and rhinoscleroma.

246
This category for drug safety in pregnancy indicates that risk to the human fetus can not be ruled
out, studies are lacking, animal studies are equivocal:
A. Category C
B. Category A
C. Category B
D. Category X
E. Category D
►A

Category C means that risk to the human fetus cannot be ruled out, studies are lacking; animal
studies are equivocal.

247
Side effects from this antihistamine include gynecomastia, impotence, and loss of libido:
A. Doxepin
B. Cyproheptadine
C. Promethazine
D. Fexofenadine
E. Cimetidine
►E

Cimetidine, an H2 antihistamine, also competitively inhibits dihydrotestosterone at the androgen


receptor site, with resultant antiandrogen side effects including gynecomastia, impotence, and
loss of libido.

248
Terfinadine is an antihistamine that has been used in the past. However, it is no longer available
in the USA due to issues with :
A. Life-threatening cardiac arrhythmias

586
B. Thyroid cancer
C. Seizures
D. Cyanosis
E. Agranulocytosis
►A

Terfenadine is an anti-histamine that was one of the first nonsedating antihistamines. It is no


longer available in the USA because of its propensity to cause life-threatening cardiac
arrhythmias. The problem is worsen by interaction with macrolide antibiotics and imidazole
antifungals.

249
All of the following topical antioxidants have demonstrated cutaneous anticarcinogenic effects in
mice except:
A. Zinc
B. Vitamin C
C. Tea polyphenois
D. Vitamin E
E. Silymarin
►A

Anti-oxidants are thought to be protective against photoinjury by neutralizing oxygen radicals.


Vitamin C, Vitamin E, tea polyphenois, and silymarin are all anti-oxidants.

250
Which of the following is known to induce lichen planus-like eruptions?
A. Dapsone
B. Doxepin
C. Minocycline
D. Gold
E. Mercury
►D

Mucocutaneous side effects of gold include stomatitis, cheilitis, lichen planus- like eruptions,
and pityriasis rosea-like eruptions.

251
What is the mechanism utilized by the co-administration of probenicid to raise blood levels of
penicillins in patients with infections that require high blood levels?
A. Inhibition of cytochrome P-450 hepatic biotransformation system

587
B. Competitive inhibition of b-lactam binding sites
C. Displacement of plasma proteins
D. Synergistic effect of probenicid with penicillins
E. Prolongs the half-life of penicillins by decreasing renal tubular secretion
►E

Probenicid is co-administered with penicillin to prolong its half-life through decreased renal
tubular secretion when higher blood levels are warranted.

252
Which of the following drugs binds iron and thereby significantly prevents absorption?
A. Methotrexate
B. Azathioprine
C. Cyclosporine
D. Mycophenolate mofetil
E. Tacrolimus
►D

Mycophenolate mofetil binds with Fe preparations preventing its absorption. Oral iron
supplements markedly reduce absorption of mycophenolate mofetil (CellCept®). It is
recommended that iron be taken four to six hours before, or two hours after mycophenolate
mofetil.

253
This drug is the most common drug known to cause a skin reaction:
A. Ampicillin
B. Potassium chloride
C. Digoxin
D. Meperidine
E. Prednisone
►A

The most common drugs that have been known to cause a skin reaction is ampicillin, Penicillin
G, Cephalosporins, and Heparin. All the other drugs have a very low incidence of drug reaction
and do not have a high incidence of a rash.

254
Which of the following antibiotics is the only drug that is bactericidal to Mycobacterium leprae?
A. Rifampin
B. Levofloxacin

588
C. Minocycline
D. Amikacin
E. Pentavalent antimony
►A

Rifampin inhibits RNA synthesis by blocking DNA-dependent RNA polymerase. It is effective


in tuberculosis and atypical mycobacterial infections. It is the only drug that is bactericidal to
Mycobacterium leprae. Cutaneous leishmaniasis and rhinoscleroma also respond to rifampin.

255
The SPF of a sunscreen is based on applying the sunscreen at what concentration?
A. 1 mg/cm2
B. 2 mg/cm2
C. 3 mg/cm2
D. 4 mg/cm2
E. 5 mg/cm2
►B

A sunscreen SPF is based on using it at a concentration of 2 mg/cm2 which is about 1 ounce or


30 grams for the entire average sized body. It also is about 3-5 grams for the head and neck.

256
The anti-CD 20 antibody rituximab is FDA-approved for treatment of which of the following?
A. Mycosis fungoids
B. Metastatic melanoma
C. Psoriasis
D. Paraneoplastic pemphigus
E. Non-Hodgkin‟s lymphoma
►E

Rituximab (brand name Rituxan) is a monoclonal antibody is approved for the treatment of
CD20 non-Hodgkin‟s lymphoma. Rituximab is a monoclonal antibody directed against B
lymphocytes which are CD20.

257
Which of the following systemic agents has been shown to be the most effective in the treatment
of toenail onychomycosis?
A. Ketoconazole
B. Griseofulvin
C. Itraconazole

589
D. Fluconazole
E. Terbinafine
►E

Craford et al. reviewed the available literature examining the efficacy of systemic anti-fungals
and performed a meta-analysis. Pooled analysis of cure rates at 11 and 12 months suggested that
terbinafine was more effective than itraconazole.

258
This azole can cause photosensitivity and accelerated photoaging:
A. Voriconazole
B. Fluconazole
C. Ketoconazole
D. Terbinafine
E. Oxiconazole
►A

Voriconazole can cause photosensitivity and accelerated photoaging. Is a triazole antifungal


medication that is generally used to treat serious, invasive fungal infections. These are generally
seen in patients who are immunocompromised, and include invasive candidiasis, invasive
aspergillosis, and certain emerging fungal infection.

259
The antifungal drug that acts by disrupting microtubule mitotic spindle formation causing
metaphase arrest is:
A. Griseofulvin
B. Fluconazole
C. Ketoconazole
D. Itraconazole
E. Terbinafine
►A

Griseofulvin disrupts microtubule mitotic spindle formation causing metaphase arrest. It is


effective against dermatophytes but not yeast. It is resistant to Trichophyton.

260
A patient being treated for aspergillosis develops altered vision. What is the most likely
medication he is taking?
A. Caspofungin
B. Voriconazole

590
C. Amphotericin B
D. Itraconazole
E. Griseofulvin
►B

Voriconazole blocks ergosterol synthesis by inhibiting 14-alpha-demethylase. Side effects


include visual disturbances in 30%, headaches, nausea, vomiting, fever, and peripheral edema.

261
What is the most likely congenital defect associated with isotretinoin therapy?
A. Atrial septal defect
B. Ventral septal defect
C. Cranial synostosis
D. Spina bifida
E. Phocomelia
►C

Isotretinoin is thought to cause congenital defects by interfering with neural crest development.
The most likely congenital defect is cranial synostosis.

262
Which of the following antiparasitic agents is an organophosphate cholinesterase inhibitor?
A. Precipitated sulfur
B. Thiabendazole
C. Ivermectin
D. Lindane
E. Malathion
►E

Malathion is an organophosphate cholinesterase inhibitor.

263
Which member of the tetracycline family is most likely to have caused this photomediated
reaction?
A. Demeclocycline
B. Doxycycline
C. Minocycline
D. Oxytetracycline
E. Tetracycline
►A

591
The tetracycline family of antibiotics are bacteriostatic and act by inhibiting protein synthesis.
Each member of the family may cause photosensitivity, but demeclocycline is the most
photosensitizing.

264
A patient is on potassium iodide for the treatment of erythema nodosum. The patient is starts to
experience signs of hypothyroidism. This effect is called the:
A. Wolff-Chaikoff effect
B. Nitritoid reaction
C. Alkaloid reaction
D. Glutamate reaction
E. Potassium reaction
►A

Patients on potassium iodide can experience the Wolff- Chaikoff effect. It can impair
autoregulatory mechanisms and lead to hypothyroidism. TSH should be checked a month after
therapy is initiated. It competes with organic iodides from binding to thyroid.

265
Which of the following biologic agents is pregnancy category C?
A. Alefacept
B. Infliximab
C. Efalizumab
D. Etanercept
E. All of these answers are correct
►C

Efalizumab is pregnancy category C. The other drugs listed are pregnancy category B.

266
Which of the following is a side effect of hydroquinone cream?
A. Telangiectasia
B. Photosensitivity
C. Ochronosis
D. Tachyphylaxis
E. Atrophy
►C

592
Exogenous ochronosis is an uncommon complication of irreversible pigmentation due to overuse
of topical hydroquinone (1,4 dihydroxybenzene). Hydroquinone acts to by melanocyte pigment
production by auto-oxidation of melanin, tyrosinase and phenol oxidases.

267
The class of antibiotics that block bacterial cell wall synthesis through inhibition of penicillin-
binding proteins that catalyze transpeptidation is:
A. Cephalosporins
B. Tetracyclines
C. Penicillins
D. Macrolides
E. Rifampin
►A

Cephalosporins resembles the penicillins and possesses a beta lactam ring. It block bacterial cell
wall synthesis through inhibition of penicillin binding proteins that catalyze transpeptidation. It
treats soft tissue infections that are caused by staphylococci and non-enterococcal streptococci.

268
Which one of the following antifungals requires an acidic environment for its absorption?
A. amphotericin B
B. fluconazole
C. griseofulvin
D. terbinafine
E. itraconazole
►E

Itraconazole is a triazole whose mechanism, like the other azoles, is inhibition of cytochrome P-
450 enzyme lanosterol 14-alpha demethylase, with resultant inhibition of lanosterol conversion
to ergosterol. Ergosterol is an essential component of fungal cell membranes. Itraconazole is a
weak base, which is highly lipophilic and virtually insoluble in water. It is ionized only at a low
pH. Griseofulvin is administered in divided doses for the treatment of tinea capitus. Its
absorption is optimized when given with a fatty food. Oral fluconazole is very well absorbed
when given orally with over 90% bioavailability. Terbinafine, though highly lipophilic, has 70 to
80% absorption, when administered orally, with a bioavailability not significantly impacted by
food intake.

269
This antiviral is a nucleotide analogue and does not require phosphorylation by virus but is
converted by host cell kinases to a diphosphate. The antiviral is:

593
A. Cidofovir
B. Xidovudine
C. Foscarnet
D. Gancyclovir
E. Famciclovir
►A

Cidofovir is a nucleotide analogue and does not require phosphorylation by virus and is
converted by host cell kinases to a diphosphate. It is usually active against CMV isolates that is
resistant to ganciclovir and foscarnet.

270
Which one of the following sunscreens has an absorption spectrum primarily in the UVA range?
A. Padimate O
B. Octyl Salicylate
C. Parsol 1789 (butyl dibenzoylmethane)
D. PABA (para-aminobenzoic acid)
E. Cinnamates
►C

Avobenzone (Parsol 1789) is primarily a UVA blocker. Photostability of avobenzone may be a


problem if it is combined with octyl methoxycinnamate. Salicylates, PABA, Padimate O, and
cinnamates are primarily UVB blockers.

271
This antifungal drug has been know to exacerbate lupus erythematosus, photoallergy, and
porphyria. This drug is contraindicated in patients with a history of porphyria:
A. Griseofulvin
B. Fluconazole
C. Ketoconazole
D. Itraconazole
E. Terbinafine
►A

Griseofulvin should not be used in patients that have porphyria. It can exacerbate lupus
erythematosus and photoallergy. It is also absorbed by fatty meals. It is not effective in
candidiasis.

272
What is a possible metabolic abnormality with long term systemic glucocorticosteroid therapy?

594
A. Hyperlipidemia
B. Hypoglycemia
C. Hyperkalemia
D. Hypercalcemia
E. Hypokalemic acidosis
►A

Long term systemic steroid therapy can result in hyperlipidemia, especially


Hypertriglyceridemia. Hyperglycemia, not hypoglycemia may occur with steroid therapy.
Hypokalemic alkalosis may result from long term steroid use. Uncommonly, hypocalcemia may
develop with resultant tetany.

273
How long after isotretinoin therapy can one safely begin trying to conceive?
A. Immediately
B. Two weeks
C. One month
D. One year
E. Three years
►C

A woman should wait one month before trying to conceive after taking isotretinoin to prevent
birth defects. After taking acitretin a woman should wait three years before trying to conceive.

274
What drug can induce native SLE?
A. Hydralazine
B. Procainamide
C. Isoniazide
D. TNF inhibitor
E. Minocycline
►D

Drug induced lupus consists of systemic lupus type symptoms with anti-histone antibodies.
Common culprit drugs are hydralazine, procainamide, isoniazid, and minocycline. Cessation of
the medication usually results in improvement of symptoms. The TNF inhibitors also induce
systemic lupus, but this typically is an unmasking of native lupus that is not associated with anti-
histone antibodies and does not remit with cessation of treatment.

275

595
The only biologic that is pregnancy category C is:
A. Adalumimab
B. Etanercept
C. Efaluzimab
D. Alefacept
E. Infliximab
►C

The only biologic that is pregnancy category C is efaluzimab. The other biologics are category
B. Efaluzimab is a humanized antibody that binds CD11a.

276
Weekly CD4 T-cell counts are recommended for psoriasis patients treated with which biologic
agent?
A. Alefacept
B. Etanercept
C. Efalizumab
D. Infliximab
E. None of the above
►A

Alefacept eliminates activated memory T-cells, so weekly CD4 T-cell counts are recommended.

277
What is the best medication to lower isotretinoin induced hypertriglyceridemia?
A. Simvastatin
B. Gemfibrozil
C. Niacin
D. Cholestyramine
E. All are equally effective
►B

Gemfibrozil generally reduces trygliceride levels to a greater extent than niacin, cholestyramine,
and the HMG-CoA reductase inhibitors.

278
Which biologic agent is infused intravenously?
A. Alefacept
B. Etanercept
C. Efalizumab

596
D. Infliximab
E. None of these answers are correct
►D

Infliximab is infused intravenously.

279
This can increase the levels of fluoroquinolones and lead to inhibition of CYP 1A2:
A. Warfarin
B. ACE inhibitors
C. Calcium
D. Iron
E. Magnesium
►A

Warfarin can lead to elevated levels of fluoroquinolones by inhibition of CYP 1A2. It should be
taken two hours apart from each other. All the others reduce levels of fluoroquinolones except
ACE inhibitors.

280
Which of the following chemotherapeutic agents is not cell cycle specific?
A. Methotrexate
B. 5-fluorouracil
C. Cyclophosphamide
D. Azathioprine
E. Hydroxyurea
►C

Cyclophosphamide is an alkylating agent that cross links DNA at any point during the cell cycle.
Methotrexate, 5-fluorouracil, azathioprine, and hydroxyurea are S-phase specific cytotoxic
agents. Methotrexate is a folic acid analog that blocks dihydrofolate reductase. 5-FU is a
pyrimidine analog that prevents the conversion of deoxyuridine monophosphate to
deoxythymidine monophosphate in DNA synthesis. Azathioprine is a purine analog that is
converted into the active metabolite 6-thioguanine bit hypoxanthine guanine
phosphoribosyltransferase. Hydroxyurea inhibits ribonucleotide reductase which normally
converts ribonucleotides to deoxyribonucleotides in DNA synthesis.

281
Which of the following inhibits bacterial cell wall synthesis by complexing with the carrier
protein C55-prenol pyrophosphatase.

597
A. Bacitracin
B. Polymyxin
C. Neomycin
D. Mupirocin
E. Silver sulfadiazine
►A

Bacitracin is a polypeptide antibiotic produced by the Tracey I stain of Bacillus subtilis. It


inhibits bacterial cell wall synthesis by complexing with the carrier protein C55-prenol
pyrophosphatase, which is involved in the transfer of polysaccharides, liposaccharides, and
peptidoglycans to a growing cell wall.

282
What antifungal is known to cause gynecomastia and impotence?
A. Griseofulvin
B. Itraconazole
C. Terbinafine
D. Ketoconazole
E. Fluconazole
►D

Ketoconazole is known to cause gynecomastia and impotence, by interfering with androgen and
glucocorticoid synthesis.

283
A middle aged gentleman with pemphigus vulgaris is managed with azathioprine. Which of the
following substances is the active metabolite?
A. 6-thioguanine
B. Azathioprine
C. 6-mercaptopurine
D. Thiouric acid
E. 6-thioinosine monophosphate
►A

Azathioprine is a purine analog that is used as a steroid-sparing agent in a variety of


dermatologic inflammatory disorders. Azathioprine is converted into 6-mercaptopurine before
being converted into the active metabolite, 6-thioguanine, by the hypoxanthine guanine
phosphoribosyltransferase pathway. Other inactive metabolites are produced via xanthine
oxidase and thiopurine methyltransferase.

598
284
The most common side effect of treatment with interferon-alpha is:
A. Weight loss
B. Nausea
C. Liver toxicity
D. Flu-like symptoms
E. Spastic diplegia
►D

The most common side effect of treatment with interferon-alpha is flu-like symptoms of fever,
chills, myalgias, headache and arthralgias. Prophylactic administration of non steroidal anti-
inflammatory medications may alleviate some of these symptoms.

285
Which of the following medications would be safe for use in pregnancy?
A. Tazarotene
B. Finasteride
C. Etretinate
D. Azeleic acid
E. 5-fluorouracil topically
►D

All of the listed medications except azeleic acid are pregnancy category X. Azeleic acid is a
pregnancy category B medication, meaning that there are controlled human studies that show no
risk to fetus but may show risk to animals, or no risk in animals with no human studies
conducted.

286
The treatment of choice for scabies in pregnant women is:
A. Malathion
B. 1% permethrin
C. Thiabendazole
D. Precipitated sulfur 6%
E. Ivermectin
►D

Many authors consider precipitated sulfur to be the drug of choice for the treament of scabies in
pregnant women, although there are no published studies for this indication (see Wolverton).
Permethrin of note is category B in pregnancy.

599
287
The mechanism of action of podophyllin most closely resembles that of what other drug listed
below?
A. Chlorambucil
B. Dactinomycin
C. Colchicine
D. 5-fluorouracil
E. Permethrin
►C

Both podophyllin and colchicine have antimitotic activity. They bind to tubulin dimers,
interfering with mitotic spindle and microtubule assembly.

288
Which of the following chemotherapeutic agents causes increased growth of eyelashes?
A. Interleukin 2
B. Interferon alpha
C. Mitomycin
D. Cytarabine
E. Methotrexate
►B

Interferons can cause increased growth of eyelashes. Trichomegaly has been reported after
treatment with interferon-alpha in patients with chronic hepatitis, B-cell lymphoma, chronic
granulocytic leukemia, and cutaneous melanoma. Trichomegaly has also been reported in
associatoin with latanoprost, minoxidil, cyclosporine, phenytoin, psoralen, and penicillamine.

289
Which of the following agents is NOT a UVA blocker?
A. Dioxybenzone
B. Amyl p-dimethylaminobenzoate
C. Dibenzoylmethane
D. Avobenzone
E. Red veterinary petrolatum
►B

Dioxybenzone is one of the benzophenones (as in oxybenzone and sulisobenzone), which are
UVA blockers. Amyl p-dimethylaminobenzoate is a UVB blocker. Dibenzoylmethane
(avobenzone; Parsol 1789) is a UVA/UVB blocker. Red veterinary petrolatum is a UVA blocker.

600
290
After exposure to a tick, a patient develops a rickettsial infection, requiring a tetracycline-class
antibiotic. A review of systems reveals that he has a history of chronic renal insufficiency.
Which is the only tetracycline that can be used in patients with renal failure?
A. Doxycycline
B. Minocycline
C. Tetracycline
D. Demeclocycline
E. Azithromycin
►A

Tetracycline antibiotics inhibit protein synthesis by binding to the 30S ribosomal subunit. They
are effective against gram positive and negative organisms, Mycoplasma, Chlamydia, Rickettsia,
and others. Doxycycline, which is excreted by the Gi tract, is the only tetracycline for use in
patients with renal failure.

291
A 66 year old female presents with palpable purplish lesions and small necrotic ulcers of the
extremities. Additional workup reveals renal and hepatic involvement, and a diagnosis of
Wegener's granulomatosis is made. Which of the following cytotoxic agents is the treatment of
choice for Wegener's granulomatosis?
A. Cyclophosphamide
B. Chlorambucil
C. Methotrexate
D. Bleomycin
E. Mycophenolate mofetil
►A

Cyclophosphamide is a nitrogen mustard derivative. It is cell-cycle nonspecifc and works by


producing DNA cross-linkages at any point in the cell cycle. It is the treatment of choice for
Wegener's granulomatosis.

292
All of the following are associated with fixed drug eruption except:
A. Tetracyclines
B. Barbiturates
C. NSAIDs, naproxen
D. Sulfonamides
E. Azathioprine
►E

601
Azathioprine has been known to cause SCLE like eruption. Tetracyclines, Barbiturates, NSAIDs,
naproxen, sulfonamides, and phenopthalein in laxatives have been known to cause fixed drug
eruptions.

293
Which antifungal medication has a black box warning for CHF?
A. Terbinafine
B. Griseofulvin
C. Itraconazole
D. Ketoconazole
E. Caspofungin
►C

Itraconazole should be avoided in heart patients due to its black box warning against CHF. Other
reported adverse effects from itraconazole include photosensitivity and COPD.

294
Patients on azathioprine can have bone marrow suppression due to low genetic activities of:
A. Thiopurine methyltransferase activity (TPMT)
B. Deoxyuridine monophosphate
C. Ribonucleotide reductase
D. Inosine monophosphate dehydrogenase
E. HGPRT
►A

Patients that are on azathioprine that have low levels of xanthine oxidase or thiopurine
methyltransferase activity (TPMT) and on allopurinol use can have bone marrow suppression.
Allopurinol inhibits xanthine oxidase activity or in patients with genetically low TPMT allele
activity.

295
Imiquimod is FDA approved for the treatment of which of the following?
A. Actinic keratosis
B. Squamous cell carcinoma in situ
C. Infiltrative basal cell carcinoma
D. Superficial spreading melanoma
E. Psoriasis vulgaris
►A

602
Imiquimod is FDA approved for the treatment of actinic keratoses, superficial basal carcinomas,
and condyloma acuminate. It is not used to treat infiltrative basal cell carcinomas, squamous cell
carcinoma in situ, superficial spreading melanoma, or psoriasis. Off label uses include common
warts, molluscum contagiosum, Bowen‟s disease (SCCIS), and keloids.

296
Dihydroxyacetone is found in which of the following products?
A. Rubber accelerators
B. Shampoos
C. Artificial nails
D. Hair dyes
E. Sunless tanning preparations
►E

Dihydroxyacetone is the active ingredient in sunless tanning preparations. Upon oxidation it


turns orange-brown and binds to the stratum corneum. Rubber accelerators contain
mercaptobenthothiazole, carba mix, thiuram mix, or mercapto mix. Shampoos, especially "tear-
free" ones, may contain cocamidopropyl betaine. Artificial nails may contain methyl
methacrylate or ethyl acrylate. Hair dyes often have paraphenylenediamine.

603
604

You might also like